Вы находитесь на странице: 1из 464

MATHS

Quest
General Mathematics
HSC COURSE
SE

ITION
D
E
D
CON

Robert Rowland

Second edition published 2007 by


John Wiley & Sons Australia, Ltd
42 McDougall Street, Milton, Qld 4064
First edition published 2001
Typeset in 10.5/12.5 pt Times

John Wiley & Sons Australia, Ltd 2001, 2007

The moral rights of the author have been asserted.


National Library of Australia
Cataloguing-in-Publication data
Rowland, Robert, 1963.
Maths quest: general mathematics HSC.
2nd ed.
Includes index.
For secondary school students.
ISBN 978 0 7314 0569 5. (Student edition)
ISBN 978 0 7314 0568 8. (Teacher edition)
1. Mathematics - Problems, exercises, etc. 2. Mathematics.
I. Title.
510

Reproduction and communication for educational purposes


The Australian Copyright Act 1968 allows a maximum of one
chapter or 10% of the pages of this work, whichever is the greater,
to be reproduced and/or communicated by any educational
institution for its educational purposes provided that the
educational institution (or the body that administers it) has given a
remuneration notice to Copyright Agency Limited (CAL).
Reproduction and communication for other purposes
Except as permitted under the Act (for example, a fair dealing for
the purposes of study, research, criticism or review), no part of this
book may be reproduced, stored in a retrieval system,
communicated or transmitted in any form or by any means without
prior written permission. All inquiries should be made to the
publisher.
Cover photograph and internal design images: Digital Vision
Cartography by MAPgraphics Pty Ltd, Brisbane
Illustrated by the Wiley Art Studio
Printed in China by
Printplus Limited
10 9 8 7 6 5 4 3

Contents
Introduction vi
About eBookPlus viii
Acknowledgements ix

Summary 73
Chapter review 75
Practice examination questions

CHAPTER 3

CHAPTER 1
Credit and borrowing

Are you ready?

2
Flat rate interest 3
Exercise 1A 6
Home loans 9
Exercise 1B 12
10 Quick Questions 1 16
The cost of a loan 16
Exercise 1C 19
Investigation Researching home loans 22
Credit cards 22
Exercise 1D 26
Investigation Researching credit cards 28
10 Quick Questions 2 28
Loan repayments 29
Exercise 1E 31
Summary 34
Chapter review 36
Practice examination questions 39

CHAPTER 2
Further applications of area
and volume 41
Are you ready?

78

42
Area of parts of the circle 43
Exercise 2A 45
Area of composite shapes 48
Exercise 2B 50
10 Quick Questions 1 52
Simpsons rule 53
Exercise 2C 55
Surface area of cylinders and spheres 57
Exercise 2D 59
Investigation Packaging 62
Volume of composite solids 62
Exercise 2E 65
Investigation Maximising volume 67
10 Quick Questions 2 68
Error in measurement 69
Exercise 2F 71

Applications of
trigonometry 79
Are you ready?

80
Review of right-angled triangles 81
Exercise 3A 85
Bearings 86
Exercise 3B 89
Investigation Trigonometric ratios for obtuse
angles 91
The sine rule 91
Investigation Derivation of the sine rule 92
Exercise 3C 95
Exercise 3D 99
10 Quick Questions 1 101
Area of a triangle 102
Exercise 3E 103
The cosine rule 106
Investigation Derivation of the cosine
rule 106
Exercise 3F 109
Exercise 3G 115
10 Quick Questions 2 117
Radial surveys 118
Exercise 3H 121
Investigation Conducting a radial
survey 122
Summary 123
Chapter review 125
Practice examination questions 127

CHAPTER 4
Interpreting sets of data
Are you ready?

129

130

Measures of location and spread 131


Exercise 4A 136
Skewness 140
Exercise 4B 142
10 Quick Questions 1 145
Displaying multiple data sets 145
Investigation Examining exam results 146
Exercise 4C 150

iv
Comparison of data sets 152
Exercise 4D 155
Investigation Developing a two-way
table 159
Summary 160
Chapter review 161
Practice examination questions 165

CHAPTER 5
Algebraic skills and
techniques 167
Are you ready?

168

Substitution 169
Exercise 5A 171
Algebraic manipulation 172
Exercise 5B 174
10 Quick Questions 1 175
Equations and formulas 175
Exercise 5C 178
Solution by substitution 180
Exercise 5D 182
Investigation Repeated enlargements 183
10 Quick Questions 2 183
Scientific notation 184
Exercise 5E 186
Summary 187
Chapter review 188
Practice examination questions 189

CHAPTER 6
Multi-stage events
Are you ready?

191

192

Tree diagrams 193


Exercise 6A 195
Counting techniques 196
Investigation Ordered arrangements 196
Investigation Tree diagrams and ordered
arrangements 198
Investigation Committee selections 199
Investigation Unordered selection 200
Exercise 6B 200
Probability and counting techniques 201
Investigation Popular gaming 203
Exercise 6C 203
10 Quick Questions 1 204
Probability trees 205
Exercise 6D 209
Summary 212
Chapter review 213
Practice examination questions 215

CHAPTER 7
Applications of
probability 217
Are you ready?

218

Expected outcomes 219


Investigation Rolling a die 219
Exercise 7A 221
Financial expectation 223
Exercise 7B 225
10 Quick Questions 1 226
Two-way tables 227
Exercise 7C 229
Summary 232
Chapter review 233
Practice examination questions 235

CHAPTER 8
Annuities and loan
repayments 237
Are you ready?

238

Future value of an annuity 239


Exercise 8A 242
10 Quick Questions 1 245
Present value of an annuity 246
Exercise 8B 248
Future and present value tables 250
Exercise 8C 253
10 Quick Questions 2 254
Loan repayments 255
Exercise 8D 257
Investigation Types of loan
arrangements 259
Summary 260
Chapter review 261
Practice examination questions 263

CHAPTER 9
Modelling linear and
non-linear relationships
Are you ready?

265

266

Linear functions 267


Exercise 9A 272
Investigation Conversion of
temperature 274
Quadratic functions 274
Exercise 9B 278
Investigation Maximising areas 280
10 Quick Questions 1 280

v
Other functions 281
Exercise 9C 284
Investigation Compound interest 285
Variations 285
Exercise 9D 288
Graphing physical phenonema 289
Exercise 9E 292
Investigation Force of gravity 293
Summary 294
Chapter review 295
Practice examination questions 297

CHAPTER 10
Depreciation
Are you ready?

299

300

Modelling depreciation 301


Investigation Depreciation of motor
vehicles 301
Exercise 10A 304
Straight line depreciation 307
Exercise 10B 309
Declining balance method of
depreciation 310
Exercise 10C 312
Investigation Rates of depreciation 313
10 Quick Questions 1 314
Depreciation tables 314
Exercise 10D 319
Summary 322
Chapter review 323
Practice examination questions 325

CHAPTER 11
The normal distribution
Are you ready?

327

328

z-scores 329
Exercise 11A 332
Comparison of scores 334
Exercise 11B 336
10 Quick Questions 1 338
Investigation Comparison of subjects 338
Distribution of scores 339
Exercise 11C 342
Investigation Examining a normal
distribution 343
Summary 344
Chapter review 345
Practice examination questions 346

CHAPTER 12
Correlation
Are you ready?

349

350

Scatterplots 351
Exercise 12A 355
Investigation Collecting bivariate data 357
Regression lines 357
Exercise 12B 359
Exercise 12C 365
Investigation Relationship between
variables 369
10 Quick Questions 1 369
Correlation 370
Investigation Causality 372
Exercise 12D 374
Summary 378
Chapter review 379
Practice examination questions 381

CHAPTER 13
Spherical geometry
Are you ready?

383

384

Arc lengths 385


Exercise 13A 386
Great circles and small circles 389
Exercise 13B 390
10 Quick Questions 1 392
Latitude and longitude 393
Exercise 13C 396
Investigation Important parallels of
latitude 397
Distances on the Earths surface 397
Exercise 13D 399
10 Quick Questions 2 401
Time zones 401
Investigation Australian time zones 402
Exercise 13E 404
Investigation The keepers of time 405
Summary 406
Chapter review 407
Practice examination questions 409
Glossary 411
Formula sheet 414
Answers 417
Index 449

Introduction
Maths Quest General Mathematics HSC course is the second book in a
series specifically designed for the General Mathematics Stage 6 Syllabus
starting in 2000. This course replaces the current syllabuses for Mathematics
in Society (1981) and Mathematics in Practice (1989).
There are five new areas of study:
Financial mathematics
Data analysis
Measurement
Probability
Algebraic modelling.
This resource contains:
a student textbook with accompanying eBookPLUS and
a teacher edition with accompanying eGuidePLUS.

Student textbook
Full colour is used throughout to produce clearer graphs and diagrams, to provide bright, stimulating photos and to make navigation through the text easier.
Clear, concise theory sections contain worked examples, highlighted important text and remember boxes.
Worked examples in a Think/Write format provide a clear explanation of key
steps and suggest a presentation for solutions.
Exercises contain many carefully graded skills and application problems,
including multiple-choice questions. Cross-references to relevant worked
examples appear beside the first matching question throughout the exercises.
Investigations, including spreadsheet investigations, provide further learning
opportunities through discovery.
Sets of 10 Quick Questions allow students to quickly review the concepts
just learnt before proceeding further in the chapter.
A glossary of mathematical terms is provided to assist students understanding of the terminology introduced in each unit of the course. Words in
bold type in the theory sections of each chapter are defined in the glossary at
the back of the book.
Each chapter concludes with a summary and chapter review exercise, containing questions in a variety of forms (multiple-choice, short-answer and
analysis) that help consolidate students learning of new concepts.
Practice examination questions provide a ready source of problems for students to use to gain further confidence in each topic.

vii
Technology is fully integrated, in line with Board of Studies recommendations. As well as graphics calculators, Maths Quest features spreadsheets,
dynamic geometry software and several graphing packages. Not only does
the text promote these technologies as learning tools, but demonstration
versions of the programs (with the exception of Microsoft Excel) are also
included, as well as hundreds of supporting files available online.
Graphics calculator tips are incorporated throughout the text.
All formulas, which are given on the examination formula sheet, are marked
with the symbol .

Programs included
Graphmatica: an excellent graphing utility
Equation grapher and regression analyser: like a graphics calculator for
the PC
GrafEq: graphs any relation, including complicated inequalities
Poly: for visualising 3D polyhedra and their nets
Tess: for producing tessellations and other symmetric planar illustrations
TI Connect: calculator screen capture and program transfer
CASIO Software FA-123: calculator screen capture and program transfer
Cabri Geometry II: dynamic geometry program
Adobe Acrobat Reader 4.0

Teacher edition with accompanying


eGuidePLUS
The teacher edition textbook contains everything in the student textbook and
more. To support teachers assisting students in class, answers appear in red
next to most questions in the exercises. Each exercise is annotated with relevant study design dot points. A readily accessible Work program lists all
available resources and provides curriculum coverage information.
The accompanying online resources contain everything in the student
eBookPLUS and more. Four tests per chapter, fully worked solutions to
WorkSHEETs, the work program and other curriculum advice in editable
Word 2000 format are provided.
Maths Quest is a rich collection of teaching and learning resources within
one package.
Maths Quest General Mathematics HSC course provides ample material,
such as exercises, analysis questions, investigations, worksheets and technology files, from which teachers may set assessment tasks.

Next generation teaching and learning


About eBookPLUS
Using the JacarandaPLUS website

This book features eBookPLUS: an electronic version


of the entire textbook and supporting multimedia
resources. It is available for you online at the
JacarandaPLUS website (www.jacplus.com.au).
These additional resources include:
Word documents designed for easy customisation
and editing
interactive activities and a wealth of ICT resources
weblinks to other useful resources and information
on the internet.

To access your eBookPLUS resources, simply log on


to www.jacplus.com.au. There are three easy steps for
using the JacarandaPLUS system.
Step 1. Create a user account
The first time you use the JacarandaPLUS system,
you will need to create a user account. Go to the
JacarandaPLUS home page (www.jacplus.com.au)
and follow the instructions on screen.
Step 2. Enter your registration code
Once you have created a new
account and logged in, you will be
prompted to enter your unique
registration code for this book,
which is printed on the inside
front cover of your textbook.

LOGIN
Once you have created your account,
you can use the same email address and
password in the future to register any
JacarandaPLUS books.

Step 3. View or download eBookPLUS resources


Your eBook and supporting resources are provided
in a chapter-by-chapter format. Simply select the
desired chapter from the drop-down list and navigate
through the tabs to locate the appropriate resource.

Work

Key to the icons in the book


T
SHEE

In this student text, the resource icons, such as


the one shown, indicate that digital support
materials for this section of the text are
available online at JacarandaPLUS.

Minimum requirements

Troubleshooting
Go to the JacarandaPLUS help page at
www.jacplus.com.au
Contact John Wiley & Sons Australia, Ltd.
Email: support@jacplus.com.au
Phone: 1800 JAC PLUS (1800 522 7587)

Internet Explorer 7, Mozilla Firefox 1.5 or Safari 1.3


Adobe Flash Player 9
Javascript must be enabled (most browsers are
enabled by default).

Acknowledgements
The Maths Quest project began in 1997, and the first edition of this book
was printed in 2001. In that time we believe that Maths Quest has become
the best resourced mathematical database in Australian education. I would
like to thank all of those people who have supported us with our first edition.
I hope that we have been able to help you in achieving your goals and have
also played a part in your successes.
Technology has evolved greatly since our first edition was published. The
second edition has evolved from being a textbook in its first edition, into an
interactive resource for both students and teachers. I would like to thank
everyone at John Wiley & Sons Australia, Ltd for giving me the opportunity
to do this.
There are three people in particular that I would like to single out for
special mention: Jennifer Nolan, whose support for the Maths Quest project
and for me personally has made everything possible; Ingrid Kemp, the
newest addition to our team. Ingrid has brought a new set of eyes to our
project and kept the ball rolling thanks, Ingrid; and finally, Keith Hartmann, who has tirelessly reviewed all of the new material and has completed
all of the answer checking. Thanks, Keith hope youre enjoying
retirement!
Finally and most importantly to my family thank you. Without your
support this book and online resources would never have been completed.
The author and publisher would like to thank the following copyright
holders, organisations and individuals for their assistance and for permission
to reproduce copyright material in this book.

Illustrative material
Banana Stock: p. 302 Brand X Pictures: p. 134 Comstock:
p. 143 Corbis Corporation: p. 67, p. 72, p. 77, p. 100, p. 126, p. 377
Digital Stock: p. 20/Corbis Corporation, p. 37/Corbis Corporation, p. 41/
Corbis Corporation, p. 58/Corbis Corporation, p. 81/Corbis Corporation,
p. 167/Corbis Corporation, p. 179/Corbis Corporation, p. 249, Corbis Corporation, p. 304/Corbis Corporation, p. 327/Corbis Corporation, p. 332/Corbis
Corporation, p. 397/Corbis Corporation, p. 408/Corbis Corporation Digital
Vision: p. 29, p. 144 (lower), p. 163, p. 231, p. 293, p. 383, p. 388, p. 392
Image Addict: p. 12 IT StockFree: p. 66, p. 198 John Wiley &
Sons Australia: p. xi/Photo by Jo Patterson, p. 193/Photo by Werner Langer,
p. 299/Taken by Kari-Ann Tapp MAPgraphics: p. 394/Map by MAPgraphics Pty Ltd, Brisbane. Photodisc: p. 1, p. 7, p. 8, p. 17, p. 32, p. 47,
p. 53 (all), p. 55, p. 69, p. 79, p. 105, p. 116, p. 118, p. 125, p. 129, p. 139,
p. 144 (upper), p. 158, p. 159, p. 172, p. 191, p. 195, p. 196, p. 205, p. 208,
p. 210, p. 211, p. 213, p. 215, p. 217, p. 219, p. 221, p. 225, p. 234, p. 237,
p. 239, p. 263, p. 265, p. 279, p. 289, p. 307, p. 312, p. 323, p. 337, p. 343,
p. 346, p. 347, p. 349, p. 351, p. 356, p. 357 (both), p. 368, p. 370, p. 372,
p. 380, p. 396 Purestock: p. 305 Stockbyte: p. 90

x
Software
The authors and publisher would like to thank the following software providers
for their assistance and for permission to use their materials. However, the use of
such material does not imply that the providers endorse this product in any way.
Third party software registered full version ordering information
Full versions of third party software may be obtained by contacting the
companies listed below.
Texas Instruments TI Connect and TI-GRAPHLINK software
TI Connect and TI-GRAPHLINK software reproduced with permission of
the publisher Texas Instruments Incorporated.
TI Connect software available from Texas Instruments
Web: http://education.ti.com/us/product/software.html
Note: The TI Connectivity cable can be purchased from educational booksellers or calculator suppliers.
Casio FA-124
Software used with permission of Casio Computer Co. Ltd. 2001
All rights reserved.
Distributed by Shriro Australia Pty Ltd
2327 Chaplin Drive
Lane Cove
NSW 2066
Web: www.casioed.net.au and find the calculator product range
If you are interested in this product after expiry, please contact Shriro
Australia Pty Ltd.
Graphmatica
Reproduced with permission of kSoft, Inc.
345 Montecillo Dr., Walnut Creek, CA 94595-2654.
e-mail: ksoft@graphmatica.com
Web: http://www.graphmatica.com
Software included is for evaluation purposes only. The user is expected to
register share-ware if use exceeds 30 days. Order forms are available at
www.graphmatica.com/register. txt
Cabri Geometry II PLUS
Reproduced with permission of Cabrilog.
Cabrilog 6, Robert Schuman Place
38000 Grenoble FRANCE
Web: http://www.cabri.com
1. Due to copyright restrictions, the demo version of Cabri Geometry II
Plus must not be used in the classroom for presentation on a regular basis.
2. For site licences contact Cabrilog Grenoble-France at
sales@cabri.com or www.cabri.com

xi
GrafEq and Poly
Evaluation copies of GrafEq and Poly have been included with permission from Pedagoguery Software, Inc.
e-mail: peda@peda.com
Web: http://www.peda.com
Microsoft Excel, Microsoft Word and Microsoft PowerPoint
Microsoft Excel, Microsoft Word and Microsoft PowerPoint are registered
trademarks of the Microsoft Corporation in the United States and/or other
countries.
Screenshots reproduced throughout with permission from Microsoft.
Every effort has been made to trace the ownership of copyright material.
Information that will enable the publisher to trace the copyright holders or to
rectify any error or omission in subsequent reprints will be welcome. In such
cases, please contact the Permission Section of John Wiley & Sons Australia,
who will arrange for the payment of the usual fee.

About the author


Robert Rowland has been teaching Mathematics for over 20 years and currently holds the position of Head teacher, Teaching and learning at Ulladulla
High School. He taught at Cabramatta High School from 1985 to 1988 before
taking up his appointment at Ulladulla High School in 1989. Robert has successfully taught all levels of Mathematics to Year 12 as well as Computing
Studies 712 and Information Processes and Technology. Robert is the coauthor of New South Wales Maths Year 9 Standard and New South Wales
Maths Year 10 Standard as well as being the author of Maths Quest General
Mathematics Preliminary Course and Maths Quest General Mathematics
HSC Course.

Credit and
borrowing

1
syllabus reference
Financial mathematics 4
Credit and borrowing

In this chapter
1A
1B
1C
1D
1E

Flat rate interest


Home loans
The cost of a loan
Credit cards
Loan repayments

areyou

READY?

Are you ready?

Try the questions below. If you have difficulty with any of them, extra help can be
obtained by completing the matching SkillSHEET. Either click on the SkillSHEET icon
next to the question on the Maths Quest HSC Course CD-ROM or ask your teacher for
a copy.

1.1

Converting a percentage to a decimal

1 Convert each of the following percentages to decimals.


a 40%
b 12%
c 8%
1
e 0.3%
f 7 --- %
g 1--- %
2

1.2

Finding a percentage of a quantity (money)

2 Find:
a 30% of $5000
d 0.45% of $3600

1.3

d 2.4%
h 0.02%

b 5% of $7390
e 1--- % of $82 000
2

c 7.4% of $125 000


f 0.06% of $78 000

Calculating simple interest

3 Calculate the simple interest earned on an investment of:


a $7000 at 9% p.a. for 4 years
b $57 500 at 6.5% p.a. for 2 years
c $90 000 at 7 1--- % for 2 1--- years
d $60 000 at 5.2% p.a. for 9 months
2

1.4

Finding values of n and r in financial formulas

4 Find the value of n and r for each of the following investments


a Interest of 6% p.a. for 5 years with interest calculated annually
b Interest of 9% p.a. for 4 years with interest calculated six-monthly
c Interest of 8.8% p.a. for 3 years with interest calculated quarterly
d Interest of 7.2% p.a. for 10 years with interest calculated monthly
e Interest of 21% p.a. for June with interest calculated daily

1.5

Calculating compound interest

5 Use the formula A = P(1 + r)n to calculate the amount to which each of the following investments
will grow.
a $7000 at 9% p.a. for 4 years with interest compounded annually
b $75 000 at 6.2% p.a. for 6 years with interest compounded six-monthly
Calculate the amount of compound interest earned on an investment of:
c $18 000 at 9.2% p.a. for 3 years with interest compounded annually
d $150 000 at 8.4% p.a. for 10 years with interest compounded quarterly

1.6

Substitution into a formula

6 Evaluate each of the following by substituting into the given formula.


m
a If d = ---- , find d when m = 30 and v = 3.
v
b If A = 1--- (x + y)h, find A when h = 10, x = 7 and y = 2.
2

c If s = ut + 1--- at2, find s when u = 0.8, t = 5 and a = 2.3.


2

Chapter 1 Credit and borrowing

Flat rate interest


During the preliminary course we calculated the simple interest earned on investments.
Flat rate interest is the borrowing equivalent of simple interest. Flat rate interest applies
to many small loans and hire purchase agreements.
When money is borrowed from a lending institution such as a bank at a flat rate of
interest, the total amount of interest is calculated as a percentage of the initial amount
borrowed and then this is multiplied by the term of the loan. The term of the loan is
the length of time which the loan is agreed to be repaid over.
The formula for calculating the amount of flat interest to be paid on a loan is the
same formula as for simple interest (I):
I = Prn
where P = initial quantity
r = percentage interest rate per period expressed as a decimal
n = number of periods
As you work through the financial mathematics strand there are several formulas that
use the same pronumerals.
While the initial quantity (P) will be the principal in an investing scenario, it will
represent the amount borrowed in a loans situation.
All of these formulas use the same pronumerals and all of them require r to be
expressed as a decimal. It should be part of your normal practice when doing such
questions to convert the interest rate, expressed as a percentage, to a decimal. In simple
or flat rate interest, r will always be a rate per annum or per year and there will be no
variation on this regardless of how often interest is paid.
Similarly, n will always be the number of years of the investment or loan.

WORKED Example 1

Calculate the flat interest to be paid on a loan of $20 000 at 7.5% p.a. flat interest if the
loan is to be repaid over 5 years.
THINK

WRITE

Convert the interest rate to a decimal.

r = 7.5 100
= 0.075

Write the formula.

I = Prn

Substitute the values of P, r (as a


decimal) and n.

= $20 000

Calculate.

= $7500

0.075

Once the interest has been calculated, we can calculate the total amount that must be
repaid in a loan. This is calculated by adding the principal and the interest.

Maths Quest General Mathematics HSC Course

WORKED Example 2

Alvin borrows $8000 to buy a car at a flat rate of 9% p.a. interest. Alvin is to repay the
loan, plus interest, over 4 years. Calculate the total amount that Alvin is to repay on this
loan.
THINK

WRITE

Convert the interest rate to a decimal.

Write the interest formula.


Substitute the values of P, r and n.
Calculate the interest.
Calculate the total repayments by
adding the interest and principal.

3
4
5

r = 9 100
= 0.09
I = Prn
= $8000 0.09 4
= $2880
Total repayments = $8000 + $2880
Total repayments = $10 880

Graphics Calculator tip! Calculate simple interest


Your Casio graphics calculator can perform a number of financial functions by using
the TVM mode.
One of the options in this mode is to calculate simple interest. Examples such as
worked example 2 above are more simply done using the arithmetic method as shown
above. However, for some of the more complex questions later in this chapter it is
worth familiarising yourself with this method.
1. From the MENU select TVM.

2. Press F1 to select Simple Interest.

3. The calculator has two modes of calculating interest:


360 day mode or 365 day mode. You need to make
sure that it is on 365 day mode. If not, press SHIFT
[SET UP], select Date Mode and press F1 for 365.
4. Press EXE to return to the previous screen and
enter the data for worked example 2.
n = 4 365 (as n is in days)
I% = 9
PV = 8000
(principal or present value is entered as a negative)

Chapter 1 Credit and borrowing

5. The calculator gives you two options.


Press F1 (SI) for simple interest
Press F2 (SFV) for future value (in other words, the
principal plus interest).
In this example we want the total repayments, so we
press F2 (SPV).
Most loans are repaid on a monthly basis. Once the total amount to be repaid has been
calculated, this can be divided into equal monthly, fortnightly or weekly instalments.

WORKED Example 3
Narelle buys a computer on hire purchase. The cash price of the computer is $3000, but
Narelle must pay a 10% deposit with the balance paid at 8% p.a. flat rate interest in equal
monthly instalments over 3 years.
a Calculate the deposit.
b Calculate the balance owing.
c Calculate the interest on the loan.
d Calculate the total amount to be repaid.
e Calculate the amount of each monthly instalment.
THINK

WRITE

a Find 10% of $3000.

a Deposit = 10% of $3000


Deposit = $300

b Subtract the deposit from the cash price


to find the amount borrowed.

b Balance = $3000
Balance = $2700

c I = Prn

Write the interest formula.

Substitute for P, r and n.

I = $2700

Calculate the interest.

I = $648

0.08

$300

d Add the interest to the amount


borrowed.

d Total repayments = $2700 + $648


Total repayments = $3348

e Divide the total repayments by 36


(the number of monthly instalments in
3 years).

e Monthly repayments = $3348 36


Monthly repayments = $93.00

If given the amount to be repaid each month, we can calculate the interest rate. The
interest on the loan is the difference between the total repaid and the amount borrowed.
This is then calculated as a yearly amount and written as a percentage of the amount
borrowed.

Maths Quest General Mathematics HSC Course

WORKED Example 4

Theresa borrows $12 000 to buy a car. This is to be repaid over 5 years at $320 per month.
Calculate the flat rate of interest that Theresa has been charged.
THINK
1
2
3
4

WRITE

Calculate the total amount that is


repaid.
Subtract the principal from the total
repayments to find the interest.
Calculate the interest paid each year.
Write the annual interest as a
percentage of the amount borrowed.

Total repayments = $320 60


Total repayments = $19 200
Interest = $19 200 $12 000
Interest = $7200
Interest per year = $7200 5
Interest per year = $1440
$1440
Interest rate = ------------------- 100%
$12 000
Interest rate = 12%

remember
1. Flat rate interest is the borrowing equivalent of simple interest. It is calculated
based on the initial amount borrowed.
2. The simple interest formula is used to calculate the amount of flat rate interest
to be paid on a loan. The simple interest formula is I = Prn .
3. The total amount to be repaid on a loan is the principal plus interest. To
calculate the amount of each instalment, we divide the total amount by the
number of repayments.
4. When given the amount of each instalment, we can calculate the flat rate of
interest.

SkillS

1A
HEET

1.1
WORKED

SkillS

Converting Example
a
1
percentage
to a decimal
HEET

1.2

SkillS

Finding a
percentage
of a
WORKED
quantity Example
2
HEET

Flat rate interest

1.3

Calculating
simple
interest

1 Calculate the amount of flat rate interest paid on each of the following loans.
a $5000 at 7% p.a. for 2 years
b $8000 at 5% p.a. for 3 years
c $15 000 at 10% p.a. for 5 years
d $9500 at 7.5% p.a. for 4 years
e $2500 at 10.4% p.a. for 18 months
2 Roula buys a used car that has a cash price of $7500. She has saved a deposit of
$2000 and borrows the balance at 9.6% p.a. flat rate to be repaid over 3 years.
Calculate the amount of interest that Roula must pay.
3 Ben borrows $4000 for a holiday. The loan is to be repaid over 2 years at 12.5% p.a.
flat interest. Calculate the total repayments that Ben must make.
4 Calculate the total amount to be paid on each of the following flat rate interest loans.
a $3500 at 8% p.a. over 2 years
b $13 500 at 11.6% p.a. over 5 years
c $1500 at 13.5% p.a. over 18 months
d $300 at 33% p.a. over 1 month
e $100 000 at 7% p.a. over 25 years

Chapter 1 Credit and borrowing

Example

GC

asio

WORKED

sheet

5 Mr and Mrs French purchase a new lounge suite, which has a cash price of $5500.
L Spre
XCE ad
They purchase the lounge on the following terms: 30% deposit with the balance to be
Simple
repaid at 9% p.a. flat interest over 2 years. Calculate:
interest
a the deposit
b the balance owing
c the interest to be paid
am
progr C
d the total amount that they pay for the lounge.

TI

6 Yasmin borrows $5000 from a credit union at a flat interest rate of 8% p.a. to be Interest
repaid over 4 years in equal monthly instalments. Calculate:
a the interest that Yasmin must pay on the loan
b the total amount that Yasmin must repay
program
GC
c the amount of each monthly repayment.
7 Ian borrows $2000 from a pawnbroker at 40% p.a. interest. The loan is to be paid over
1 year in equal weekly payments.
a Calculate the interest on the loan.
b Calculate the total that Ian must repay.
c Calculate Ians weekly payment.
8 The Richards family purchase an entertainment system for their home. The total cost
of the system is $8000. They buy the system on the following terms: 25% deposit
with the balance repaid over 3 years at 12% p.a. flat interest in equal monthly
instalments. Calculate:
a the deposit
b the balance owing
c the interest on the loan
d the total repayments
e the amount of each monthly repayment.
9 Sam buys an electric guitar with a cash price of $1200. He
buys the guitar on the following terms: one-third deposit, with
the balance at 15% p.a. flat interest over 2 years in equal
monthly instalments. Calculate the amount of each monthly
repayment.
10 multiple choice
The amount of flat rate interest on a loan of $10 000 at 10% p.a.
for 2 years is:
A $1000
B $2000
C $11 000
D $12 000
11 multiple choice
A refrigerator with a cash price of $1800 is bought on the
following terms: 20% deposit with the balance paid in
12 equal monthly instalments at 12% p.a. flat interest.
The total cost of the refrigerator when purchased on
terms is:
A $172.80
B $216.00
C $1972.80
D $2016.00

Interest

Maths Quest General Mathematics HSC Course

WORKED

Example

12 Andy borrows $4000, which is to be repaid over 4 years at $110 per month. Calculate
the flat rate of interest that Andy has been charged.
13 Sandra buys a used car with a cash price of
$12 000 on the following terms: 20%
deposit with the balance paid at $89.23 per
week for 3 years. Calculate:
a the deposit
b the balance owing
c the total cost of the car
d the flat rate of interest charged.
14 Calculate the flat rate of interest charged on
a lounge suite with a cash price of $5000 if
it is purchased on the following terms: 15%
deposit with the balance paid at $230.21
per month for 2 years.

Computer Application 1 Flat rate interest loan calculator


EXCE

et

reads
L Sp he

Flat
interest

Access the spreadsheet Flat Interest from the Maths Quest General Mathematics HSC
Course CD-ROM. This spreadsheet will demonstrate how to calculate a deposit, the
total repayments on a loan and the size of each repayment.

Monthly payment calculator


Consider a $5000 loan to be repaid at 9% p.a. flat rate interest over 3 years.
1. On the sheet titled Monthly Payments, in cell B5 enter the amount which has been
borrowed ($5000), or the balance owing on a purchase after the deposit has been
paid.
2. In cell B7 enter the interest rate as a percentage (9%).
3. In cell B9 enter the number of years over which the loan is to be repaid (3).

Chapter 1 Credit and borrowing

4. The total interest paid on the loan will be displayed in cell B11. The formula for this
will be displayed in this cell.
5. Cell B13 shows the total amount to be repaid and cell B15 shows the amount of each
repayment.

Flat interest rate calculator


The worksheet Flat Interest Rate will calculate the flat rate of interest charged given
the amount of each repayment. Consider a $15 000 loan that is repaid over 5 years at
$350 per month.
1. In cell B5 enter the amount borrowed ($15 000).
2. In cell B7 enter the amount of each monthly payment ($350).
3. In cell B9 enter the total number of monthly payments (60).

4. Displayed will be the total amount to be repaid (cell B11), the total interest paid on
the loan (cell B13), the amount of interest paid per year (cell B15) and the flat rate of
interest (cell B17).
Check your answers to the previous exercise using this spreadsheet.

Home loans
The biggest loan that most people will ever take out will be for a home. These loans are
usually for large amounts of money and are taken over long periods of time. Most commonly they are taken over 10, 15, 20 or 25 years but they can be taken over even longer
periods of up to 35 years.
Home loans are not charged at a flat rate of interest. The interest on these loans is
reducible, which means that the interest is calculated on the amount of money owing on
the loan at the time rather than on the amount initially borrowed. This is known as a
reducing balance loan.
The interest on a home loan is usually calculated at the beginning of each month, and
payments are calculated on a monthly basis. So each month interest is added to the loan
and a payment is subtracted from the balance owing. The balance increases by the
amount of interest and then decreases by the amount of each payment.

10

Maths Quest General Mathematics HSC Course

Consider the case of a person who borrows $250 000 to buy a home at 9% p.a.
reducible interest. The monthly repayment on this loan is $2500 per month. The
interest rate of 9% p.a. converts to 0.75% per month.
First months interest = 0.75% of $250 000
= $1875
Balance owing = $250 000 + $1875 $2500
= $249 375
In the second month the interest is calculated on the balance owing at the end of the
first month.
Second months interest = 0.75% of $249 375
= $1870.31
Balance owing = $249 375 + $1870.31
= $248 745.31

$2500

The progress of this loan can be followed in the following computer application.

Computer Application 2 Home loan calculator


EXCE

et

reads
L Sp he

Home
loan

Access the spreadsheet Home Loan from the Maths Quest General Mathematics HSC
Course CD-ROM. This spreadsheet will allow you to follow the progress of a home
loan as it is paid off.

Use the Edit and then the Fill and Down functions on columns A, B, C and D. Look
down column D to find when the balance owing becomes 0 or when it becomes
negative. At this time the loan will have been fully repaid.
Examine other loans by changing the data in C4, C5 and C6.

Chapter 1 Credit and borrowing

11

WORKED Example 5
Mr and Mrs Chan take out a $100 000 home loan at 8% p.a. reducible interest over
25 years. Interest is calculated and added on the first of each month. They make a
payment of $775 each month. Calculate:
a the interest added after one month
b the balance owing after one month.
THINK

WRITE

a 8% p.a. = 2--- % per month

1
2

Convert 8% p.a. to a monthly rate.

Calculate

2
--- %
3

of $100 000 to find the

Interest =

interest for one month.


b Add the interest to the principal and
subtract the repayment.

2
--- %
3

of $100 000

Interest = $666.67
b Balance owing = $100 000 + $666.67
Balance owing = $99 891.67

$775

the interest in
Graphics Calculator tip! Calculate
a one-month period
We can use the TVM function to calculate the interest for a one-month period but great
care needs to be taken. Remembering that the interest is calculated for a number of
days, to calculate monthly interest we need to enter n = 365 12. Consider the method
shown below for worked example 5.
1. From the MENU select TVM.

2. Press F1 to select Simple Interest.

3. n = 365 12 (as n is in days)


I% = 8
PV = 100000

4. Press F1 (SI) to find the interest for one month.

When interest is calculated every year for such a long


period of time, as with many home loans, the amount of
money required to pay off such a loan can be a great
deal more than the initial loan.

12

Maths Quest General Mathematics HSC Course

WORKED Example 6
A loan of $120 000 is paid off at 9% p.a. reducible interest over a period of 25 years.
The monthly repayment is $1007.04. Calculate the total amount made in repayments on
this loan.
THINK
1
2

WRITE

Calculate the number of repayments by


multiplying the number of years by 12.
Multiply the monthly repayment by the
number of repayments.

No. of repayments = 25 12
No. of repayments = 300
Total repayments = $1007.04 300
Total repayments = $302 112.00

remember
1. The interest on home loans is calculated at a reducible rate. This means that the
interest is calculated on the balance owing rather than the initial amount
borrowed.
2. Interest is calculated each month; this is then added to the principal and a
payment is made. The interest next month is then calculated on the new amount
owing.
3. To calculate the total amount to be repaid on a home loan, we multiply the
monthly payment by the number of repayments made.

1B
SkillS

HEET

1.4

WORKED

Example

Finding
values of
n and r in
financial
formulas
SkillS

HEET

1.5
Calculating
compound
interest

EXCE

et

reads
L Sp he

Interest

Home loans

1 Mr and Mrs Devcich borrow


$80 000 to buy a home. The interest
rate is 12% p.a. and their monthly
payment is $850 per month.
a Calculate the interest for the first
month of the loan.
b Calculate the balance owing at
the end of the first month.
2 The repayment on a loan of
$180 000 at 7.5% p.a. over a
15-year term is $1668.62 per
month.
a Calculate the interest for the first
month of the loan and the balance owing at the end of the first
month.
b Calculate the amount by which
the balance has reduced in the
first month.

Chapter 1 Credit and borrowing

13

c Calculate the interest for the second month of the loan and the balance at the end
of the second month.
d By how much has the balance of the loan reduced during the second month?
3 The repayment on a loan of $150 000 over a 20-year term at 9.6% p.a. is $1408.01 per
month. Copy and complete the table below.
Month

Principal ($)

Interest ($)

Balance ($)

150 000.00

1200.00

149 791.99

149 791.99

3
4
5
6
7
8
9
10
4 Mr and Mrs Roebuck borrow $255 000 to purchase a home. The interest rate is
9% p.a. and over a 25-year term the monthly repayment is $2294.31.
a Copy and complete the table below.
Month

Principal ($)

Interest ($)

Balance ($)

255 000.00

1912.50

254 618.19

254 618.19

3
4
5
6
7
8
9
10
11
12

14

Maths Quest General Mathematics HSC Course

b Mr and Mrs Roebuck decide to increase their monthly payment to $2500.


Complete the table below.
Month

Principal ($)

Interest ($)

Balance ($)

255 000.00

1912.50

254 412.50

254 412.50

3
4
5
6
7
8
9
10
11
12
c How much less do Mr and Mrs Roebuck owe at the end of one year by increasing
their monthly repayment?
WORKED

Example

5 The repayments on a loan of $105 000 at 8% p.a. reducible interest over 25 years are
$810.41 per month. Calculate the total repayments made over the life of the loan.
6 The Taylors borrow $140 000 over 20 years at 9% p.a.
a The monthly repayment on this loan is $1259.62. Calculate the total repayments.
b The Taylors attempt to pay the loan off quickly by increasing their monthly
payment to $1500. The loan is then paid off in 161 months. Calculate the total
repayments made under this plan.
c How much will the Taylors save by increasing each monthly payment?
7 multiple choice
The first months interest on a $60 000 home loan at 12% p.a. reducible interest is:
A $600
B $7200
C $60 600
D $67 200
8 multiple choice
A $95 000 loan at 8% p.a. reducible interest over a 15-year term has a monthly
payment of $907.87. The total amount of interest paid on this loan is:
A $7600
B $68 416.60
C $114 000
D $163 416.60
9 Mr and Mrs Chakraborty need to borrow $100 000 to purchase a home. The interest
rate charged by the bank is 7% p.a. Calculate the total interest paid if the loan is taken
over each of the following terms:
a $706.78 per month over a 25-year term
b $775.30 per month over a 20-year term
c $898.83 per month over a 15-year term
d $1161.08 per month over a 10-year term.

Chapter 1 Credit and borrowing

15

10 The Smith and Jones families each take out a $200 000 loan at 9.5% p.a. reducible
interest. The Smith family repay the loan at $2000 per month and the Jones family
repay the loan at $3000 per month.
a How much does each family make in repayments in the first year?
b Complete the table below for each family.
Smith family
Month

Principal ($)

Interest ($)

Balance ($)

200 000.00

1583.33

199 583.33

199 583.33

3
4
5
6
7
8
9
10
11
12
Jones family
Month

Principal ($)

Interest ($)

Balance ($)

200 000.00

1583.33

198 583.33

198 583.33

3
4
5
6
7
8
9
10
11
12
c After one year how much less does the Jones family owe than the Smith family?

16

Maths Quest General Mathematics HSC Course

1
1 Calculate the amount of flat rate interest payable on a loan of $1500 at 14% p.a. to be
repaid over 2 years.
2 Calculate the amount of flat rate interest payable on a loan of $2365 at 19.2% p.a. to
be repaid over 2 1--- years.
2

3 Calculate the total repayments on a loan of $5000 at 13.5% p.a. to be repaid over
3 years.
4 Susan buys a lounge suite on terms. The cash price of the lounge is $6500 and she
pays a 15% deposit. Calculate the amount of the deposit.
5 Calculate the balance that Susan owes on the lounge suite.
6 Calculate the interest that Susan will pay at 17% p.a. flat rate interest for a period of
3 years.
7 Calculate the total amount that Susan will have to repay.
8 Calculate the monthly repayment that Susan will need to make.
9 Harry and Sally borrow $164 000 to purchase a home. The interest rate is 12% p.a.
Calculate the amount of interest payable for the first month.
10 A $175 000 loan that is repaid over 25 years has a monthly repayment of $1468.59.
Calculate the total amount of interest that is paid on this loan.

The cost of a loan


Because of the different ways that interest can be calculated, the actual interest rate
quoted may not be an accurate guide to the cost of the loan. By using a flat rate of
interest, a lender can quote an interest rate less than the equivalent reducible interest
rate.
To compare flat and reducible rates of interest, we need to calculate the effective rate
of interest for a flat rate loan. The effective rate of interest is the equivalent rate of
reducible interest for a flat rate loan.
The formula for effective rate of interest is:
( 1 + r )n 1
E = ---------------------------n
where E = effective rate of interest, expressed as a decimal
r = stated rate of flat interest expressed as a decimal
n = term of the loan in years
Note: This formula for effective rate of interest is not on your formula sheet. This does
not mean that you have to memorise it as the formula will be given to you as a part of
any question that requires you to use it.

Chapter 1 Credit and borrowing

17

WORKED Example 7
Andrea takes out an $8000 loan for a car over 5 years at 6% p.a. flat rate interest.
Calculate the effective rate of interest charged on the loan.
THINK

WRITE

Write the formula.

Substitute r = 0.06 and n = 5.

Calculate.

Write the interest rate as a percentage.

( 1 + r )n 1
E = ---------------------------n
( 1.06 ) 5 1
E = -------------------------5
E = 0.068
The effective rate of interest is 6.8% p.a.

A loan with a reducible rate of interest can be compared to a flat rate of interest if we
are able to calculate the total repayments made over the term of the loan.

WORKED Example 8
An $85 000 loan at 10% p.a. reducible interest is to be repaid over 15 years at $913.41 per
month.
a Calculate the total repayments on the loan.
b Calculate the total amount of interest paid.
c Calculate the equivalent flat rate of interest on this loan.
THINK

WRITE

a Multiply the monthly repayments by the


number of months taken to repay the loan.

a Total repayments = $913.41 180


Total repayments = $164 413.80

b Subtract the initial amount borrowed from the


total repayments.

b Interest = $164 413.80


Interest = $79 413.80

c Annual interest = $79 413.80 15


Annual interest = $5294.25

Calculate the amount of interest paid per


year.

Write the yearly interest as a percentage of


the amount borrowed.

$85 000

$5294.25
Flat interest rate = ---------------------$85 000
Flat interest rate = 6.2% p.a.

100%

The most accurate way to compare loans is to calculate the total repayments made in
each loan.

18

Maths Quest General Mathematics HSC Course

WORKED Example 9
Allison borrows $6000 and has narrowed her choice of loans down to two options.
Loan A: At 8% p.a. flat rate interest over 4 years to be repaid at $165.00 per month.
Loan B: At 12% p.a. reducible interest over 3 years to be paid at $199.29 per month.
Which of the two loans would cost Allison less?
THINK
1
2
3

WRITE

Calculate the total repayments on


Loan A.
Calculate the total repayments on
Loan B.
Write a conclusion.

Loan A repayments = $165.00 48


Loan A repayments = $7920
Loan B repayments = $199.29 36
Loan B repayments = $7174.44
Loan B would cost $745.56 less than Loan A.

In the above example Allison should take Loan B even though it has a much higher
advertised interest rate. This of course would depend upon Allisons ability to meet the
higher monthly payments.
Generally the more quickly that you can pay off a loan the cheaper the loan will be.
The savings are particularly evident when examining home loans. Some home loans
that offer a lower interest rate allow for you to make only the minimum monthly repayment. This will maximise the amount of interest that the customer will pay.
If a person can afford to pay more than the minimum amount, they may be better off
over time by paying a slightly higher rate of interest and paying the loan off over a
shorter period of time.

WORKED Example 10
Mr and Mrs Beasley need to borrow $100 000 and have the choice of two home loans.
Loan X: 6% p.a. over 25 years with a fixed monthly repayment of $644.30. No extra
repayments are allowed on this loan.
Loan Y: 7% p.a. over 25 years with a minimum monthly payment of $706.78.
Mr and Mrs Beasley believe they can afford to pay $800 per month on this loan. If they
do, the loan will be repaid in 18 years and 9 months. Which loan would you recommend?
THINK
1
2
3

WRITE

Calculate the total repayments on


Loan X.
Calculate the total repayments on
Loan Y.
Make a recommendation.

Loan X repayments = $644.30 300


= $193 290
Loan Y repayments = $800 225
= $180 000
Mr and Mrs Beasley should choose Loan Y
as they will save $13 290 provided they can
continue to pay $800 per month.

With loans such as the one in the above example, the savings depend upon the ability to
make the extra repayments. If this is doubtful, Loan X would have been the safer
option.

Chapter 1 Credit and borrowing

19

The other factor to consider when calculating the cost of a loan is fees. Many loans
have a monthly management fee attached to them. This will need to be calculated into
the total cost and may mean that a loan with a slightly higher interest rate but no fee
may be a cheaper option.

remember
1. The actual cost of a loan is calculated by the total cost in repaying the loan.
The interest rate is a guide but not the only factor in calculating cost.
2. A loan that is quoted at a flat rate of interest can be compared to a reducible
rate of interest only by calculating the effective rate of interest on the flat rate
loan. The effective rate of interest is the equivalent reducible rate of interest
and is found using the formula:
( 1 + r )n 1
E = ---------------------------n
3. By calculating the total repayments on a loan, we can calculate the equivalent
flat rate of interest paid on the loan.
4. A loan that is repaid over a shorter period of time will usually cost less than
one where the repayments are made over the full term of the loan.
5. The flexibility of a loan, which includes factors such as whether extra
repayments can be made, is important when considering the cost of a loan.
6. When calculating the cost of a loan, any ongoing fees need to be calculated.

WORKED

Example

1 A $15 000 loan is to be repaid at 8% p.a. flat rate interest over a 10-year term.
( 1 + r )n 1
Use the formula E = ---------------------------- to calculate the effective rate of interest.
n

Substitution
into a
formula

3 A bank offers loans at 8% p.a. flat rate of interest. Calculate the effective rate
of interest for a loan taken over:
a 2 years
b 3 years
c 4 years
d 5 years
e 10 years
f 20 years.
WORKED

Example

SkillS

4 An $85 000 home loan at 9% p.a reducible interest is to be repaid over 25 years at
$713.32 per month.
a Calculate the total repayments on the loan.
b Calculate the total amount of interest paid.
c Calculate the equivalent flat rate of interest on the loan.
5 Calculate the equivalent flat rate of interest paid on a $115 000 loan at 12% p.a.
reducible interest to be repaid over 30 years at $1182.90 per month.

L Spre
XCE ad

Effective
rate of
interest

sheet

2 Calculate the effective rate of interest on each of the following flat rate loans.
a 10% p.a. over 4 years
b 8% p.a. over 2 years
c 12% p.a. over 5 years
d 7.5% p.a. over 10 years e 9.6% p.a. over 6 years

1.6

HEET

The cost of a loan

1C

20

Maths Quest General Mathematics HSC Course

WORKED

Example

6 Kim borrows $12 000 for a holiday to South-East


Asia. She is faced with a choice of two loans.
Loan I: At 10% p.a. flat rate of interest over 2 years
to be repaid at $600 per month.
Loan II: At 12.5% p.a. reducible interest over
3 years to be repaid at $401.44 per month.
Which loan will cost Kim the least money?
7 Calculate the total cost of repaying a loan
$100 000 at 8% p.a. reducible interest:
a over 25 years with a monthly repayment
$771.82
b over 20 years with a monthly repayment
$836.44
c over 10 years with a monthly repayment
$1213.28.

WORKED

Example

10

of
of
of
of

8 Masako and Toshika borrow $125 000 for their


home. They have the choice of two loans.
Loan 1: A low interest loan at 7% p.a. interest over
25 years with fixed repayments of $833.47 per
month.
Loan 2: A loan at 7.5% p.a interest over 25 years
with minimum repayments of $923.74 per month.
Masako and Toshika believe they can afford to pay
$1000 per month. If they do, Loan 2 will be repaid
in 20 years and 4 months.
Which loan should they choose if they could afford to pay the extra each month?
9 multiple choice
A loan can be taken out at 8% p.a. flat interest or 9% p.a. reducible interest. Using the
( 1 + r )n 1
formula E = ---------------------------- , the number of years of the loan (n) after which the effective
n
rate of interest on the flat rate loan becomes greater than the reducible rate loan is:
A 2 years
B 3 years
C 4 years
D 5 years
10 Glenn and Inge are applying for a $150 000 loan to be repaid over 25 years.
a Bank A charges 7.8% p.a. interest, no fees, with the loan to be repaid at $1137.92
per month. Calculate the total cost of this loan.
b Bank B charges 7.6% p.a. interest, a $600 loan application fee, a $5 per month
management fee and repayments of $1118.26 per month. Calculate the total cost of
this loan.
11 multiple choice
A $50 000 loan is to be taken out. Which of the following loans will have the lowest
total cost?
A 5% p.a. flat rate interest to be repaid over 10 years
B 8% p.a. reducible interest to be repaid over 10 years at $606.64 per month
C 6% p.a. reducible interest to be repaid over 12 years at $487.93 per month
D 6.5% p.a. reducible interest to be repaid over 10 years at $567.74 per month, with
a $600 loan application fee and $8 per month account management fee

Chapter 1 Credit and borrowing

21

12 A home loan of $250 000 is taken out over a 20-year term. The interest rate is 9.5% p.a.
and the monthly repayments are $2330.33.

b Calculate the equivalent flat rate of interest on the loan. (Consider the extra
payments as part of the interest.)
c If the loan is repaid at $3000 per month, it will take 11 1--- years to repay the loan.
2
Calculate the equivalent flat rate of interest if this repayment plan is followed.

Graphics Calculator tip! Loan repayment function


Your Casio graphics calculator can calculate the amount of each monthly repayment
on a home loan when given the term of the loan and the interest rate. The PMT
function, which is under the compound interest menu, allows for such calculations to
be made.
Consider a loan of $250 000 to be repaid over 25 years at 8% p.a. with interest added
and repayments made monthly. We wish to find the amount of each monthly
repayment.
1. From the MENU select TVM.

2. Press F2 to select Compound Interest.

3. Enter the following settings.


n = 25 12
I% = 8
PV = 250000
PMT = 1
FV = 0
P/Y = 12 (payments per year)
C/Y = 12 (You will need to scroll to see this.)
4. Press F4 (PMT) to find the amount of each monthly
repayment, which will be displayed as a negative.

Work

a The mortgage application fee on this loan was $600 and there is a $10 per month
account management fee. Calculate the total cost of repaying this loan.
T
SHEE

1.1

22

Maths Quest General Mathematics HSC Course

Researching home loans


1 Suppose that you wish to borrow $100 000 to buy a home. Go to a bank or other
lender and gather the following information.
a The annual interest rate
b The loan application fee and any other costs such as stamp duty, legal costs
etc. associated with establishing the loan
c Is there a monthly account keeping or management fee?
d The monthly repayment if the loan is repaid over:
i 15 years
ii 20 years
iii 25 years
e The total cost of repaying the loan in each of the above examples
2 There are many ways that people can reduce the overall cost of repaying a
mortgage. Research and explain why people are able to save money by adopting
the following repayment strategies.
a Repaying the loan fortnightly or weekly instead of monthly
b Using an account where the whole of a persons net pay is deposited on the
mortgage and then a redraw is used to meet living expenses

Credit cards
Credit cards are the most common line of day-to-day credit that most people use. A
credit card works as a pre-approved loan up to an amount agreed upon by the customer
and the bank. The card can then be used until the amount of the debt reaches this limit.
As with other types of loan, the bank charges interest upon the amount that is owed
on the card and repayments must be made monthly. The way in which the interest is
calculated varies with different types of credit cards.
Some cards have interest charged from the day on which the purchase was made.
Others have what is called an interest-free period. This means that a purchase that is
made will appear on the next monthly statement. Provided that this amount is paid by
the due date, no interest is charged. Hence, the customer can repay the loan within a
maximum of 55 days and be charged no interest.
Generally, credit cards without an interest-free period have a lower interest rate than
those with an interest-free period. These cards, however, generally attract an annual fee.
This annual fee can in some cases be waived if a certain amount is spent on the card
over the year.
The minimum monthly repayment on most credit cards is 5% of the outstanding
balance, or $10, whichever is greater.

WORKED Example 11

On Trevors credit card statement he has an outstanding balance of $1148.50.


The minimum monthly payment is 5% of the outstanding balance, or $10, whichever
is greater. Calculate the minimum repayment that Trevor must make.
THINK
1 Calculate 5% of the outstanding balance.
2 Decide which repayment is greater and
give a written answer.

WRITE
5% of $1148.50 = $57.43
The minimum repayment is $57.43.

Chapter 1 Credit and borrowing

23

Credit card interest is quoted as an annual amount but is added monthly. To calculate
the interest due, calculate one months interest on the outrstanding balance..

WORKED Example 12
The outstanding balance on a credit card is $2563.75. If the full balance is not paid by the
due date, one months interest will be added at a rate of 18% p.a. Calculate the amount of
interest that will be added to the credit card.
THINK

WRITE

Use the simple interest formula to calculate


one months interest.

I = Prn
I = $2563.75
I = $38.46

0.18

1
-----12

In practice, most credit cards calculate interest on the outstanding balance at a daily
rate and then add the interest monthly. If a credit card advertises its interest rate as
18% p.a., the daily rate is 0.049 315%. To work out the interest, you will need to count
the number of days that the credit card has each different balance over the month.

WORKED Example 13
An extract from a credit card statement is shown below.
Interest rate = 15% p.a.
Daily rate = 0.041 096%
Date

Credit

Debit

Balance

1 June
10 June

$900
$400 repayment

$500

15 June

$350 purchase

$850

22 June

$140 purchase

$990

1 July

??? interest

Calculate the interest that will be due for the month of June.
THINK
1
2
3
4
5

For 1 June 9 June inclusive (9 days), the


balance owing is $900. Calculate the interest.
For 10 June 14 June inclusive (5 days), the
balance owing is $500. Calculate the interest.
For 15 June 21 June inclusive (7 days), the
balance owing is $850. Calculate the interest.
For 22 June 30 June inclusive (9 days), the
balance owing is $990. Calculate the interest.
Add each amount of interest to calculate the
total interest for the month.

WRITE
I = 0.041 096% of $900 9
I = $3.33
I = 0.041 096% of $500 5
I = $1.03
I = 0.041 096% of $850 7
I = $2.45
I = 0.041 096% of $990 9
I = $3.66
Total interest = $3.33 + $1.03
Total interest = + $2.45 + $3.66
Total interest = $10.47

24

Maths Quest General Mathematics HSC Course

Graphics Calculator tip! Calculating interest on a daily basis


When doing this type of question where we need to consider interest calculated on a
daily basis the TVM mode of your calculator is very useful. Consider the method shown
below for worked example 13.
1. From the MENU of your calculator select TVM.

2. Press F1 to select Simple Interest.

3. For 9 days the balance is $900, so enter:


n=9
I% = 15
PV = 900

4. Press F1 (SI) to get the interest for these 9 days.

Interest = $3.33
5. For 5 days the balance is $500. Press EXIT to
return to the previous screen; change the values of n
and PV.
n=5
I% = 15
PV = 500

Interest = $1.03

Then press F1 for the simple interest.


6. For 7 days the balance is $850. Press EXIT to
return to the previous screen; change the values of n
and PV.
n=7
I% = 15
PV = 850

Then again press F1 for the simple interest.

Interest = $2.45

Chapter 1 Credit and borrowing

7. For 9 days the balance is $990. Press EXIT to


return to the previous screen; change the values of n
and PV.
n=9
I% = 15
PV = 990
Then again press F1 for the simple interest.
8. Add each amount of interest to find the total
amount of interest for the month.

25

Interest = $3.66

Total interest
= $3.33 + $1.03 + $2.45 + $3.66
= $10.47

When deciding which credit card is most suitable for your needs, consider if you will
generally be able to pay most items off before the interest-free period expires. The total
cost in interest over a year will vary according to the repayment pattern.

WORKED Example 14

Kerry has a credit card with an interest-free period and interest is then charged on the
outstanding balance at a rate of 18% p.a. Kerry pays a $1200 bill for her council rates on
her credit card.
a Kerry pays $600 by the due date. What is the outstanding balance on the card?
b Calculate the interest Kerry must then pay for the second month.
c An alternative credit card charges 12% p.a. interest with no interest-free period.
Calculate the interest that Kerry would have been charged on the first month.
d Calculate the balance owing after Kerry pays $600 then calculate the interest for the
second month.
e Which credit card would be the cheapest to use for this bill?
THINK

WRITE

a Subtract the repayment from the


balance.

a Balance owing = $1200


Balance owing = $600

b Use the simple interest formula to


calculate one months interest.

b I = Prn
= $600 0.18
= $9.00

c Use the simple interest formula to


calculate the first months interest.

c I = Prn
= $1200 0.12
= $12.00

d Balance owing = $1200 + $12


= $612
I = Prn
1
= $612 0.12 ----12
= $6.12

1
2

Add the interest to the amount of the


bill and subtract the repayment.
Use the simple interest formula to
calculate the second months
interest.

e Add the two months of interest together


for the second card and compare with
the interest for the first card.

$600

1
-----12

1
-----12

$600

e The interest on the second card is $18.12 and


therefore the card with the interest-free
period is cheaper in this case.

26

Maths Quest General Mathematics HSC Course

remember
1. A credit card is a source of an instant loan to the card holder.
2. The card is repaid monthly with the minimum payment usually 5% of the
outstanding balance, or $10, whichever is the greater.
3. There are many different types of credit card. The main difference between
them is that some have an interest-free period while others charge interest from
the date of purchase.
4. Cards without an interest-free period generally have a lower rate of interest
than those with an interest-free period.
5. The interest on a credit card is usually calculated as a daily rate. This is found
by dividing the annual rate by 365.
6. The TVM function on the graphics calculator can be used to calculate the
monthly interest on a credit card.
7. To calculate the cheaper credit card, we need to consider the repayment plan
that would be used.

1D

Credit cards

1 Roy has a credit card with an outstanding balance of $2730. Calculate the minimum
payment if he must pay 5% of the balance, or $10, whichever is greater.
11
2 The minimum monthly repayment on a credit card is 5% of the balance, or $10, whichever is greater. Calculate the minimum monthly repayment on a balance of:
a $3500
b $1194.50
c $492.76
d $150
e $920.52.

WORKED

Example

3 Leonie has a credit card with an outstanding balance of $1850. If the interest rate is
18% p.a., calculate the amount of interest that Leonie will be charged for one month if
12
the balance is not paid by the due date.

WORKED

Example

4 Hassim buys a refrigerator for $1450 with his credit card. The card has no interest-free
period and interest is charged at a rate of 15% p.a. Calculate one months interest on
this purchase.
5 Michelle has a $2000 outstanding balance on her credit card. The interest rate charged
is 21% p.a. on the balance unpaid by the due date.
a If Michelle pays $200 by the due date, calculate the balance owing.
b Calculate the interest that Michelle will owe for the next month.
c What will be the balance owing on Michelles next credit card statement?
d What will be the total amount owing on the credit card after another months interest
is added?
6 Chandra has a credit card which charges interest at a rate of 12% p.a. but has no
interest-free period. He makes a purchase of $1750 on the credit card.
a After one month Chandras credit card statement arrives. What will be the outstanding balance on the statement?
b The minimum repayment will be 5% of the outstanding balance. Calculate the
amount that Chandra will owe if he makes only the minimum payment.
c In the next month Chandra makes purchases totalling $347.30. Calculate the interest
charged and the balance owing for the next months statement.

Chapter 1 Credit and borrowing

WORKED

Example

13

27

7 An extract of a credit card statement is shown below. Take 1 year = 365.25 days.
Interest rate = 18% p.a.
Date

Daily rate = 0.049 28%

Credit ($)

Debit ($)

1 July

256.40

10 July
20 July

Balance ($)

40 purchase
40 repayment

1 August

??? interest

a Complete the balance column. Calculate the balance owing on 10 July and 20 July.
b Calculate the interest due on 1 August and the balance on that date.
8 Study the credit card statement below.
Interest rate = 16.5% p.a.
Date

Daily rate = ______

Credit ($)

Debit ($)

1 Jan.
8 Jan.

1548.50
500 repayment

15 Jan.

399 purchase

1 Feb.
8 Feb.
1 March

Balance ($)

??? interest
??? repayment
??? interest

a Calculate the daily rate of interest, correct to 4 decimal places (take 1 year = 365.25
days).
b Calculate the interest added to the account on 1 February.
c On 8 February the minimum repayment of 5% is made. Calculate the amount of this
repayment.
d Calculate the outstanding balance on the account on 1 March.
9 Kai has two credit cards. One has an interest-free period and interest is then charged on
the outstanding balance at a rate of 18% p.a. The other has no interest-free period with
14
interest added from the date of purchase at a rate of 14% p.a. Kai has $1500 worth of
bills to pay in the coming month and intends to use one of the cards to pay them, then
pay the balance off in monthly instalments of $500.
a If Kai uses the card with the interest-free period and pays $500 by the due date, what
is the outstanding balance on the card?
b Calculate the interest Kai must then pay for the second month.
c Calculate the balance owing at the end of the second month and the balance owing
at the end of the third month, at which time Kai pays off the entire balance.
d Calculate the interest payable in the first month if Kai uses the card without the
interest-free period.
e Calculate the balance owing after Kai pays $500 then calculate the interest for the
second month.
f Calculate the balance owing at the end of the second month and the balance owing
at the end of the third month, at which time Kai pays off the entire balance.
g Which card should Kai use for these bills?

WORKED

Example

28

Maths Quest General Mathematics HSC Course

Researching credit cards


Find out about the costs associated with two credit cards. One of the cards should
have an interest-free period and the other no interest-free period. Find out:
1 if there is an annual fee associated with having the card
2 the interest rate on each card
3 the minimum monthly payment to be made on each card
4 what credit limits apply to a first-time credit card holder
5 what benefits such as Fly-Buys or Frequent Flyer Points can be obtained
from use of the card
6 any other relevant information about the card.

2
1 Calculate the amount of flat rate interest payable on a loan of $4500 at 21% p.a. over
a 3 year term.
2 A loan of $2000 is repaid over 1 year at a rate of $100 per week. Calculate the rate of
interest charged on the loan.
3 A loan of $120 000 at 11% p.a. reducible over 20 years is repaid at $1238.63 per
month. The bank also charges an $8 per month account management fee. Calculate
the total cost of repaying the loan.
4 A loan of $5000 is advertised at a rate of 9% p.a. flat rate interest for a term of
( 1 + r )n 1
4 years. Use the formula E = ---------------------------- to calculate the effective rate of interest on
n
this loan (correct to 1 decimal place).
5 A loan of $10 000 at 11% p.a. reducible interest is repaid over 4 years at a rate of
$258.46 per month. Calculate the equivalent flat rate of interest charged on the loan
(correct to 1 decimal place).
6 With reference to credit cards, what is meant by the term interest-free period?
7 The minimum repayment on a credit card is 5% or $10, whichever is greater.
Calculate the minimum repayment for July that is to be made on a card with an
outstanding balance of $3297.50.
8 On the credit card in question 7, a repayment of $500 is made by the due date.
Calculate the interest that will be charged for August at the rate of 18% p.a.
9 An alternative credit card with no interest-free period has an interest rate of 12% p.a.
Calculate the interest on the above credit card for July at this rate.
10 Calculate the total interest that would have been charged for 2 months assuming a
$500 payment was made on 1 August.

Chapter 1 Credit and borrowing

29

Loan repayments
With a reducing balance loan, an amount of interest is added to the principal each
month and then a repayment is made which is then subtracted from the outstanding
balance. Consider the case below of a $2000 loan at 15% p.a. to be repaid over 1 year
in equal monthly instalments of $180.52.
Month

Opening
balance

Interest

Closing
balance

$2000.00

$25.00

$1844.48

$1844.48

$23.06

$1687.02

$1687.02

$21.09

$1527.59

$1527.59

$19.09

$1366.17

$1366.17

$17.08

$1202.73

$1202.73

$15.03

$1037.25

$1037.25

$12.97

$ 869.70

$ 869.70

$10.87

$ 700.05

$ 700.05

$ 8.75

$ 528.29

10

$ 528.29

$ 6.60

$ 354.37

11

$ 354.37

$ 4.43

$ 178.29

12

$ 178.29

$ 2.23

0.00

The actual calculation of the amount to be repaid each month to pay off the loan plus
interest in the given period of time is beyond this course. The most practical way to find
the amount of each monthly repayment is to use a table of repayments.

30

Maths Quest General Mathematics HSC Course

Monthly repayment per $1000 borrowed


Interest rate
Year

5%

6%

7%

8%

9%

10%

11%

12%

13%

14%

15%

$85.61 $86.07 $86.53 $86.99 $87.45 $87.92 $88.38 $88.85 $89.32 $89.79 $90.26

$43.87 $44.32 $44.77 $45.23 $45.68 $46.14 $46.61 $47.07 $47.54 $48.01 $48.49

$29.97 $30.42 $30.88 $31.34 $31.80 $32.27 $32.74 $33.21 $33.69 $34.18 $34.67

$23.03 $23.49 $23.95 $24.41 $24.89 $25.36 $25.85 $26.33 $26.83 $27.33 $27.83

$18.87 $19.33 $19.80 $20.28 $20.76 $21.25 $21.74 $22.24 $22.75 $23.27 $23.79

$16.10 $16.57 $17.05 $17.53 $18.03 $18.53 $19.03 $19.55 $20.07 $20.61 $21.15

$14.13 $14.61 $15.09 $15.59 $16.09 $16.60 $17.12 $17.65 $18.19 $18.74 $19.30

$12.66 $13.14 $13.63 $14.14 $14.65 $15.17 $15.71 $16.25 $16.81 $17.37 $17.95

$11.52 $12.01 $12.51 $13.02 $13.54 $14.08 $14.63 $15.18 $15.75 $16.33 $16.92

10

$10.61 $11.10 $11.61 $12.13 $12.67 $13.22 $13.78 $14.35 $14.93 $15.53 $16.13

11

$ 9.86 $10.37 $10.88 $11.42 $11.96 $12.52 $13.09 $13.68 $14.28 $14.89 $15.51

12

$ 9.25 $ 9.76 $10.28 $10.82 $11.38 $11.95 $12.54 $13.13 $13.75 $14.37 $15.01

13

$ 8.73 $ 9.25 $ 9.78 $10.33 $10.90 $11.48 $12.08 $12.69 $13.31 $13.95 $14.60

14

$ 8.29 $ 8.81 $ 9.35 $ 9.91 $10.49 $11.08 $11.69 $12.31 $12.95 $13.60 $14.27

15

$ 7.91 $ 8.44 $ 8.99 $ 9.56 $10.14 $10.75 $11.37 $12.00 $12.65 $13.32 $14.00

16

$ 7.58 $ 8.11 $ 8.67 $ 9.25 $ 9.85 $10.46 $11.09 $11.74 $12.40 $13.08 $13.77

17

$ 7.29 $ 7.83 $ 8.40 $ 8.98 $ 9.59 $10.21 $10.85 $11.51 $12.19 $12.87 $13.58

18

$ 7.03 $ 7.58 $ 8.16 $ 8.75 $ 9.36 $10.00 $10.65 $11.32 $12.00 $12.70 $13.42

19

$ 6.80 $ 7.36 $ 7.94 $ 8.55 $ 9.17 $ 9.81 $10.47 $11.15 $11.85 $12.56 $13.28

20

$ 6.60 $ 7.16 $ 7.75 $ 8.36 $ 9.00 $ 9.65 $10.32 $11.01 $11.72 $12.44 $13.17

21

$ 6.42 $ 6.99 $ 7.58 $ 8.20 $ 8.85 $ 9.51 $10.19 $10.89 $11.60 $12.33 $13.07

22

$ 6.25 $ 6.83 $ 7.43 $ 8.06 $ 8.71 $ 9.38 $10.07 $10.78 $11.50 $12.24 $12.99

23

$ 6.10 $ 6.69 $ 7.30 $ 7.93 $ 8.59 $ 9.27 $ 9.97 $10.69 $11.42 $12.16 $12.92

24

$ 5.97 $ 6.56 $ 7.18 $ 7.82 $ 8.49 $ 9.17 $ 9.88 $10.60 $11.34 $12.10 $12.86

25

$ 5.85 $ 6.44 $ 7.07 $ 7.72 $ 8.39 $ 9.09 $ 9.80 $10.53 $11.28 $12.04 $12.81
The table shows the monthly repayment on a $1000 loan at various interest rates
over various terms. To calculate the repayment on a loan, we simply multiply the repayment on $1000 by the number of thousands of dollars of the loan.

Chapter 1 Credit and borrowing

31

WORKED Example 15
Calculate the monthly repayment on a loan of $85 000 at 11% p.a. over a 25-year term.
THINK
1
2

WRITE

Look up the table to find the monthly repayment


on $1000 at 11% p.a. for 25 years.
Multiply this amount by 85.

Monthly repayment = $9.80


Monthly repayment = $833

85

This table can also be used to make calculations such as the effect that interest rate rises
will have on a home loan.

WORKED Example 16
The Radley family borrow $160 000 for a home at 8% p.a. over a 20-year term. They
repay the loan at $1400 per month. If the interest rate rises to 9%, will they need to
increase their repayment and, if so, by how much?
THINK
1
2
3

WRITE

Look up the table to find the monthly repayment


on $1000 at 8% p.a. for 20 years.
Multiply this amount by 160.
If this amount is greater than $1400, state the
amount that the repayment needs to rise.

Monthly repayment = $9.00 160


Monthly repayment = $1440.00
The Radley family will need to increase
their monthly repayments by $40.

remember
1. The amount of each monthly repayment is best determined by using a table of
repayments.
2. The amount of each repayment is calculated by multiplying the monthly
repayment on a $1000 loan by the number of thousands of the loan.

1E

Loan repayments

1 Use the table of repayments on page 30 to calculate the monthly repayment on a 1.7 S
killS
$75 000 loan at 7% p.a. over a 15-year term.
15
2 Use the table of repayments to calculate the monthly repayment on each of the Reading
tables
following loans.
a $2000 at 8% p.a. over a 2-year term
L Spre
b $15 000 at 13% p.a. over a 5-year term
XCE ad
c $64 000 at 15% p.a. over a 25-year term
Reducing
d $100 000 at 12% p.a. over a 20-year term
balance
e $174 000 at 9% p.a. over a 22-year term
loans

WORKED

Example

sheet

HEET

32

Maths Quest General Mathematics HSC Course

3 Jenny buys a computer for $4000 on the following terms: 10% deposit with the balance
paid in equal monthly instalments over 3 years at an interest rate of 14% p.a.
a Calculate Jennys deposit.
b Calculate the balance owing on the computer.
c Use the table of repayments to calculate the amount of each monthly repayment.
4 Mr and Mrs Dubois borrow $125 000 over 20 years at 10% p.a. to purchase a house.
They repay the loan at a rate of $1500 per month. If the interest rate rises to 12% p.a.,
16
will Mr and Mrs Dubois need to increase the size of their repayments and, if so, by how
much?

WORKED

Example

5 Mr and Mrs Munro take out a $180 000 home


loan at 9% p.a. over a 25-year term.
a Calculate the amount of each monthly
repayment.
b After 5 years the balance on the loan has
been reduced to $167 890. The interest rate
then rises to 10% p.a. Calculate the new
monthly repayment required to complete
the loan within the existing term.
6 A bank will lend customers money only if
they believe the customer can afford the
repayments. To determine this, the bank has a
rule that the maximum monthly repayment a
customer can afford is 25% of his or her gross
monthly pay. Darren applies to the bank for a
loan of $62 000 at 12% p.a. over 15 years.
Darren has a gross annual salary of $36 000.
Will Darrens loan be approved? Use calculations to justify your answer.
7 Tracey and Barry have a combined gross income of $84 000.
a Calculate Tracey and Barrys gross monthly income.
b Using the rule applied in the previous question, what is the maximum monthly
repayment on a loan that they can afford?
c If interest rates are 11% p.a., calculate the maximum amount (in thousands) that they
could borrow over a 25-year term.
8 Mr and Mrs Yousef borrow $95 000 over 25 years at 8% p.a. interest.
a Calculate the amount of each monthly repayment on the loan.
b Mr and Mrs Yousef hope to pay the loan off in a much shorter period of time. By
how much will they need to increase the monthly repayment to pay the loan off in
15 years?
9 Mr and Mrs Bath borrow $375 000 at 8% p.a. reducible over a 25-year term, with
repayments to be made monthly.
a Calculate the amount of each monthly repayment.
b Calculate the total amount that Mr and Mrs Bath will repay over the term of the loan.
c What is the total amount of interest that Mr and Mrs Bath will pay on the loan?
d Calculate the average amount of interest that Mr and Mrs Bath will pay each year.
e Calculate the equivalent flat rate of interest by expressing your answer to part d as
a percentage of the amount borrowed.

33

10 A loan of $240 000 is taken out over a 25-year term at an interest rate of 7% p.a.
reducible.
a Calculate the amount of each monthly repayment.
b Calculate the total repayments made on the loan.
c Calculate the amount of interest paid on the loan.
d Find the equivalent flat rate of interest.
e By following steps a to d above calculate the equivalent flat rate of interest if the
term of the loan is:
ii 20 years
ii 15 years.

Work

Chapter 1 Credit and borrowing

T
SHEE

1.2

Computer Application 3 Loan repayments

1. $80 000
2. $50 000

L Spre
XCE ad

Loan
repayments

3. $20 000.
Next, change the amount borrowed in the spreadsheet to $200 000. Does it take the
same length of time for the outstanding balance to be halved?
Change the interest rate to 12% p.a. and the amount borrowed back to $100 000.
Does it still take the same length of time for the balance to be halved?
Experiment with different loans and look for a pattern in the way in which the
balance of the loan reduces.

sheet

Access the spreadsheet Loan Repayments from the Maths Quest General Mathematics
HSC Course CD-ROM. This spreadsheet shows the graph of a home loan of $100 000
at 9% p.a. that is repaid over 25 years. Use the graph to determine how long it takes for
the outstanding balance to reduce to:

34

Maths Quest General Mathematics HSC Course

summary
Flat rate interest
A flat rate loan is one where interest is calculated based on the amount initially
borrowed.
Flat rate loans have the interest calculated using the simple interest formula:
I = Prn
The total repayments on a flat rate loan are calculated by adding the interest to the
amount borrowed.
The monthly or weekly repayments on a flat rate loan are calculated by dividing the
total repayments by the number of weeks or months in the term of the loan.

Home loans
The interest of home loans is calculated at a reducible rate. This means that the
interest is calculated on the outstanding balance at the time and not on the initial
amount borrowed.
The interest on home loans is usually calculated and added monthly while
repayments are calculated on a monthly basis.
To calculate the total cost of a home loan, we multiply the amount of each monthly
payment by the number of payments.

The cost of a loan


To compare a flat rate loan with a reducing balance loan, the equivalent reducing
balance interest rate can be calculated using the formula:
( 1 + r )n 1
E = ---------------------------n
When comparing two or more loans, the most accurate comparison is done by
calculating the total cost of repaying the loan.
A loan that is repaid over a shorter period of time will generally cost less even if the
interest rate may be slightly higher.
The flexibility of loan repayments is an important consideration when calculating
the cost of a loan.
When calculating the cost of a loan, fees such as application fee and account
management fees must be considered along with the interest payable.

Credit cards
A credit card is a pre-approved loan up to a certain amount called the credit limit.
There are many kinds of credit cards and the most important difference is that some
cards have an interest-free period while others attract interest from the date of
purchase.
Credit cards without an interest-free period generally have a lower rate of interest
than those with an interest-free period.

Chapter 1 Credit and borrowing

35

Each credit card will have a monthly statement and will require a minimum
payment each month.
When evaluating the best credit card for your circumstances, you need to consider
if you will be able to pay most bills by the due date and consider any fees attached
to the card.

Loan repayments
The amount of each monthly repayment is best calculated using a table of monthly
repayments.
The monthly repayment on a $1000 loan at the given rate over the given term is
then multiplied by the number of thousands of the loan to find the size of each
repayment.

36

Maths Quest General Mathematics HSC Course

CHAPTER
review
1A

1 Calculate the amount of flat rate interest that will be paid on each of the following loans.
a $8000 at 7% p.a. for 2 years
b $12 500 at 11.5% p.a. for 5 years
c $2400 at 17.8% p.a. for 3 years
d $800 at 9.9% p.a. over 6 months
e $23 400 at 8.75% p.a. over 6 years

1A

2 Calculate the total repayments made on a loan of $4000 at 23% p.a. flat rate interest to be
repaid over 3 years.

1A

3 Noel borrows $5600 at 7.6% p.a. flat rate interest to be repaid in monthly instalments over
3 years. Calculate the amount of each monthly instalment.

1A

4 Shane borrows $9500 to purchase a new car. He repays the loan over 4 years at a rate of
$246.60 per month. Calculate the flat rate of interest charged on the loan.

1B

5 Mr and Mrs Smith borrow $125 000 to purchase a home. The interest rate is 12% p.a. and
the monthly repayments are $1376.36. Calculate:
a the first months interest on the loan
b the balance of the loan after the first month.

1B

6 Mr and Mrs Buckley borrow $130 000 to purchase a home. The interest rate is 8% p.a. and
over a 20-year term the monthly repayment is $1087.37.
a Copy and complete the table below.
Month

Principal ($)

Interest ($)

Balance ($)

130 000.00

866.67

129 779.29

129 779.29

3
4
5
6
7
8
9
10
11
12

Chapter 1 Credit and borrowing

37

b Mr and Mrs Buckley decide to increase their monthly payment to $1500. Complete the
table below.
Month

Principal ($)

Interest ($)

Balance ($)

130 000.00

866.67

129 366.67

129 366.67

3
4
5
6
7
8
9
10
11
12
c How much less do Mr and Mrs Buckley owe at the end of one year by increasing their
monthly repayment?
7 Mr and Mrs Stone borrow $225 000 for their home. The interest rate is 9.6% p.a. and the
term of the loan is 25 years. The monthly repayment is $1989.48.
a Calculate the total repayments made on this loan.
b If Mr and Mrs Stone increase their monthly payments to $2000, the loan will be repaid in
24 years and 1 month. Calculate the amount they will save in repayments with this
increase.
( 1 + r )n 1
8 Use the formula E = ---------------------------- to calculate the effective interest rate on each of the
n
following flat rate loans (answer correct to 2 decimal places).
a $4000 at 7% p.a. over 2 years
b $12 000 at 11% p.a. over 5 years
c $1320 at 23% p.a. over 2 years
d $45 000 at 9.2% p.a. over 10 years
9 Yu-Ping borrows $13 500 for a holiday to Africa at 12.5% p.a. reducible interest over a
5-year term. The monthly repayments on the loan are $303.72.
a Calculate the total repayments on the loan.
b Calculate the amount of interest that Yu-Ping pays on the loan.
c Calculate the equivalent flat rate of interest on the loan.

1B

1C

1C

38

Maths Quest General Mathematics HSC Course

1C

10 Kristen and Adrian borrow $150 000 for their home. They have the choice of two loans.
Loan 1: At 8% p.a. interest over 25 years with fixed repayments of $1157.72.
Loan 2: At 8.25% p.a interest over 25 years with minimum repayments of $1182.68 and an
$8 per month account management fee.
Kristen and Adrian believe they can afford to pay $1500 per month. If they do, Loan 2
will be repaid in 14 years and 2 months. Which loan should Kristen and Adrian choose if
they can afford to pay the extra each month?

1C

11 Stephanie has a credit card with an outstanding balance of $423. Calculate the minimum
payment that must be made if she must pay 5% of the balance, or $10, whichever is greater.

1D

12 Lorenzo has a credit card with an outstanding balance of $850. If the interest rate is 24%
p.a., calculate the amount of interest that Lorenzo will be charged if the balance is not paid
by the due date.

1D

13 Jessica pays for her car repairs, which total $256.50, using her credit card. The credit card
has an interest rate of 15% p.a. and interest is charged from the date of purchase. Calculate
the amount of interest charged after one month on this card.

1D

14 Study the extract from the credit card statement below.


Interest rate = 19.5% p.a.
Daily rate = ______
Date

Credit ($)

Debit ($)

1 Jan.
6 Jan.

2584.75
600 repayment

15 Jan.

39.99 purchase

1 Feb.
8 Feb.
15 Feb.
1 March

Balance ($)

??? interest
??? repayment
425.85 purchase
??? interest

a Calculate the daily rate of interest. (Take 1 year = 365.25 days and answer correct to
4 decimal places.)
b Calculate the interest due for January.
c If the minimum monthly payment of 5% of the outstanding balance is made on
8 February, calculate the amount of this repayment.
d Calculate the interest for February.

1E

15 Use the table of repayments on page 30 to calculate the monthly repayment on each of the
following loans.
a $25 000 at 9% p.a. over a 10-year term
b $45 000 at 14% p.a. over a 15-year term
c $164 750 at 15% p.a. over a 25-year term
d $425 000 at 12% p.a. over a 15-year term

1E

16 Mr and Mrs Rowe take out a $233 000 home loan at 12% p.a. over a 25-year term.
a Use the table of repayments to calculate the amount of each monthly repayment.
b After 3 years the balance on the loan has been reduced to $227 657. The interest rate then
rises to 13% p.a. Calculate the new monthly repayment required to complete the loan
within the existing term.

Chapter 1 Credit and borrowing

39

Practice examination questions


1 multiple choice
The total repayments for a $3400 loan on a flat rate interest of 8.5% p.a. over a 3-year period are:
A $867
B $942.78
C $4267
D 4342.78
2 multiple choice
A $115 000 loan is repaid over a 25-year term at the rate of $1211.21 per month. The total
amount of interest that is paid on this loan is:
A $30 280.25
B $145 280.25
C $248 363.00
D $363 363.00
3 multiple choice
A $150 000 loan is to be taken out. Which of the following loans will have the lowest total cost?
A 4% p.a. flat rate interest to be repaid over 20 years
B 8% p.a. reducible interest to be repaid over 20 years at $1254.66 per month
C 9% p.a. reducible interest to be repaid over 15 years at $1521.40 per month
D 8.5% p.a. reducible interest to be repaid over 15 years at $1512.49 per month with a $900
loan application fee and $12 per month account management fee
4 multiple choice
Look at the table of loan repayments per $1000 shown below.
Interest rate (p.a.)
Term

9%

10%

11%

12%

10

$12.67

$13.22

$13.78

$14.35

15

$10.14

$10.75

$11.37

$12.00

20

$9.00

$9.65

$10.32

$11.01

25

$8.39

$9.09

$9.80

$10.53

Daniel has an $80 000 mortgage at 10% p.a. over 10 years. After interest rates rise to 12%
Daniel extends the term of his loan to 15 years. What is the change in Daniels monthly
repayments?
A They increase by $1.13 per month.
B They decrease by $1.22 per month.
C They increase by $90.40 per month.
D They decrease by $97.60 per month.
5 David buys a computer that has a cash price of $4600. David pays 10% deposit with the
balance in weekly instalments at 13% p.a. flat rate interest over a period of 4 years.
a Calculate the balance owing after David has paid the deposit.
b Calculate the total repayments that David must make on this loan.
c Calculate the amount of each weekly instalment
( 1 + r )n 1
d Use the formula E = ---------------------------- to calculate the equivalent reducible interest rate on this
n
loan.

40

Maths Quest General Mathematics HSC Course

6 Mr and Mrs Tarrant borrow $186 500 to purchase a home. The interest rate is 9% p.a. and the
loan is over a 20-year term.
a Use the table below to calculate the amount of each monthly repayment.
Interest rate (p.a.)
Term

9%

10%

11%

12%

10

$12.67

$13.22

$13.78

$14.35

15

$10.14

$10.75

$11.37

$12.00

20

$9.00

$9.65

$10.32

$11.01

25

$8.39

$9.09

$9.80

$10.53

b Calculate the total amount that they can expect to make in repayments.
c After 10 years the outstanding balance is $132 463 and the interest rate is increased to
11%. Calculate the amount of the monthly repayment they will need to make to complete
the loan within the term.
d The loan has a $5 per month account management fee. The Tarrants also had a $400 loan
application fee and $132.75 in stamp duty to pay in establishing the loan. Calculate the
total cost of the loan after 20 years.

CHAPTER

test
yourself

7 Paul has a credit card that has an interest-free period. The interest rate is 21% p.a.
a If Paul has an outstanding balance of $275.50, calculate the minimum payment he must
make by the due date if it is 5% of the balance, or $10, whichever is greater.
b If Paul pays only the minimum balance by the due date, calculate the balance owing for the
next month.
c Calculate the interest that Paul will be charged on his next months statement.
d If Paul pays the whole balance off next month, is this card cheaper than a card without an
interest-free period but an interest rate of 15% p.a.? Use calculations to justify your
answer.

Further
applications
of area and
volume

syllabus reference
Measurement 5
Further applications of
area and volume

In this chapter
2A
2B
2C
2D

Area of parts of the circle


Area of composite shapes
Simpsons rule
Surface area of cylinders
and spheres
2E Volume of composite solids
2F Error in measurement

areyou

READY?

Are you ready?

Try the questions below. If you have difficulty with any of them, extra help can be
obtained by completing the matching SkillSHEET. Either click on the SkillSHEET icon
next to the question on the Maths Quest HSC Course CD-ROM or ask your teacher for
a copy.

2.1

2.2

Area of a circle

1 Find the area of a circle with:


a radius 4 cm
b radius 19.6 cm

c diameter 9 cm

d diameter 19.7 cm

Areas of squares, rectangles and triangles

2 Find the area of each of the following.


10.9 m
a
b

c
7.6 m

3.7 m

13.8 m
4.5 cm

2.6

Volume of cubes and rectangular prisms (3a, 3b); Volume of triangular prisms (3c)

3 Find the volume of:


a

c
11 cm

2.7

8 cm
9 cm
24 cm

2.8

6 cm
26 cm

18 cm
Volume of cylinders (4a); Volume of a sphere (4b)

4 Find the volume of:


a
19 cm

2.9

b
12 m

8 cm

Volume of a pyramid

2.10

Find the volume of:


10 cm

7 cm

2.11

Error in linear measurement

For each of the following linear measurements, state the limits between which the true limits
actually lie.
a 15 cm (measured correct to the nearest centimetre)
b 8.3 m (measured correct to 1 decimal place)
c 4800 km (measured correct to the nearest 100 km)

Chapter 2 Further applications of area and volume

43

Area of parts of the circle


From previous work you should know that the area of a circle can be calculated using
the formula:
A = r2

WORKED Example 1

Calculate the area of a circle with a radius of 7.2 cm. Give your answer correct to
2 decimal places.
THINK
1
2
3

WRITE

Write the formula.


Substitute for the radius.
Calculate the area.

A = r2
A=
(7.2)2
A = 162.86 cm2

A sector is the part of a circle between two radii as


shown on the right.
To calculate the area of a sector we find the fraction
of the circle formed by the sector. For example, a semicircle is half of a circle and so the area of a semicircle is
half the area of a full circle. A quadrant is a quarter of
a circle and so the area is quarter that of a full circle.
For other sectors the area is calculated by using the
angle between the radii as a fraction of 360 and then
multiplying by the area of the full circle. This can be
written using the formula:
A = --------- r 2
360
where

is the angle between the two radii.

WORKED Example 2

Calculate the area of the sector drawn


on the right. Give your answer correct to
1 decimal place.
5 cm
80

THINK

WRITE

Write the formula.

A=

Substitute for

A=

Calculate the area.

and r.

--------360
80
--------360

r2

A = 17.5 cm2

52

44

Maths Quest General Mathematics HSC Course

An annulus is the area between two circles that have the same centre (i.e. concentric
circles). The area of an annulus is found by subtracting the area of the smaller circle
from the area of the larger circle. This translates to the formula A = (R2 r2) , where
R is the radius of the outer circle and r is the radius of the inner circle.

WORKED Example 3
Calculate the area of the annulus on the right.
Give your answer correct to 1 decimal place.

5.7 cm
3.2 cm

THINK

WRITE

Write the formula.

A = (R2 r 2)

Substitute R = 5.7 and r = 3.2.

A = (5.72 3.22)

Calculate.

A = 69.9 cm2

An ellipse is an oval shape and therefore


does not have a constant radius. The greatest
distance from the centre of the ellipse to the
circumference is called the semi-major axis,
a, while the smallest distance is called the
semi-minor axis, b, as shown in the figure on
the right. The area of an ellipse is calculated
using the formula, found on the formula
sheet:

semi-minor
axis (b)
semi-major
axis (a)

A = ab

WORKED Example 4
Calculate the area of the ellipse drawn
on the right. Give your answer correct
to 2 decimal places.

4.2 m
6.6 m

THINK

WRITE

Write the formula.

A = ab

Substitute for the values of a and b.

A=

Calculate the area.

6.6

A = 87.08 m

4.2
2

Chapter 2 Further applications of area and volume

45

remember

1. The area of a circle is found using the formula A = r2.


2. The area of a sector can be found by multiplying the area of a full circle by the
fraction of the circle given by the angle in the sector. You can use the formula
A = --------r 2.
360
3. An annulus is the area between two concentric circles. The area is found by
using the formula A = (R2 r2) , where R is the radius of the outer circle
and r is the radius of the inner circle.
4. An ellipse is an oval shape. The area is calculated using the formula
A = ab , where a is the length of the semi-major axis and b is the length of
the semi-minor axis.

2A
WORKED

Example

1 Calculate the area of the circle


drawn on the right, correct to
1 decimal place.

2.1
6.4 cm

Area
of a
circle

2 Calculate the area of each of the circles drawn below, correct to 2 decimal places.
a
b
c
33 mm
9 cm

7.4 m

26.5 cm

f
6.02 m
3.84 m

3 Calculate the area of a circle that has a diameter of 15 m. Give your answer correct to
1 decimal place.
WORKED

Example

SkillS

HEET

Area of parts of the circle

4 Calculate the area of the sector drawn on the right.


Give your answer correct to 1 decimal place.
7.2 m

46

Maths Quest General Mathematics HSC Course

5 Calculate the area of each of the sectors drawn below. Give each answer correct to
2 decimal places.
a
b
c

23 m

5.2 cm

135
74 mm

60

20

9.2 mm
150

39 mm
240
19.5 m

72

6 Calculate, correct to 1 decimal place, the area of a semicircle with a diameter of


45.9 cm.
WORKED

Example

7 Calculate the area of the annulus


shown at right, correct to 1 decimal
place.

12 cm

6 cm

8 Calculate the area of each annulus drawn below, correct to 3 significant figures.
a
b
c
9.7 m

77 mm

20 cm
18 cm

13 mm

4.2 m

9 A circular garden of diameter 5 m is to have concrete laid around it. The concrete is
to be 1 m wide.
a What is the radius of the garden?
b What is the radius of the concrete circle?
c Calculate the area of the concrete, correct to 1 decimal place.
10 Calculate the area of the ellipse
drawn on the right, correct to
4
1 decimal place.

WORKED

Example

6 cm
10 cm

Chapter 2 Further applications of area and volume

47

11 Calculate the area of each of the ellipses drawn below. Give each answer correct to the
nearest whole number.
a
b
c
7.2 m

34 mm

14 cm

13.6 m

56 mm

21 cm

12 multiple choice
The area of a circle with a diameter of 4.8 m is closest to:
A 15 m2
B 18 m2
C 36 m2

D 72 m2

13 multiple choice
Which of the following calculations
will give the area of the sector shown
on the right?

45
8m

1
--8

42

1
--8

82

C 1-4 4 2

D 1-4 8 2

14 multiple choice
The area of the ellipse drawn on the
right is closest to:

86 cm
1.2 m

A 32 400 cm2

B 324 m2

C 5900 cm2

D 59 m2

15 A circular area is pegged out and has a diameter of 10 m.


a Calculate the area of this circle, correct to
1 decimal place.
b A garden is to be dug which is 3 m wide
around the area that has been pegged out.
Calculate the area of the garden to be dug.
Give your answer correct to 1 decimal place.
c In the garden a sector with an angle of 75 at
the centre is to be used to plant roses. Calculate
the area of the rose garden, correct to 1 decimal
place.
16 A circle has a diameter of 20 cm.
a Calculate the area of this circle, correct to 2 decimal places.
b An ellipse is drawn such that the radius of the circle forms the semi-major axis.
The semi-minor axis is to have a length equal to half the radius of the circle.
Calculate the length of the semi-minor axis.
c Calculate the area of the ellipse, correct to 2 decimal places.

48

Maths Quest General Mathematics HSC Course

Area of composite shapes


A composite shape is a shape that is made up of two or more regular shapes. The area
of a composite shape is found by splitting the area into two or more regular shapes and
calculating the area of each separately before adding them together. In many cases it
will be necessary to calculate the length of a missing side before calculating the area.
There will sometimes be more than one way to split the composite shape.

WORKED Example 5

6 cm

Find the area of the figure


at right.
18 cm
10 cm
12 cm

THINK
1

WRITE

Copy the diagram and divide the shape into


two rectangles.

6 cm
A1 8 cm
18 cm
10 cm

A2
12 cm

Calculate the length of the missing side in


rectangle 1. (Write this on the diagram.)
Calculate the area of rectangle 1.

Calculate the area of rectangle 2.

Add together the two areas.

18

10 = 8 cm

A1 = 6 8
A1 = 48 cm2
A2 = 10 12
A1 = 120 cm2
Area = 48 + 120
Area = 168 cm2

Composite areas that involve triangles may require you to also make a calculation using
Pythagoras theorem.

WORKED Example 6

13 m

Find the area of the figure


on the right.
10 m
24 m

THINK
1

Draw the triangle at the top and cut the


isosceles triangle in half.

WRITE
13 m
a
12 m

Chapter 2 Further applications of area and volume

THINK

49

WRITE

Calculate the perpendicular height using


Pythagoras theorem.

a2 = c2 b2
= 132 122
= 169 144
= 25
a = 25
=5m

Calculate the area of the triangle.

A=

1
--2

24

Calculate the area of the rectangle.

Add the two areas together.

= 60 m
A = 24 10
= 240 m2
Area = 60 + 240
Area = 300 m2

Composite areas can also be calculated by using subtraction rather than addition. In
these cases we calculate the larger area and subtract the smaller area in the same way as
we did with annuluses in the previous section.

WORKED Example 7

Find the shaded area in the figure


on the right.

6 cm

20 cm

30 cm

THINK

WRITE

Calculate the area of the rectangle.

Calculate the area of the circle.

Subtract the areas.

A = 30 20
A = 600 cm2
A=
62
A = 113.1 cm2
Area = 600 113.1
Area = 486.9 cm2

remember
1. To find the area of any composite figure, divide the shape into smaller regular
shapes and calculate each area separately.
2. You may have to use Pythagoras theorem to find missing pieces of
information.
3. Check if the best way to solve the question is by adding two areas or by
subtracting one area from the other to find the remaining area.

50

Maths Quest General Mathematics HSC Course

SkillS

2B
HEET

2.2

WORKED

Example

5
Areas of
squares,
rectangles
and triangles

Area of composite shapes

1 Copy the figure on the right into your


workbook and calculate its area by
dividing it into two rectangles.

4m

18 m
11 m
20 m

2 Find the area of each of the figures below. Where necessary, give your answer correct
to 1 decimal place.
a

7 cm b

5 cm

18 cm
12 cm

19 cm

16 cm
25 cm

6 cm
40 cm

5 cm
22 cm

12 cm

4 cm

8 cm

8 cm

4 cm
4 cm

SkillS

16 cm

HEET

2.3
Using
Pythagoras
theorem

Cabr

omet
i Ge ry

Pythagoras
calculations

3 Look at the triangle on the right.


a Use Pythagoras theorem to find
the perpendicular height of the triangle.
b Calculate the area of the triangle.

10 cm

6 cm

17 cm

15 cm

4 Below is an isosceles triangle.

8m

12 m

a Use Pythagoras theorem to find the perpendicular height of the triangle, correct to
1 decimal place.
b Calculate the area of the triangle.

51

Chapter 2 Further applications of area and volume

sheet

5 Calculate the area of each of the triangles below. Where necessary, give your answer
L Spre
XCE ad
correct to 1 decimal place.
Pythagoras
a
b
c
25 cm
26 m

24 m

Mensuration
124 mm

WORKED

Example

6 Find the area of each of the composite figures drawn below.


a
b
c 25 mm
13 cm
17 m

52 mm

15 mm

48 mm
54 mm

13 m

12 cm
30 m

7 multiple choice
The area of the composite figure on the right
is closest to:
A 139 m2
B 257 m2
2
C 314 m
D 414 m2
10 m

8 multiple choice
The area of the figure drawn on the right is:
A 36 m2
B 54 m2
2
C 72 m
D 144 m2

12 m

6m

9 A block of land is in the shape of a square with an equilateral triangle on top. Each
side of the block of land is 50 m.
a Draw a diagram of the block of land.
b Find the perimeter of the block of land.
c Find the area of the block of land.
10 In each of the following, find the area of the shaded region. Where necessary, give
your answer correct to 1 decimal place.
7
a
b
c
12 m

WORKED

Example

4 cm

10 cm

9 cm
3 cm

16 cm

8m

9 cm

7.
1

1.9 m
7.4 m

36 mm
40 mm

10 cm
95 mm

3.1 m

f
112 mm

cm

am
progr C

asio

GC

48 cm

Maths Quest General Mathematics HSC Course

11 An athletics track consists of a rectangle with two semicircular


ends. The dimensions are shown in the diagram on the right.
The track is to have a synthetic running surface laid. Calculate the
area which is to be laid with the running surface, correct to the
nearest square metre.

70 m
90 m

12 A garden is to have a concrete path laid around it. The garden is rectangular in shape
and measures 40 m by 25 m. The path around it is to be 1 m wide.
a Draw a diagram of the garden and the path.
b Calculate the area of the garden.
c Calculate the area of the concrete that needs to be laid.
d If the cost of laying concrete is $17.50 per m2, calculate the cost of laying the path.

1
Calculate the area of each of the figures drawn below. Where necessary, give your
answer correct to 1 decimal place.
1

3
5.8 cm
9.4 cm

12 cm

6.3 m

10 cm

91 mm
30 cm

25 cm
62 mm

20 cm
4 cm
25 cm

24 cm

9
12 cm
76

mm

20 m

32

40 m
40 cm

10
6 cm

52

12

cm

82 m

15
cm

Chapter 2 Further applications of area and volume

53

Simpsons rule
Simpsons rule is a method used to approximate the
area of an irregular figure. Simpsons rule approximates an area by taking a straight boundary and
dividing the area into two strips. The height of each
strip (h) is measured. Three measurements are then
taken perpendicular to the straight boundary, as
shown in the figure on the right. The formula for
Simpsons rule is:
h
A --- ( d f + 4d m + d l )
3
where h = distance between successive measuements
df = first measurement
dm = middle measurement
dl = last measurement.

dm
df
h

WORKED Example 8
Use Simpsons rule to approximate
the area shown on the right.
30 m
10 m
90 m

THINK
1

Calculate h.

Write down the values of df , dm and dl.

Write the formula.

Substitute.

Calculate.

dl

WRITE
h = 90 2
= 45
df = 10, dm = 30, dl = 0
h
A --- ( d f + 4d m + d l )
3
45
A ------ ( 10 + 4 30 + 0 )
3
= 15 130
1950 m2

Could Simpsons rule be used to estimate the areas of these irregular shapes from nature?

54

Maths Quest General Mathematics HSC Course

Simpsons rule can be used to approximate an irregular area without a straight edge.
This is done by constructing a line as in the diagram below and approximating the area
of each section separately.

WORKED Example 9
Use Simpsons rule to find an approximation
for the area shown on the right.

30 m
30 m

30 m

10 m

17 m

THINK

WRITE

Write down the value of h.


For the top area, write down the values
of df , dm and dl .

Write the formula.

Substitute.

Calculate the top area.

For the bottom area, write down the


values of df , dm and dl .

Write down the formula.

Substitute.

Calculate the bottom area.

10

Add the two areas together.

h = 30
df = 0, dm = 30, dl = 10
h
--- ( d f + 4d m + d l )
3
30
A ------ ( 0 + 4 30 + 10 )
3
10 130
1300 m2
df = 0, dm = 17, dl = 0
A

h
--- ( d f + 4d m + d l )
3
30
A ------ ( 0 + 4 17 + 0 )
3
10 68
680 m2
Area 1300 + 680
Area 1980 m2
A

Simpsons rule approximates an area, it does not give an exact measurement. To obtain
a better approximation, Simpsons rule can be applied several times to the area. This is
done by splitting the area in half and applying Simpsons rule separately to each half.

30 m

29 m

24 m

31 m

Use two applications of Simpsons rule to


approximate the area on the right.

32 m

WORKED Example 10
105 m

THINK
1
2

Calculate h by dividing 105 by 4.


(We are using 4 sub-intervals.)
Apply Simpsons rule to the left half.
Write the values of df , dm and dl .

WRITE
h = 105 4
= 26.25
df = 32, dm = 31, dl = 24

Chapter 2 Further applications of area and volume

THINK

WRITE

Write the formula.

Substitute.

Calculate the approximate area of the


left half.
Apply Simpsons rule to the left half.
Write the values of df , dm and dl .

h
--- ( d f + 4d m + d l )
3
26.25
A ------------- ( 32 + 4 31 + 24 )
3
8.75 180
1575 m2
df = 24, dm = 29, dl = 30
A

Write the formula.

Substitute.

Calculate the approximate area of the


right half.
Add the areas together.

10

55

h
--- ( d f + 4d m + d l )
3
26.25
A ------------- ( 24 + 4 29 + 30 )
3
8.75 170
1487.5 m2
Area 1575 + 1487.5
Area 3062.5 m2
A

remember
1. Simpsons rule is a method of approximating irregular areas.
h
2. The Simpsons rule formula is A --- ( d f + 4d m + d l ) , where h is the
3
distance between successive measurements, df is the first measurement, dm is
the middle measurement and dl is the last measurement.
3. A better approximation of an area can be found by using Simpsons rule twice.

2.4

60 m

18 m

1 The diagram on the right is of a part of a


creek.
a State the value of h.
b State the value of df , dm and dl .
c Use Simpsons rule to approximate
the area of this section of the creek.

9m

Example

Substitution
into
formulas

Simpsons rule

40 m

WORKED

HEET

2C

SkillS

56

Maths Quest General Mathematics HSC Course

72 m

40 m

16 m

0m

12 m

28 m

6m

12 m

10 m

35 m

2 Use Simpsons rule to approximate each of the areas below.


a
b
c

54 m

48 m
A1

30 m

30 m

5 m 18 m

3 The irregular area on the right has been divided into two
areas labelled A1 (upper area) and A2 (lower area).
a Use Simpsons rule to find an approximation for Al .
b Use Simpsons rule to find an approximation for A2.
c What is the approximate total area of the figure?

19 m 11 m

Example

7m

WORKED

A2

21 m

31 m
27 m

7m

51 m

22 m

5 multiple choice
Consider the figure drawn on the right.
Simpsons rule gives an approximate area of:
A 1200 m2
B 2400 m2
2
C 3495 m
D 6990 m2

27 m
40 m

16 m

21 m

27 m

16 m

12 m 10 m

23 m

45 m

14 m 6 m

17 m

45 m
12 m

22 m 11 m

4 Use Simpsons rule to find an approximation for each of the areas below.
a
b
c

90 m

6 multiple choice
If we apply Simpsons rule twice, how many measurements from the traverse line
need to be taken?
A4
B 5
C7
D9

25 m

36 m

10 m

45 m

50 m

18 m 18 m 18 m 18 m

10 m 10 m 10 m 10 m

60 m 60 m 60 m 60 m

22 m

33 m

11 m

50 m

44 m

20 m

71 m

42 m

87 m

102 m

8 Use Simpsons rule twice to approximate each of the areas drawn below.
a
b
c

45 m

10

7 Use Simpsons rule twice to approximate


the area on the right.

63 m

Example

54 m

WORKED

21 m 21 m 21 m 21 m

Chapter 2 Further applications of area and volume

36 m

27 m

32 m
15 m

9 The figure on the right is of a cross-section of a


waterway.
a Use Simpsons rule once to find an approximate
area of this section of land.
b Use Simpsons rule twice to obtain a better
approximation for this area of land.

57

Work

20 m
30 m
35 m
36 m
38 m
41 m
45 m
30 m
24 m

10 Apply Simpsons rule four times to


approximate the area on the right.

9m 9m 9m 9m 9m 9m 9m 9m

Surface area of cylinders and spheres


From earlier work you should remember that surface area
is the area of all surfaces of a 3-dimensional shape.
Consider a closed cylinder with a radius (r) and a
perpendicular height (h). The surface of the cylinder
h
consists of two circles and a rectangle.
Area of top = r 2
r
Area of bottom = r 2
The rectangular side of the cylinder will have a length
equal to the circumference of the circle (2 r) and a width
equal to the height (h) of the cylinder.
Area of side = 2 rh
The surface area of the closed cylinder can be calculated using the formula:
SA = 2 r 2 + 2 rh

WORKED Example 11

Calculate the surface area of the closed cylinder drawn on


the right. Give your answer correct to 1 decimal place.
10 cm
9 cm

THINK
1
2
3

WRITE

Write the formula.


Substitute the values of r and h.
Calculate the surface area.

SA = 2 r 2 + 2 rh
SA = 2
92 + 2
SA = 1074.4 cm2

10

For cylinders, before calculating the surface area you need to consider whether the
cylinder is open or closed. In the case of an open cylinder there is no top and so the
formula needs to be written as:
SA = r 2 + 2 rh

T
SHEE

2.1

58

Maths Quest General Mathematics HSC Course

Note: On the formula sheet in the exam only the formula for the closed cylinder is
provided. You will need to adapt the formula yourself for examples such as this.

WORKED Example 12
Calculate the surface area of an open cylinder with a radius of 6.5 cm and a height of
10.8 cm. Give your answer correct to 1 decimal place.
THINK
1
2
3

WRITE

Write the formula.


Substitute the values of r and h.
Calculate the surface area.

SA = r 2 + 2 rh
SA =
(6.5)2 + 2
SA = 573.8 cm2

A sphere is a round 3-dimensional shape, and the


only measurement given is the radius (r). The surface
area of a sphere can be calculated using the formula:
SA = 4 r 2

WORKED Example 13
Calculate the surface area of the
sphere drawn on the right. Give the
answer correct to 1 decimal place.

THINK
1
2
3

2.7 cm

WRITE

Write the formula.


Substitute the value of r.
Calculate the surface area.

SA = 4 r 2
SA = 4
(2.7)2
2
SA = 91.6 cm

remember
1. The surface area of a closed cylinder is
found using the formula
SA = 2 r 2 + 2 rh .
2. If the cylinder is an open cylinder, the
surface area formula becomes
SA = r 2 + 2 rh.
3. The surface area of a sphere is found
using the formula SA = 4 r 2 .

The Atomium, Brussels

6.5

10.8

59

Chapter 2 Further applications of area and volume

Surface area of cylinders and


spheres

2D

2.5

Example

11

1 Calculate the surface area of a closed cylinder with a radius of 5 cm and a height of
11 cm. Give your answer correct to 1 decimal place.
Circumference
of a circle

12 cm

5 cm

1.6 m

2 Calculate the surface area of each of the closed cylinders drawn below. Give each
answer correct to 1 decimal place.
a
b
c
20 cm

1.1 m
3 cm

f
5.9 cm

20 cm

1.5 m

5.9 cm
r

2.3 m

r = 5 cm

3 Calculate the surface area of a closed cylinder with a diameter of 3.4 m and a height
of 1.8 m. Give your answer correct to 1 decimal place.
Example

12

4 Calculate the surface area of an open cylinder with a radius of 4 cm and a height of
16 cm. Give your answer correct to the nearest whole number.
5 Calculate the surface area of each of the following open cylinders. Give each answer
correct to 1 decimal place.
a
b
c

30 cm

22 cm

13.3 cm
9.6 cm
r

20 cm

r = 4.1 cm

f
50 cm

3.2 m

WORKED

23.2 cm
4m

2.4 cm
4 cm

6 A can of fruit is made of stainless steel. The can has a radius of 3.5 cm and a height
of 7 cm. A label is to be wrapped around the can.
a Calculate the amount of steel needed to make the can (correct to the nearest whole
number).
b Calculate the area of the label (correct to the nearest whole number).

SkillS
HEET

WORKED

60

Maths Quest General Mathematics HSC Course

WORKED

Example

13

7 Calculate the surface area of a sphere with a radius of 3 cm. Give your answer correct
to the nearest whole number.
8 Calculate the surface area of each of the spheres drawn below. Give each answer
correct to 1 decimal place.
a
b
c
2.1 cm
14 cm

8 cm

e
1m

f
3.4 cm

1.8 m

9 Calculate the surface area of a sphere with a diameter of 42 cm. Give your answer
correct to the nearest whole number.
10 multiple choice
An open cylinder has a diameter of 12 cm and a height of 15 cm. Which of the
following calculations gives the correct surface area of the cylinder?
A
62 + 2
6 15
B 2
62 + 2
6 15
2
C
12 + 2
12 15
D2
122 + 2
12 15
11 multiple choice
Which of the following figures has the greatest surface area?
A A closed cylinder with a radius of 5 cm and a height of 10 cm
B An open cylinder with a radius of 6 cm and a height of 10 cm
C A cylinder open at both ends with a radius of 7 cm and a height of 10 cm
D A sphere with a radius of 6 cm
12 An open cylinder has a diameter and height of 12 cm.
a Calculate the surface area of the cylinder (correct to the nearest whole number).
b A sphere sits exactly inside this cylinder. Calculate the surface area of this sphere
(correct to the nearest whole number).
13 A cylindrical can is to contain three tennis balls each having a diameter of 6 cm.
a Calculate the surface area of each ball.
b The three balls fit exactly inside the can. State the radius and height of the can.
c The can is open and made of stainless steel, except the top which will be plastic.
Calculate the area of the plastic lid (correct to the nearest whole number).
d Calculate the amount of stainless steel in the can (correct to the nearest whole
number).
e Calculate the area of a paper label that is to be wrapped around the can (correct to
the nearest whole number).

Chapter 2 Further applications of area and volume

61

Computer Application 1 Minimising surface area


E

1. In cell B3 enter the volume of the cylinder, 1000.


2. In cell A6 enter a radius of 1. In cell A7 enter a radius of 2 and so on up to a radius
of 20.
3. The formula that has been entered in cell B6 will give the height of the cylinder
corresponding to the radius for the given volume.
4. The surface area of each possible cylinder is in column D. Use the charting function
on the spreadsheet to graph the surface area against the radius.
5. What are the most cost-efficient dimensions of the drink container?

Challenge exercise
Use one of the other worksheets to find the most efficient dimensions to make a
rectangular prism of volume 1000 cm3 and a cone of volume 200 cm3.

sheet

L Spre
XCE ad

Access the spreadsheet Volume from the Maths Quest General Mathematics HSC
Volume
Course CD-ROM.
A cylindrical drink container is to have a capacity of 1 litre (volume = 1000 cm3).
We are going to calculate the most cost efficient dimensions to make the container. To
do this, we want to make the container with as little material as possible, in other words
we want to minimise the surface area of the cylinder. The spreadsheet should look as
shown below.

62

Maths Quest General Mathematics HSC Course

Packaging
A company makes tennis balls that have a diameter of 6.5 cm. The tennis balls are
to be sold in packs of four.
1 Calculate the surface area of the
packaging needed if the balls are
packed in a cylindrical tube that just
fits all four balls as shown on the
right.

2 Calculate the amount of packaging


needed if the balls are packed in a
rectangular prism.

3 Calculate the amount of packaging needed if the balls


are packed in a 2 2 design as shown on the right.

4 Design the most effective way of packaging nine tennis balls.

Volume of composite solids


Many solid shapes are a composition of two or more regular solids. To calculate the
volume of such a figure, we need to determine the best method for each particular part.
Many irregular shapes may still be prisms.
A prism is a shape in which every cross-section taken parallel to the base shape is
equal to that base shape.
The formula for the volume of a prism is:
V = Ah
where A is the area of the base shape and h is the height.
Remember that the base of the prism is not necessarily the bottom. The base is the
shape that is constant throughout the prism and will usually be drawn as the front of the
prism. This means that the height will be drawn perpendicular to the base. To calculate
the volume of any prism, we first calculate the area of the base and then multiply by the
height.

63

Chapter 2 Further applications of area and volume

WORKED Example 14
Find the volume of the figure
drawn on the right.

6 cm

12 cm

4 cm

10 cm

Divide the front face into two


rectangles.

4 cm
12 cm

WRITE

A1
A2

6 cm

THINK

3 cm

10 cm
2

Calculate the area of each.

Add the areas together to find the value


of A.
Write the formula.
Substitute A = 84 and h = 3.
Calculate.

4
5
6

A1 = 4 12
= 48 cm2
A = 48 + 36
= 84 cm2
V=A h
= 84 3
= 252 cm3

A2 = 6 6
= 36 cm2

If the shape is not a prism, you may need to divide it into two or more regular 3dimensional shapes. You could then calculate the volume by finding the volume of each
shape separately. You will need to use important volume formulas that appear on the
formula sheet:
Cone: V = 1--- r 2h
Cylinder: V = r 2h
Pyramid: V = 1--- Ah
Sphere: V = 4--- r 3
3

WORKED Example 15
Calculate the volume of the figure
drawn on the right, correct to 2 decimal
places.
2.4 cm
1.2 cm

THINK
1
2
3
4

The shape is a cylinder with a


hemisphere on top.
Write down the formula for the volume
of a cylinder.
Substitute r = 1.2 and h = 2.4.
Calculate the volume of the cylinder.

WRITE

V = r 2h
V=
(1.2)2 2.4
V = 10.857 cm3
Continued over page

64

Maths Quest General Mathematics HSC Course

THINK

WRITE
r3 2

Write down the formula for the volume


of a hemisphere. (This is the formula
for the volume of a sphere divided by
2.)

V=

4
--3

Substitute r = 1.2.

V=

4
--3

Calculate the volume of the


hemisphere.

V = 3.619 cm3

Add the two volumes together.

Volume = 10.857 + 3.619


Volume = 14.48 cm3

(1.2)3 2

In many cases a volume question may be presented in the form of a practical problem.

WORKED Example 16
A water storage tank is in the shape of a cube of side length 1.8 m, surmounted by a
cylinder of diameter 1 m with a height of 0.5 m. Calculate the capacity of the tank, correct
to the nearest 100 litres.
THINK
1

WRITE

Draw a diagram of the water tank.

0.5 m
1m

1.8 m
2

Calculate the volume of the cube using


the formula V = s3.

V = s3
V = 1.83
V = 5.832 m3

Calculate the volume of the cylinder


using the formula V = r 2h.

V = r 2h
V=
0.52 0.5
V = 0.393 m3

Add the volumes together.

Volume = 5.832 + 0.393


Volume = 6.225 m3

Calculate the capacity of the tank using


1 m3 = 1000 L.

Capacity = 6.225 1000


Capacity = 6225 L

Give an answer in words.

The capacity of the tank is approximately


6200 litres.

65

Chapter 2 Further applications of area and volume

remember
1. To find the volume of any prism, use the formula V = A h, where A is the area
of the base and h is the height.
2. Important volume formulas:
Cone: V = 1--- p r 2h
Cylinder: V = p r 2h
3
1
Pyramid: V = --- Ah
Sphere: V = 4--- p r 3
3
3
where r = radius, h = perpendicular height, A = area of base
3. For other shapes, calculate the volume of each part of the shape separately, then
add together each part at the end.
4. Remember to begin a worded or problem question with a diagram and finish
with a word answer.

2E

Volume of composite solids


6 cm

2.6
Volume of
cubes and
rectangular
prisms

4 cm
20 cm

12 cm

5 cm

40 cm

0.7 m

2.3 m
0.4 m

0.6 m
1m

5m

1.5 m
2m

2.1 m

WORKED

Example

3 Consider the figure on the right.


The shape consists of a cube with a
square pyramid on top.
a What is the volume of the cube?
b What is the volume of the square
pyramid?
c What is the total volume of this
figure?

1.5 m

2.7

SkillS
HEET

15

3 cm

f
4 m 0.5 m

20 cm

12 cm
25 cm

15 cm

4 cm
20 cm

10 cm

2 Calculate the volume of each of the figures drawn below.


a
b
c
5 cm
12 cm

SkillS
HEET

14

1 Look at the figure drawn on


the right.
a Find the area of the front face.
b Use the formula V = A h
to calculate the volume of
the prism.

5 cm

Example

18 cm

WORKED

Volume of
triangular
prisms

2m

66

Maths Quest General Mathematics HSC Course

4 The figure on the right is a cylinder with a cone


mounted on top.
a Calculate the volume of the cylinder, correct to
the nearest cm3.
b Calculate the volume of the cone, correct to the
nearest cm3.
c What is the total volume of the figure?

40 cm

50 cm

SkillS

12 cm

HEET

2.8

5 Calculate the volume of each of the figures drawn below, correct to 1 decimal place.
a
b
c

Volume of
a cylinder

3 cm
34 cm
5 cm

SkillS

HEET

2.9
Volume of
a sphere

r =12 cm

50 cm

6 multiple choice
Which of the figures drawn below is not a prism?
A
B

7 multiple choice
The volume of the figure drawn
on the right is closest to:
A 718 cm3
B 1437 cm3
3
C 2155 cm
D 2873 cm3
8 A fish tank is in the shape of a
rectangular prism. The base measures
45 cm by 25 cm. The tank is filled to
a depth of 15 cm.
a Calculate the volume of water in
the tank in cm3.
b Given that 1 cm3 = 1 mL calculate,
in litres, the amount of water in
the tank.

14 cm
7 cm

67

Chapter 2 Further applications of area and volume

Example

16

9 A hemispherical wine glass of radius 2.5 cm is joined to a cylinder of radius 1 cm and


height 5 cm. The glass then rests on a solid base.
a Draw a diagram of the wine glass.
b Calculate the capacity of the glass, to the nearest 10 mL.
c How many glasses of wine can be poured from a 1 litre bottle?
10 The figure on the right is the cross-section of a concrete pipe
used as a sewage outlet.
a Calculate the area of a cross-section of the pipe, correct
to 2 decimal places.
b Calculate the amount of concrete needed to make a 10 m
length of this pipe.

3m
2.5 m

11 A commemorative cricket ball has a diameter of 7 cm. It is to be preserved in a cubic


case that will allow 5 mm on each side of the ball.
a What will the side length of the cubic case be?
b Calculate the amount of empty space inside the case, to the nearest whole number.
c Calculate the percentage of space inside the case occupied by the ball, to the
nearest whole number.

6 mm

Maximising volume
You have been given a piece of sheet metal that is in the shape of
a square with a side length of 2 m. The corners are to be cut and
9 cm
the sides bent upwards to form a rectangular prism, as shown in
the figure on the right.
3 cm
1 If a square of side length 1 cm is cut from each corner, what
will be the length and width of the rectangular prism?
2 What will be the volume of this rectangular prism?
3 What will be the volume of the prism if a square of side length 2 cm is cut from
each corner?
4 Find the size of the square to be cut from each corner that will make a prism of
maximum volume.
This exercise can be modelled using a spreadsheet or a graphics calculator.

2.10 SkillS
HEET

12 A diamond is cut into the shape of two square-based pyramids as


shown on the right. Each mm3 of the diamond has a mass of 0.04 g.
Calculate the mass of the diamond.

6 mm

WORKED

Volume
of a
pyramid

Maths Quest General Mathematics HSC Course

2
1 Calculate the area of a circle
with a diameter of 8.6 cm,
correct to 1 decimal place.

2 Calculate the area of the annulus shown


below, correct to 2 decimal places.
9 cm
3 cm

4 Calculate the area of the figure below.


10 cm

9 cm

29 cm

3 Calculate the area of the sector below,


correct to 1 decimal place.

13.2 cm
28 cm

85

5 Calculate the shaded area in the


figure drawn below, correct to
2 decimal places.

6 Use Simpsons rule to approximate


the area shown below.
43 m

70 m
21 m
4.6 cm
70 m
9.7 cm
32 m

7 Calculate the surface area of a closed


cylinder with a radius of 10 cm and a
height of 23 cm. Give your answer
correct to the nearest whole number.
9 Calculate the volume of the prism
drawn below.

8 Calculate the surface area of a sphere


with a radius of 1.3 m. Give your
answer correct to 3 decimal places.
10 Calculate the volume of the solid
below, correct to the nearest whole
number.
4 cm

20.3 cm

68

13.4 cm

9.1 cm
13.7 cm

8 cm

Chapter 2 Further applications of area and volume

69

Error in measurement
As we saw in the preliminary course, all measurements are approximations. The degree
of accuracy in any measurement is restricted by the accuracy of the measuring device
and the degree of practicality.
We have previously seen that the maximum error in any measurement is half of the
smallest unit of measurement. This error is compounded when further calculations such
as surface area or volume are made.

In the rectangular prism on the right, the length,


breadth and height have been measured, correct to
the nearest centimetre.
a Calculate the volume of the rectangular prism.
b Calculate the greatest possible error in the volume.

8 cm

WORKED Example 17

15 cm
20 cm

THINK

WRITE

a Calculate the volume of the rectangular


prism.

a V=l w h
= 20 15 8
= 2400 cm3

b Smallest possible dimensions:


l = 19.5, w = 14.5, h = 7.5
V=l w h
= 19.5 14.5 7.5
= 2120.625 cm3
Largest possible dimensions:
l = 20.5, w = 15.5, h = 8.5
V=l w h
= 20.5 15.5 8.5
= 2700.875 cm3
Maximum error = 2700.875 2400
Maximum error = 300.875 cm3

Write the smallest possible


dimensions of the prism.
Calculate the smallest possible
volume.
Write the largest possible
dimensions of the prism.
Calculate the largest possible
volume.
Calculate the maximum error.

As can be seen in the above example, a


possible error of 0.5 cm in the linear
measurement compounds to an error
of 300.875 cm3 in the volume
measurement.
Mismeasurements that are
made will compound all further
calculations.

70

Maths Quest General Mathematics HSC Course

WORKED Example 18
A swimming pool is built in the shape of a rectangular prism with a length of 10.2 m,
a width of 7.5 m and a depth of 1.5 m. The floor and the sides of the pool need to be
cemented.
a Calculate the area that is to be cemented.
b The concreter mismeasured the length of the pool as 9.4 m. Calculate the error
in the area calculation.
c Calculate the percentage error (correct to 1 decimal place) in the area
calculation.
THINK

WRITE

a Area of floor = 10.2 7.5


Area of floor = 76.5 m2
Area of ends = 7.5 1.5
Area of ends = 11.25 m2
Area of sides = 10.2 1.5
Area of sides = 15.3 m2
Total area = 76.5 + 2 11.25 + 2
Total area = 129.6 m2

Calculate the area of the pool floor.

Calculate the area of the ends.

Calculate the area of the sides.

Calculate the total area to be


cemented.

Use the incorrect measurement to


repeat all the above calculations.

Find the difference between the two


answers.

c Write the error as a percentage of the


correct answer.

b Area of floor = 9.4 7.5


Area of floor = 70.5 m2
Area of ends = 7.5 1.5
Area of ends = 11.25 m2
Area of sides = 9.4 1.5
Area of sides = 14.1 m2
Total area = 70.5 + 2 11.25 + 2
Total area = 121.2 m2
Error = 129.6 121.2
Error = 8.4 m2
8.4
c Percentage error = ------------129.6
Percentage error = 6.5%

15.3

14.1

100%

remember
1. All measurements are approximations. The accuracy of any measurement
is limited by the instrument used and the most practical degree of
accuracy.
2. The maximum error in any linear measurement is half the smallest unit
used.
3. Any error made in linear measurement will compound when used in further
calculations such as those for surface area or volume.

Chapter 2 Further applications of area and volume

WORKED

Example

1 In the figure on the right each measurement has


been taken to the nearest centimetre.
a Calculate the volume of the figure.
b Calculate the maximum error in the volume
calculation.

2.11 SkillS
HEET

17

Error in measurement
12 cm

2F

71

6 cm
16 cm

Error in
linear
measurement

2 The radius of a circle is measured as 7.6 cm, correct to 1 decimal place.


a What is the maximum possible error in the measurement of the radius?
b Calculate the area of the circle. Give your answer correct to 1 decimal place.
c Calculate the maximum possible error in the area of the circle.
d Calculate the maximum possible error in the area of the circle as a percentage of
the area.
3 A cube has a side length of 16 mm, correct to the nearest millimetre.
a Calculate the volume of the cube.
b Calculate the smallest possible volume of the cube.
c Calculate the largest possible volume of the cube.
d Calculate the maximum possible percentage error in the volume of the cube.
e Calculate the surface area of the cube.
f Calculate the smallest possible surface area of the cube.
g Calculate the largest possible surface area of the cube.
h Calculate the maximum possible percentage error in the surface area of the cube.
4 A cylinder has a radius of 4 cm and a height of 6 cm with each measurement being
taken correct to the nearest centimetre.
a Calculate the volume of the cylinder (correct to the nearest whole number).
b Calculate the smallest possible volume of the cylinder (correct to the nearest whole
number).
c Calculate the largest possible volume of the cylinder (correct to the nearest whole
number).
d Calculate the greatest possible percentage error in the volume of the cylinder.
5 For the cylinder in question 4, calculate the greatest possible percentage error in the
surface area of the cylinder.
6 The radius of a sphere is 1.4 m with the measurement taken correct to 1 decimal
place.
a Calculate the volume of the sphere, correct to 1 decimal place.
b Calculate the maximum possible error in the volume of the sphere.
c Calculate the maximum percentage error in the volume.
d Calculate the surface area of the sphere, correct to 1 decimal place.
e Calculate the maximum possible error in the surface area of the sphere.
f Calculate the maximum percentage error in the surface area.

72

Maths Quest General Mathematics HSC Course

WORKED

Example

18

7 An open cylindrical water tank has a radius of 45 cm and a height of 60 cm.


a Calculate the capacity of the tank, in litres (correct to the nearest whole number).
b If the tanks radius is given as 50 cm, correct to the nearest 10 cm, calculate the
error in the capacity of the tank.
c Calculate the percentage error in the capacity of the tank.
8 A rectangular prism has dimensions 56 cm 41 cm 17 cm.
a Calculate the volume of the prism.
b Calculate the surface area of the prism.
c If the dimensions are given to the nearest 10 cm, what will the dimensions of the
prism be given as?
d Calculate the percentage error in the volume when the dimensions are given to the
nearest 10 cm.
e Calculate the percentage error in the surface area when the dimensions are given to
the nearest 10 cm.

Work

9 The four walls of a room are to be painted. The length of the room is 4.1 m and the
width is 3.6 m. Each wall is 1.8 m high.
a Calculate the area to be painted.
b One litre of paint will paint an area of 2 m2. Each wall will need two coats of paint.
Calculate the number of litres of paint required to complete this job.
c Karla incorrectly measures the length of the room to be 3.9 m. If Karla does all her
calculations using this incorrect measurement, how many litres will she be short of
paint at the end of the job?

T
SHEE

2.2

10 The dimensions of a rectangular house are 16.6 m by 9.8 m.


a Simon takes the dimensions of the house to the nearest metre for all his calculations. What dimensions does Simon use?
b Simon plans to floor the house in slate tiles. What is the area that needs to be tiled?
c The tiles cost $27.50/m2 and Simon buys an extra 10% to allow for cutting and
breakage. Calculate the cost of the tiles.
d How much extra has Simon spent than would have been necessary had he used the
original measurements of the house?

Chapter 2 Further applications of area and volume

73

summary
Area of parts of the circle
The area of a circle can be calculated using the formula A = r 2.
The area of a sector is found by multiplying the area of the full circle by the
fraction of the circle occupied by the sector. This is calculated by looking at the
angle that the sector makes with the centre.
An annulus is the area between two circles. The area is calculated by subtracting
the area of the smaller circle from the area of the larger circle or by using the
formula A = (R2 r2) , where R is the radius of the large circle and r is the
radius of the small circle.
The area of an ellipse is calculated using the formula A = ab, where a is the length
of the semi-major axis and b is the length of the semi-minor axis.

Area of composite figures


The area of a composite figure is calculated by dividing the figure into two or more
regular figures.
When calculating the area of a composite figure, some side lengths will need to be
calculated using Pythagoras theorem.

Simpsons rule
Simpsons rule is used to find an approximation for an irregular area.
The formula for Simpsons rule is A

h
--- ( d f + 4d m + d l )
3

To obtain a better approximation for an area, Simpsons rule can be applied twice.
This is done by dividing the area in half and applying Simpsons rule separately to
each half.

Surface area of cylinders and spheres


The surface area of a closed cylinder is found by using the formula
SA = 2 r 2 + 2 rh .
If the cylinder is an open cylinder, the surface area is found using
SA = r 2 + 2 rh.
The surface area of a sphere is calculated using the formula
SA = 4 r 2 .

Volume of composite solids


The volume of solid prisms is calculated using the formula V = A
The volume of a cone is found using the formula V =

1
--3

r 2h

The volume of a cylinder is found using the formula V = r 2h

.
.

h.

74

Maths Quest General Mathematics HSC Course

The volume of a sphere is found using the formula V =

r3

4
--3

The volume of a pyramid is found using the formula V = 1--- Ah


3

.
.

Other solids have their volume calculated by dividing the solid into regular solid
shapes.

Error in measurement
All measurements are approximations. The maximum error in any measurement is
half the smallest unit used.
Any error in a measurement will compound when further calculations using the
measurement need to be made.

Chapter 2 Further applications of area and volume

75

CHAPTER
review
1 Calculate the area of each of the circles below. Give each answer correct to 1 decimal place.
a
b
c
3.7 cm

52 mm

1.7 m

2 Calculate the area of each of the figures below. Give each answer correct to 1 decimal place.
a
b
c

92 mm

2A

2A

237
12.5 cm

30
4.8 m

3 Calculate the area of each of the annuluses below. Give each answer correct to 1 decimal
place.
a
b
c
34 cm

3.7 m

81 mm

17 cm

1.3 m

94 mm

4 Calculate the area of each of the ellipses below, correct to 1 decimal place.
a
b
c
9.2 m

30 mm
45 mm

2A

2A
3.6 cm

11.4 m
7 cm

5 Calculate the area of the figure below.

15 cm

35 cm

10 cm
12 cm

10 cm

2B

76
2B

Maths Quest General Mathematics HSC Course

6 Calculate the area of each of the figures below. Where appropriate, give your answer correct
to 2 decimal places.
a 0.7 m
b
c
4.1 m

1.5 cm

7 Use Simpsons rule to approximate


the area on the right.

13 m
42 m
42 m

21 m

8 Use Simpsons rule to find an approximation for each of the areas below.
a
b
c

31 m

57 m

36 m
14 m

30 m

2m

62 m

96 m
24 m

42 m

50 m
30 m

30 m

9m
15 m 15 m 15 m 15 m

2D

23 m

38 m

10 Use Simpsons rule twice to find an


approximation for the area on the
right.

44 m

2C

33 m

62 m

25 m

19 m 11 m

9 By dividing the area shown on the right into two


sections, use Simpsons rule to find an
approximation for the area.

27 m

2C

57 m

2C

36 cm
6 cm

29 m

2C

1.5 cm

50 m

3.9 m

0.9 m

3 cm

11 Calculate the surface area of each of the closed cylinders drawn below, correct to 1 decimal
place.
a
b
c
60 cm

25 cm

10 cm

1.1 m

7 cm
4 cm

Chapter 2 Further applications of area and volume

77

12 Calculate the surface area of an open cylinder with a diameter of 9 cm and a height of
15 cm. Give your answer correct to the nearest whole number.

2D

13 Calculate the surface area of a sphere with:


a a radius of 5 cm
b a radius of 2.4 m
Give each answer correct to the nearest whole number.

2D

c a diameter of 156 mm.

0.5 m

2E
1.9 m

3.1 m

14 Calculate the volume of the solid drawn


on the right.

0.6 m
2.7 m

15 Calculate the volume of each of the solids drawn below. Where necessary, give your answer
correct to the nearest whole number.
a
b
c
12 cm

2E

22 cm
17 cm

9 cm
15 cm

20 cm

12 cm

19 cm

3 cm

10 cm

3 cm
3 cm
10 cm

40 cm

16 Calculate the volume of the figure drawn on the


right, correct to 2 decimal places.

2E
15 cm
9 cm

17 A sphere has a diameter of 16 cm when measured to the nearest centimetre.


a State the maximum error made in the measurement of the radius.
b Calculate the volume of the sphere. Answer correct to the nearest whole number.
c Calculate the maximum percentage error in the volume of the sphere.

2F

18 An aluminium soft drink can has a diameter of 8 cm and a height of 10 cm.


a Calculate the capacity of the
can, in millilitres, correct to the
nearest 10 millilitres.
b The machine that cuts the
aluminium for the can is
mistakenly set to 12 cm.
Calculate the percentage error
in the capacity of the can
(correct to the nearest whole
number).

2F

78

Maths Quest General Mathematics HSC Course

Practice examination questions


1 multiple choice
6.2 mm

Which of the following calculations will correctly


give the area of the ellipse drawn on the right?
A
6.22
B
8.52
C
10.82
D
10.8 6.2

10.8 mm

20 m

25 m

15 m

30 m

The field drawn on the right is to have its area


approximated by two applications of Simpsons
rule. The value of h is:
A 16
B 20
C 40
D 80

10 m

2 multiple choice

80 m

3 multiple choice
The figure drawn on the right is an open cylinder.
Which of the calculations below will correctly
give the surface area of the cylinder?
A
52 + 2
5 20
2
B 2
5 +2
5 20
C
102 + 2
10 20
D2
102 + 2
10 20

20 cm
10 cm

4 multiple choice
A closed cylinder is measured as having a radius of 1.2 m and a height of 1.4 m. The
maximum error in the calculation of the surface area is:
A 1.2 m2
B 1.5 m2
C 1.6 m2
D 19.6 m2
5 The figure on the right shows a section of a
concrete drainage pipe.
a Calculate the area of the annulus, correct to 1 decimal
2.5 m
place.
b Calculate the volume of concrete needed to make a
1.5 m
5 m length of this pipe (correct to 1 decimal place).
c Calculate the volume of water that will flow through the
5 m length of the pipe (in litres, to the nearest 100 L).
d Calculate the surface area of a 5 m section of pipe (correct to the nearest m2).
(Hint: Include the area of the inside of the pipe.)

CHAPTER

4.9 m

5.1 m

60 m
9.2 m

test
yourself

6 The diagram on the right shows the cross-section of a river.


a Use two applications of Simpsons rule to find the
approximate area of the rivers cross-section.
b If the river flows with this cross-section for
approximately 800 m, calculate the volume of the river.
c The length of the river has been approximated to the
nearest 100 m. Calculate the maximum percentage error
in calculating this volume.

Applications of
trigonometry

3
syllabus reference
Measurement 6
Applications of
trigonometry

In this chapter
3A Review of right-angled
triangles
3B Bearings
3C Using the sine rule to find
side lengths
3D Using the sine rule to find
angles
3E Area of a triangle
3F Using the cosine rule to
find side lengths
3G Using the cosine rule to
find angles
3H Radial surveys

areyou

READY?

Are you ready?

Try the questions below. If you have difficulty with any of them, extra help can be
obtained by completing the matching SkillSHEET. Either click on the SkillSHEET icon
next to the question on the Maths Quest HSC Course CD-ROM or ask your teacher for
a copy.

3.1

Right-angled trigonometry finding a side length

1 In each of the following find the length of the side marked with the pronumerals correct to two
decimal places.
d
a
b
c
f
40

71

16 m

19.5 m

25.2 km
63

3.2

Using the inverse trigonometric ratios

2 Find angle , where


a sin = 0.7

3.4

is acute, correct to the nearest degree.


b tan = 1.5

c cos

= 0.8

Right-angled trigonometry finding an angle

3 In each of the following find the size of the angle marked with the pronumerals correct to the
nearest degree.
8m
a
b
c
46.1 mm

20 cm

25 cm

16 m

31.2 mm

3.5

Converting nautical miles to kilometres

4 Use 1 nautical mile = 1.852 km to convert:


a 4 nautical miles to kilometres.
c 1.2 nautical miles to metres.

3.6

b 50 kilometres to nautical miles.


d 3560 metres to nautical miles.

Angle sum of a triangle

5 Find the angle marked with the pronumeral in each of the following.
a
b
c
132
41
58

3.7

63

71

Solving fractional equations

6 Solve each of the following equations, where appropriate give your answer correct to 2 decimal
places.
x
x 3
x
9.5
9 2
a --- = 3
b --- = --c ------- = ------d --- = --5
4 8
3.6 2.4
x 5

Chapter 3 Applications of trigonometry

81

Review of right-angled triangles


Previously we have studied right-angled triangles and discovered that we can calculate
a side length of a triangle when given the length of one other side and one of the acute
angles.
To do this we need to use the formulas for the three trigonometric ratios.
sin
cos
tan

opposite side
= -------------------------------hypotenuse
adjacent side
= --------------------------------hypotenuse
opposite side
= --------------------------------adjacent side

WORKED Example 1

Find the length of the side marked x


in the figure on the right (correct
to 1 decimal place).

42
x

29.2 cm

THINK
1

WRITE

Label the sides of the diagram.


42
adj

hyp
x

29.2 cm
opp

Choose the sine ratio and write the


formula.
Substitute for the opposite side and
hypotenuse.
Make x the subject of the formula.

Calculate the value of x.

2
3

opposite side
= ------------------------------hypotenuse
29.2
sin 42 = ---------x
x sin 42 = 29.2
29.2
x = ----------------sin 42
x = 43.6 m
sin

82

Maths Quest General Mathematics HSC Course

equation solver to find


Graphics Calculator tip! Using
side lengths
Consider worked example 1. Once you have chosen the correct trigonometric ratio and
substituted, you can finish the solution using the equation solver on your graphics
calculator.
1. From the MENU select EQUA.

2. Press F3 (SOLV).

3. Delete any equation, enter the equation


sin 42 = 29.2 X and press EXE .
Note: Your calculator may display a different value
of X at this stage. This is just the last value of X
stored in the calculators memory.
4. Press F6 (SOLV) to solve the equation.

The same formulas can be used to calculate the size of an angle if we are given two
side lengths in the triangle.

WORKED Example 2

Calculate the size of the angle marked in the


figure on the right (correct to the nearest degree).

47 mm
35 mm

THINK
1 Label the sides of the triangle.

Choose the tangent ratio and write the


formula.
Substitute for the opposite side and the
adjacent side.
Make the subject of the formula.

Calculate .

2
3

WRITE
Opposite = 47 mm
Adjacent = 35 mm
opposite side
tan = ------------------------------adjacent side
47
tan = -----35
47
= tan 1 -----35
= 53

83

Chapter 3 Applications of trigonometry

equation solver to find


Graphics Calculator tip! Using
the size of an angle
We can use the equation solver when we are finding the size of an angle.
1. From the MENU select EQUA.

2. Press F3 (SOLV).

3. Delete any existing equation, then enter the equation


tan X = 47 ab/c 35 and press EXE .
Note: Your calculator may display a different value
of X at this stage. This is just the last value of X
stored in the calculators memory.
4. Press F6 (SOLV) to solve the equation.

Using these results, we are able to solve problems that involve more than one rightangled triangle.

WORKED Example 3
Greg stands 70 m from the base of a building and measures the
angle of elevation to the top of the building as being 35. Julie is
standing 40 m from the base of the building on the other side of
the building as shown in the figure on the right.
a Calculate the height of the building, correct to 2 decimal places.
b Calculate the angle of elevation of the top of the building that
Julie would measure, correct to the nearest degree.
THINK
a

Draw the triangle showing the angle


of elevation from where Greg is
standing and label the sides.

h
35
40 m

70 m

WRITE
a
h

35
70 m
Continued over page

84

Maths Quest General Mathematics HSC Course

WRITE

Choose the tangent ratio and write


the formula.

Substitute for and the adjacent


side.
Make h the subject of the formula.

Calculate the value of h.

Draw the triangle from where Julie


is standing and label the sides.

tan

opposite side
= ------------------------------adjacent side

h
tan 35 = -----70
h = 70

tan 35

h = 49.01 m
b
49.01 m

THINK

40 m
2

Choose the tangent ratio and write


the formula.

tan

opposite side
= ------------------------------adjacent side

Substitute for the opposite side and


the adjacent side.

tan

49.01
= ------------40

Make

Calculate , correct to the nearest


degree.

the subject of the formula.

= tan

49.01
------------40

= 51

remember
1. The formulas for the three trigonometric ratios are:
opposite side
sin = ------------------------------hypotenuse
adjacent side
cos = ------------------------------hypotenuse
opposite side
tan = ------------------------------adjacent side
2. To calculate the length of a side we need to be given one side length and one
acute angle.
3. To calculate the size of an angle we need to be given two side lengths.
4. Many problems involve solving two or more right-angled triangles.
5. After substitution, the value of the unknown can be found using the equation
solver on a graphics calculator.

85

Chapter 3 Applications of trigonometry

3A

Review of right-angled
triangles

1 Calculate the length of the side marked with the pronumerals in each of the following, 3.1
correct to 1 decimal place.
1
a
b
c
Right-angled
b

WORKED

Example

38

142 mm

23
a

trigonometry
finding a
side length

61

Cabri Geo

314 mm

ry
met

11.4 m

HEET

13.2 cm

SkillS

Sine,
cosine and
tangent

f
17
5
e

d
50

19.2 cm

9.1 m

2 Calculate the size of each of the angles marked with the pronumerals, correct to the
nearest degree.
2
a
b
c
113 cm
9.5 m

WORKED

cm

61

71 mm

11.4 m

3.2

SkillS
HEET

36 mm

Example

Using the
inverse
trigonometric
ratios

3 From the top of a cliff the angle of depression


to a boat sailing 100 m offshore is 32. Calculate
the height of the cliff, correct to the nearest metre.

100 m
32
h

SkillS

Rounding
angles
to the
nearest
degree

3.4

SkillS
HEET

5 A lighthouse is 40 m tall and the beacon atop the


lighthouse is sighted by a ship 150 m from shore, as
shown in the figure on the right. Calculate the angle 40 m
of elevation at which the lighthouse is sighted,
correct to the nearest degree.

3.3

HEET

4 Andrew walks 5 km from point P to point Q. At the same time Bianca walks from
P to R such that PQ is perpendicular to PR. Given that PQR = 28:
a draw a diagram of DPQR
b calculate the distance walked by Bianca, correct to the nearest metre
c calculate the distance that Andrew would need to walk in a straight line to
meet Bianca, correct to the nearest metre.

150 m

Right-angled
trigonometry
finding an angle

86

Maths Quest General Mathematics HSC Course

6 From a point 65 m above the ground, a point is sighted on the ground at a distance of
239 m.
a Draw a diagram of this situation.
b Calculate the angle of depression at which the point is sighted.
7 Sally and Tim are both sighting the top of a
building, as shown in the figure on the right.
3
Sally is 40 m from the base of the building
and sights the angle of elevation to the top of
the building as 35. Tim is 60 m from the
base of the building.
a Calculate the height of the building, correct to 2 decimal places.
b Calculate the angle of elevation at which
Tim will sight the building.

WORKED

Example

8 George and Diego are both flying a kite


from the same point. Georges kite is flying
on 50 m of string and the string makes a 70
angle with the ground. Diegos kite is flying
on a 60 m piece of string and is at the same
height as Georges kite, as shown in the
figure on the right. Calculate the angle that
the string from Diegos kite makes with the
ground. Give your answer correct to the
nearest degree.

35
40 m

60 m

50 m

60 m

70

Bearings
A bearing is an angle used to describe direction. Bearings are used in navigation and
are a common application of trigonometry to practical situations. We can therefore
apply our trigonometrical formulas to make calculations based upon these bearings.
There are two types of bearing that we need to be able to work with: compass bearings
and true bearings.

Compass bearings
Compass bearings use the four points of the
compass. With compass bearings there are four
main directions: north, south, east and west. In
between each of these main directions there are
four others: north-east, south-east, south-west and
north-west. Each of these directions is at 45 to
two of the four main directions.
Trigonometry can then be used to solve
problems about distances and angles using these
eight basic directions.

N
NW

NE

SW

SE
S

Chapter 3 Applications of trigonometry

87

WORKED Example 4

A ship (A) is 10 nautical miles due east of a lighthouse. A second ship (B) bears SE of the
lighthouse and is due south of the first ship. Calculate the distance of the second ship from
the lighthouse, correct to 1 decimal place.
THINK
1

WRITE

Draw a diagram labelling the sides of


the triangle.

adj
10 M

N
L

45

opp

hyp
x
B
2

Choose the cosine ratio and write the


formula.
Substitute for and the adjacent
side.
Make x the subject of the equation.

Calculate the value of x, correct to


1 decimal place.
Give a written answer.

adj
= --------hyp
10
cos 45 = -----x
x cos 45 = 10
10
x = -----------------cos 45
= 14.1 M
cos

The second ship is 14.1 nautical miles from the


lighthouse.

These eight compass points do not allow us to make calculations about more precise
directions. For this reason an alternative method of describing bearings is needed for
any direction other than these basic eight points.

True bearings
A true bearing is an angle measured from north in a
clockwise direction. As there are 360 in a revolution,
all true bearings are represented as a three-digit number
between 000 and 360. For example, east is at a
bearing of 090, south has a bearing of 180 and west
270.
When given information about a bearing, we can
solve problems using trigonometry by constructing a
right-angled triangle. As most questions involving bearings are in problem form, a diagram is necessary to
solve the problem and an answer in words should be
given.

270

090

180

88

Maths Quest General Mathematics HSC Course

WORKED Example 5
A ship sails on a bearing of 130 for a distance of 10 nautical miles. Calculate how far
south of its starting point the ship is, correct to 2 decimal places.
THINK
1

WRITE

Draw a diagram completing a


right-angled triangle and label the
sides.

N
130
50

hyp
10 M

adj
x

opp

Choose the cosine ratio and write the


formula.
Substitute for and the hypotenuse.

Make x the subject of the equation.

Calculate.
Give a written answer.

adj
= --------hyp
x
cos 50 = -----10
x = 10 cos 50
x = 6.43 M
The ship is 6.43 nautical miles south of its
starting point.
cos

We can also use our methods of calculating angles to make calculations about bearings.
After solving the right-angled triangle, however, we need to provide the answer as a
bearing.

WORKED Example 6
On a hike Lisa walked south for 3.5 km and then turned west for 1.2 km. Calculate Lisas
bearing from her starting point.

WRITE

Draw a diagram and label the sides of


the triangle.

hyp

3.5 km
adj

THINK

1.2 km
opp
2

Choose the tangent ratio and write the


formula.

tan

opp
= --------adj

Chapter 3 Applications of trigonometry

THINK

89

WRITE

Substitute for the opposite and adjacent


sides and simplify.

Make

Calculate .

From the diagram we can see the angle


lies between south and west. South has
a bearing of 180, and so we must add
19 to 180 to calculate the true
bearing.

Bearing = 180 + 19
= 199

Give a written answer.

Lisa is at a bearing of 199 from her starting


point.

tan

1.2
= ------3.5
= 0.3429
= tan 1(0.3429)

the subject of the equation.

= 19

remember
1. Bearings are used to describe a direction. We have used two types of bearings.
Compass bearings use the four main points of the compass, north, south, east
and west, as well as the four middle directions, north-east, north-west, southeast and south-west.
True bearings describe more specific direction by using a three-digit angle,
which is measured from north in a clockwise direction.
2. Bearing questions are usually given in written form so you will need to draw a
diagram to extract all the information from the question.
3. Read carefully to see if the question is asking you to find a side or an angle.
4. Always give a written answer to worded questions.
5. Use 1 M = 1.852 km to convert beteen nautical miles and kilometres.

3B
WORKED

Example

1 A road runs due north. A hiker leaves the road and walks for 4.2 km in a NW
direction.
a Draw a diagram of this situation.
b How far due east must the hiker walk to get back to the road? (Give your
answer correct to 3 decimal places.)
2 A driver heads due south for 34 km, then turns left and drives until he is SE of
his starting point.
a Draw a diagram to show the drivers journey.
b Calculate the distance the driver travelled in an easterly direction from his
starting point.

3.5

SkillS

HEET

Bearings

Converting
nautical
miles to
kilometres

90

Maths Quest General Mathematics HSC Course

3 Two boats, A and B, sail from a port. A heads due west, while B heads NW for a
distance of 43 nautical miles, where it drops anchor. Boat A drops anchor due south of
boat B.
a Draw a diagram showing the positions of boats A and B.
b Calculate the distance between boats A and B in nautical miles, correct to
1 decimal place.
c Calculate the distance in kilometres between A and B.
4 multiple choice
A true bearing of 315 is equivalent to a
compass bearing of:
A NE
B NW
C SE
D SW
5 multiple choice
A compass bearing of SE is equivalent to a true bearing of:
A 045
B 135
C 225
Example

6 Two hikers, Adrian and Bertrand, set out on a


walk. Adrian walks 5 km due north to a point,
A, and Bertrand walks on a bearing of 052 to
a point, B. Bertrand lets off a flare and Adrian
notices Bertrand is now due east of him, as
shown in the diagram on the right. Calculate
the distance between the two hikers, correct to
1 decimal place.

5 km

WORKED

D 315

52

7 A yacht sights a lighthouse on a bearing of 060. After sailing another eight nautical
miles due north, the yacht is due west of the lighthouse.
a Draw a diagram of this situation.
b Calculate the distance from the yacht to the lighthouse when it is due west of it
(correct to 1 decimal place).
8 An aeroplane takes off from an airport and flies on a bearing of 220 for a distance of
570 km. Calculate how far south of the airport the aeroplane is (correct to the nearest
kilometre).
9 A camping ground is due east of a car park. Eden and Jeff walk 3.8 km due south
from the camping ground until the car park is on a bearing of 290.
a Draw a diagram showing the car park, the camping ground, and Eden and Jeffs
position.
b Calculate the distance Eden and Jeff need to walk directly back to the car park,
correct to 1 decimal place.
10 multiple choice
A ship is on a bearing of 070 from a lighthouse. The bearing of the lighthouse from
the ship will be:
A 070
B 160
C 200
D 250

91

Chapter 3 Applications of trigonometry

11 multiple choice
A camping ground is SW of a car park. The bearing of the car park from the camping
ground will be:
A NE
B NW
C SE
D SW
12 A search party leaves its base and head 4 km due west before turning south for 3.5 km.
a Draw a diagram of this situation.
6
b Calculate the true bearing of the search party from its base, correct to the nearest
degree.

WORKED

Example

13 A ship is two nautical miles due west of a harbour. A yacht that sails 6.5 nautical miles
from that harbour is due north of the ship. Calculate the true bearing (correct to the
nearest degree) of the course on which the yacht sails from the harbour.

Trigonometric ratios for


obtuse angles
Many non-right-angled triangles have one obtuse angle. In the following sections
we will be solving non-right-angled triangles and will need to investigate the
trigonometric ratios for obtuse angles.
1 Use your calculator to give each of the following, correct to 3 decimal places.
a sin 100
b cos 100
c tan 100
d sin 135
e cos 135
f tan 135
g sin 179
h cos 179
i tan 179
2 Which of the answers to question 1 are positive and which are negative?
3 Calculate the sine, cosine and tangent of several other obtuse angles and see if
the established pattern continues.
4 Can you develop a rule for the sign of trigonometric ratios of obtuse angles?

The sine rule


Finding side lengths
The trigonometry we have studied so far has been
applicable to only right-angled triangles. The sine rule
allows us to calculate the lengths of sides and the size of
angles in non-right-angled triangles. Consider the
triangle drawn on the right.

C
C
b

A
A

B
c

The sine rule states that in any triangle, ABC, the ratio of each side to the sine of its
opposite angle will be equal.
a
b
c
------------- = ------------- = ------------sin A
sin B
sin C

92

Maths Quest General Mathematics HSC Course

Derivation of the sine rule


A, B and C represent the three angles in the triangle ABC and a, b and c represent
the three sides, remembering that each side is named with the lower-case letter of
the opposite vertex.
Construct a line from C to a point, D, perpendicular to AB. CD is the
perpendicular height of the triangle, h.
C

a
h

D
c

Now consider ACD and BCD separately.


C

a
h

Use the formula for the sine ratio:


opp
opp
= --------sin = --------hyp
hyp
h
h
sin A = --sin B = --b
a
h = b sin A
h = a sin B
We are now able to equate these two expressions for h.
a sin B = b sin A
Dividing both sides by sin A sin B we get:
sin

a sin B
b sin A
--------------------------- = --------------------------sin A sin B sin A sin B
a
b
------------ = -----------sin A sin B
c
Similarly, we are able to show that each of these is also equal to ------------- . Try it!
sin C

This formula allows us to calculate the length of a side in any triangle if we are given
the length of one other side and two angles. When using the formula we need to use
only two parts of it.

Chapter 3 Applications of trigonometry

93

WORKED Example 7

Calculate the length of the side marked x in the


triangle on the right, correct to 1 decimal place.

A
80
16 cm
40
B

THINK

WRITE

Write the formula.

Substitute a = x, b = 16, A = 80 and


B = 40.

Make x the subject of the equation by


multiplying by sin 80.
Calculate.

a
b
------------ = -----------sin A sin B
x
16
----------------- = ----------------sin 80 sin 40
16 sin 80
x = ------------------------sin 40
x = 24.5 cm

equation solver to solve


Graphics Calculator tip! Using
sine rule problems (sides)
As with right-angled trigonometry, you can use the equation solver function on your
graphics calculator to solve the equation formed immediately after you substitute into
the equation. Consider worked example 7 above.
1. From the MENU select EQUA.

2. Press F3 (SOLV).

3. Delete any existing equation, enter the equation


X sin 80 = 16 sin 40, and then press EXE .
Note: Your calculator may display a different value
of X at this stage. This is just the last value of X
stored in the calculators memory.
4. Press F6 (SOLV) to solve the equation.

94

Maths Quest General Mathematics HSC Course

Note: Some questions may ask for you to give the answer in a form other than a
number and as such the graphics calculator method can not be used. For example, the
16 sin 80
question above could be worded to, say, show x = ------------------------ , in which case you must
sin 40
manipulate the equation to arrive at the desired expression.
To use the sine rule we need to know the angle opposite the side we are finding and
the angle opposite the side we are given. In some cases these are not the angles we are
given. In such cases we need to use the fact that the angles in a triangle add to 180 to
calculate the required angle.

WORKED Example 8

Calculate the length of the side labelled


m in the figure on the right, correct to
4 significant figures.

65
m

75
B

THINK
1 Calculate the size of angle C.
2

Write the formula.

3
4

Substitute a = 16, c = m, A = 65 and


C = 40.
Make m the subject of the equation.

Calculate.

16 m

WRITE
C = 180 65
= 40
a
c
------------ = ------------sin A sin C
16
m
----------------- = ----------------sin 65 sin 40
16 sin 40
m = ------------------------sin 65
= 11.35 m

75

As mentioned in the previous investigation, we need to apply the sine rule to obtuseangled triangles. In such examples the method used is exactly the same with the substitution of an obtuse angle.
Using the sine rule allows us to solve a number of more complex problems. As with
our earlier trigonometry problems, we begin each with a diagram and give a written
answer to each.

WORKED Example 9

Georg looks south and observes an aeroplane at an angle of elevation


of 60. Henrietta is 20 km south of where Georg is and she faces
north to see the aeroplane at an angle of elevation of 75.
Calculate the distance of the aeroplane from Henriettas
observation point, to the nearest metre.
60
G

THINK
1 Calculate the size of

GAH.

WRITE
A = 180
= 45

75
20 km

60

75

Chapter 3 Applications of trigonometry

THINK

95

WRITE

Write the formula.

Substitute g = x, a = 20, G = 60 and


H = 75.

4
5

Make x the subject.


Calculate.

Give a written answer.

g
a
------------- = -----------sin G sin A
x
20
----------------- = ----------------sin 60 sin 45
20 sin 60
x = ------------------------sin 45
x = 24.495 km
The distance of the aeroplane from Henriettas
observation point is 24.495 km.

remember
a
b
c
1. The sine rule formula is ------------ = ------------ = ------------- .
sin A
sin B
sin C
2. The sine rule is used to find a side in any triangle when we are given the length
of one other side and two angles.
3. We need to use only two parts of the sine rule formula.
4. For written problems, begin by drawing a diagram and finish by giving a
written answer.
5. You can use the equation solver on a graphics calculator to find the value of the
unknown after substituting into the formula.

Using the sine rule to find


side lengths

3C

1 Write down the sine rule formula as it applies to each of the triangles below.
a
b X
c P
A

WORKED

Example

2 Use the sine rule to calculate the length of the side marked with the pronumeral in
each of the following, correct to 3 significant figures.
a
b
c
L
A
R
x
50
B

16 cm

1.9 km

63

45
C
M

52

59

84
N

89 mm

3.7

SkillS

HEET

Angle
sum of a
triangle

SkillS

HEET

3.6

Solving
fractional
equations

96
Triangle

WORKED

Example

3 In each of the following, use the sine rule to calculate the length of the side marked
with the pronumeral, correct to 1 decimal place, by first finding the size of the third
angle.
a G
b
c
x
H
B
N
74 74
80
18.2 mm

19.4 km

Cabr

omet
i Ge ry

Maths Quest General Mathematics HSC Course

m
62 P

85

y
27

A
35.3 cm
I

4 multiple choice
42 cm
Look at the figure drawn on the right.
Which of the following expressions gives
28
35
m
the value of m?
42 sin 117
42 sin 117
A m = ---------------------------B m = ---------------------------sin 28
sin 35

42 sin 28
C m = ------------------------sin 117

42 sin 35
D m = ------------------------sin 117

5 multiple choice
Look at the figure drawn on the right.
Which of the following expressions gives
the value of n?

n
28

35
42 m

42 sin 117
A n = ---------------------------sin 28

42 sin 117
B n = ---------------------------sin 35

42 sin 28
C n = ------------------------sin 117

42 sin 35
D n = ------------------------sin 117

6 ABC is a triangle in which BC = 9 cm, BAC = 54 and ACB = 62. Calculate the
length of side AB, correct to 1 decimal place.
7 XYZ is a triangle in which y = 19.2 m, XYZ = 42 and XZY = 28. Calculate x,
correct to 3 significant figures.
WORKED

Example

8 X and Y are two trees, 30 m apart on one side of a river. Z is a tree on the opposite
side of the river, as shown in the diagram below.
Z

59
X

72
30 m

It is found that XYZ = 72 and YXZ = 59. Calculate the distance XZ, correct to
1 decimal place.

97

Chapter 3 Applications of trigonometry

9 From a point, M, the angle of elevation to the top of a building, B, is 34. From a
point, N, 20 m closer to the building, the angle of elevation is 49.
a Draw a diagram of this situation.
b Calculate the distance NB, correct to 1 decimal place.
c Calculate the height of the building, correct to the nearest metre.
10 Look at the figure on the right.
a Show that XY can be given by the
80 sin 30
expression ------------------------- .
sin 40
b Show that h can be found using the
80 sin 30 sin 70
expression -------------------------------------------- .
sin 40
c Calculate h, correct to 1 decimal place.

h
30
80 m

70
X

Finding angles
Using the sine rule result, we are able to calculate angle sizes as well. To do this, we
need to be given the length of two sides and the angle opposite one of them. For
simplicity, in solving the triangle we invert the sine rule formula when we are using it
to find an angle. The formula is written:
sin A
sin B
sin C
------------ = ------------ = ------------a
b
c
Your formula sheet has the sine rule to find a side length. You need to invert this
formula when finding an angle. As with finding side lengths, we use only two parts of
the formula.

WORKED Example 10
Find the size of the angle, , in the figure on the right,
correct to the nearest degree.

A
6 cm

110
C
20 cm

THINK
1

Write the formula.

Substitute A = 110, C = , a = 20 and


c = 6.

3
4

Make sin the subject of the equation.


Calculate a value for sin .

Calculate sin 1(0.2819) to find .

WRITE
sin A sin C
------------ = ------------a
c
sin 110 sin
-------------------- = -----------20
6
6 sin 110
sin = ------------------------20
sin = 0.2819
= 16

98

Maths Quest General Mathematics HSC Course

equation solver to solve


Graphics Calculator tip! Using
sine rule problems (angles)
The same graphics calculator method can be used when finding an angle using the sine
rule. Consider worked example 10.
1. From the MENU select EQUA.

2. Press F3 (SOLV).

3. Delete any existing equation, enter the equation


sin 110 20 = sin X 6 and press EXE .
Note: Your calculator may display a different value
of X at this stage. This is just the last value of X
stored in the calculators memory.
4. Press F6 (SOLV) to solve the equation.

Note: When using the graphics calculator, you do not need to remember to invert the
sine rule. If you enter 20 sin 110 = 6 sin x, the graphics calculator will still solve the
equation.
As with finding side lengths, some questions will be problems that require you to
draw a diagram to extract the required information and then write the answer.

WORKED Example 11

From a point, P, a ship (S) is sighted 12.4 km from P on a bearing of 137. A point, Q, is
due south of P and is a distance of 31.2 km from the ship. Calculate the bearing of the ship
from Q, correct to the nearest degree.
THINK
1

Draw a diagram.

WRITE
P

137
12.4 km
43 S

31.2 km

Chapter 3 Applications of trigonometry

THINK

99

WRITE

Write the formula.

Substitute for p, q and P.

Make sin Q the subject.

Calculate a value for sin Q.

sin Q sin P
------------- = -----------q
p
sin Q sin 43
------------- = ----------------12.4
31.2
12.4 sin 43
sin Q = -----------------------------31.2
sin Q = 0.271

Calculate sin (0.271) to find Q.

Give a written answer.

Q = 16
The bearing of the ship from Q is 016.

remember
sin A
sin B
sin C
1. The sine rule formula for finding an angle is ------------ = ------------ = ------------- .
a
b
c
2. The formula sheet gives the sine rule in the form used to find a side. You have
to invert the formula when finding angles.
3. We can use this formula when we are given two sides and the angle opposite
one of them.
4. Worded questions should begin with a diagram and finish with a written
answer.

Using the sine rule to find


angles

3D

1 Find the size of the angle marked with a pronumeral in each of the following, correct to
the nearest degree.
10
a
b
c L
P
A

WORKED

Example

32 cm

100

29.5 m
B

46 cm

153 mm
79 mm

C
R
Q 60
18.9 m

117

N
27 mm

23.6 km

75

23.6 km

16.5 cm

170

27.6 cm
86

156 mm

100

Maths Quest General Mathematics HSC Course

2 multiple choice

36

36 sin 13
= ------------------------7

C sin

13

Which of the statements below give the


correct value for sin ?
13 sin 36
A sin = ------------------------7

B sin

7 sin 36
= ---------------------13

D sin

7 sin 13
= ---------------------36

3 multiple choice
In which of the triangles below is the information insufficient to use the sine rule?
A

12.7 m

14.8 m

45

57
16.2 m

12.6 m

D
115

12.7 m

6.2 m

8.7 m

12.9 m

4 In PQR, q = 12 cm, r = 16 cm and


the nearest degree.

PRQ = 56. Find the size of

5 In KLM, LM = 4.2 m, KL = 5.6 m and


to the nearest degree.

KML = 27. Find the size of

PQR, correct to

LKM, correct

6 A, B and C are three towns marked on a map. Judy calculates that the distance between
A and B is 45 km and the distance between B and C is 32 km. CAB is 45. Calculate
11
ACB, correct to the nearest degree.

WORKED

Example

7 A surveyor marks three points X, Y and Z in the ground. The surveyor measures XY to
be 13.7 m and XZ to be 14.2 m. XYZ is 60.
a Calculate XZY to the nearest degree.
b Calculate YXZ to the nearest degree.

Work

8 Two wires support a flagpole. The


first wire is 8 m long and makes a
65 angle with the ground. The
second wire is 9 m long. Find the
angle that the second wire makes
with the ground.
T
SHEE

3.1

101

Chapter 3 Applications of trigonometry

1
1 Find a in the triangle below,
correct to 1 decimal place.

2 Find b in the triangle below,


correct to the nearest
millimetre.

23
a

346 mm
11.4 m

63
b

3 Find c in the triangle below,


correct to 3 significant figures.

4 Find in the triangle below,


correct to the nearest degree.

42 cm
12 m
37
c
7m

In questions 5 to 7 find the size of the side marked with a pronumeral, correct to
2 significant figures.
5

6
80

46 m

12

75

150
6.1 cm

23

30
1700 mm

In questions 8 to 10 find the size of the angle marked , correct to the nearest degree.
8

10
44 cm

65 cm
41 m

23
4.9 m

31
60
32 m

3.6 m

102

Maths Quest General Mathematics HSC Course

Area of a triangle
You should be familiar with finding the area of a triangle using
the formula Area = 1--2- bh . In this formula, b is the base of the
triangle and h is the perpendicular height. This formula cant
be used in triangles where we do not know the perpendicular
height. Trigonometry allows us to find the area of such
triangles when we are given the length of two sides and the
B
included angle.
Consider the triangle drawn on the right. In this triangle:
Area = 1--2- ah

b
h
C

D
a

[1]

(a = base of triangle, h = height)


Now consider ACD. Since this triangle is right angled:
opp
sin C = --------hyp
h
sin C = --b
h = b sin C
Substituting for h in [1]:
Area = 1--2- ab sin C
This becomes the formula for the area of a triangle. There are three equivalent formulas
for the area of a triangle.
Area = 1--2- ab sin C
Area = 1--2- ac sin B
Area = 1--2- bc sin A
The formula sheet gives the first version of this formula only. The others are an
adaptation of the same rule. These formulas allow us to find the area of any triangle
where we are given the length of two sides and the included angle. The included angle
is the angle between the two given sides. The formula chosen should be the one that
uses the angle you have been given.

WORKED Example 12

Find the area of the triangle on the right, correct to


2 decimal places.

12 cm

60

THINK
2

Write the formula that uses sin B.


Substitute a = 16, c = 12 and B = 60.

Calculate.

16 cm

WRITE
Area = 1--- ac sin B
Area =

2
1
--2

16

Area = 83.14 cm

12
2

sin 60

Chapter 3 Applications of trigonometry

103

As with all other trigonometry we can use this formula to solve practical problems.

WORKED Example 13
Two paths diverge at an angle of 72. The paths lengths are 45 m and 76 m respectively.
Calculate the area between the two paths, correct to the nearest square metre.
THINK
1

WRITE

Draw a diagram.
45 m
76 m

72
2

Write the formula.

Area = 1--- ab sin C

Substitute a = 45, b = 76 and C = 72.

Area =

Calculate.

Give a written answer.

Area = 1626 m2
The area between the paths is 1626 m2.

2
1
--2

45 76 sin 72

remember
1. The area of a triangle can be found when you are given the length of two sides
and an included angle.
2. The formulas to use are:

Area = 1--- ab sin C


2

Area = 1--- ac sin B


2

Area = 1--- bc sin A


2

3. Where possible you should still use Area = 1--- bh.


2

4. Begin worded problems with a diagram and finish them with a written answer.

3E

Area of a triangle

1 Write down the formula for the area of a triangle in terms of each of the triangles
drawn below. Write the formula using the boldfaced angle.
a
b X
c
B
A

104

Maths Quest General Mathematics HSC Course

2 For each of the triangles drawn below, state whether the area would be best found
using the formula Area = 1--- ab sin C or Area = 1--- bh.
2

b
6 cm
1.9 m

60
12 cm

2.6 m

d
6.2 m
9.1 m

8.3 m

60
12.4 m
Example

12

3 Find the area of each of the following triangles, correct to 1 decimal place.
a
b
c
11 cm
196 mm

207 mm
117 mm

40

120
92 mm

10

12 cm

4 Use either Area = 1--- ab sin C or Area = 1--- bh to find the area of each of the following
2
2
triangles. Where necessary, give your answer correct to 1 decimal place.
a
b
c
32 cm

38 cm

WORKED

19 cm
66
14 cm

38 cm

32 cm

5 multiple choice
In which of the following triangles can the formula Area = 1--- ab sin C not be used to
2
find the area of the triangle?
A
B
4 cm
4 cm
60
9 cm
9 cm

D
4 cm

75

9 cm
120
4 cm

9 cm

Chapter 3 Applications of trigonometry

105

6 multiple choice
The area of the triangle on the right
(correct to 1 decimal place) is:
A 4.4 cm2
B 14.7 cm2
C 17.1 cm2
D 20.5 cm2

5 cm

7 In PQR, p = 4.3 cm, q = 1.8 cm and


correct to 4 significant figures.

7 cm

78
6 cm

PRQ = 87. Calculate the area of PQR,

8 The figure on the right is of a parallelogram, ABCD.


a Copy the diagram into your workbook and draw
the diagonal AC on your diagram.
b By considering the parallelogram as two equal
triangles, calculate its area, correct to 1 decimal
place.

2.5 m
70
D

5.2 m

9 On the right is a diagram of a pentagon


inscribed in a circle of radius 5 cm.
a Calculate the size of each of the angles
made at the centre.
b Calculate the area of the pentagon,
correct to the nearest square centimetre.
10 A surveyor sights the four corners of a
block of land and makes the following
13
notebook entry. Calculate the area of the
block of land, correct to the nearest
square metre.

WORKED

Example

18 m
20 m
90
80 70
15 m
120
25 m

106

Maths Quest General Mathematics HSC Course

The cosine rule


Finding side lengths
When given the length of one side and two angles in a triangle, we can use the sine rule
to find another side length. However, in many cases we do not have this information
and need another method of calculating the side lengths. The cosine rule allows us to
calculate the length of the third side of a triangle when we are given the length of the
other two sides and the included angle.
a2 = b2 + c2

2bc cos A

2ac cos B

2ab cos C

b =a +c
c =a +b

The formula sheet gives the third version of this formula only. The others are an
adaptation of the same rule.
It is important to notice that the formula is given in terms of a2, b2 or c2. This means
that to find the value of a, b or c we need to take the square root of our calculation.

Derivation of the cosine rule


Consider ABC on the right. In this
triangle, h is the perpendicular
height of the triangle and meets
AB at D. We will let AD = x, and
therefore BD = c x.

Using Pythagoras theorem on BCD:


From ACD:
Therefore:

Therefore:

b
h

c x

a2 = (c
a2 = c2

D
c

x)2 + h2
2cx + x2 + h2 [1]

b2 = x2 + h2
h2 = b2 x2

Substituting for h2 in [1]:

Now in ACD:

a2 = c2
a2 = c2

2cx + x2 + b2 x2
2cx + b2 [2]

x
cos A = --b
x = b cos A

Substituting for x in [2]:

a2 = c2 2c(b cos A) + b2
a2 = c2 + b2 2bc cos A

This becomes the formula for the cosine rule. A similar formula can be used for
finding sides b and c. You may like to try it for yourself.
1 Start with ABC and draw a perpendicular line from A to BC.
2 Use this diagram and follow the method shown to obtain the following version
of the cosine rule: b2 = a 2 + c 2 2ac cosB.
3 Can you obtain c2 = a2 + b 2

2ab cosC?

Chapter 3 Applications of trigonometry

WORKED Example 14

Find the length of the side marked b in the triangle


on the right, correct to 1 decimal place.

70
10 m

12 m

THINK
1

2
3
4

107

WRITE

Write the formula with b2 as the


subject.
Substitute a = 12, c = 10 and B = 70.
Calculate the value of b2.
Find b by taking the square root of b2.

b2 = a2 + c2

2ac cos B

= 122 + 102 2
= 161.915
b = 161.915
= 12.7 m

12

10

cos 70

equation solver to solve


Graphics Calculator tip! Using
cosine rule problems (sides)
Using the equation solver method for the cosine rule is a very useful method as many
students forget the final step of the solution, which is to take the square root of a2, b2
or c2.
In the same way as with earlier questions we write the formula and then substitute the
appropriate values, leaving one unknown. Hence we have an equation, which can be typed
into the equation solver of the graphics calculator. Consider worked example 14 above.
1. From the MENU select EQUA.

2. Press F3 (SOLV).

3. Delete any existing equation, enter


B2 = 122 + 102 2 12 10 cos 70
and then press EXE .

4. Press F6 (SOLV) to solve the equation.

108

Maths Quest General Mathematics HSC Course

As with sine rule questions, we can apply the cosine rule to obtuse-angled triangles.
You should recall from the earlier investigation that the cosine ratio of an obtuse angle
is negative. The method of solution remains unchanged.

WORKED Example 15
Find the length of side PQ in the triangle
on the right, correct to the nearest
millimetre.

68 mm
122
R

THINK
1

2
3
4

92 mm

WRITE

Write the formula with r 2 as the


subject.
Substitute p = 92, q = 68 and R = 122.
Calculate the value of r 2.
Find r by taking the square root of r 2.

r 2 = p2 + q2

2pq cos R

= 922 + 682 2
= 19 718.35
r = 19 718.35
= 140 mm

92

68

cos 122

The cosine rule also allows us to solve a wider range of practical problems. The
important part of solving such problems is marking the correct information on your
diagram. If you can identify two side lengths and the included angle, you can use the
cosine rule.

WORKED Example 16
A surveyor standing at a point, X, sights a point, M, 50 m away and a point, N, 80 m away.
If the angle between the lines XM and XN is 45, calculate the distance between the points
M and N, correct to 1 decimal place.
THINK
1

WRITE

Draw a diagram and mark all given


information on it.

X
45
50 m

80 m
N

M
2

3
4
5

Write the formula with x2 as the


subject.
Substitute m = 80, n = 50 and X = 45.
Calculate the value of x2.
Calculate x by taking the square root of
x2.
Give a written answer.

x2 = m2 + n2

2mn cos X

= 802 + 502 2
= 3243.15
x = 3243.15
= 56.9 m

80

50

cos 45

109

Chapter 3 Applications of trigonometry

remember
1. To use the cosine rule to find a side length, you need to be given the length of
two sides and the included angle.
2. The cosine rule formulas are:
a2 = b2 + c2 - 2bc cos A
b2 = a2 + c2 - 2ac cos B
c2 = a2 + b2 - 2ab cos C.
3. In the solution to cosine rule questions, your final answer is found by taking the
square root of the calculation.
4. Begin worded questions by drawing a diagram and finish them by giving a
written answer.

Using the cosine rule to find


side lengths

3F

1 Write down the cosine rule formula as it applies to each of the triangles below. In
each case, make the boldfaced pronumeral the subject.
a

P
q

r
c

m
l

N
WORKED

Example

14

2 Find the length of the side marked with a pronumeral in each of the following, correct
to 3 significant figures.
a

A
x

12 m

12 m
42

14 m

13 cm

r
C

35

21 cm

Example

15

12 m

R
Y

WORKED

60

3 In each of the following obtuse-angled triangles, find the length of the side marked
with the pronumeral, correct to 1 decimal place.
a

bA

X
112 cm

110
x

6.1 m
B

130
9.7 m

R
q

b
Z

114 cm

C P

160
43 mm Q

63 mm

110

Maths Quest General Mathematics HSC Course

4 multiple choice
In which of the following triangles are we unable to use the cosine rule to find x?
A
B
14.8 cm
16.4 m

132

16.2 cm

x
32
18.2 m

D
63
8.3 km

9.6 km

10.5 m

9.7 m

63
x

5 multiple choice
Look at the triangle drawn on the right.
The value of x, correct to 1 decimal place, is:
A 7.2 m
B 7.3 m
C 52.4 m
D 52.5 m
6 multiple choice

50
8m

9m

Lieng is asked to find the value of a, correct to 1 decimal place,


in the figure drawn on the right. Below is Liengs solution.
Line 1: a2 = 122 + 82 2 12 8 cos 60
60
Line 2:
= 144 + 64 192 cos 60
8 cm
Line 3:
= 208 192 cos 60
Line 4:
= 16 cos 60
Line 5:
=8
a
Line 6:
a = 2.8 m
Liengs solution is incorrect. In which line did she make her error?
A Line 2
B Line 3
C Line 4
D Line 5

12 cm

7 In ABC, a = 14 cm, c = 25 cm and ABC = 29. Calculate b, correct to 1 decimal place.


8 In PQR, PQ = 234 mm, QR = 981 mm and
PR, correct to 3 significant figures.
WORKED

Example

16

PQR = 128. Find the length of side

9 Len and Morag walk separate paths that diverge from one another at an angle of 48.
After three hours Len has walked 7.9 km and Morag 8.6 km. Find the distance
between the two walkers at this time, correct to the nearest metre.
10 A cricketer is fielding 20 m from the batsman and at an angle of 35 to the pitch. The
batsman hits a ball 55 m and straight behind the bowler. How far must the fieldsman
run to field the ball? (Give your answer to the nearest metre.)
11 The sides of a parallelogram are 5.3 cm and 11.3 cm. The sides meet at angles of 134
and 46.
a Draw a diagram of the parallelogram showing this information and mark both
diagonals on it.
b Calculate the length of the shorter diagonal, correct to 1 decimal place.
c Calculate the length of the long diagonal, correct to 1 decimal place.

Chapter 3 Applications of trigonometry

12 The cord supporting a picture frame is 58 cm


long. It is hung over a single hook in the
centre of the cord and the cord then makes an
angle of 145 as shown in the figure on the
right. Calculate the length of the backing of
the picture frame, to the nearest centimetre.

111

58 cm
145

Finding angles
We can use the cosine rule to find the size of the angles within a triangle. Consider the
cosine rule formula.
a2 = b2 + c2

2bc cos A

We now make cos A the subject of this formula.


a2 = b2 + c2 2bc cos A
a + 2bc cos A = b2 + c2
2bc cos A = b2 + c2 a2
b2 + c2 a2
cos A = ---------------------------2bc
2

In this form, we can use the cosine rule to find the size of an angle if we are given all
three side lengths. We should be able to write the cosine rule in three forms depending
upon which angle we wish to find.
b2 + c2 a2
cos A = ---------------------------2bc
a2 + c2 b2
cos B = ---------------------------2ac
a2 + b2 c2
cos C = ---------------------------2ab
Again, the formula sheet gives the third version of this formula only. The others are an
adaptation of the same rule.

WORKED Example 17

Find the size of angle B in the triangle


on the right, correct to the nearest degree.
7 cm

THINK
1

Write the formula with cos B as the


subject.

5 cm

9 cm

WRITE
a2 + c2 b2
cos B = ---------------------------2ac
Continued over page

112

Maths Quest General Mathematics HSC Course

THINK

WRITE

Substitute a = 9, b = 5 and c = 7.

Calculate the value of cos B.

Make B the subject of the equation.


Calculate B.

92 + 72 52
cos B = ---------------------------2 9 7
105
cos B = --------126
= 0.8333
B = cos 1(0.8333)
B = 34

equation solver to solve


Graphics Calculator tip! Using
cosine rule problems (angles)
As with all of the trigonometric applications we can use the equation solver to find the
required answer. Consider worked example 17.
1. From the MENU select EQUA.

2. Press F3 (SOLV).

3. Delete any existing equation, enter the equation


cos B = (92 + 72 52) (2 9 7),
and then press EXE .

4. Press F6 (SOLV) to solve the equation.

Your formula sheet will give you two versions of the cosine rule, one for finding a side
length and one for finding an angle. When using the equation solver it does not matter
which version you use to find a side or an angle.
Try using the solver on the equation 52 = 92 + 72 2 9 7 cos B.
As we found earlier, the cosine ratio for an obtuse angle will be negative. So, when
we get a negative result to the calculation for the cosine ratio, this means that the angle
we are finding is obtuse. Your calculator will give the obtuse angle when we take the
inverse.

Chapter 3 Applications of trigonometry

113

WORKED Example 18
Find the size of angle Q in the triangle
on the right, correct to the nearest
degree.

Q
4 cm
P

THINK

6 cm

WRITE

Write the formula with cos Q as the


subject.

Substitute p = 3, q = 6 and r = 4.

Calculate the value of cos Q.

Make Q the subject of the equation.


Calculate Q.

3 cm

p2 + r 2 q2
cos Q = ---------------------------2 pr
32 + 42 62
cos Q = ---------------------------2 4 3
11
cos Q = --------24
= 0.4583
Q = cos 1( 0.4583)
Q = 117

In some cosine rule questions, you need to work out which angle you need to find. For
example, you could be asked to calculate the size of the largest angle in a triangle. To
do this you do not need to calculate all three angles. The largest angle in any triangle
will be the one opposite the longest side. Similarly, the smallest angle will lie opposite
the shortest side.

WORKED Example 19
Find the size of the largest angle in
the triangle drawn on the right.

R
3.4 m

4.9 m

S
5.7 m

THINK
1 ST is the longest side, therefore angle R
is the largest angle.
2

Write the formula with cos R the


subject.

Substitute r = 5.7, s = 4.9 and t = 3.4.

Calculate the value of cos R.

Make R the subject of the equation.


Calculate R.
Give a written answer.

6
7

WRITE

s2 + t 2 r 2
cos R = -------------------------2st
4.9 2 + 3.4 2 5.7 2
cos R = -----------------------------------------2 4.9 3.4
3.08
cos R = ------------33.32
= 0.0924
R = cos 1(0.0924)
R = 85
The largest angle in the triangle is 85.

114

Maths Quest General Mathematics HSC Course

Many problems that require you to find an angle are solved using the cosine rule. As
always, these begin with a diagram and are finished off by giving a written answer.

WORKED Example 20
Two paths diverge from a point, A. The first path goes for 1.25 km to a point, B. The
second path goes for 1.4 km to a point, C. B and C are exactly 2 km apart. Find the angle
at which the two paths diverge.
THINK
1

WRITE

Draw a diagram.

2 km

1.25 km

A
1.4 km
2

Write the formula with cos A as the


subject.

Substitute a = 2, b = 1.4 and c = 1.25.

Calculate the value of cos A.

Make A the subject of the equation.


Calculate the value of A.
Give a written answer.

6
7

b2

c2

a2

+
cos A = ---------------------------2bc

1.4 2 + 1.25 2 2 2
cos A = ---------------------------------------2 1.4 1.25
0.4775
cos A = ------------------3.5
= 0.1364
A = cos 1( 0.1364)
= 98
The roads diverge at an angle of 98.

remember
1. The cosine rule formulas are:
b2 + c2 a2
cos A = ---------------------------2bc
a2 + c2 b2
cos B = ---------------------------2ac
a2 + b2 c2
cos C = ---------------------------2ab
2. If the value of the cosine ratio is negative, the angle is obtuse.
3. In any triangle, the largest angle lies opposite the largest side and the smallest
angle lies opposite the smallest side.
4. Worded problems begin with a diagram and end with a written answer.

115

Chapter 3 Applications of trigonometry

3G

Using the cosine rule to find


angles

1 For each of the following, write the cosine rule formula as it applies to the triangle
drawn with the boldfaced angle as the subject.
a

b P

c
A

P
B

WORKED

Example

17

2 Find the size of the angle marked with the pronumeral in each of the following
triangles, correct to the nearest degree.
a

8 cm

11 cm

2.8 m

3.2 m

4.5 m

5.4 m

C
B

WORKED

Example

18

13 cm

4.0 m

6.2 m

3 In each of the obtuse-angled triangles below find the size of the angle marked with the
pronumeral, to the nearest degree.
a

6m

c
9.6 m

8m

12.9 m

4.2 m

9.2 m
6.1 m

11 m

4.2 m

4 multiple choice
Look at the figure drawn below.
5 cm

3 cm

7 cm

Which of the following correctly represents the value of cos ?


A cos

32 + 72 52
= ---------------------------2 3 7

B cos

32 + 72 52
= ---------------------------2 5 7

C cos

32 + 52 72
= ---------------------------2 3 5

D cos

52 + 72 32
= ---------------------------2 5 7

116

Maths Quest General Mathematics HSC Course

5 multiple choice
In which of the following is the angle
A

obtuse?
B

3 cm

4 cm

3 cm

5 cm

4 cm

4 cm

3 cm

4 cm

3 cm

4 cm

6 cm
4 cm

6 In PQR, p = 7 m, q = 9 m and r = 6 m. Find

QRP, correct to the nearest degree.

7 In KLM, k = 85 mm, l = 145 mm and m = 197 mm. Find the size of the smallest
angle, correct to the nearest degree.
WORKED

Example

19

8 Calculate the size of all three angles (correct to the nearest degree) in a triangle with
side lengths 12 cm, 14 cm and 17 cm.
9 WXYZ is a parallelogram. WX = 9.2 cm and XY = 13.6 cm. The diagonal
WY = 14 cm.
a Draw a diagram of the parallelogram.
b Calculate the size of WXY, correct to the nearest degree.

10 Two roads diverge from a point, P. The


first road is 5 km long and leads to a
20
point, Q. The second road is 8 km long
and leads to a point, R. The distance
between Q and R is 4.6 km. Calculate
the angle at which the two roads
diverge.

WORKED

Example

11 A soccer goal is 8 m wide.


a A player is directly in front of the
goal such that he is 12 m from each
post. Within what angle must he kick
the ball to score a goal?
b A second player takes an angled shot.
This player is 12 m from the nearest
post and 17 m from the far post.
Within what angle must this player
kick to score a goal?
12 The backing of a picture frame is 50 cm long and is
hung over a picture hook by a cord 52 cm long as
shown in the figure on the right. Calculate the angle
made by the cord at the picture hook.

52 cm

50 cm

Chapter 3 Applications of trigonometry

117

2
1 Find the size of the side marked x,
correct to the nearest millimetre.

2 Find the size of the side marked y,


correct to 3 significant figures.
4.1 m
11

y
40
x

346 mm

3 Find the angle marked , correct


to the nearest degree.

5.8 km

4.9 km

4 Write down the sine rule formula as used to find a side.


5 Use the sine rule to find a,
correct to 1 decimal place.

6 Use the sine rule to find ,


correct to the nearest degree.

68

83

4.2 km
a

57

7.9 km
14 m

7 Write down the cosine rule formula as used to find a side length.
8 Use the cosine rule to find m,
correct to 2 significant figures.

250 m

40
320 m

9 Write down the cosine rule as used to find an angle.


10 Use the cosine rule to find ,
correct to the nearest degree.

9m
13 m

17 m

118

Maths Quest General Mathematics HSC Course

Radial surveys
In the preliminary course we examined the offset survey. In this survey method an area
is measured by drawing a traverse line and measuring offsets at right angles to the traverse line. Because the offset survey created right-angled triangles, the length of each
boundary could be calculated using Pythagoras theorem and the area could be calculated using the formula Area = 1--- bh.
2
An alternative survey method to this is a radial survey. One type of radial survey is
the plane table radial survey. The following steps are taken in a plane table survey.
1. A table is placed in the centre of the
field to be surveyed, each corner of
the field is sighted and a line is ruled
on the paper along the line of sight.

2. The distance from the plane


table to each corner is then
measured.

25

20
m

26 m

28

3. The angle between each radial


line is then measured and the
radial lines joined to complete
the diagram.

25

20
m

115
60

115

28
m

26 m

70

The field will then be divided into triangles. The length of each side of the field can
then be calculated by using the cosine rule. The perimeter of the field is then found by
adding the lengths of each side.

Chapter 3 Applications of trigonometry

WORKED Example 21

119

A
23

30 m

The figure on the right is a plane table survey


of a block of land. Calculate the perimeter of
the block of land, correct to the nearest metre.

110
125 4017 m
85

28 m

C
X

THINK
1

WRITE

Apply the cosine rule in AXB to


calculate the length of AB.

Apply the cosine rule in BXC to


calculate the length of BC.

Apply the cosine rule in CXD to


calculate the length of CD.

Apply the cosine rule in DXA to


calculate the length of DA.

Calculate the perimeter by adding the


length of each side and rounding the
answer to the nearest metre.

For AXB:
x2 = a2 + b2 2ab cos X
= 302 + 232 2 30 23
= 1900.99
x = 43.6 m
The length of AB is 43.6 m.
For BXC:
x2 = b2 + c2 2bc cos X
= 172 + 302 2 17 30
= 407.63
x = 20.2 m
The length of BC is 20.2 m.
For CXD:
x2 = c2 + d 2 2cd cos X
= 282 + 172 2 28 17
= 990.03
x = 31.5 m
The length of CD is 31.5 m.
For DXA:
x2 = d2 + a2 2da cos X
= 232 + 282 2 23 28
= 2051.77
x = 45.3 m
The length of DA is 45.3 m.

cos 110

cos 40

cos 85

cos 125

Perimeter = 43.6 + 20.2 + 31.5 + 45.3


Perimeter = 140.6 m
Perimeter = 141 m (correct to the nearest metre)

A similar approach is used to calculate the area of such a field. The area of each
triangle is found using the formula Area = 1--- ab sin C. The total area is then found by
2
adding the area of each triangle.

120

Maths Quest General Mathematics HSC Course

WORKED Example 22

m
52

Calculate the area of the field on the right.


Give your answer correct to the nearest
square metre.

m
48

96
144 120

67 m

THINK
1

WRITE

Calculate the area of AXB.

For AXB: Area = 1--- ab sin X


=

Calculate the area of BXC.

2
1
--2

48

52

sin 96

= 1241.2 m2
For BXC: Area = 1--- bc sin X
=

2
1
--2

67

48

sin 120

Calculate the area of CXA.

= 1392.6 m
For CXA: Area = 1--- ca sin X
=

Calculate the total area by adding the


area of each triangle.

2
1
--2

52

67

sin 144

= 1023.9 m2
Total area = 1241.2 + 1392.6 + 1023.9
Total area = 3657.7 m2
Total area = 3658 m2 (correct to the nearest m2)

An alternative to the plane table radial survey is the compass radial survey. In this
survey the bearing of each radial line is calculated and this bearing is used to calculate
the angle between each radial, as in the worked example below. The method of calculating the perimeter and area of the field is then the same as for the plane table radial
survey.

WORKED Example 23

A
338

B
067

49

m
58

The figure on the right shows a compass radial


survey of a block of land.
a Calculate the size of AXB.
b Hence, calculate the distance AB,
correct to the nearest metre.

55

57 m

m
X

D
239

THINK

WRITE

a A is 22 west of North, B is 67 east


of North.

a 22 + 67 = 89

C
114

Chapter 3 Applications of trigonometry

121

THINK

WRITE

b For DAXB: x2 = a2 + b2 - 2ab cos X


= 492 + 582 - 2 49 58 cos 89
= 5665.8
x = 75 m (correct to the nearest metre)
The distance AB is 75 m.

1
2
3
4
5

Write the cosine rule formula.


Substitute for a, b and X.
Calculate the value of x2.
Calculate x.
Write your answer.

remember
1. In a radial survey, radial lines are drawn and measured from a point in the
centre of an area.
2. In a plane table radial survey, radial lines are drawn on a table by sighting
each corner of the field. The length of each line and the angle between the
lines is then measured.
3. A compass radial survey is similar but the bearing of each radial line is
measured.
4. Each survey divides the area into triangles and the length of each boundary
can be calculated using the cosine rule.
5. The area of each triangle can be calculated using the formula
Area = 1--- ab sin C.
2

3H

1 The figure on the right is a plane table radial


survey of a block of land. Use the cosine
21
rule to calculate the perimeter of the block
of land, correct to the nearest metre.

WORKED

Example

15
m

10
m

Radial surveys
100
70 80
110

m
20

25

2 Calculate the perimeter of each of the following areas, correct to the nearest metre.

45
m
m

115

80

70
85 80
125
45

95 75 m
150

92 m

m
1

m
114

m
60

55 m

c
12

b
100 m

90 40
60
89 m
140 30

78

122

Maths Quest General Mathematics HSC Course

3 The figure on the right is a plane table survey


of a block of land. Calculate the area of the
22
block, correct to the nearest square metre.

WORKED

160
0m
60 100 8
40

90 m

4 For each of the plane table surveys shown in


question 2 calculate the area, correct to the
nearest square metre.

100 m

11
0m

Example

315

5 The figure on the right is a compass radial


Example
survey of a field.
23
a Calculate the size of AXB.
b Hence, use the cosine rule to calculate the
distance AB, correct to the nearest metre.

B
040

50

WORKED

40 m

60

m
X
70 m
110 C

170

D
350

100 m

6 Calculate the perimeter of the field given by


the compass radial survey on the right. Give
your answer correct to the nearest metre.

30 m

90 m

110

250

7 Calculate the perimeter of each of the compass radial surveys shown below.
a 327
b 339
c 319
030

020

Work

63
m

10
8

m
42

99 m

097

196

m
29
226

085

3.2

49 m

38 m

38

T
SHEE

m
72

m
215

m
114

53
24 m

052

170

8 For each of the compass radial surveys in question 7 calculate the area, correct to the
nearest square metre.

Conducting a radial survey


Choose an appropriate area in or near your school to conduct a radial survey.
1 Set up a table in the centre of the area and tape a large piece of paper to the table.
2 Mark a point in the middle of the piece of paper and sight each corner of the
field from this point, ruling a line from the point in that direction.
3 Use a tape or trundle wheel to measure the distance from the table to each
corner of the field.
4 Use your protractor to measure the angle between each radial line.
5 Calculate the area and the perimeter of the field.

123

Chapter 3 Applications of trigonometry

summary
Right-angled triangles
The formulas to be used when solving right-angled triangles are:
opposite side
sin = ------------------------------hypotenuse
adjacent side
cos = ------------------------------hypotenuse
opposite side
tan = ------------------------------adjacent side
To calculate a side length, you need to be given the length of one other side and one
angle.
To calculate the size of an angle, you need to be given two side lengths.
If a question is given as a problem, begin by drawing a diagram and give a written
answer.

Bearings
Bearings are a measure of direction.
A compass bearing uses the four main points of the compass, north, south, east and
west, as well as the four intermediate directions, north-east, north-west, south-east,
south-west.
More specific directions are given using true bearings. A true bearing describes a
direction as a three-digit angle taken in a clockwise direction from north.
Most bearing questions will require you to draw a diagram to begin the question
and require a written answer.

Sine rule
The sine rule allows us to calculate sides and
b
angles in non-right-angled triangles.
When finding a side length you need to
A
be given the length of one other side and two angles. A
a
b
c
The sine rule formula is ------------ = ------------ = ------------sin A
sin B
sin C

C
C
a
B
c

When finding an angle you need to be given two side lengths and one angle.
sin A
sin B
sin C
The sine rule formula when finding an angle is ------------ = ------------ = ------------- .
a
b
c

Area of a triangle
When you do not know the perpendicular height of a triangle, you can calculate the
area using the formula Area = 1--- ab sin C .
2
To calculate the area using this formula, you need to be given the length of two
sides and the included angle.

Maths Quest General Mathematics HSC Course

Cosine rule
The cosine rule allows you to calculate the length of sides and size of angles of
non-right-angled triangles where you are unable to use the sine rule.
To find a side length using the cosine rule, you need to be given the length of two
sides and the included angle and use the formula c2 = a2 + b2 2ab cos C .
To find an angle using the cosine rule, you need to be given the length of all three
a2 + b2 c2
sides and use the formula cos C = ---------------------------- .
2ab

Surveying
A plane table radial survey sights each corner of a
field and draws a radial line in that direction. This
divides the field into triangles. The length of each
radial line and the angle between radial lines are
then measured.
The cosine rule can then be used to calculate the
length of each boundary.

25

20

115
60

115

28

26 m

70

The formula Area = 1--- ab sin C can be then used to


2

calculate the area of the field.


A compass radial survey takes the bearing of each radial line and this is then used
to calculate the angles between them.
A
338

m
58

067

49

124

55
D
239

57 m

m
X

C
114

Chapter 3 Applications of trigonometry

125

CHAPTER
review
1 Find the length of the side marked with the pronumeral in each of the right-angled triangles
below, correct to 1 decimal place.
a
b
c
t
7.9 cm

72
17.2 cm

3A

42 km
45

60
x

2 In each of the following right-angled triangles, find the size of the angle marked with the
pronumeral, correct to the nearest degree.
a
b
c
8.3 km

3A

35 cm

16 m

24.8 cm

20.1 km

9m

3 An aeroplane at an altitude of 2500 m sights a ship


at an angle of depression of 39. Calculate, to the
nearest metre, the horizontal distance from the
aeroplane to the ship.

3A

4 When a yacht is 500 m from shore, the top of a cliff


is sighted at an angle of elevation of 12.
a Calculate the height of the cliff, correct to the
nearest metre.
b Calculate what the angle of elevation of the top of
the cliff will be when the yacht is 200 m from
shore.

3A

5 Two aircraft are approaching an airport. The Qantas


plane (Q) is 40 km due north of the runway (R),
while a Jetstar plane (J) is due east of the Qantas
plane and north-east of the runway. Calculate the
distance of the Jetstar plane from the runway. (Give
your answer correct to the nearest metre.)

3B

6 A car rally requires cars to travel for 25 km on a


bearing of 240. The cars are then required to travel
due north until they are due west of the starting
point. Calculate the distance from the cars to the
starting point. (Give your answer correct to
1 decimal place.)

3B

126

Maths Quest General Mathematics HSC Course

3B

7 A yacht sails due west for 45 nautical miles before turning north for 23 nautical miles.
a Calculate the bearing of the yacht from its starting point.
b On what bearing must the yacht sail to return to its starting point?

3C

8 Use the sine rule to calculate each of the sides marked with a pronumeral, correct to
3 significant figures.
a
b
c
e
a
d

19

117
70

28

31

4.6 cm

4.6 km

136 mm
20

3C
3D

9 In XYZ: x = 9.2 cm,

XYZ = 56 and

YXZ = 38. Find y, correct to 1 decimal place.

10 Use the sine rule to calculate the size of the angle marked with a pronumeral, correct to the
nearest degree.
a
b
c

8 cm

9.7 cm

7.1 m 9

9 cm

123
4.1 cm

63

1.2 m

3E

11 In ABC: b = 46 cm, c = 37 cm and


the nearest square centimetre.

3E

12 Find the area of a triangular field with two sides of 80 m and 98 m, which meet at an angle
of 130 (correct to the nearest hundred square metres).

3F

13 Use the cosine rule to find each of the following unknown sides, correct to 3 significant
figures.
a
b
c
6.9 cm

BAC = 72. Find the area of the triangle, correct to

6.2 cm

9m

128

b
a

5.7 m

50
117
11 m

3F

14 In LMN: LM = 63 cm, MN = 84 cm and


1 decimal place.

3F

15 During a stunt show two


aeroplanes fly side by side until
they suddenly diverge at an
angle of 160. After both planes
have flown 500 m what is the
distance between the planes,
correct to the nearest metre?

4.6 m

LMN = 68. Find the length of LN, correct to

Chapter 3 Applications of trigonometry

127

16 Use the cosine rule to find the size of the angle in each of the following, correct to the
nearest degree.
a
b
c
9 cm
7 cm

4.2 m

3G

5.3 m

6 cm
15 cm

6 cm
7.9 m
6 cm

17 In XYZ: x = 8.3 m, y = 12.45 m and z = 7.2 m. Find

YZX, to the nearest degree.

18 Two wooden fences are 50 m and 80 m long respectively. Their ends are connected by a
barbed wire fence 44 m long. Find the angle at which the two wooden fences meet.
19 The figure below is a plane table radial survey of a field.

3G
3G
3H

60 m
m

70 m

30

40
80
120 50
110

a Use the cosine rule to calculate the perimeter of the field.


b Calculate the area of the field.
340
0m
15

260

160 m

90 m

140 m

20 The figure on the right is a compass


radial survey.
a Calculate the perimeter of the field.
b Calculate the area of the field.

190

Practice examination questions


1 multiple choice
In the figure on the right, which of the
following will give the value of x?
13 sin 36
A x = ------------------------sin 64
13 sin 64
B x = ------------------------sin 36
13 sin 64
C x = ------------------------sin 80
13 sin 80
D x = ------------------------sin 64

13 m
64
x
36

080

3H

128

Maths Quest General Mathematics HSC Course

2 multiple choice
In the figure on the right, which of the
following will give the value of cos ?
62 + 72 82
A cos = ---------------------------B cos
2 6 7
C cos

72 + 82 62
= ---------------------------2 7 8

D cos

7m

6m

62

82

72

+
= ---------------------------2 6 8

62 + 72 82
= ---------------------------2 7 8

8m

3 multiple choice
Maurice walks 3 km on a true bearing of 225. To return to his starting point he must walk on
a compass bearing of:
A north-east
B north-west
C south-east
D south-west
4 multiple choice

B 80
305 A

The figure on the right is a compass


radial survey.
AXB is:
A 35
B 55
C 85
D 135

C 174

5 The distance between football goal posts is 7 m. If Soon Ho is 20 m from one goal post and
25 m from the other:
a draw a diagram showing the goal posts and Soon Hos position.
b calculate the angle within which Soon Ho must kick to score a goal. (Give your answer
correct to the nearest degree.)
6 An observer sights the top of a building at an angle of
elevation of 20. From a point 30 m closer to the building,
the angle of elevation is 35 as shown in the figure on
20
the right.
A
30 m
a Calculate the size of ATB.
b Show that the distance BT can be given by the expression
30 sin 20
BT = ------------------------- .
sin 15
c Show that the height of the building can be given by the expression
30 sin 20 sin 35
h = -------------------------------------------sin 15
d Calculate the height of the building correct to 1 decimal place.

CHAPTER

h
35
B

A 345

110 m

test
yourself

7 The figure on the right shows a compass radial survey


of a field.
a Calculate the length of the boundary CD, correct to
1 decimal place.
b Calculate the area of LAXB, correct to the nearest square
metre.

X 30 m
80 m

D 250

30
m

B
085

125 C

Interpreting
sets of data

4
syllabus reference
Data analysis 5
Interpreting sets of data

In this chapter
4A Measures of location and
spread
4B Skewness
4C Displaying multiple data
sets
4D Comparison of data sets

areyou

READY?

Are you ready?

Try the questions below. If you have difficulty with any of them, extra help can be
obtained by completing the matching SkillSHEET. Either click on the SkillSHEET icon
next to the question on the Maths Quest HSC Course CD-ROM or ask your teacher for
a copy.

4.1

4.2

Finding the mean

1 Find the mean of the following sets of scores.


a 3, 5, 8, 3, 9, 4, 3, 5
b Stem Leaf
0 9
1 22367
2 457
3 00

Score

Frequency

14

32

27

18

Finding the mode

2 For each of the data sets in question 1 find the mode.

4.3

Finding the median

3 Find the median of the data sets in question 1.

4.4

Finding the range

4 Find each of the data sets in question 1 find the range.

4.5

Finding the interquartile range

5 For each of the data sets in question 1 find the interquartile range.

4.6

Choosing the appropriate standard deviation

6 For each of the following choose and find the appropriate measure of the standard deviation.
a At the end of a movie 10 viewers were chosen and asked to rate the movie from 1 to 5 stars.
The results were: 3, 4, 2, 3, 1, 5, 2, 4, 3, 5.
b At Yass High School there are 80 students who study General Mathematics. They all sat for a
test scored out of 20, and the results obtained are given in the table below.
Score
Frequency

14

15

16

17

18

19

20

15

24

18

4.7
Compiling a stem-and-leaf plot

4.9

7 The scores below show the number of customers each day in a car yard.
23, 32, 27, 31, 19, 45, 22, 26, 38, 41, 27, 40, 9, 34, 37, 21,
22, 30, 39, 19, 14, 32, 20, 40, 23, 27, 26, 28, 11, 15, 28, 33
Display the data in a stem-and-leaf plot.
Drawing a box-and-whisker plot

8 For the data set in question 6, display the results using a box-and-whisker plot.

Chapter 4 Interpreting sets of data

131

Measures of location and spread


Consider the following set of scores that are the exam results for 10 students.
55, 57, 57, 58, 60, 60, 62, 63, 63, 65
To identify a score that is typical in this data set, we can use the mean or median.
The mean is calculated by adding all the scores and dividing by the number of scores
in the set.
When the data is a small set of scores the mean is found using the formula
x = -----xn
where x = mean
x = individual scores (Therefore, x represents the sum of individual scores.)
n = number of scores
Where the data is presented in a frequency table we use the formula
fx
x = ------f
where x = mean
x = individual scores
f = frequency
In this formula fx represents the sum of the frequency score column on the frequency table and f represents the sum of frequency column.
The median is the middle score (odd number of scores) or the average of the two
middle scores (even number of scores).
For this set of scores:
Mean = 600 10
= 60
Median = 60
Both the mean and median are a measure of location within a data set.

WORKED Example 1

For the set of scores 13, 19, 31, 40, 55, 65, 90, 92, 95, 100 calculate:
a the mean
b the median.
THINK

WRITE

a Total = 600
Mean = 600 10
Mean = 60

1
2

Find the total of the scores.


Divide the total by the number of
scores.

b Average the two middle scores.

b Median = (55 + 65) 2


= 60

132

Maths Quest General Mathematics HSC Course

We have now examined two data sets. Look at these data sets side by side.
Set A: 55, 57, 57, 58, 60, 60, 62, 63, 63, 65
Set B: 13, 19, 31, 40, 55, 65, 90, 92, 95, 100
Although both sets of scores have the same mean and median, they are very different
sets of scores. Clearly, in Set B the scores are more spread out than in Set A. To
measure the spread of a set of scores, we use one or all of the following.
Range: Highest score lowest score
Set A: Range = 65
Set A: Range = 10

55

Set B = 100
Set B = 87

13

Interquartile range (IQR): The difference between the upper quartile and lower quartile.
Set A: Interquartile range = 63
Set A: Interquartile range = 6

57

Standard deviation: Found using the


calculator.
Set A:

Set B: Interquartile range = 92


Set B: Interquartile range = 61

31

(population) or sn (sample) functions on the

= 3.07

Set B:

= 31.51

Each of these measures of spread show that in Set B the scores are more scattered than
in Set A.

WORKED Example 2

For the set of scores 45, 62, 75, 69, 50, 87, 92 calculate:
a the range
b the interquartile range
c the standard deviation.
THINK

WRITE

a Subtract the lowest score from the


highest score.

a Range = 92
Range = 47

b 45, 50, 62, 69, 75, 87, 92


45, 50, 62,
75, 87, 92

1
2

Write the scores in ascending order.


Divide the data in two halves,
leaving the middle score out of both
sets.
The lower quartile is the median of
the lower half; the upper quartile is
the median of the upper half.
Subtract the lower quartile from the
upper quartile.

c Enter the set of scores into your


calculator using the statistics function.

45

Lower quartile = 50 Upper quartile = 87

Interquartile range = 87
Interquartile range = 37
c

50

= 16.36

Graphics Calculator tip! Finding all summary statistics


Your graphics calculator can be used to find all of the important measures of central
tendency and spread. This is demonstrated in worked example 3.

Chapter 4 Interpreting sets of data

133

WORKED Example 3
Nadia is a gymnast. For a routine she is given the following scores by 10 judges.
9.0 8.7 9.2 9.3 9.8
9.2 8.8 9.4 9.0 9.1
Use your graphics calculator to find
a the mean
b the median
c the mode
d the range
e the interquartile range
f the population standard deviation
g the sample standard deviation.
THINK
1

From the MENU select STAT.

Delete any existing data, and enter the


scores above in List 1.

Press F2 (CALC). You may need to


first press F6 for more options.

Press F6 (SET). Check that 1Var Xlist


is set to List 1 and 1Var Freq is set to 1.

Press EXE to return to the previous


screen, and then press F1 (1Var). All
statistics will now be on display using
the scroll function.

WRITE

Mean
Population standard
deviation
Sample standard
deviation
Number of scores
Lowest score
Lower quartile
Median

Upper quartile
Highest score
Mode

Continued over page

134

Maths Quest General Mathematics HSC Course

THINK
a The mean is denoted by the symbol x .

WRITE
a x = 9.15

b The median is denoted by Med.

b Median = 9.15

c The mode is displayed by Mod. Check


the scores for yourself as the data is
bimodal only the largest mode is
displayed.

c Mode = 9.0 and 9.2

d The range is the highest score (maxX)


minus the lowest score (minX).

d Range = 9.8 8.7


Range = 1.1

e The interquartile range is the upper


quartile (Q3) minus the lower quartile
(Q1).

e Interquartile range = 9.3 9


Interquartile range = 0.3

f The population standard deviation is


denoted by x n.

g The sample standard deviation is


denoted by x n1.

n1

0.297
0.314

Having identified that the mean and median are measures of location and that range,
interquartile range and standard deviation are measures of spread, it is important that
you can recognise the effect that the members of a set have on these measures.
Consider the case of a
basketball team. There are five
players on the team, whose
heights are: 1.91 m, 1.85 m,
1.52 m, 1.93 m and 1.99 m.
The teams mean height is
1.84 m. Only one of the five
players in the team is shorter
than the mean height. This is
because there is one member
of the data set whose height is
much less than the others. A
score in a data set that is
either much less or much
greater than all others is called
an outlier.
An outlier will either
reduce or increase the mean
such that the mean is no
longer typical of the data set.
In such cases, the median is a
better measure of location
than the mean.

Chapter 4 Interpreting sets of data

135

WORKED Example 4

In a small street there are five houses. The values of these houses are:
$450 000, $465 000, $465 000, $480 000, $495 000.
A new house is built and valued at $750 000. Describe the effect that this outlier
has on the:
a mean
b median
c mode (the score that occurs most often).
THINK

WRITE

a Before new house is built:


Total = $2 355 000
Mean = $2 355 000 5
= $471 000
After new house is built:
Total = $3 105 000
Mean = $3 105 000 6
= $517 500
The outlier has caused the mean to increase
by $46 500. Only the new house is valued
at more than the mean and, as such, has
made the mean a poor measure of the
typical price.

Calculate the mean before the new


house is built.

Calculate the mean after the new


house is built.

Comment on the change in the mean


caused by the outlier.

Calculate the median before the new


house is built.

Calculate the median after the new


house is built.

Comment on the change in the


median caused by the outlier.

Calculate the mode before the new


house is built.

Calculate the mode after the new


house is built.

After new house is built:


Mode = $465 000

Comment on the change in the mode


caused by the outlier.

The outlier has had no effect on the mode.

b Before new house is built:


Median = $465 000
After new house is built:
Median = ($465 000 + $480 000) 2
Median = $472 500
The outlier has caused only a small increase
in the median and, as such, the median
remains a good measure of the typical score
in this data set.
c Before new house is built:
Mode = $465 000

Generally the mean is the most vulnerable measure of location when an outlier is added
to a data set. The median is affected only by the addition of the extra score and is not
affected by the size of that score. The outlier will have no effect on the mode.

136

Maths Quest General Mathematics HSC Course

remember
1. The mean and median are measures of location in a data set.
The mean is calculated by adding the scores and then dividing by the number
of scores. The mean is calculated using the formulas:
x = Sx
x = Sfx
------------or
n
Sf
The median is the middle score or the average of the two middle scores in a
data set.
2. The range, interquartile range and standard deviation are measures of spread.
The range is the difference between the highest and lowest scores.
The interquartile range is the difference between the upper and lower
quartiles.
The standard deviation is found using the sn (population) or sn (sample)
functions on the calculator.
3. An outlier is a score in a data set that is either much less or much greater than
all other scores in the set.
4. All important summary statistics can be found by entering data into a graphics
calculator.

4A
SkillS

HEET

4.1

WORKED

Example

1
Finding
the
mean

SkillS

HEET

4.2
Finding
the
mode

SkillS

HEET

4.3
Finding
the
median

EXCE

et

reads
L Sp he

One
variable
statistics

Measures of location and


spread

1 The number of goals scored by a team in 10 games of soccer are:


2, 1, 3, 1, 0, 0, 1, 1, 6, 1.
a Calculate the mean number of goals scored.
b Calculate the median number of goals scored.
2 For each of the following sets of scores, calculate the mean, median and mode (if one
exists).
a 56, 75, 88, 20, 37, 23, 44
b 2, 1, 7, 4, 6, 1, 1, 4, 5, 3
c 9.9, 9.4, 9.8, 9.6, 9.0, 9.2, 9.8, 9.9
d 13, 15, 16, 17, 10, 13, 15, 14, 19, 20
3 The table at right shows the scores out of 10 by
a class of 30 students on a spelling test.
a Use the statistics function on your calculator
to find the mean score.
b Add a cumulative frequency column to the
table and use it to calculate the median score.
c State the mode.

Score

Frequency

10

Chapter 4 Interpreting sets of data

137

Score

Class centre

Frequency
2

610

1115

1620

2125

2630

UV Stats

program
GC

TI

15

Cumulative
frequency

am
progr C

asio

GC

4 The table below shows the scores achieved by a football team over a season.

UV Stats

a Copy and complete the table.


b Calculate the mean.
c Draw a cumulative frequency histogram and polygon and use them to estimate the
median.
Example

No. of cars

Frequency

15

10

11

11

12

4.5

SkillS

HEET

6 The table below shows the number of cars sold in a car yard each week over one year.

SkillS

HEET

5 Below is the number of students in each class at a small primary school.


4.4
28, 29, 27, 28, 30, 28, 25, 27, 23, 28, 27, 28
a Calculate the range of the distribution.
Finding
b Calculate the interquartile range.
the
c Use the statistics function on your calculator to find the mean and standard range
deviation.
Finding
the
interquartile
range

4.6

Skil

HEET

lS
a Calculate the range of the number of cars sold.
b Add a cumulative frequency column to the table and use the table to calculate: Choosing the
i the median
appropriate
standard
ii the upper and lower quartiles
deviation
iii the interquartile range.
L Spre
c Use the statistics function on the calculator to find:
XCE ad
i the mean
Boxplots
ii the standard deviation.
d Draw a box-and-whisker plot of the data.

sheet

WORKED

138

Maths Quest General Mathematics HSC Course

7 The table below shows crowds at each match for a team during football season.
Crowd

a
b
c
d
WORKED

Example

Class centre

Frequency

10 00015 000

15 00020 000

20 00025 000

25 00030 000

30 00035 000

Cumulative
frequency

Copy and complete the table.


Draw a cumulative frequency histogram and polygon.
Use the graph in part b to estimate the interquartile range.
Find the mean and standard deviation. (Give your answer correct to 2 significant
figures.)

8 Below are the scores of two rugby league teams over a period of 10 matches.
Team A: 14, 16, 16, 20, 10, 12, 18, 16, 18, 20
Team B: 28, 12, 32, 2, 0, 8, 40, 10, 12, 16
a For each team calculate the mean score.
b For each team calculate:
i the range
ii the interquartile range
iii the standard deviation.
c Comment on the difference between the performance of the two teams over this
10-game period.
The information below is to be used for questions 9 to 12.
A basketball squad has eight players. The mean height of the eight players is 1.8 m, and
the standard deviation in the heights of the players is 0.1 m. In the first game the tallest
player, who is 1.9 m tall, is injured and replaced in the squad by a player who is 1.98 m tall.
9 multiple choice
The mean height of the basketball squad will now be:
A 1.8 m
B 1.81 m
C 1.86 m
10 multiple choice
As a result of the substitution:
A the standard deviation will increase
B the standard deviation will decrease
C the standard deviation will be unchanged
D the effect on the standard deviation cannot be calculated
11 multiple choice
As a result of the substitution:
A the range will increase
B the range will decrease
C the range will be unchanged
D the effect on the range cannot be calculated

D 1.96 m

Chapter 4 Interpreting sets of data

139

12 multiple choice
As a result of the substitution:
A the interquartile range will increase
B the interquartile range will decrease
C the interquartile range will be unchanged
D the effect on the interquartile range cannot be calculated
13 James recorded the following five marks on his Maths tests during the year:
78, 77, 80, 85 and 80.
4
a Calculate:
i the mean
ii the median
iii the mode.
b In James final exam he scored only 20. For the six test results calculate:
i the mean
ii the median
iii the mode.
c Describe the effect that the outlier had on the mean, median and mode.

WORKED

Example

14 The mean of a set of five scores is 60. A score of 90 is added to the data set. Describe
the effect that this outlier will have on the mean.
15 multiple choice
Julie is currently in Year 12. The table below shows the number of days that Julie has
been absent from school in each
of the previous five years.
Year

No. of days
absent

10

11

During Year 12, Julie became seriously ill and was forced to have 37 days off school.
According to statistics calculated on Julies absences over six years, this outlier will
have the greatest effect on:
A the mean
B the median
C the mode
D all of the above
16 A small company has four employees who each earn $397.50 per week. Later, a
manager is employed who earns $1645.00 per week.
a Calculate the mean, median and mode wages.
b What effect does the managers wage have on the:
i mean?
ii median?
iii mode?
c A wage debate is conducted with the employees asking for a rise. Would the mean,
median or mode be quoted:
i in support of a wage rise by the employees?
ii against a wage rise by the employer?
Explain your answers.

140

Maths Quest General Mathematics HSC Course

Skewness

8
7
6
5
4
3
2
1
0

Frequency

Frequency

Frequency

By looking at a graph, we can make judgements


about the nature of a data set. Consider the first
graph shown on the right.
This graph is symmetrical and we can see that
the mean, median and mode are all equal to 3. The
1 2 3 4 5
majority of scores are clustered around the mean.
This is an example of a normal distribution.
We can compare the standard deviation of data
6
sets by looking at such graphs. The more clustered
5
the data set, the smaller the standard deviation.
4
The second graph is still normally distributed
3
with the mean, median and mode still equal to 3.
2
1
However, there are more scores which are further
0
away from the mean and, hence, the standard
1 2 3 4 5
deviation of the data set is greater.
The third graph shows a data set where the
scores are not clustered and there are two modes
6
at either end of the distribution.
5
In this example, although it is still symmetrical
4
there are two modes, 1 and 5, while the mean and
3
2
median are still 3. The standard deviation in this
1
distribution is greater than either of the two pre0
vious examples as there are more scores further
1 2 3 4 5
away from the mean.
The mean and median can be seen from the graph only because it is symmetrical.

The figure on the right shows the distribution of


a set of scores on a spelling test.
a Is the graph symmetrical?
b What is the mode(s)?
c Can the mean and median be seen from the
graph?

Frequency

WORKED Example 5
5
4
3
2
1
0
6 7 8 9 10
Score

THINK

WRITE

a The columns either side of the middle


are equal.

a The graph is symmetrical.

b The scores that occur the most often are


7 and 9.

b Mode = 7 and 9

c The middle score will be the mean and


median.

c Mean = 8, median = 8

Chapter 4 Interpreting sets of data

141

8
7
6
5
4
3
2
1
0

Frequency

Frequency

When a graph is not symmetrical, the mean and median cannot be easily seen from the
graph. Consider the distribution in the graph below left.
The way in which the data are gathered to one end of the distribution is called the
skewness. A greater number of scores are distributed at the lower end of the distribution.
In this case, the data are said to be positively skewed. Similarly, when most of the scores
are distributed at the upper end, the data are said to be negatively skewed, as shown in
the graph below right.

1 2 3 4 5

8
7
6
5
4
3
2
1
0

1 2 3 4 5

20
16
12
8
4
0
51
6
61 0
7
71 0

81 80
91 90
1
00

The distribution on the right shows the results of the


Maths Trial HSC at a certain school.
a What is the modal class?
b Describe the skewness of the data set shown
on the right.

Frequency

WORKED Example 6

Maths results

THINK

WRITE

a The class occurring the most often is the


8190 class.

a Modal class = 81

b The majority of data are at the upper end


of the distribution.

b The data are negatively skewed.

90

remember
1. A distribution is symmetrical when the data are equally distributed around the
mean.
2. When the data are symmetrical, the median and mean will both be the middle
score.
3. When the data are clustered around the mean, the standard deviation is smaller.
4. When the majority of scores are at the lower end of a distribution, it is said to
be positively skewed.
5. When the majority of scores are at the upper end of the distribution, it is said to
be negatively skewed.

Maths Quest General Mathematics HSC Course

4B
WORKED

Example

Skewness
Frequency

142

1 In the distribution on the right:


a is the graph symmetrical?
b what is the modal class(es)?
c can the mean and median be seen from the
graph? and, if so, what are their values?

12
10
8
6
4
2
0
1 2 3 4 5
7
6
5
4
3
2
1
0
0
4
5
10 9

15 14
1
20 9

25 24
2
9

Frequency

2 For the distribution shown on the right:


a are the data symmetrical?
b what is the modal class(es)?
c can the mean and median be seen from the
graph? and, if so, what are their values?

Example

Frequency

4 For the distribution shown on the right:


a what is the modal class(es)?
b describe the skewness of the distribution.

12
10
8
6
4
2
0
0
1
1
2
2
3
3
4
4
5

WORKED

No. of goals

Frequency

3 The table on the right shows the number of


goals scored by a hockey team throughout
a season.
a Show this information in a frequency
histogram.
b Are the data symmetrical?
c What is the mode(s)?
d Can the mean and median be seen for
this distribution? and, if so, what are their
values?

5 For each of the following dot plots describe the skewness of the distribution.
a
b
c

0 1 2 3 4 5


14 15 16 17 18 19 20

9.5 9.6 9.7 9.8 9.9 10

6 For the stem-and-leaf plots drawn below describe the distribution


a Key 3|5 = 35
b Key 4|3 = 4.3 4*|6 = 4.6
Stem Leaf
Stem Leaf
2 259
2* 9
3 0012589
3* 0 4
4 2289
3* 5 5 8 8
5 09
4* 0 0 0 1 1 3 4 4
6 0
4* 5 5 5 6 7 8 8 9 9

Chapter 4 Interpreting sets of data

7 The table below shows the number of


goals scored by a basketball team
throughout a season.
No. of goals

Frequency

1120

2130

3140

4150

23

5160

21

a Draw a frequency histogram of the


data.
b Describe the data set in terms of its
skewness.
8 multiple choice
Which of the distributions below has the smallest standard deviation?

Frequency

B
Frequency

10
8
6
4
2
0

1 23 4 5

6
5
4
3
2
1
0

12345

6
5
4
3
2
1
0
1 23 4 5

Frequency

Frequency

8
7
6
5
4
3
2
1
0
1 2 3 4 5

The distribution represented by the graph on the


right is:
A positively skewed
B negatively skewed
C symmetrical
D normally distributed

Frequency

9 multiple choice
16
14
12
10
8
6
4
2
0

1 2 3 4 5

143

144

Maths Quest General Mathematics HSC Course

10 A movie is shown at a cinema 30 times during the week. The number of people
attending each session of the movie is shown in the table below.

a
b
c
d

No. of people

Frequency

150

51100

101150

151200

10

201250

10

Present the data in a frequency histogram.


Are the data symmetrical?
What is the modal class(es)?
Describe the skewness of the distribution.

11 Year 12 at Wallarwella High School sit exams in Chemistry and Maths. The results are
shown in the table below.
Mark

Chemistry

Maths

3140

4150

5160

6170

7180

8190

91100

Work

a Is either distribution symmetrical?


b If either distribution is not symmetrical, state whether it is positively or negatively
skewed.
c State the mode of each distribution.
d In which subject is the standard deviation greater? Explain your answer.
T
SHEE

4.1

12 Draw an example of a graph which is:


a symmetrical
b positively skewed with one mode
c negatively skewed with two modes.

Chapter 4 Interpreting sets of data

145

1
Consider the following set of scores: 23, 45, 24, 19, 22, 16, 16, 27, 20, 21. Calculate the
following measures of location and spread.
1 Mean
2 Median
3 Mode
4 Range
5 Interquartile range
6 Standard deviation
7 For the data set, describe the skewness of the distribution.
8 Does the data set have an outlier?
9 Which measure of central tendency is the best measure of location in this data set?
10 Explain why the interquartile range is a better measure of spread than the range.

Displaying multiple data sets


Two data sets can be compared using a number of the displays that have been studied
in earlier parts of this course. Presenting both sets of data on the same display gives a
quick and easy comparison.

Stem-and-leaf plots
Two sets of data can be displayed on the same stem-and-leaf plot. This is done by
having the stem in the centre of the plot, with both sets of data back to back.

WORKED Example 7

The data shown below display the marks of 15 students in both English and Maths.
English: 45 67 81 59 66 61 78 71 74 91 60 49 58 62 70
Maths: 85 71 49 66 64 68 75 71 69 60 63 80 87 54 59
Display the data in a back-to-back stem-and-leaf plot.
THINK
1
2
3
4

WRITE

Write a key at the top of the


stem-and-leaf plot.
Draw the stem showing categories of
10 in the centre of the page.
Display the information for English on
the left of the stem.
Display the information for Maths on
the right of the stem.

Key: 4 5 = 45
English
95 4
98 5
76210 6
8410 7
1 8
1 9

Maths
9
49
034689
115
057

This stem-and-leaf plot allows for both distributions to be easily seen, and for a judgement on the skewness of the distribution to be made.

146

Maths Quest General Mathematics HSC Course

Box-and-whisker plots
A single scale can be used. Draw two box-and-whisker plots on that scale that will
allow the comparison of the median, range and interquartile range of two distributions
to be compared.

WORKED Example 8
Use the back-to-back stem-and-leaf plot drawn in worked example 7 to:
a calculate the median of each distribution
b calculate the range of each distribution
c calculate the interquartile range of each distribution
d draw a box-and-whisker plot of each distribution on the same scale.
THINK

WRITE

a The median will be the eighth score in


each distribution.

a English median = 66
Maths median = 68

b To calculate the range of each


distribution, subtract the lowest score
from the highest score.

b English range = 91 45
English range = 46
Maths range = 87 49
Maths range = 38

The lower quartile will be the fourth


score.
The upper quartile will be the
twelfth score.
The interquartile range is the
difference between the quartiles.

c English lower quartile = 59


Maths lower quartile = 60
English upper quartile = 74
Maths upper quartile = 75
English interquartile range = 74 59
English interquartile range = 15
Maths interquartile range = 75 60
Maths interquartile range = 15

Draw a scale.
Draw the English box-and-whisker
plot.
Draw the Maths box-and-whisker
plot.

1
2
3

1
2
3

English
Maths

0 10 20 30 40 50 60 70 80 90 100 Scale

Examining exam results


Collect data on the most recent exam that has been done in your class.
1 Display the data in a stem-and-leaf plot.
2 Find all the information needed to display the data in a box-and-whisker plot.
3 Is there any skewness evident in the data?
4 Which measure of location best describes the typical score in this data set?

Chapter 4 Interpreting sets of data

147

Graphics Calculator tip! Storing multiple data sets


Your graphics calculator can store two sets of data at the same time. Consider worked
example 8, which uses the data from worked example 7. By graphing each data set
using a box-and-whisker-plot we can also easily retrieve the rest of the required information by using the trace function.
1. From the MENU select STAT.

2. Enter the data for English in List 1 and the data for
Maths in List 2.

3. Press F1 (GRPH), and then F6 (SET). We will set


the English data as GPH1 and the Maths data as
GPH2. Press F1 (GPH1) and enter the settings
shown in the screen at right.

4. Press EXE once these settings have been entered.


Press F6 (SET) and F2 (GPH2), and again enter
the settings shown at right.

5. To show both graphs on the same screen press


EXE after entering the settings above, press F4
(SEL), and set both StatGraph1 and StatGraph2 to
DrawOn as shown.

6. Press F6 (DRAW) to draw both graphs.

7. Press SHIFT F1 (TRACE), and use the arrow keys


to move around to the five key points on each graph.
The screen at right displays the median for the Maths
data.

148

Maths Quest General Mathematics HSC Course

Multiple sets of data can be displayed on the same set of axes for many different
graphs. This is particularly useful when comparing data suitable for display on a radar
chart.

WORKED Example 9

The table below shows the number of admissions to two hospitals, each month, over a
one-year period. Display both sets of data on a radar chart.
Month

Hospital A

Hospital B

January

15

February

12

March

April

10

May

10

June

15

July

14

August

16

September

10

October

November

December

THINK
1
2
3

Draw the radar with a 30 angle


between the months.
Draw a scale around the radar.
Plot each set of points.

WRITE
Hospital A

Jan
Dec 20
15
Nov
10
5
Oct
0

Hospital B

Feb
March
April

Sep

May
Aug

July

June

Chapter 4 Interpreting sets of data

149

Area charts are another method of comparing information. In an area chart, line
graphs are stacked on top of each other, thus allowing the area between each line graph
to serve as the comparison between the data sets.

WORKED Example 10
The table below shows the amount of rainfall, in millimetres, in Sydney, Melbourne and
Brisbane each month throughout a year.

Sydney
Melbourne
Brisbane

January

February

March

April

May

June

103

117.1

133.7

126.6

120.4

131.7

49

47.7

51.8

58.4

57.2

50.2

159.6

158.3

140.7

92.5

73.7

67.8

July

August

September

October

November

December

Sydney

98.2

79.8

69.9

77.5

83.1

79.6

Melbourne

48.7

50.6

59.4

67.7

60.2

59.9

Brisbane

56.5

45.9

45.7

75.4

97

133.3

Show this information in an area chart.

Draw a pair of axes. The vertical axis will need


to be at least the rainfall total of all three cities
in the wettest month.
Draw a line graph of Sydneys rainfall, and
shade the area below it.
Add Melbournes rainfall to Sydneys rainfall,
and draw a line graph showing these figures.
Colour the area between the two graphs, as this
area represents Melbournes rainfall.
Add Brisbanes rainfall to the previous total.
Colour the area above the previous line, as this
area represents Brisbanes rainfall.

Brisbane
Rainfall (mm)

WRITE

Melbourne

Sydney

350
300
250
200
150
100
50
0
Ja
n
Fe
b
M
a
Apr
Mr
ay
Ju
n
Ju
Au l
g
Se
p
Oc
No t
v
De
c

THINK

remember
1. Data can be compared by showing two sets of data on the same display.
2. Two sets of data are shown on a stem-and-leaf plot by displaying the data back
to back.
3. Two box-and-whisker plots can be drawn on the same scale to compare the
ranges, interquartile ranges and medians.
4. A radar chart can be used to compare trends over a period of time by plotting
two sets of data on one radar chart.
5. An area graph can be used to compare multiple sets of data. The area in each
section of the graph allows for comparison between quantities.

150

Maths Quest General Mathematics HSC Course

SkillS

4C
HEET

4.7

WORKED

Example

SkillS

Compiling a
stem-andleaf plot

HEET

4.8

SkillS

4.9

WORKED

Example

Drawing a
box-andwhisker plot

1 In a class of 30 students there are 15 boys and 15 girls. Their heights are measured
and are listed below.
Boys: 1.65, 1.71, 1.59, 1.74, 1.66, 1.69, 1.72, 1.66, 1.65, 1.64, 1.68, 1.74, 1.57, 1.59,
1.60
Girls: 1.66, 1.69, 1.58, 1.55, 1.51, 1.56, 1.64, 1.69, 1.70, 1.57, 1.52, 1.58, 1.64, 1.68,
1.67
Display this information in a back-to-back stem-and-leaf plot.
2 The number of points scored in each match by two rugby union teams are shown
below.
Team 1: 34, 32, 24, 25, 8, 18, 17, 23, 29, 40, 19, 42
Team 2: 23, 20, 35, 21, 46, 7, 9, 24, 27, 38, 41, 30
Display these data in a back-to-back stem-and-leaf plot.

Finding the
mean, median,
mode from a
stem-and-leaf
plot

HEET

Displaying multiple data sets

3 The stem-and-leaf plot below is used to display the number of vehicles sold by the
Ford and Holden dealerships in a Sydney suburb each week for a three-month period.
Key: 1 5 = 15
Ford
Holden
74 0 39
952210 1 111668
8544 2 2279
0 3 5
a State the median of both distributions.
b Calculate the range of both distributions.
c Calculate the interquartile range of both distributions.
d Show both distributions on a box-and-whisker plot.
4 A motoring organisation tests two different brands of tyres. Twenty tyres of each
brand are tested to find out the number of kilometres each tyre could travel before the
tread had worn down. The results are shown in the stem-and-leaf plot below.
Key: 1 2 = 12 000 km
1* 7 = 17 000 km
Brand A
Brand B
9 8 0*
43110 1 0011224
7 7 7 6 6 5 1* 5 6 7 8 8 8 9
4431100 2 0134
2* 5 5
Draw two box-and-whisker plots on the same scale to display this information.
5 The figures below show the ratings of two radio stations each week over a threemonth period.
Station A: 9.2, 9.4, 9.2, 9.5, 9.7, 9.9, 10.1, 9.1, 8.8, 8.7, 9.0, 8.5, 9.3
Station B: 8.5, 8.1, 8.2, 8.9, 9.0, 9.2, 8.4, 8.7, 8.8, 10.5, 11.2, 11.4, 8.7
a Display the information in a back-to-back stem-and-leaf plot.
b Use the stem-and-leaf plot to display both sets of data on the same box-andwhisker plot.

151

Chapter 4 Interpreting sets of data

6 The box-and-whisker plot drawn on the


Team A
right displays statistical data for two AFL
Team B
teams over a season.
50 60 70 80 90 100110120130140 150 160 Scale
a Which team had the higher median
score?
b What was the range of scores for each team?
c For each team calculate the interquartile range.
7 The two five-number summaries below show the performance of Emad and Larry on
their Mathematics exams throughout the year.
Emad: 45, 64, 68, 76, 80
Larry: 51, 58, 65, 72, 75
a Compare the performance of Emad and Larry on a box-and-whisker plot.
b What is the range for both students?
c What is the interquartile range for both students?
8 multiple choice
The box-and-whisker plot drawn on the right
Physics
shows Emmas performance in her Physics
Chemistry
and Chemistry exams. Which of the following
0 10 20 30 40 50 60 70 80 90 100 Scale
statements is correct?
A The median of Emmas mark in Physics is greater than for Chemistry.
B The range of Emmas marks in Physics is greater than in Chemistry.
C The interquartile range of Emmas marks in Physics is greater than in Chemistry.
D All of the above.
WORKED

Example

9 This radar chart shows the average daily maximum temperature


for both Sydney and Melbourne for each month of a year.
a Which month had the lowest temperature in Sydney?
b What was the range of temperatures in Melbourne?
c What was the average of the temperatures in Sydney?

Sydney temperature (C)


Melbourne temperature (C)
J
F
D 30

M
A

WORKED

Brisbane

2 pm

8 am
12 noon

Melbourne

10 am

Sydney

40
35
30
25
20
15
10
5
0

Jan
Fe
b
Ma
Apr
Ma r
y
Jun
Ju
Au l
g
Se
p
Oc
No t
Dev
c

Average no. of rainy days


per month

Supermarket X
Supermarket Y
12 midnight
2 am
10 pm 120
100
80
60
8 pm
4 am
40
20
0
6 pm
6 am
4 pm

11 This area chart shows the average number of rainy days


each month in Sydney, Melbourne and Brisbane. Display
10
this information as a table.

10
0

10 This radar chart shows the number of customers in two different


supermarkets at two-hour intervals.
a Find the range for each supermarket.
b Describe the general pattern at each supermarket.

Example

20

152

Maths Quest General Mathematics HSC Course

Computer Application 1 Displaying statistical data


EXCE

et

reads
L Sp he

1. From the Maths Quest General Mathematics HSC Course CD-ROM, access the
spreadsheet Fast Food Sales.

Fast
Food
Sales

2. In cell B12 use the spreadsheets inbuilt statistical function to find McDonalds
average daily sales. [=AVERAGE(B4:B10)]
3. In cell B13 use the spreadsheets inbuilt statistical function to find the standard deviation of McDonalds daily sales. [=STDEV(B4:B10)]
4. Under Edit, use the Fill and Right functions to copy these formulas for KFC and Pizza
Hut.
5. Use the charting facility to draw an area chart of the figures presented.

Comparison of data sets


When multiple data displays are used to display similar sets of data, comparisons and
conclusions can then be drawn about the data.
Multiple displays such as stem-and-leaf plots and box-and-whisker plots allow for
comparison of statistics such as the median, range and interquartile range, while radar
charts and area charts allow for trends and overall quantities to be compared.

Chapter 4 Interpreting sets of data

153

WORKED Example 11

A bank surveys the average morning and


Key: 1 2 = 1.2 minutes
Morning
Afternoon
afternoon waiting time for customers.
7 0 788
The figures were taken each Monday to
86311 1 1124456667
Friday in the morning and afternoon for
9
6
6
6
5
5
4331 2 2558
one month. The stem-and-leaf plot at right
952 3 16
shows the results.
5 4
a Find the median morning waiting time
5 7
and the median afternoon waiting time.
b Calculate the range for morning waiting times and the range for afternoon waiting
times.
c What conclusions can be made from the display about the average waiting time at the
bank in the morning compared with the afternoon?
THINK
a There are 20 scores in each set and so
the median will be the average of the
10th and 11th scores.

WRITE
a Morning:

b For each data set, subtract the lowest


score from the highest score.

b Morning:

Afternoon:

Afternoon:
c Conclude that waiting time in the
afternoon is generally less and more
consistent except for one outlier.

Median = (2.4 + 2.5) 2


= 2.45 minutes
Median = (1.6 + 1.6) 2
= 1.6 minutes
Range = 4.5 0.7
= 3.8 minutes
Range = 5.7 0.7
= 5 minutes

c The waiting time is generally shorter in the


afternoon. There is one outlier in the
afternoon scores which causes the range to
be larger. However, apart from this outlier
the afternoon scores are less spread.

Two-way tables can also be a meaningful way of displaying data. A two-way table
allows for two variables to be compared.

WORKED Example 12

A survey of 25 000 people is taken. The sex of each respondant is noted and whether they are
a smoker or non-smoker is also noted. The results are displayed in the two-way table below.
Males

Females

Totals

Smokers

4 125

4 592

8 717

Non-smokers

8 436

7 847

16 283

12 561

12 439

25 000

Totals

a What percentage of the females surveyed were smokers?


b What percentage of the smokers surveyed were female?
Continued over page

154

Maths Quest General Mathematics HSC Course

THINK

WRITE

a Write 4592 as a percentage of 12 439.

a Percentage of females who smoke


4592
= ---------------- 100%
12 439
= 36.9%

b Write 4592 as a percentage of 8717.

b Percentage of smokers who are female


4592
= ------------ 100%
8717
= 52.7%

The most common method, however, for comparing data sets is to compare the summary statistics from the data sets. The measures of location such as mean and median
are used to compare the typical score in a data set. Measures of spread such as range,
interquartile range and standard deviation are used to make assessments about the consistency of scores in the data set.

WORKED Example 13
Below are the scores for two students in eight Mathematics tests throughout the year.
Jane: 45, 62, 64, 55, 58, 51, 59, 62
Pierre: 84, 37, 45, 80, 74, 44, 46, 50
a Use the statistics function on the calculator to find the mean and standard deviation for
each student.
b Which student had the better overall performance on the eight tests?
c Which student was more consistent over the eight tests?
THINK

WRITE

a Enter the statistics into your calculator


and use the x function for the mean and
the n function for the standard
deviation.

a Jane: x = 57, n = 6
Pierre: x = 57.5, n = 17.4

b The student with the higher mean


performed better overall.

b Pierre performed slightly better overall, as


his mean mark was higher than Janes.

c The student with the lower standard


deviation was more consistent.

c Jane was the more consistent student, as her


standard deviation was much lower than
Pierres.

Chapter 4 Interpreting sets of data

155

remember
1. When multiple displays are used for two or more sets of data, we can compare
and contrast the data sets and determine if any relationship exists between
them.
2. A multiple stem-and-leaf plot allows for a quick comparison of medians,
ranges and interquartile ranges.
3. The summary statistics from two data sets can be compared quickly on a
box-and-whisker plot.
4. Two-way tables can be used to make a comparison of data where two variables
are involved.
5. The most commonly used comparisons are summary statistics to compare what
is a typical score and what the spread of the data is.

4D
WORKED

Example

11

Comparison of data sets

1 The stem-and-leaf plot drawn below shows the marks obtained by 20 students in both
English and Maths.
Key: 7 1 = 71
English
7410
9976653110
87752
2

4
5
6
7
8
9

Maths
17
24799
133466
4448
36
4

a Calculate the median mark for both English and Maths.


b Calculate the range of marks for both English and Maths.
c Comment on the distribution of marks in each of the subjects.
2 Tracey measures the heights of twenty Year 10 boys and twenty Year 10 girls and
produces the following five-number summaries for each data set.
Boys: 1.47, 1.58, 1.64, 1.72, 1.81
Girls: 1.55, 1.59, 1.62, 1.66, 1.73
a Draw a box-and-whisker plot for both sets of data and display them on the same
scale.
b What is the median of each distribution?
c What is the range of each distribution?
d What is the interquartile range for each distribution?
e Comment on the spread of the heights among the boys and the girls.

156

Maths Quest General Mathematics HSC Course

3 The box-and-whisker plots on the right


show the heights of a sample of Year 7
boys and a similar-sized sample of Year
12 boys.

Year 7
Year 12
1.0 1.1 1.2 1.3 1.4 1.5 1.6 1.7 1.8 1.9 2.0 m Scale

a Calculate the range of heights among both the Year 7 and Year 12 boys.
b Calculate the interquartile range of the heights among both the Year 7 and Year 12
boys.
c Comment on the relationship between the two data sets, both in terms of measures
of location and measures of spread.
4 The values of hardware and software sales for a chain
of computer stores are shown for each month in the
radar chart on the right. Comment on any relationship
observed in this chart between the sales of hardware
and the sales of software.

Hardware

N
O

D 2.5
2
1.5
1
0.5
0

Software

F
M
A

M
A

WORKED

Example

12

1200

Western region

1000
Rainfall (mm)

5 The area chart on the right shows the


rainfall in four areas of New South
Wales throughout the year.
a Which region has the greatest
rainfall?
b In which region is the range of
rainfall figures least?
c What relationship exists between
the rainfall in each of the areas?

Southern region

800
600

North/Eastern
region

400

North/Western
region

200

0
mer utumn Winter Spring
m
A
Su

6 The two-way table below shows the results of random breath testing by Sydney police
over one weekend. A driver is charged if they record a reading of 0.05% prescribed
concentration of alcohol (PCA).
Males

a
b
c
d
e

Females

Totals

Over 0.05 PCA

26

33

Below 0.05 PCA

962

743

1705

Totals

988

750

1738

What percentage of the drivers tested were female?


What percentage of the drivers tested had a PCA over 0.05?
What percentage of female drivers had a PCA over 0.05?
What percentage of male drivers had a PCA over 0.05?
Based on the above results, can any conclusion be drawn concerning the prevalence
of drink driving among males and females? Explain your answer.

Chapter 4 Interpreting sets of data

157

7 Ashley is the star player of a football team. To analyse the importance of Ashley to
the team, the coach prepares the two-way table below showing the results of games
over three years both when Ashley is playing and not playing.
Won

Lost

Totals

Ashley playing

38

42

Ashley not
playing

10

18

Totals

48

12

60

a What percentage of games were won when Ashley played?


b What percentage of games were won when Ashley did not play?
c Do you think that Ashley has a significant impact on the performance of the team?
Explain your answer.
8 To compare the performance of city and country students in the HSC, the number of
students achieving a UAI of at least 90 is studied in six city and six country high
schools.
City

Country

Totals

90

58

61

119

UAI < 90

551

569

1120

Totals

609

630

1239

UAI

a What percentage of city students achieved a UAI of at least 90?


b What percentage of country students achieved a UAI of at least 90?
c Of those students who achieved a UAI of at least 90, what percentage were from:
i the city?
ii the country?
d Based on the above results, could any conclusion be drawn about the performance
of city and country students in the HSC?
WORKED

Example

13

9 Calvin recorded his marks for each test that he did in Physics and Chemistry
throughout the year.
Physics:
65, 74, 69, 66, 72, 64, 75, 60
Chemistry: 45, 85, 91, 42, 47, 72, 87, 85
a In which subject did Calvin achieve the better average mark?
b In which subject was Calvin more consistent? Explain your answer.
10 The police set up two radar speed checks in a country town. In both places the speed
limit is 60 km/h. The results of the first 10 cars that have their speed checked are
given below.
Point A: 60, 62, 58, 55, 59, 56, 65, 70, 61, 64
Point B: 55, 58, 59, 50, 40, 90, 54, 62, 60, 60
a Calculate the mean and standard deviation of the readings taken at each point.
b At which point are drivers generally driving faster?
c At which point is the spread of the readings taken greater? Explain your answer.

158

Maths Quest General Mathematics HSC Course

11 Aaron and Sunil open the batting for the


local cricket team. The number of runs
they have scored in each innings this
season are listed below.
Aaron: 45, 43, 33, 56, 21, 38, 0, 29, 76,
40
Sunil: 5, 70, 12, 54, 68, 11, 8, 64, 32,
69
a Calculate the mean number of runs
scored for each player.
b What is the range of runs scored by
each player?
c What is the interquartile range of
runs scored by each player?
d Which player would you consider
to be the more consistent player?
Explain your answer.
12 multiple choice
Andrea surveys the age of people attending a concert given by two bands. The boxand-whisker plot shown below shows the results.
Band A
Band B

0 10 20 30 40 50 60 70 80 Scale

Which of the following conclusions could be drawn based on the above information?
A Band A attracts an older audience than Band B.
B Band A appeals to a wider age group than Band B.
C Band B attracts an older audience than Band A.
D None of the above.
13 multiple choice
Two drugs are tested to see which is more effective at fighting disease. The results are
displayed in the two-way table below.

Recovered
Not recovered
Totals

Drug 1

Drug 2

Totals

124

136

260

32

45

77

156

181

337

Of those patients who recovered, the percentage who were treated with drug 1 was:
A 46.3%
B 47.7%
C 69.0%
D 79.5%

Chapter 4 Interpreting sets of data

159

14 multiple choice

15 A company producing matches advertise that there are


50 matches in each box. Two machines are used to distribute the matches into the boxes. The results from a
sample taken from each machine are shown in the stemand-leaf plot below.
Key: 5 1 = 51
5* 6 = 56
Machine A
Machine B
4 4
9 9 8 7 7 6 6 5 4* 5 7 8 9 9 9 9 9 9 9 9
43222211100000 5 0000011111223
5 5 5* 9
a Display the data from both machines on a box-and-whisker plot.
b Calculate the mean and standard deviation of the number of matches distributed
from both machines.
c Which machine is the more dependable? Explain your answer.

Developing a two-way table


Conduct a survey to determine the number of Year 12 students at your school who
have their drivers licence. Use the results to complete the two-way table below.
Males

Females

Totals

Licensed
Unlicensed
Totals
a
b
c
d

What percentage of females have their drivers licence?


What percentage of licensed drivers are female?
Are these figures the same?
Design a data set where the percentage of females with their licence is equal to
the percentage of licensed drivers who are female.
e What conditions are necessary for these two percentages to be equal?

Work

The figures below show the ages of the mens and


womens champions at a tennis tournament.
Mens:
23, 24, 25, 26, 25, 25, 22, 23, 30, 24
Womens: 19, 27, 20, 26, 30, 18, 28, 25, 28, 22
Which of the following statements is correct?
A The mean age of the mens champions is greater
than the mean age of the womens champions.
B The range is greater among the mens champions than
among the womens champions.
C The interquartile range is greater among the mens
champions than among the womens champions.
D The standard deviation is greater among the mens
champions than among the womens champions.

T
SHEE

4.2

160

Maths Quest General Mathematics HSC Course

summary
Measures of location and spread
Measures of location give the typical score in the data set. The mean, median and
mode are measures of location.
The mean of a small data set is found using:
x = -----xn

where x = the mean, x = individual scores and n = number of scores.


Where data is in a table, the mean is found using:
fx
x = ------f
where x = the mean, x = individual scores and f = frequency.
Measures of spread describe how spread out the data are. The range, interquartile
range and standard deviation are measures of spread.
An outlier is a single score that is much greater or much less than most of the
scores. The outlier may have a great effect on the mean but has only a slight effect
on the median and no effect on the mode in a small data set. The larger the data set,
the less the effect a single outlier will have.

Skewness
When the data are symmetrical, they are said to be normally distributed.
The more clustered the data are around the mean, the smaller the standard
deviation.
When most of the data are below the mean, the data are said to be positively
skewed.
When most of the data are above the mean, the data are said to be negatively
skewed.

Displaying multiple data sets


Two sets of data can be displayed on a stem-and-leaf plot by displaying the data
back to back.
The summary statistics from two data sets can be displayed by using the same scale
and drawing two box-and-whisker plots.
Two sets of data can be displayed on a radar chart to display related trends over a
period of time.
An area chart can be drawn to display several sets of data. The area in each section
of the graph then displays the quantities for comparison.

Comparison of data sets


The summary statistics from two data sets can be compared from a stem-and-leaf
plot or box-and-whisker plot.
Two-way tables are used to compare data where there are two variables involved.
Data are most commonly compared using the mean and standard deviation.

Chapter 4 Interpreting sets of data

161

CHAPTER
review
1 Below are the ages of 15 players in a soccer squad.
23, 28, 25, 19, 17, 28, 29, 29, 22, 21, 35, 30, 22, 27, 26
a Calculate the mean age of the players in the squad.
b Find the median age of players in the squad.

4A

2 The table below shows the number of house calls that a doctor has been required to make
each day over a 32-day period.

4A

a
b
c
d

Number of house calls

Frequency

Copy the table into your workbook and add a cumulative frequency column.
Calculate the mean number of house calls per day.
Find the median number of house calls per day.
What is the modal number of house calls per day?

3 The set of figures shown below shows the number of pages in a daily newspaper every day
for two weeks.
72, 68, 76, 80, 64, 60, 132, 72, 84, 88, 60, 56, 76, 140
a What is the mean number of pages in the newspaper?
b What is the range?
c What is the interquartile range?
d Use the statistics function on your calculator to find the standard deviation.

4A

4 The table below shows the number of rescues that are made each weekend at a major beach.

4A

Number of rescues

Frequency

10

12

11

12

13

14

Use the statistics function on your calculator to find the mean and the standard deviation of
these data.

162
4A

Maths Quest General Mathematics HSC Course

5 The table below shows the customer waiting time at 10 am each morning at a bank over an
8-week period.
Waiting time

a
b
c
d

Class centre

Frequency

01 minute

12 minutes

23 minutes

10

34 minutes

13

45 minutes

56 minutes

Cumulative
frequency

Copy and complete the table.


Use the statistics function on your calculator to find the mean and standard deviation.
Draw a cumulative frequency histogram and polygon.
Use the graph to estimate the interquartile range of the data.

6 The figures below show the number of points scored by a basketball player in six matches of
a tournament.
36, 2, 38, 41, 27, 33
a Calculate the mean number of points per game.
b Calculate the median number of points per game.
c Explain why there is such a large difference between the mean and the median.

4B

7 Consider the data set represented by the frequency histogram on


the right.
a Are the data symmetrical?
b Can the mean and median of the data be seen?
c What is the mode of the data?

4B

8 The table below shows the number of attempts that 20 members of


a Year 12 class took to obtain a drivers licence.
Number of attempts

Frequency

11

Frequency

4A

8
7
6
5
4
3
2
1
0

15 16 17 18 19 20

a Show these data in a frequency histogram.


b Are the data positively or negatively skewed?

4B

9 Draw an example of a frequency histogram for which the data are negatively skewed.

Chapter 4 Interpreting sets of data

163

10 The figures below show the marks obtained by 20 students in English and Maths.
English: 56, 45, 57, 56, 65, 82, 74, 80, 91, 84, 68, 52, 67, 64, 60, 66, 74, 77, 77, 66
Maths: 65, 66, 58, 60, 61, 70, 74, 66, 69, 68, 71, 55, 51, 49, 50, 71, 99, 85, 70, 66
a Display the data in a back-to-back stem-and-leaf plot.
b For each subject find the median.
c For each subject state the range.
d For each subject find the interquartile range.

4C

11 Betty runs a surf and ski shop. The table below shows
the monthly sales of both types of equipment.

4C

Month

Surf sales ($)

Ski sales ($)

January

20 000

5 000

February

18 000

6 000

March

12 000

8 000

April

9 000

10 000

May

6 000

12 000

June

4 000

12 000

July

5 000

9 000

August

8 000

8 000

September

10 000

6 000

October

11 000

3 000

November

15 000

4 000

December

22 000

9 000

a Display both sets of data on the same radar chart.


b Use the chart to compare trends in the sales.
12 The data below give the cost per minute of a long-distance telephone call with three
companies.
Telecomm

Omtus

Tel One

Day

21c

25c

17.5c

Economy

18c

15c

17.5c

Night

12c

12c

17.5c

Display this information in an area chart.

4C

164

Maths Quest General Mathematics HSC Course

4D

13 The stem-and-leaf plot below compares the crowds (correct to the nearest thousand) at a
football teams home and away matches.
Key: 2 5 = 25 000
Home
Away
8 0 67
732 1 0116899
6632 2 45
552 3
a Calculate the median of both data sets.
b Calculate the range of both data sets.
c Calculate the interquartile range of both data sets.
d Display both sets of data on a box-and-whisker plot.

4D

14 The figure on the right shows a


2003
box-and-whisker plot showing
2004
the average number of weekly
car sales made in 2003 and
0 1 2 3 4 5 6 7 8 9 10 11 12 13 14 15 Scale
2004.
a What was the median for each year?
b In which year was the range of sales greatest?
c In which year was the interquartile range of sales greatest?
d In which year did the car yard perform better? Explain your answer.

4D

15 The two-way table below compares the number of men and women who are right- and lefthanded.

Right-handed
Left-handed
Totals

Men

Women

Totals

158

172

330

17

15

32

175

187

362

a What percentage of males are left-handed?


b What percentage of females are left-handed?
c Based on the above data, is there any significant difference between the percentage of
male and female left-handers?

4D

16 Hsiang compares her marks in 10 English exams and 10 Maths exams.


English: 76, 74, 80, 77, 73, 70, 75, 37, 72, 76
Maths: 80, 56, 92, 84, 65, 58, 55, 62, 70, 71
a Calculate Hsiangs mean mark in each subject.
b Calculate the range of marks in each subject.
c Calculate the standard deviation of marks in each subject.
d Based on the above data, in which subject would you say that Hsiang performs more
consistently?

Chapter 4 Interpreting sets of data

165

Practice examination questions


1 multiple choice
The table below shows the number of patients seen each day by a local doctor.
No. of patients

Frequency

12

13

14

15

15

23

16

18

17

13

Which of the following statements are correct?


A The range of the data is 20.
B The mean of the data is 15.05.
C The standard deviation of the data is 1.34.
D The median of the data is 15.
2 multiple choice
The data below show the number of people that live in each house in a small street.
4, 4, 5, 3, 2, 5, 11, 2
The outlier in this data set has:
A the greatest effect on the mean.
B the greatest effect on the median.
C the greatest effect on the mode.
D an equal effect on the mean, median and mode.
3 multiple choice

Frequency

The two data sets below show the number of goals scored in 15 matches by two soccer teams.
Manchester: 0, 2, 1, 2, 1, 6, 0, 0, 1, 5, 0, 0, 1, 1, 1
Liverpool:
5
4
3
2
1
0

0 1 2 3 4 5
Number of goals

Which of the following statements is correct?


A The Manchester data are negatively skewed, while the Liverpool data are positively
skewed.
B The Liverpool data are negatively skewed, while the Manchester data are positively
skewed.
C Both sets of data are positively skewed.
D Both sets of data are negatively skewed.

166

Maths Quest General Mathematics HSC Course

4 The two-way table below shows the number of men and women who work in excess of
45 hours per week.
Men

Women

Totals

45 hours

132

128

260

> 45 hours

69

34

103

Totals

201

162

363

The percentage of men who work greater than 45 hours per week is closest to:
A 28%
B 34%
C 51%
D 67%
5 multiple choice
The figures below show the number of attempts that the boys and girls in a Year 12 class take
to get their drivers licence.
Boys: 1, 2, 4, 1, 1, 2, 1, 1, 2, 2, 3, 1
Girls: 2, 2, 4, 2, 1, 2, 2, 3, 1, 1, 1, 2
When comparing the performance of the boys and the girls, it is found that the boys have:
A a lower mean and a lower standard deviation
B a lower mean and a higher standard deviation
C a higher mean and a lower standard deviation
D a higher mean and a higher standard deviation
6 The data below show the weekly income among ten Year 12 boys and girls.
Boys: $80, $110, $75, $130, $90, $125, $100, $95, $115, $150
Girls: $50, $80, $75, $90, $90, $60, $250, $80, $100, $95
a Calculate the median of both sets of data.
b Calculate the range of both sets of data.
c Calculate the interquartile range of both sets of data.
d Display both sets of data on a box-and-whisker plot.
e Use the statistics function on the calculator to find the mean and standard deviation of
both sets of data.
f Discuss whether the boys or girls have a more consistent average weekly income.

CHAPTER

test
yourself

7 In the week leading up to the NRL grand final, Kylie records the number of points scored by
both teams in each game throughout the season and displays the information on the stem-andleaf plot below.
Key: 1 8 = 18
Sharks
Bulldogs
8 0
84422 1 5558889
88644432200 2 0022226668889
886200 3 000222
862 4
a Find the median of both sets of data.
b Which teams scores are the more consistent?
c Describe the skewness of the Sharks scores.
d Find the mean and standard deviation of the Bulldogs scores.

Algebraic
skills and
techniques

5
syllabus reference
Algebraic modelling 3
Algebraic skills and
techniques

In this chapter
5A
5B
5C
5D
5E

Substitution
Algebraic manipulation
Equations and formulas
Solution by substitution
Scientific notation

areyou

READY?

Are you ready?

Try the questions below. If you have difficulty with any of them, extra help can be
obtained by completing the matching SkillSHEET. Either click on the SkillSHEET icon
next to the question on the Maths Quest HSC Course CD-ROM or ask your teacher for
a copy.

5.1

Substitution into a formula

1 For each of the following formulas, find the value of the subject, given the variables.
a I = Prn given that P = 5750, r = 0.08 and n = 5 (answer correct to 1 decimal place)
b A = ab given that a = 5.6 and b = 9.2
h
c A = --- (df + 4dm + dl) given that h = 60, df = 0, dm = 32 and dl = 28
3
d A = 1--- ab sin C given that a = 23.4, b = 37.1 and C = 60 (answer correct to 1 decimal place)
2

5.2

Simplifying like terms

2 Simplify each of the following expressions.


a r+r+r+r+r
b 7m + 9m 6m
d 9a + 2b 8a 7b
e 2x + 4y 3x

5.3

Multiplication using indices

3 Simplify each of the following expressions.


a r4 r6
b 6a3 3
2
4
d 5q 7q
e 12m 4m5

5.4

c 9x + 7 + 8 + 7x
f 5m 5n + 4m 3n

c 4p5 7p
f 3r2s5 9rs6

Division using indices

4 Simplify each of the following expressions.


6

4q
d -------32q

5.5

42x
c ----------x

4k
f --------32k

30m
b ------------5

d
a ----2d

56rs
e ------------7rs

c (2c2)4

Solving linear equations

6 Solve each of the following equations.


a z 42 = 76
b 4y = 96
9v
d ------ = 8
e 6(t 5) = 54
6

5.7

Raising a power to a power

5 Simplify each of the following expressions.


a (a2)4
b (4b)3

5.6

c 6w 9 = 69
f 20 + 2n = n + 54

Changing the subject of a formula

7 Write each of the following in scientific notation.


a 25 000
b 236 000 000
c 400 000

d 26 000 000 000 000

Chapter 5 Algebraic skills and techniques

169

Substitution
During the preliminary course we studied substitution. Substitution involves replacing
a pronumeral in an expression with a numerical value. There are many different types
of expressions that may need substitution.
A linear expression such as 3x + 5 involves no index other than 1. When graphed,
these expressions form a straight line. When performing a substitution, we write the
expression and the values of the known pronumerals, rewrite the expression having
substituted the given values, and finally calculate the value of the expression.

WORKED Example 1

h
The formula for the area of a trapezium is given by A = --- ( a + b ) , where a and b are the
2
parallel sides and h is the height. Find the area of a trapezium with parallel sides 4.2 cm
and 7.9 cm and a height of 5.1 cm.
THINK

WRITE

Write down the given expression.

Write down the variables where the values


are known.
Substitute the given values into the
formula.
Evaluate.

3
4

h
A = --- ( a + b )
2
a = 4.2, b = 7.9, h = 5.1
5.1
A = ------- ( 4.2 + 7.9 )
2
A = 30.855

the graphics calculator


Graphics Calculator tip! Using
to substitute
You can use a graphics calculator to assist with substitutions. Consider worked
example 1.
1. From the MENU select RUN.

2. Assign the value a = 4.2 by entering


4.2
ALPHA [A] and then pressing EXE .

3. Repeat step 2 to assign the values b = 7.9 and h = 5.1.

170

Maths Quest General Mathematics HSC Course

4. Now enter the expression H 2


press EXE .

(A + B) and then

Many expressions involve higher powers or indices. An expression involving a power


of 2 is called a quadratic expression, an expression involving a power of 3 is called a
cubic expression.
Consider the expression M = 6r2. In this expression, only the pronumeral r is raised
to the power of 2. In the expression M = (6r)2 the product 6 r is raised to the power
of 2.

WORKED Example 2
The expression V = 4--3- p r 3 is used to calculate the volume of a sphere. Find the volume of
the sphere with a radius of 4.2 cm, giving your answer correct to 1 decimal place.
THINK

WRITE
4
--3

r3

Write down the given expression.

V=

Write down the variables where the values


are known.
Substitute the given values into the
formula.
Evaluate.

r = 4.2

3
4

V=

4
--3

( 4.2 ) 3

V = 310.3 cm3

Other expressions may involve taking square roots and cube roots. Care must be taken
to use the calculator correctly. The square or cube root must be taken of the entire part
of the expression that is under the root sign. This may involve using brackets.

WORKED Example 3
3V
------- is used to calculate the radius of a sphere, given the volume.
4
Find the radius of a sphere with a volume of 200 cm3. (Give your answer correct to
1 decimal place.)
The expression r =

THINK
1

Write down the given expression.

Write down the variables where the


values are known.

Substitute the given values into the


formula.
Evaluate.

WRITE
3V
------4
V = 200
r=

r=

3 200
-----------------4

r = 3.6 cm

Chapter 5 Algebraic skills and techniques

171

remember
1. Substitution involves replacing a pronumeral or pronumerals in an expression
with numerical values.
2. Linear expressions involve only powers of 1.
3. Quadratic and cubic expressions involve powers of 2 and 3 respectively. In
these expressions be sure to raise only the relevant part of the expression to the
power.
4. Expressions that involve square and cube roots must be solved by correctly
using a calculator and brackets.

5A
Example

5.1

2 Find the value of each of the following by substituting into the formula.
a A = 1--- bh, if b = 5 and h = 12.3
2

Substitution
into a
formula
L Spre
XCE ad

Substitution

3 The formula P = 2l + 2w is used to find the perimeter of a rectangle. Use the formula
to find the perimeter of a rectangle, where l = 3.5 and w = 9.7.
4 The formula C = p d is used to calculate the circumference of a circle. Use the formula
to find the circumference of a circle with a diameter of 9.5 m. Give your answer
correct to 1 decimal place.
WORKED

Example

5 In the formula A = 6s2, find the value of A when s = 5.5.


6 Find the value of each of the following by substituting into the formula. (Give your
answers correct to 2 decimal places.)
a V = r2h, if r = 0.75 and h = 2.5
b A = p (R2 - r2), if R = 2.2 and r = 1
c V = 4--- p r 3, if r = 3.2
3
2

d P = I R, where I = 0.6 and R = 230


e E = 1--- mv2, where m = 23 and v = 4.7
2

7 The formula A = 2p r 2 + 2p rh is used to calculate the surface area of a cylinder.


Calculate the surface area of a cylinder with a radius of 2.3 cm and a height of 6.4 cm.
Give your answer correct to the nearest whole number.

WORKED

Example

8 Use the formula S = ut + 1--- at2 to calculate the value of S, when u = 9, t = 5 and
2
a = 4.5.
A
9 Use the formula r = ------ to find the value of r (correct to 1 decimal place) when
4
p
A = 500.

sheet

PRT
b A = P + ----------- , if P = 2000, R = 6.55 and T = 2.5
100
c S = 2(lw + lh + wh), where l = 3, w = 2.5 and h = 1.1
d V = u + at, where u = 20, a = 4 and t = 25
e T = a + (n - 1)d, if a = 66, n = 56 and d = -4

SkillS
HEET

1 In the formula A = l b, find the value of A, given that l = 6.7 and b = 4.8.

WORKED

Substitution

172

Maths Quest General Mathematics HSC Course

10 Find the value of each of the following by substitution into the formula. Where
necessary, give your answer correct to 1 decimal place.
a c =

a 2 b 2 , when a = 17 and b = 8

b T = 2

3V
------- , if V = 600 and h = 25
h

c S =
d r =
e m =

L
--- , when L = 65 and g = 9.8
g

3V
------- , if V = 900
4
2xy + y 3 , when x = 2 and y = 3

A
--- is used to calculate the side
6
length of a cube having been given the area. Calculate
the side length of a cube with a surface area
of 162.24 cm2.

11 The formula s =

12 The formula A = s ( s a ) ( s b ) ( s c ) can be used to find the area of any triangle,


where a, b and c are the side lengths and s is half the perimeter of the triangle. Given
that the side lengths of a triangle are 4 cm, 8 cm and 9 cm:
a+b+c
a calculate the value of s s = --------------------2
b find the area of the triangle, correct to 1 decimal place.

Algebraic manipulation
Basic manipulation of algebraic expressions was covered in the preliminary course. We
need to be able to add and subtract algebraic expressions as well as multiply and divide
them. Algebraic expressions are added and subtracted by collecting like terms. Only the
same pronumeral or combination of pronumerals can be added together.

WORKED Example 4

Simplify each of the following.


a 8x + 2x 11x
b 9a2 + 2a + 4a2 7a
THINK

WRITE

a Each term uses the same pronumeral and


so we add and subtract the coefficients.

a 8x + 2x

b 9a2 + 2a + 4a2 7a
= (9a2 + 4a2) + (2a 7a)
= 13a2 5a

Rewrite the expression, grouping the


like terms together. Remember that
a2 and a are not like terms.
Complete each addition and
subtraction separately.

11x = x

Chapter 5 Algebraic skills and techniques

173

To multiply and divide algebraic expressions, we need to remember the index laws
covered in the preliminary course.
First Index Law:

ax

ay = ax + y

Second Index Law: a x a y = a x


Third Index Law:

ax
or -----y = a x y
a

(a x) y = a xy

When multiplying and dividing algebraic expressions it is important to remember to


apply the index laws separately to each pronumeral.

WORKED Example 5
Simplify each of the following fully.
48 p 2 q 4
a 6m7 7m3 b ----------------c (5x4)3
6 pq 3
THINK

WRITE

a Multiply the coefficients and add the indices.

a 6m7

b Divide the coefficients and subtract the indices


for each pronumeral separately.

48 p 2 q 4
b ------------------ = 8 pq
6 pq 3

c Calculate 53 and multiply the indices.

c (5x4)3 = 125x12

7m3 = 42m10

The manipulation of algebraic expressions will also involve the expansion of brackets.
When expanding brackets, we multiply every term inside the brackets by the term
immediately outside the brackets.

WORKED Example 6
Expand 2x3(6xy

9y4).

THINK

WRITE

Multiply both terms inside the brackets by 2x3.

2x3(6xy

9y4) = 12x4y

18x3y4

remember
1. Algebraic expressions are added and subtracted by collecting like terms.
2. Algebraic expressions are multiplied and divided using the index laws.
First Index Law:
ax ay = ax + y
ax
Second Index Law: a x a y = a x y or -----y = a x y
a
Third Index Law:
(a x) y = a xy
3. When using the index laws, apply each law to each pronumeral separately.
4. When expanding brackets, multiply every term inside the brackets by what is
immediately outside.

174

Maths Quest General Mathematics HSC Course

5B
SkillS

HEET

5.2

1 Simplify each of the following expressions.


a 4a + 8a
b 14b + 4b
4a
d 35d + 6d
e 5e - e
g g - 8g
h -4h + 9h
j 7j - 5j + 9j
k -3k + 8k - k

WORKED

Example

Simplifying
like terms

SkillS

HEET

5.3

et

EXCE

c
f
i
l

23c - 9c
16f - 15f
-2i - 7i
5l - 15l + 8l

2 Simplify each of the following expressions.


a 8m + 3m - 9
b n - 4 + 7n
4b
d 5r + 9s - 2r + 2s
e 7t + 1 - 4t - 7
g 4w2 - 3w3 + 2w2 - w3
h 2xy + 4xz - 3xy + xz
j 4x + 3y - 2xy + 6x - 4yx + y

c 7p2 + 6p + 3p2 - 2p
f 6u - 8v - 7u + 2v
i 4p2 - 12 + p2 - 4

3 Simplify each of the following.


a a5 a8
b b b3
5a
d d3 7d
e 4p4q3 3p5q2
4
5
g 4mn 7m n
h 4p5 5q4
3 3
2 4
5
j 6u v 4v w 2uw

c 3c2 4c5
f 7gh 9g2h3
i 6xyz 4x2y2

WORKED

Example

Multiplication
using indices

reads
L Sp he

Algebraic manipulation

WORKED

Example

Index
laws

4 Simplify each of the following.


a k4 k
b 15m7 5m2
5b
14x 5
56m 4 n 3
d ----------e -----------------7x
7m 2 n
g m6n7 mn
h 48p3 6q3
3 4 6
2
2 2
j 32p q r 4pqr 2p q

WORKED

SkillS

Example

HEET

5.4

SkillS

Division
using
indices
HEET

5.5

5 Simplify each of the following.


a (a3)4
b (2b4)2
5c
2 3 2
d (4x y )
e (2pq2)4

Work

5.1

45x 5
----------9
i 121a 11b
f

WORKED

Example

6 Expand each of the following.


a 2(m + 5)
b x(x + 2)
6
d 3q2(6q4 - 2)
e 5n(m - 5n)
g -3(d + 5)
h -3m(m - 2n)
j 6pqr(3pq - r)

c (3m2)3

WORKED

Example

Raising
a power
to a power

T
SHEE

c 48n7 8n2

c 3a(3a + 2b)
f 7a2b4(2a4 - 3b6)
i -6r3(2 - 3r3)

7 Expand and simplify each of the following.


a 4(x + 2) + 2(3x - 1)
b a(a + 7) + 2(3a - 5)
d 5(4x - 7) - 2(x + 5)
e 2p(2p - 5) - 5(p - 6)

c 2m(m - n) + 6n(m - 2n)


f 2xy(3x - 4y) - x(y - xy)

8 Fully simplify each of the following.


a a 4 a 5 a2
b (m2)3 m4 m

c 32m2n3 4mn 2n3

d 4x6y7 5xy4 2x6

e (2z3)4 8z5 2z7

9m 2 n 4 4mn 2
----------------------------------( 6mn 3 ) 2

Chapter 5 Algebraic skills and techniques

175

1
a
1 Calculate the value of S = ----------- , when a = 8 and r = 0.2.
1r
2 Calculate the value of S = ut + --1- at2, when u = 4.5, t = 6.1 and a = 4.
2

3 Calculate the value of S =

3V
------- , when V = 352.6 and h = 4.5. (Give your answer
h

correct to 1 decimal place.)


Simplify the following expressions.
4 6x

7x + x

5 4a + 2b
6 3b4

3a

5x
8b

5b2

42g 3 h 4
7 ---------------7h 2
8 (5p3q4)2
9 5x(2
10 3a(2a

x)
5b)

4b(a

6b)

Equations and formulas


During the preliminary course we studied equations. The object of solving an equation
was to find the value of an unknown pronumeral that made that statement true. In
solving the equation we reversed every process that had been performed on the
pronumeral until it became the subject of the equation. In many cases, an equation
arose as the result of substitution into a formula.

WORKED Example 7

In the formula C = 2 r, find the value of r when C = 100, correct to 2 significant figures.
THINK
1
2
3

Write the formula.


Substitute the value of C.
Divide each side by 2 and round the
answer off to 2 significant figures.

WRITE
C=2 r
100 = 2 r
100
r = --------2
= 16

176

Maths Quest General Mathematics HSC Course

equations that arise


Graphics Calculator tip! Solving
from substitution
You can use the equation solver function on a graphics calculator to solve equations
that arise as the result of a substitution. Consider worked example 7.
1. From the MENU select EQUA.

2. Press F3 (Solver).

3. Delete any existing equation, and enter the equation


that arises after the substitution is made. To enter
100 = 2 r press 1
0
0
SHIFT [=]
2 SHIFT [ ] ALPHA [R].
Note: At this stage you may have a different value of
R, but this is to be ignored.
4. Press F6 to solve the equation.

Some equations involve powers and roots. In the solution to an equation, remember
that the opposite function to taking a square is to take the square root and vice versa.
When solving such an equation, both the positive and negative square roots are possible
solutions.
For example, the equation x2 = 9 has the solution x = 3. This differs from 9 ,
which equals 3. The square root symbol indicates to take the positive square root only.

WORKED Example 8
In the equation d = 5t2, find the value of t when d = 320.
THINK
1
2
3
4

Write the formula.


Substitute the value of d.
Divide each side by 5.
Take the square roots of each side,
considering both the positive and
negative answers.

WRITE
d = 5t2
320 = 5t2
t2 = 64
t = 8

Chapter 5 Algebraic skills and techniques

177

Note: If you use the solver function on your graphics calculator, only the positive solution is given. It is important that you remain aware that equations of this type have a
positive and negative solution.
With such examples, we consider both the positive and negative cases only where
appropriate. In practical cases where measurements are being considered, only the
positive answer is given.
Using the same process as this we can change the subject of a formula. The subject
of the formula is the single pronumeral usually written on the left-hand side of the
formula. For example, in the formula A = r 2, A is the subject. We are able to make
another pronumeral the subject of the equation by moving all other numbers and
pronumerals to the other side of the formula, as if we were solving an equation.
Formulas that need the subject changed include those with both linear and quadratic
terms.

WORKED Example 9
Make x the subject of the formula y = 5x
THINK

2.
WRITE

Write the equation.

y = 5x

Add 2 to each side.

y + 2 = 5x

Divide each side by 5 (and write the


new subject of the formula on the
left-hand side).

y+2
x = -----------5

This method is also used for quadratic formulas but, as with equation solving, we must
remember to use both the positive and negative square root where appropriate.

WORKED Example 10
The formula A = 4 r 2 is used to find the surface area of a sphere. Make r the subject of the
formula.
THINK
1

Write the formula.

Divide both sides by 4 .

Take the square root of each side.


As r is the radius, a length, we consider
only the positive square root.

WRITE
A = 4 r2
A
------ = r 2
4
r=

A
-----4

178

Maths Quest General Mathematics HSC Course

remember
1. An equation can be formed after substitution into a formula.
2. When solving an equation, the object is to find the value of the unknown.
3. When an equation involves taking a square, the opposite function used to solve
the equation is a square root.
4. Both the positive and negative square root should be taken unless the context of
the equation means that only the positive should be used.
5. To make another pronumeral the subject of an equation, the same methods as
for equation solving are used although we use pronumerals rather than make
actual calculations.

SkillS

5C
HEET

5.6

WORKED

Example

SkillS

Solving
linear
equations
HEET

Equations and formulas

1 The formula C = d is used to calculate the circumference of a circle. Find the


diameter of a circle that has a circumference of 40 cm. Give your answer correct to
3 significant figures.
2 The formula P = 2l + 2w is used to calculate the perimeter of a rectangle. Calculate
the length of a rectangle that has a perimeter of 152 m and a width of 38 m.

5.7

3 In each of the following, find the value of the unknown after substitution into the
formula. Where appropriate, give your answer correct to 1 decimal place.
h
a A = --- ( a + b ) ; find h when A = 145, a = 15 and b = 25.
2
b A = l w; find w when A = 186 and l = 15.
c V = r 2h; find h when V = 165.2 and r = 3.6.
d T = a + (n 1)d; find n when T = 260, a = 15 and d = 11.
e v2 = u2 + as; find s when v = 5.5, u = 2.4 and a = 1.2.

Changing
the subject
of a formula

WORKED

Example

4 In the formula A = 6s2, find the value(s) of s when A = 150.


5 The formula A = r 2 is used to calculate the area of a circle. Find the radius of a
circle, correct to 2 decimal places, given that the area of the circle is 328 cm2.
6 Substitute into each of the formulas and solve the equation to find the value of the
unknown. Where necessary, give your answer correct to 2 decimal places.
a V = r2 h; find r when V = 1.406 25 and h = 2.5.
b A = (R2 r2); find R when A = 12 and r = 1.
c V=

4
--3

r 3, find r when V = 136.

d E = --1- mv2; find v when E = 254 and m = 23.


2

e P = I 2R; find I when P = 0.54 and R = 1.5.


WORKED

Example

7 Make x the subject of the formula y = 2x + 1.


8 Make l the subject of the formula A = l

b.

179

Chapter 5 Algebraic skills and techniques

h
9 In the formula A = --- ( a + b ) :
2
a make a the subject of the formula
b make h the subject of the formula.
WORKED

Example

10 Make r the subject of the formula A = r 2.

10

11 In the formula E = mc2:


a make m the subject of the formula
b make c the subject of the formula.
Questions 12 to 14 refer to the following information.
The volume of a square-based pyramid with the side of the base, s, and the height, h, is
given by the formula V = 1--- s2h.
3

12 multiple choice
The side length of the base of a square-based pyramid with the height, h, and volume,
V, is given by:
V
A s = 3 ---h

B s =

h
------3V

V
C s = -----3h

D s =

3V
------h

13 multiple choice
The height of a square-based pyramid with the side of the base 5 cm and the volume
75 cm3 is:
A 8 cm
B 9 cm
C 10 cm
D 12 cm
14 multiple choice
If both the side of the base and the height are doubled the volume is:
A doubled
B increased by 4 times
C increased by 6 times
D increased by 8 times
15 In each of the following, make the subject of the formula the pronumeral indicated in
brackets.
a V = r 2 [r]
d T = 2

L
--g

[L]

b v2 = u2 + as

[u]

e c2 = a2 + b2

[a]

c V=

4
--3

r 3 [r]

180

Maths Quest General Mathematics HSC Course

Solution by substitution
From our earlier work on equations, we have found that substituting the solution back
into the original equation can check the answer to the equation. If the solution is correct, then the value that is substituted will satisfy the equation. For example, consider
the following equation.
4x 5 = 19
4x = 24
x=6
Substituting x = 6 into 4x + 5 = 19
Left-hand side (LHS) = 4 6 5
Left-hand side (LHS) = 19
Left-hand side (LHS) = Right-hand side (RHS)
By substitution we can see that x = 6 is the correct solution to this equation.
Some more difficult equations can have an approximate solution found by substituting a first guess into the equation and gradually refining the solution.

WORKED Example 11

Find an approximate solution to the equation 2x = 20 (correct to 1 decimal place).


THINK
1
2
3
4
5
6

WRITE

Make a first guess (x = 4) and substitute


into the equation.
As 24 < 20, make a second guess that is
greater than 4 (x = 5).
As 25 > 20, make the next estimate
between 4 and 5 (x = 4.5).
As 24.5 > 20, make the next estimate
between 4 and 4.5 (x = 4.3).
As 24.3 < 20, make the next estimate
between 4.3 and 4.5 (x = 4.4).
Since x = 4.3 gives a result closer to
20 than x = 4.4, the solution, correct
to 1 decimal place, is x = 4.3.

Test x = 4 24 = 16
Test x = 5 25 = 32
Test x = 4.5 24.5 = 22.6
Test x = 4.3 24.3 = 19.7
Test x = 4.4 24.5 = 21.1
Solution is x = 4.3.

Many such equations will arise from a practical situation such as investments.

WORKED Example 12

Terry has $1000 to invest; however, he needs $1500 to purchase the electric guitar that he
wants. If Terry invests his $1000 at 6% p.a., the amount in the account at any time can be
found using the formula A = 1000(1.06)n, where n is the number of years for which the
money has been invested. Find how long it will take (correct to the nearest year) for
Terrys $1000 to grow to $1500.

Chapter 5 Algebraic skills and techniques

THINK
1
2
3
4
5
6

181

WRITE

Write the formula.


Substitute A = 1500.
Divide both sides by 1000.
Make a first estimate for the solution
(n = 5).
As (1.06)5 < 1.5, make a second
estimate greater than n = 5 (n = 8).
As (1.06)8 > 1.5, make the next
estimate between n = 5 and n = 8 (n =
7).
The solution must be n = 7 as
(1.06)7 = 1.5, correct to 2 decimal
places.

A = 1000(1.06)n
1500 = 1000(1.06)n
1.5 = (1.06)n
(1.06)5 = 1.34
(1.06)8 = 1.59
(1.06)7 = 1.50

It will take 7 years for the $1000 to grow to


$1500 at 6% p.a

This type of question can be solved using a graphics calculator and by setting up a table
of values.

Graphics Calculator tip! Using the table function


We can use the table function on a graphics calculator to take the repetition out of solution by substitution. Consider worked example 12 above.
1. From the MENU select TABLE.

2. For Y1 enter the function 1000


EXE .

1.06^X and press

3. Press F5 (RANG) and then enter the settings shown


on the screen opposite. These determine the start and
end values and the pitch of the table. The pitch is the
increment by which x changes from the start value.
4. Press EXE to return to the previous screen, and
then press F6 (TABL). You will then need to scroll
down to find the value of Y1 that is closest to 1500.

182

Maths Quest General Mathematics HSC Course

remember

1. Equations such as 2x = 10 have no opposite operation that you can use easily.
Find an approximate solution by substitution and then improve on the first
estimate.
2. The first estimate is taken and substituted into the equation. A second estimate,
either higher or lower than the first depending on the result of the substitution,
is then taken.
3. Further estimates can then be taken by dividing the range within which we
know the solution lies.
4. Most equations of this type can be solved using the table function on a graphics
calculator.

5D
WORKED

Example

11

Solution by substitution

1 Solve the equation 2x = 100, correct to 1 decimal place.


2 Solve the equation 1.1x = 2, correct to the nearest whole number.
3 Solve the equation 0.9x = 0.5, correct to the nearest whole number.

4 The amount of time that it will take for an investment to double when invested at
5% p.a. can be calculated using the equation (1.05)n = 2. Find the value of n, correct to
12
the nearest whole number.

WORKED

Example

5 It is anticipated that the value of a house will keep pace with inflation. Judy purchased
a house in 2001 for $265 000. The future value of the house can be calculated using the
r n
formula A = P 1 + --------- , where A is the future value, P is the present value, r is the
100
inflation rate and n is the number of years. Judy wants to know how many years it will
take for the value of her property to exceed $500 000 given that the inflation rate will
average 4% p.a.
a Substitute the known values into the formula to create an equation.
b Solve the equation for n, correct to the nearest whole number.
6 The value of a computer decreases at the rate of 30% p.a. A new computer purchased
for $3000 can have its value after n years calculated using the formula V = 3000(0.7)n,
where n is the age of the computer in years. Calculate when the value of the computer
will equal $500, correct to the nearest year.
7 The distance through which an object will fall in t seconds can be calculated using the
formula d = 5t2.
a Copy and complete the table below.
t

10

d
b Calculate the length of time that it will take an object to fall 300 m, correct to the
nearest second.

Chapter 5 Algebraic skills and techniques

183

8 Kayla has 80 m of fencing in which to enclose a rectangular area for a garden.


a Copy and complete the table below.
Length

10

15

20

25

30

35

Width
Area
b What dimensions should the garden be if it is to enclose the maximum possible
area?
c The garden is to use an existing fence for one side and use the 80 m of fencing to
build the other three sides. Draw up a table to calculate the dimensions Kayla should
now build the garden to maximise the area.

Repeated enlargements
Consider the following problem. I need to enlarge a diagram on my photocopier to
twice its original size. My photocopier can enlarge to only 150% of the original.
Explain how I can make the enlargement that I need by using a repeated
enlargement.

2
1 Given that y = 25

5x, find the value of y when x = 3.

2 Given that T = 6n + 5n2, find the value of T when n = 2.


3 In the formula y =

r 2 x 2 , find y when r = 5 and x = 4.

4 Given that C = d find d, correct to 1 decimal place, when C = 400.


5 Given that d = 5t 2, find t when d = 2000.
Simplify the following expressions.
6 4a + 6b
7 4x2y3

3a

9b

5x5y

8 (4m3n4)3
18m 2 n 4
9 -----------------6mn 2
10 Use the method of substitution to solve the equation (1.1)x = 3, correct to the
nearest whole number.

184

Maths Quest General Mathematics HSC Course

Scientific notation
Scientific notation is used to express very large or very small numbers in terms of a
power of 10. It is particularly useful in branches of science such as astronomy, where
large distances are measured, or in biology, where very small measurements of
microbes are taken.
As we found in the preliminary course, numbers are written in scientific notation by
rewriting the number with a decimal point after the first significant figure. This decimal
is then multiplied by the appropriate power of 10. This power of 10 is found by
counting the number of places that the decimal point has been moved. When moving
the decimal point left, the power of 10 is positive; it is negative when moving the
decimal point to the right.

WORKED Example 13
Write each of the following in scientific notation.
a 8 000 000
b 13 400 000 000

c 0.000 034 51

THINK

WRITE

Move the decimal point after the first significant


figure.
The decimal point has been moved 6 places left.

a 8 000 000 = 8

Move the decimal point after the first significant


figure.
The decimal point has been moved 10 places left.

b 13 400 000 000 = 1.34

Move the decimal point after the first significant


figure.
The decimal point has been moved 5 places right.

c 0.000 034 51 = 3.451

106

1010

10

In many examples we are required to round such measurements off to a given number
of significant figures.

WORKED Example 14
Write each of the following measurements in scientific notation, correct to 3 significant
figures.
a 97 856 472 124 km
b 0.000 000 124 117 23 mg
THINK

WRITE

a 9.79

1
2
3

Move the decimal point after the first significant


figure.
The decimal point has been moved 10 places left.
Round the decimal off after the third significant
figure.

1010 km

Chapter 5 Algebraic skills and techniques

THINK

WRITE

b 1.24

Move the decimal point after the first significant


figure.

The decimal point has been moved 7 places left.

Round the decimal off after the third significant


figure.

10

185

mg

To change a number from scientific notation back to a decimal number, move the
decimal point to the right if the power of 10 is positive. If the power of 10 is negative,
move the decimal point to the left. Zeros will need to be added if there are insufficient
decimal places.

WORKED Example 15
Write each of the following as a decimal number:
a 3.85 108
b 8.654 106
THINK

WRITE

a Move the decimal point eight places to the right. You


will need to add six zeros to do this.

a 3.85

b Move the decimal point six places to the left. You will
need to add five zeros after the decimal point to do
this.

b 8.654

108 = 385 000 000

106 = 0.000 008 654

remember
1. Scientific notation is used as a shorthand way of writing very large and very
small numbers.
2. The decimal point is placed after the first significant figure, and then this
decimal is multiplied by the appropriate power of 10.
3. The power of 10 is found as follows.
If the decimal point is moved left (for large numbers), the power of 10 is the
number of places moved left.
If the decimal point is moved right (for small numbers), the power of 10 is
negative and is the number of places moved right.
4. To write a number given in scientific notation as a decimal number, move the
decimal point:
(a) to the right for a positive power of 10
(b) to the left for a negative power of 10.

186

Maths Quest General Mathematics HSC Course

5E
SkillS

HEET

1 Write each of the following in scientific notation.


a 90 000
b 20 000 000 000
13a
Scientific WORKED 2 Write each of the following in scientific notation.
notation Example
a 1 458 000
b 23 650 000 000 000
13b

5.8

WORKED

Example

c 700
c 2589

3 Write each of the following in scientific notation.


a 0.000 000 02
b 0.004 57
c 0.000 000 000 049 321
13c
Scientific
4 Write each of the following in scientific notation, correct to 3 significant figures.
notation WORKED
Example
a 93 154 789 km
b 78 548 963 214 mm
c 45 874 t
14
d 0.003 654 7 g
e 0.213 658 mL
f 0.000 005 687 4 s
et

reads
L Sp he

EXCE

Scientific notation

WORKED

Example

5 Write each of the following as a decimal number.


a 3.4 104
b 2.87 106
15a
WORKED 6 Write each of the following as a decimal number.
Example
a 5.85 104
b 1.97 106
15b
WORKED

Example

c 3.0248 1010
c 1.002 103

7 An astronomical unit (AU) is defined to be the distance between the Earth and the sun
and is equal to approximately 150 000 000 km. The table below shows the distance
between each planet in the solar system and the sun in astronomical units. Write the
distance between each planet and the sun in kilometres in scientific notation, correct to
3 significant figures.

Work

Planet

T
SHEE

5.2

Distance
(AU)

Mercury

0.39

Venus

0.72

Earth

1.0

Mars

1.52

Jupiter

5.20

Saturn

9.54

Uranus

19.18

Neptune

30.06

Distance in km
(scientific notation)

8 Complete each of the measurement conversions.


a 2.35 107 mm = ___ m
b 8.4 107 m = ___ km
5
c 6.4 10 cm = ___ mm
d 6.58 106 kg = ___ t
6
e 7.802 10 t = ___ kg
f 8.29 1010 kg = ___ g
8
g 1.87 10 L = ___ kL
h 2.178 107 kL = ___ L
i 5.55 107 L = ___ mL

Chapter 5 Algebraic skills and techniques

187

summary
Substitution
Substitution involves the replacement of a pronumeral with a numerical value in an
expression.
These expressions include linear expressions that have only powers of 1, quadratics
that have a power of 2 and cubics that have a power of 3.
Care must be taken when using a calculator to apply the power to the correct term.

Algebraic manipulation
Algebraic terms are added or subtracted by collecting like terms.
Algebraic terms are multiplied or divided by applying the index laws to each
pronumeral separately.
First Index Law:
ax ay = ax + y
Second Index Law:
ax ay = ax y
Third Index Law:
(a x ) y = a xy

Equations and formulas


After substituting into a formula, an equation will be created when you are not
finding the subject of the formula.
The equation that you may need to solve could be linear or quadratic.
Using the same method as for solving equations, you can rearrange a formula to
make another pronumeral the subject of the formula.

Solution by substitution
Some equations have no opposite operation that allows you to easily solve the
equation. These equations can have an approximate solution found using
substitution.
To solve an equation using this method, make a first estimate of the solution and
substitute that estimate into the equation.
Use the result of that substitution to make an improved estimate, and then substitute
the improved estimate into the equation. Repeat this process until a solution to the
desired degree of accuracy is found.

Scientific notation
Scientific notation is used to write very large or very small numbers in a shorthand
way using powers of 10.
The decimal point is moved after the first significant figure and is multiplied by the
appropriate power of 10.
For large numbers, the power of 10 is the number of places the decimal point has
been moved to the left.
For small numbers, the power of 10 is negative and is the number of places the
decimal point has been moved to the right.

188

Maths Quest General Mathematics HSC Course

CHAPTER
review
5A
5A
5A
5A

1 Find the value of V = A

5B

5 Simplify each of the following by collecting like terms.


a m+m+m+m+m
b 7q + 9q
d 23t 22t
e 4m + 6n 2n
g 11k 6l + 4l 8k
h 5x2 + 20x + 3x2 6x

5B

6 Simplify each of the following.


a 4a4 7a5
b 5b 9b
d 12m5n6 mn
e 42x6 7x4
28q 2
g ----------h (3p2q4)3
4q

h, when A = 54 and h = 3.

2 Find the value of S = ut + --1- at2, when u = 4.1, t = 6.2 and a = 0.6.
2

3 Find the value of d =

( x 1 x 2 ) 2 + ( y 1 y 2 ) 2 , when x1 = 2, y1 = 7, x2 = 3, and y2 = 5.

4 Find the value of each of the following giving your answer, where necessary, correct to 2
decimal places.
a A = r(r + s), when r = 3.9 and s = 7.2
b C = 5--- (F 32), when F = 100
9
a
c S = ----------- , when a = 12 and r = 0.4
d y = r 2 x 2 , when r = 10 and x = 6
1r

5B

7 Expand each of the following.


a 2(a + 9)
b p(2p 4)
d 4m5(3m2 2n)
e 4xy(4 y)

5B

8 Expand and simplify each of the following.


a 2(m + 8) + 6(m + 4)
b 3p(p 2) + p(3 p)
d 3z(y 2z) + 4y(2y + z)
e 4pq(p q) 2p(pq 4)

5C
5C

9 In the formula P = 2l + 2b find l, when P = 78 and b = 24.

c 5p + 8p p
f 7x + 4 3x 9
i 4ab + 7a 2b 3ba
c 3g2h5 7g2h3
f 32r5s4 4r5s
i (8m2)2

m 16m3

c x2(3x3 1)
f 6a2b3(2a3 4b2)
c 7(2x

4)

3(x + 8)

10 The formula C = 2 r is used to find the circumference of a circle given the radius. Find the
radius of a circle with a circumference of 136 m. Give your answer correct to 1 decimal place.

5C
5C

11 In the formula A = 6s2, find s when A = 216.

5D

13 Use the method of substitution to solve the following equations, correct to 1 decimal place.
a 5x = 100
b (1.2)x = 2
c (0.75)x = 0.25

5D

14 The amount to which $10 000 will grow when invested at 9% p.a. can be found using the
formula A = 10 000 (1.09)n, where n is the number of years of the investment. Use the
formula to find the amount of time that it will take for $10 000 to grow into $20 000, correct
to the nearest year.

12 The volume of a square-based pyramid can be found using the formula V = 1--- s2h, where s is
3
the side length of the square base and h is the height of the pyramid. Find the side length of
a square-based pyramid with a volume of 108.864 cm3 and a height of 6.3 cm.

Chapter 5 Algebraic skills and techniques

189

15 A car depreciates at a rate of 20% p.a. The amount of time that it takes for the car to halve
in value can be found by solving the equation (0.8)n = 0.5, where n is the age of the car. Find
the length of time it takes for a car to halve in value, correct to the nearest year.

5D

16 Write each of the following in scientific notation.


a 600 000
b 0.000 000 000 2
c 78 920 000 000 000
d 0.001 25
e 0.000 004 589
f 124 589 000 000 000

5E

17 Write each of the following in scientific notation, correct to 3 significant figures.


a 12 589
b 0.000 125 478 624
c 0.032 143 68
d 586 460 484
e 12 447.151 48
f 0.000 000 051 851 58

5E

18 Write each of the following as a decimal number.


a 2.5 102
b 3.87 104
1
d 2.89 10
e 3.6702 107

5E

19 Complete each of the following.


a 2.5 105 m = ___ mm
c 3.43 104 kL = ___ L
e 4.243 107 t = ___ kg

c 9.8504 107
f 1.1 103

b 2.8 108 g = ___ kg


d 1.45 106 m = ___ km
f 1.3 108 mL = ___ L

Practice examination questions


1 multiple choice
3x(2x 4y) 2y(4y
A 6x2 8y2

6x) =
B 6x2 + 8y2

C 6x2 24xy 8y2

D 6x2 24xy + 8y2

2 multiple choice
The total surface area of a cone is given by the formula A = r(r + s), where r is the radius
and s is the slant height of the cone. The formula with s as the subject is:
A r
Ar
A
A
A s = ------ r
B s = ---------------C s = -----------D s = ------ + r
r
r
r
r
3 multiple choice
The total surface area of the square-based pyramid with side of the base b and the height of
the triangular face h is given by A = b2 + 2bh. If the total surface area of the pyramid is 64 cm
and the length of the side of the base is 4 cm, the height of the triangular face is:
A 6 cm
B 10 cm
C 20 cm
D 24 cm
4 multiple choice
The solution to the equation 10x = 200 is closest to:
A 2
B 2.3
C 2.4

D 20

5E

190

Maths Quest General Mathematics HSC Course

5 multiple choice
A square has a side length of 5.6 105 cm. The area of the square in scientific notation will be:
A 3.136 1011 cm2 B 31.36 1010 cm2 C 3.136 1025 cm2 D 31.36 1025 cm2
6 The volume of a cylinder can be found using the formula V = r 2h. The surface area of a
cylinder can be found using the formula SA = 2 r 2 + 2 rh.
a Find the volume of a cylinder with a radius of 4.2 cm and a height of 5.5 cm.
(Give your answer correct to 1 decimal place.)
b Find the height of a cylinder with a volume of 705 cm3 and a radius of 5.2 cm.
(Give your answer correct to 1 decimal place.)
c Find the radius of a cylinder with a volume of 939.4 cm3 and a height of 7.3 cm
(correct to 1 decimal place).
d Rewrite the formula for surface area to make h the subject.

CHAPTER

test
yourself

7 The time taken for an investment to double in value when invested at 7.5% p.a. can be found
by solving the equation (1.075)n = 2.
a Use the method of substitution to find the solution to this equation, correct to the nearest
whole number.
b Write an equation that can be used to find the amount of time that it will take for the value
of an item to halve in value if it depreciates at 15% p.a.
c Solve this equation, correct to 1 decimal place.

Multi-stage
events

6
syllabus reference
Probability 3
Multi-stage events

In this chapter
6A Tree diagrams
6B Counting techniques
6C Probability and counting
techniques
6D Probability trees

areyou

READY?

Are you ready?

Try the questions below. If you have difficulty with any of them, extra help can be
obtained by completing the matching SkillSHEET. Either click on the SkillSHEET icon
next to the question on the Maths Quest HSC Course CD-ROM or ask your teacher for
a copy.

6.1

Listing the sample space

6.2

Informal description of chance

6.3

1 List the sample space for each of the following events.


a A card is drawn from a standard deck and its suit is noted.
b A ball is selected from a bag containing three red, two blue and five white balls.
c A pin is stuck in the page of a book and the nearest letter is noted.
2 Describe each of the following events as being certain, probable, fiftyfifty, unlikely or
impossible.
a Winning the lottery.
b Selecting an odd number from cards labelled with numbers 1 to 55.
c Finding a $40 note in your wallet.
Equally likely events

3 For each of the events in question 2, state whether or not each outcome is equally likely.
Fundamental counting principle

6.4

6.5

4 In each of the following find the number of different ways each selection can be made.
a One person is to be chosen from each of two classes with 20 people in one class and 25 in the
other.
b From a menu an entree is to be chosen from a selection of five entrees followed by a main
course from a selection of eight, and then a dessert from a selection of six.
c Car number plates consisting of two letters, followed by two digits, followed by another two
letters.
Single event probability

5 Find the probability of each of the following events.


a Randomly selecting the winner of a swimming final with eight competitors.
b Winning a raffle when 150 tickets are sold and you purchase three tickets.
c Selecting a $2 coin from a pocket containing three $2 coins, four $1 coins and seven 20c
pieces.

6.6

Determining complementary events

6 Find the complement to each of the following events.


a Selecting a vowel from the letters of the alphabet.
b Choosing a black marble from a bag with 12 black, 23 white and 15 clear marbles.
c Selecting a number less than 10.

6.7

Calculating the probability of a complementary event

7 Find the probability of:


a winning a football match given the probability of losing is 2--- .
5
b the train being late given that it is on time four days out of every five.
c a golfer missing a putt given the probability of sinking the putt is 0.73.

Chapter 6 Multi-stage events

193

Tree diagrams
As discussed in the preliminary course, if an event has more than one stage to it, then it
is necessary to draw a tree diagram to list the sample space accurately. In a tree
diagram the tree branches out once for each stage of the experiment. At each stage the
number of branches is the same as the number of possible outcomes.
To list the sample space we then follow the tree to the end of each branch and record
the outcome at each stage.

WORKED Example 1

A coin is tossed three times. Draw a tree diagram and use it to list the sample space for
this experiment.
THINK
1
2
3

WRITE

There are three stages to the


experiment.
At each stage the outcome can be
heads or tails.
Draw the tree diagram branching out
three times with two branches at
each stage.

1st
coin

2nd
coin
Heads

Heads
Tails
Heads
Tails
Tails
4

List the sample space by following


the path to each end branch.

3rd
coin
Heads
Tails
Heads
Tails
Heads
Tails
Heads
Tails

S = {HHH, HHT, HTH, HTT, THH, THT, TTH,


S = {TTT}

To see a step-by-step construction of the tree diagram in


worked example 1, click on the PowerPoint icon.
In the above example, each stage of the
experiment (each toss of the coin) is
independent of the other stages. That is to
say, the outcome of one toss does not
affect the outcome of another toss. In
many examples, the outcome of one
stage will affect the outcome of
another. Consider worked example 2.
Here we are forming a two-digit
number such that no digit may be
repeated. Once a number has been
chosen as the first digit, it can not be
chosen as the second digit. Therefore, the
first stage of the experiment does affect the
second stage.

194

Maths Quest General Mathematics HSC Course

WORKED Example 2
A two-digit number is formed using the digits 4, 5, 7 and 9 without repetition. Draw a tree
diagram and use it to list all possible numbers that can be formed.
THINK
1
2

3
4

WRITE

There are two stages to the experiment.


For the first stage there will be four
branches and since one number is
chosen there will be three branches for
the second stage.
Draw the tree diagram.
List the sample space by following the
branches to each end point on the tree
diagram.

1st digit
4
5
7
9

2nd digit
5
7
9
4
7
9
4
5
9
4
5
7

Sample space
45
47
49
54
57
59
74
75
79
94
95
97

Click on the PowerPoint icon to see this tree diagram constructed step by step.
Once we have completed the tree diagram, the probability of an event can be
calculated using the formula:
number of favourable outcomes
P(event) = ---------------------------------------------------------------------------total number of outcomes

WORKED Example 3
A coin is tossed and a die is rolled. Calculate the probability of tossing a tail and rolling a
number greater than 4.
THINK
1
2

WRITE

There are two stages to the event.


At the first stage there are two
outcomes and at the second stage there
are six outcomes.
Draw the tree diagram.

List the sample space by following the


branches to each end point on the tree
diagram.
Calculate the probability using the
probability formula. There are two
favourable outcomes T5 and T6.
Simplify.

Coin toss Die roll


1
2
3
Heads
4
5
6
1
2
3
Tails
4
5
6

Sample space
Heads 1
Heads 2
Heads 3
Heads 4
Heads 5
Heads 6
Tails 1
Tails 2
Tails 3
Tails 4
Tails 5
Tails 6

P(tail and no. > 4) =

2
-----12

P(tail and no. > 4) =

1
--6

Again the PowerPoint icon can be used to see the tree diagram constructed step by step.

Chapter 6 Multi-stage events

195

remember
1. In any probability experiment that has more than one stage, a tree diagram
should be used to calculate the sample space.
2. The tree diagram branches once for each stage and the number of branches at
each stage is equal to the number of outcomes.
3. The sample space is found by following the path to the end of each branch.
4. Once the sample space has been found, the probability of each outcome is
calculated using the probability formula:
number of favourable outcomes
P(event) = ---------------------------------------------------------------------------total number of outcomes

6A
WORKED

Example

1 A family consists of four children. Draw a tree diagram to show all possible
combinations of boys and girls.

3 There are two bags each containing a red, blue, yellow


and green marble. One marble is to be chosen from
each bag. Draw a tree diagram that will allow you to calculate the sample space.

SkillS

Informal
description
of chance

6.3

SkillS
HEET

Example

6.2

HEET

WORKED

SkillS

Listing
the
sample
space

2 Two dice are cast. Draw a tree diagram that will allow
you to list the sample space of all possible outcomes.

4 A school is to send one male and one female representative to a conference. The
boys nominate George, Frank, Stanisa and Ian; the girls have nominated Thuy,
Petria, Joan, Wendy and Amelia. Draw a tree diagram and list the sample space for
all possible choices of representatives.

6.1

HEET

Tree diagrams

5 A two-digit number is to be formed using the digits 1, 2, 4, 5 and 7 such that no digit Equally
likely
may be repeated. Draw a tree diagram to list all possible numbers that can be formed. events
E

7 The digits 3, 5, 7 and 8 are used to form a three-digit number. If no digit can be used
more than once list the sample space.
8 multiple choice
From a group of five nominations a school captain and vice-captain are to be elected.
The number of ways that the captain and vice-captain can be chosen is:
A5
B 10
C 20
D 25
WORKED

Example

9 The four aces from a deck of cards are placed face down on a table. One card is
chosen followed by a second card without the first card being replaced. Calculate the
probability that the ace of hearts is one of the two cards chosen.

sheet

6 A committee needs to elect a president, secretary and treasurer. The four nominations
L Spre
XCE ad
for these positions are Belinda, Dean, Kate and Adrian. Given that no person is
allowed to hold more than one position, use a tree diagram to list all ways in which
Tree
diagrams
these three positions can be filled.

196

Maths Quest General Mathematics HSC Course

10 A two-digit number is formed using the digits 2, 3, 4 and 7 without repetition.


a Use a tree diagram to list the sample space.
b Calculate the probability that the number formed is greater than 35.
11 A tennis team consists of three men, Andre, Yevgeny and Jonas and two women,
Martina and Lindsay. From the team the captain and the vice-captain are to be chosen.
Calculate the probability that the captain and vice-captain are:
a Andre and Lindsay
b both men
c the same sex
d different sex.
12 Find the probability that all three
children in a family will be the same
sex.
13 multiple choice
A three-digit number is formed using
the digits 5, 6, 8 and 0. No digit can
be repeated and the 0 cant be first.
The probability of the number formed
being greater than 800 is:
A

1
--4

1
--3

3
-----16

1
--2

14 An airline offers holidays to three destinations: Brisbane, Gold Coast or Cairns. The
holiday can be taken during two seasons: Peak season or Off-peak season. The
customer has the choice of three classes: Economy, Business or First class. There is no
First class to Cairns, however.
a Use a tree diagram to list all combinations of holiday that could be taken by
choosing a destination, season and class.
b Terry takes a mystery flight, which means he is allocated a ticket at random from
the above combinations. Calculate the probability that Terrys ticket:
i goes to Brisbane
ii is First class
iii is in Peak season, flying First class.

Counting techniques
Ordered arrangements
1 Select three people and stand them in a line.
2 Now get the three people to stand in a different order.
3 In how many different orders can the three people be placed?
4 Repeat the above process with four people in the line.
5 Is there a pattern? Can you calculate the number of different ways in which five
people can be arranged?

Chapter 6 Multi-stage events

197

There are 10 people standing in a line. In how many ways can they be arranged? To
calculate this we need to consider the number of ways that each place in the line can be
filled. To do this we need to calculate the number of people remaining after we fill each
place in the line.
There are 10 people who could fill the first position.
Once the first position has been filled, there are nine people remaining to fill the
second position.
Once the second position has been filled, there are eight people remaining to fill the
third position.
This pattern continues until there is only one person left who can fill the last position.
Calculating this: 10 9 8 7 6 5 4 3 2 1 = 3 628 800.
A shorter way of writing 10 9 8 7 6 5 4 3 2 1 is to write 10!, that
is, 10 factorial.
Your calculator will have a factorial function, usually labelled x!. Make sure that you
know where this function is on your calculator.

WORKED Example 4

Calculate the value of 8!.


THINK

WRITE

Enter 8 and press 2ndF


calculator.

x! on the

8! = 40 320

Graphics Calculator tip! Factorial function


To use the factorial (!) function on your Casio graphics calculator follow the steps
below, which will calculate 8! as in worked example 4 above.
1. From the MENU select RUN.

2. Press OPTN (F6) for more options. You should be


able to see the screen at right.

3. Press F3 (PROB) and you will be able to see the


function x!. Press 8
F1 (x!) EXE .

198

Maths Quest General Mathematics HSC Course

WORKED Example 5
Six people are standing in a line. In how many ways can the six people be arranged?
THINK

WRITE

The answer is 6!.

6! = 720

Give a written answer.

The people can be arranged in 720 ways.

Tree diagrams and ordered


arrangements
Four people, Anji, Belinda, Kristen and
Summer, are to be placed in order.
1 Calculate the number of different ways
these four girls can be placed in a line.
2 Draw a tree diagram and use it to list the
ways that the four girls can be placed in
order.
3 Check that the number of elements in
the sample space found from your tree
diagram corresponds to the answer
obtained in part 1.
In worked examples 4 and 5, we have been ordering an entire group. In some cases
we may wish to order only part of the group. Consider the case of an Olympic swimming final. There are eight swimmers and we wish to know the number of ways that the
gold, silver and bronze medals can be awarded.
There are eight possible winners of the gold medal.
With the first place filled, there are seven possible winners of the silver medal.
With both first and second places filled, there are six possible winners of the bronze
medal.
Calculating this: number of arrangements = 8

Calculating this: number of arrangements = 336


This type of arrangement is known as an ordered selection. It occurs when the order
in which the choices are made is important. In the worked example below, a captain
and a vice-captain are to be chosen. If Benito is captain and Imran is vice-captain, this
is a different selection to Imran as captain and Benito as vice-captain.
To calculate the number of ordered selections that can be made, we multiply, starting
from the number of possible first selections, then reducing by one with each multiplication until each position is filled.

Chapter 6 Multi-stage events

199

WORKED Example 6
In a cricket team of eleven players, a captain and vice-captain are to be chosen. In how
many ways can this be done?
THINK
1
2

WRITE

There are 11 possible choices of captain.


Once the captain is chosen, there are 10
choices remaining for vice-captain.

No. of arrangements = 11 10
No. of arrangements = 110

Committee selections
On a committee of five people, a president and a vice-president are to be chosen.
The five committee members are Andreas, Brett, Cathy, Dharma and Emiko.
1 Use the method shown in worked example 6 to calculate the number of ways in
which the president and the vice-president can be chosen.
2 Now use a tree diagram to list the sample space of all possible selections of
president and vice-president.
3 Check that the number of elements in the sample space corresponds to the
answer obtained in part 1 of this investigation.
Consider a case where two representatives to a committee are chosen from a class of
20 students. This is an example of an unordered selection. If Sue is chosen, followed by
Graham, this is the same choice as if Graham is chosen and then Sue.
To calculate the number of unordered selections that can be made, we calculate the
number of ordered selections that can be made and then divide by the number of
arrangements of these selections. This is calculated using factorial notation as in
worked example 5. In the case of choosing the committee:
Number of ordered selections is 20 19 = 380.
Two people can be arranged in two (2!) ways.
Number of unordered selections = 380 2
Number of unordered selections = 190

WORKED Example 7
From a group of eight athletes, three are to be chosen to represent the club at a carnival.
In how many ways can the three representatives be chosen?
THINK
1
2
3

Calculate the number of ordered selections


that can be made.
Calculate the number of arrangements of the
representatives.
Divide the ordered selections by the
arrangements of the representatives.

WRITE
Ordered selections = 8 7 6
Ordered selections = 336
Arrangements = 3 2 1
Arrangements = 6
Unordered selections = 336 6
Unordered selections = 56

200

Maths Quest General Mathematics HSC Course

Unordered selection
A rowing team has six members: Mark, Norman, Olaf, Pieter, Quentin and
Raymond. Two are to be chosen to be the crew in a pairs race.
1 Use the method described in worked example 7 to calculate the number of pairs
that could be chosen.
2 Use a tree diagram to list the ordered selections and then write the sample space
of unordered selections by ignoring any repeated pair.
3 Check that the number of elements of the sample space corresponds to the
answer obtained in part 1 of this investigation.

remember
1. A group of n different items can be arranged in n! ways.
2. n! = n (n 1) (n 2) 1 and can be found as a function on your
calculator.
3. When an ordered selection is made, the number of selections can be calculated
by multiplying the number of first choices that can be made by the number of
second choices that can be made and so on.
4. To calculate the number of unordered selections that can be made, we divide
the number of ordered selections by the number of arrangements of those
selected.

SkillS

6B
HEET

6.4

WORKED

Example

Fundamental
counting
principle

WORKED

Example

Counting techniques

1 Use your calculator to calculate the value of the following.


a 3!
b 5!
c 9!
2 Four people are involved in a race. In how many different orders can they complete
the race?
3 The letters A, B, C, D and E are written on cards. In how many different orders can
the cards be placed?
4 A three-digit number is formed using the digits 3, 6 and 8. If no number can be
repeated, how many numbers is it possible to form?

WORKED

Example

5 In a race of 10 people, in how many different ways can the first three places be filled?
6 In a school, a captain and vice-captain are to be elected. The four nominations are
Geri, Reika, Melanie and Victoria. In how many different ways can the captain and
vice-captain be chosen?
7 In the Melbourne Cup there are 24 horses. In how many different ways can the three
placings be filled?

Chapter 6 Multi-stage events

WORKED

Example

201

8 Seven people try out for three places on a debating team. In how many ways can the
team of three be chosen from the group of seven?
9 How many different groups of four can be selected from ten people?
10 In his pocket Trevor has six coins: a $2 coin, $1 coin, 50c coin, 20c coin, 10c coin and
5c coin. If Trevor randomly chooses two coins, how many different sums of money
are possible?
11 On a restaurant menu there is a choice of three entrees, six main courses and four
desserts. In how many ways can a person choose an entree, main course and dessert
from the menu?
12 multiple choice
Which of the following is an example of an unordered selection?
A Five students are placed in order of their exam results.
B From a group of five students, a contestant and a reserve are chosen for a
Mathematics competition.
C From a group of five students, two are chosen to represent the class on the SRC.
D From a group of five students, two are awarded 1st and 2nd prizes in Mathematics.
13 multiple choice
The numbers 1, 2, 3 and 4 are used to form a three-digit number such that no digit can
be used more than once. The number of three-digit numbers that can be formed is:
A4
B 6
C 12
D 24
14 multiple choice
Gavin, Dion, Michael, Owen and Shane try out for two places on a tennis doubles
team. The number of teams that can be chosen is:
A5
B 10
C 20
D 25

16 At the Olympic qualifying trials, nine cyclists compete for a place on the team.
a In how many different orders can the competition finish?
b How many different ways can 1st, 2nd and 3rd place be filled?
c Two cyclists are chosen to represent Australia on the team. How many different
teams of two can be chosen?

Probability and counting techniques


Once the counting techniques done in the previous section have been completed, we
can calculate the probability of certain events occurring. To do this we go back to using
the probability formula:
number of favourable outcomes
P(event) = ---------------------------------------------------------------------------total number of outcomes

Work

15 A small play has three characters. Six people, Wendy, Rebecca, Thai, Yasmin, Andrea
and Ophelia, audition for the three parts.
a How many different groups of three can be chosen for the play?
b In how many different ways can the three parts be allocated to the three girls?
T
SHEE

6.1

202

Maths Quest General Mathematics HSC Course

WORKED Example 8
The letters A, H, M, S and T are written on cards. The cards are shuffled and then laid out
face up. Calculate the probability that the cards form the word MATHS.
THINK
1

WRITE

The five cards can be arranged in 5! ways.

No. of arrangements = 5!
No. of arrangements = 5 4
No. of arrangements = 120
1
P(MATHS) = --------

MATHS is one way of arranging the letters


and so we use the probability formula.

120

We also need to be able to calculate the probability of a particular ordered or unordered


arrangement occurring.

WORKED Example 9
From Francis, Gary, Harley, Ike and Jacinta, a school captain and vice-captain need to be
elected. Calculate the probability that Ike and Jacinta occupy the two positions.
THINK
1
2
3
4

WRITE

Calculate the number of ordered selections


that are possible.
Ike and Jacinta in the two positions can be
arranged in two ways.
Divide the ordered selections by the number
of arrangements.
Substitute into the probability formula.

No. of ordered selections = 5 4


No. of ordered selections = 20
No. of arrangements = 2 1
No. of unordered selections = 20 2
No. of unordered selections = 10
1
P(Ike and Jacinta) = ----10

WORKED Example 10
A bag contains a red, green, yellow, blue, orange and purple marble. Three marbles are
selected from the bag. Calculate the probability that the red, yellow and orange marbles
are chosen.
THINK
1
2
3
4
5

Calculate the number of ordered


selections.
Calculate the number of arrangements.
Calculate the number of unordered
selections.
The red, yellow and orange marble is
one possible selection.
Substitute into the probability formula.

WRITE
No. of ordered selections = 6 5 4
No. of ordered selections = 120
No. of arrangements = 3 2 1
No. of arrangements = 6
No. of unordered selections = 120 6
No. of unordered selections = 20

P(red, yellow and orange) =

1
-----20

Chapter 6 Multi-stage events

203

Popular gaming
There are many different forms of lottery that depend upon ordered or unordered
arrangements.
1 Lotto This requires the player to select six numbers out of 45. In how many
ways can the six numbers be chosen? Remember order does not matter.
2 Similar games to Lotto are:
a Oz Lotto seven numbers are chosen from 45.
b The Pools six numbers are chosen from 38.
In how many ways can the six numbers for each of these games be chosen?
3 Powerball This requires the player to choose five numbers from 45 in an
unordered selection. A sixth ball (the powerball) is chosen from a second barrel
containing 45 balls. In how many ways can this be selected?
4 Lotto Strike The player must select the first four balls drawn from 45 in the
correct order. In how many ways can this ordered selection be made?

remember
When we have calculated the number of arrangements and the number of ordered
or unordered selections that are possible, we can then calculate the probability of
a certain selection using the probability formula.

6C
WORKED

Example

1 Four people, Craig, Barry, Anne and Dimitri, are arranged in a line. Calculate the
probability that the four people are arranged in alphabetical order.
2 The numbers 4, 5, 6, 7 and 8 are arranged to form a five-digit number such that
no digit can be repeated. Calculate:
a how many five-digit numbers can be formed
b the probability that the number formed is 54 867
c the probability that the number formed is 86 574.

Single
event
probability

3 A three-digit number is formed using the digits 6, 8 and 9 and no digit may be
repeated. Calculate the probability that the number formed is:
a 896
b even
c greater than 800.
WORKED

Example

6.5

4 There are five candidates in an election for SRC president. The second placed
candidate will be made vice-president of the SRC. If Lauren and Meta are two of the
candidates, calculate the probability that they will occupy the two positions.
5 Seven surfers enter a competition. If two of the surfers are Kurt and Paul, calculate the
probability that:
a Kurt comes first and Paul comes second
b Paul comes first and Kurt comes second
c Kurt and Paul fill the first two places.

SkillS
HEET

Probability and counting


techniques

204

Maths Quest General Mathematics HSC Course

6 From the digits 1 to 9 a two-digit number is formed such that no digit can be repeated.
Calculate the probability that the number formed is:
a 67
b greater than 80
c less than 30.
WORKED

Example

10

7 From a deck of cards, the four aces are laid face down on a table. Two of the aces are
then turned face up. Calculate the probability that the two aces turned face up are the
ace of clubs and the ace of spades.
8 An ice-cream parlour offers a choice of 25 flavours. A triple scoop ice-cream places
three different flavours on top of each other. If the flavours are chosen randomly, find
the probability that the ice-cream is:
a vanilla, chocolate and strawberry in that order
b vanilla, chocolate and strawberry in any order.
9 Six boys try out for three places on a debating team. The boys are Gavin, David,
Andrew, Rhyse, Julius and Elliot.
a How many teams of three is it possible to choose?
b Calculate the probability that Gavin, Andrew and Elliot are on the team.
10 The letters M, A, I, D and G are written on cards and two of these are to be chosen.
Calculate the probability that the two cards chosen are:
a both vowels
b both consonants
c one vowel and one consonant.

1
1 Two coins are tossed in the air. Use a tree diagram to list the sample space of all
possible outcomes.
2 Two dice are rolled. How many possible outcomes are in the sample space?
3 In how many different ways can five cars be parked in a row?
4 A race has 10 runners. In how many different ways can the 10 runners finish?
5 A race has 10 runners. In how many different ways can the first three places be filled?
6 From a committee of nine people, a president and vice-president need to be chosen. In
how many different ways can the two positions be filled?
7 Eight people audition for four parts in a play. How many different groups of four
could be chosen?
8 Once the four people have been chosen in question 7, in how many different ways can
the four parts be allocated?
9 How many different ways can the parts be allocated among the original group of eight
in question 7?
10 Explain the difference between an ordered and unordered arrangement.

Chapter 6 Multi-stage events

205

Probability trees
In the tree diagrams studied so far, the probability of each
outcome has been equally likely. When each result is not
equally likely we can still draw the diagram in the same way,
writing the probability of each single outcome on the
branches of the tree.
Consider the case where a bag contains three green marbles and two white marbles. A marble is drawn, its colour
noted and it is then replaced in the bag. A second marble is
then drawn. We could draw a tree diagram as shown on the
right.

1st marble
Green

Green

Green

White

White

2nd marble
Green
Green
Green
White
White
Green
Green
Green
White
White
Green
Green
Green
White
White
Green
Green
Green
White
White
Green
Green
Green
White
White

Using a probability tree simplifies the diagram. In a single drawing of the marble
P(green) =

3
--5

and P(white) =

2
--- .
5

These probabilities are drawn on the branches of the

tree as shown below.


1st marble

2nd marble
3
5

Green

5
3
5

White

Green

White
Green

White

There are four elements to the sample space: (green, green), (green, white), (white,
green) and (white, white). Each element of the sample space is not equally likely. To
calculate the probability of each, we use the multiplication rule of probability.
The multiplication rule of probability states that to calculate the probability, you
multiply along the branches of the tree that lead to each event. Therefore:
P(green, green) =
=
P(white, green) =
=

3
--5

3
--5

9
-----25
2
--5
6
-----25

P(green, white) =
=

3
--5

P(white, white) =
=

3
--5

2
--5

6
-----25
2
--5

2
--5

4
-----25

This is the method that must be used to calculate the probability in any situation
where each outcome is not equally likely.

206

Maths Quest General Mathematics HSC Course

WORKED Example 11

In a bag there are seven red marbles and three green marbles. A marble is drawn, its
colour noted and it is then replaced in the bag. A second marble is then drawn. Find the
probability that both marbles are red.
THINK
1

WRITE

Draw the probability tree.

1st marble

2nd marble
7

10

10

Red

10
7

10

10

Green

Red

Green
Red

10

Green
2

Calculate the probability by


multiplying along the branches.

P(red, red) =
P(red, red) =

7
-----10
49
--------100

7
-----10

The PowerPoint icon will show you step by step how to construct this probability tree.
When asked to find the probability of an event that can occur in several ways, we
need to use the addition rule of probability. The addition rule for probability states that
for an event that can occur in several ways, the probability is the sum of the probabilities for each way that the event can occur.

WORKED Example 12

In a barrel there are four blue counters and six red counters. A counter is drawn, its
colour noted and a second counter is drawn. The first counter is not replaced in the barrel
before the second counter is drawn. Find the probability that:
a a blue counter is drawn, followed by a red counter
b two counters of a different colour are drawn.
THINK
1

Draw the probability tree.


If the first counter is blue, three blue
and six red counters remain in the
bag.
If the first counter is red, four blue
and five red counters remain in the
bag.

WRITE
1st counter 2nd counter
3
9
4

10

Blue

Blue

Red
4
9
6

10

Red

Blue

5
9

Red

Chapter 6 Multi-stage events

THINK
2

207

WRITE

a Multiply along the white, red


branches to calculate the probability.
b This outcome can occur in two ways.
Add the probabilities (blue, red) and
(red, blue).

P(blue, red) =

4
-----10

4
-----15

6
--9

b P(different colour)
= P(blue, red) + P(red, blue)
4
= ( ----10

4
-----15

8
-----15

6
--- )
9

6
+ ( ----10

4
--- )
9

4
+ ----15

The PowerPoint icon will allow you to see how this probability tree was constructed.
We must read each example carefully to see if the probabilities change throughout
the experiment. In many cases we do not need to examine each possible outcome. In
some examples we consider only one outcome. The branches of the tree then show if
this outcome occurs or not.

WORKED Example 13
Along a road there are three sets of traffic lights. The probability of catching a green light
is 0.35. Calculate the probability of catching all three green lights.
THINK
1

WRITE

Draw a probability tree. We do not need


to consider if the light is red or amber,
only whether it is green or not green.

1st lights

0.35

0.65

Calculate the probability by


multiplying along the green branches.

2nd lights
0.35

Green

0.65

Not
green

Green

Not
green

0.35

Green

0.65

Not
green

0.35
0.65
0.35
0.65
0.35
0.65
0.35
0.65

P(three green lights) = 0.35 0.35


P(three green lights) = 0.042 875

3rd lights
Green
Not
green
Green
Not
green
Green
Not
green
Green
Not
green

0.35

Click on the PowerPoint icon to see worked example 13 solved step by step.
The complementary event method is particularly important with this type of
question. Complementary events are two events that account for all possible outcomes
of an experiment. For example, when rolling a die the complement of rolling a number
less than three is to roll a number greater than two. We discovered during the preliminary course that the sum of the probability of an event and its complement is one.
It is often easier to calculate the probability of the complement rather than that of the
event itself. We can then subtract the probability of the complementary event from one.

208

Maths Quest General Mathematics HSC Course

WORKED Example 14

Three dice are rolled. What is the


probability of rolling at
least one six?

THINK
1

WRITE

Draw the probability tree. (We need to draw the


tree with only two outcomes as we are
concerned only with whether we get a 6 or not.)

1st die

2nd die

3rd die
1
6

1
6
1
6

5
6

6
5
6

Not 6

1
6

5
6
1
6

5
6

1
6

5
6

Not 6

5
6

Not 6

1
6

5
6

2
3

The complement to getting at least one six in


three rolls is getting no sixes in three rolls.
Subtract the complement from one to find the
probability.

6
Not 6
6
Not 6
6
Not 6
6
Not 6

P(at least one six) = 1

P(no sixes)

P(at least one six) = 1

( 5--6-

P(at least one six) = 1

125
--------216

P(at least one six) =

5
--6

5
--- )
6

91
--------216

As with other probability tree diagrams, you can see this example completed step by
step by clicking on the PowerPoint icon.

remember
1. If each outcome is not equally likely, draw a probability tree with the
probability of each single event on the branches.
2. To calculate a probability, multiply along the branches that give the required
outcome.
3. If an outcome can be obtained in two or more ways, add the probability of
each.
4. Read each question carefully to see if the probabilities change during the
experiment.
5. Consider carefully what outcomes you need to include in your tree. You may
need only to consider if one event occurs or not.
6. For questions that involve finding at least one, use the complementary event
method.
7. The sum of the probability of an event and its complement is one.

Chapter 6 Multi-stage events

6D
WORKED

Example

Probability trees

1 In a purse there are five 20-cent coins and three 50-cent


coins. A coin is selected from the purse and replaced,
and then a second coin is selected. The probability tree
on the right is drawn for this experiment. Find the
probability that the two coins drawn are both twenty
cent pieces.

1st coin

2nd coin
5
8

5
8

2 In a barrel there are four white marbles and five black


50c
marbles. Two marbles are drawn, the first being
replaced in the barrel before the second one is drawn.
a Draw the probability tree for this situation.
b Find the probability for each member of the sample space.
WORKED

Example

12

WORKED

Example

13

20c

20c

3
8

6.6

3
8

50c

5
8

20c

3
8

50c

SkillS
HEET

11

209

Determining
complementary
events

3 A hand of five cards contains three kings and two queens. A card is chosen and
then returned before a second card is chosen. Find the probability that:
a a queen is chosen followed by a king
b a king and a queen are chosen.
4 Jia is a shooter with an 80% chance of hitting a target. If he has three shots at a target,
find the probability that:
a he hits with all three shots
b he hits with exactly two shots.

of a
complementary
event

6 multiple choice

A bag contains four black and six white marbles. Two marbles are drawn from the bag
one after the other. If the first marble drawn is black, the probability that the second
marble drawn is white is:
A

4
--9

2
--5

2
--3

3
--5

7 multiple choice
A coin is biased such that the probability of it landing heads is 0.6. The coin is tossed
three times. Which of the following outcomes has the greatest probability of occurring?
A Tossing three heads
B Tossing two heads and one tail
C Tossing one head and two tails
D Tossing three tails

SkillS
HEET

5 A raffle has 100 tickets with two prizes. Kevin buys five tickets. Find the probability 6.7
that:
a Kevin wins 1st prize
b Kevin wins both prizes
Calculating
the probability
c Kevin does not win a prize
d Kevin wins exactly one prize.

210

Maths Quest General Mathematics HSC Course

8 A box contains three red and seven blue discs. Two discs
are chosen from the box. The probability tree for this
experiment is shown on the right. Find the probability of
selecting:
a two red discs
b two blue discs
c two discs the same colour
d two discs of a different colour.

1st disc

10

10

2nd disc
2

Red

Blue

Red

Blue

Red

Blue

9 The names of eight boys and five girls are placed into a
hat. Two people selected from the hat are to represent the
school at a function.
a Use a probability tree to find the sample space for this experiment.
b Find the probability of:
i two boys being chosen
ii two girls being chosen
iii one boy and one girl being chosen.

10 There are 25 students in class 12R and 24 students in class 12S. Two students are to
be chosen at random to attend a study skills course. Find the probability that the two
students chosen are:
a from the same class
b from different classes.
11 In a basket there are 15 balls, of which five are blue. Two are selected at random from
the basket. Find the probability that:
a two blue balls are selected
b no blue balls are selected
c exactly one blue ball is selected.
12 The probability that I will need to stop at a set of traffic lights is 0.55. If I twice travel
through this set of lights, what is the probability of:
a having to stop both times
b not having to stop either time.
13 Greg has an 80% chance of passing each Maths test. During the term he will
need to sit four tests.
14
a Find the probability that Greg will pass all four tests.
b Find the probability that Greg will fail at least one test.

WORKED

Example

14 A navy ship carries surface-to-air missiles with a probability of hitting a


target of 0.9. Two missiles are fired at an enemy warplane. Find the probability that the warplane escapes without being hit. (Hint: For the plane
to escape, both missiles that are fired must miss the target.)
15 In a certain town it is known that four-fifths of all school students
have been immunised against measles. For a medical test, four
students need to be chosen of which at least one must have been
immunised and at least one must not have been immunised. Find
the probability that if four students are chosen at random:
a at least one will have been immunised
b at least one will not have been immunised.

Chapter 6 Multi-stage events

211

16 multiple choice
Veronica rolls three dice. To win the game she needs to throw at least one six. Which
of the following will give the probability of throwing at least one six?
A 1 P(three sixes)
B 1 P(two sixes)
C 1 P(one six)
D 1 P(no sixes)
17 There are 2 classes in Year 12:
Class 12A has 15 boys and 10 girls.
Class 12B has 12 boys and 18 girls.
The principal chooses a student to make a speech by first choosing a class at random
followed by a student at random from the chosen class. Find the probability that the
student chosen is:
a from class 12A
b a boy from class 12B
c a girl.

19 A missile that is fired from the ground has a 0.8 chance of hitting its target. A missile
fired from a plane has a 0.4 chance of hitting a target. A missile is fired from both
ground and air at separate targets. Find the probability that:
a both hit their target
b one hits its target
c at least one hits its target.

Work

18 In a radio contest, to win $10 000 in prize money the contestant is faced with five
money bags. Each money bag has 10 coins in it. To win, the contestant chooses a bag
and then chooses a coin from that bag. If the coin has the station logo on it, the
contestant wins.
Bag 1 has one winning coin.
Bag 2 has three winning coins.
Bag 3 has seven winning coins.
Bags 4 and 5 have two winning coins.
Find the probability of the contestant winning the $10 000.

T
SHEE

6.2

212

Maths Quest General Mathematics HSC Course

summary
Tree diagrams
A tree diagram is used in any probability experiment where there is more than one
stage to the experiment.
The sample space can be determined from a tree diagram by following the paths to
the end of each branch.
The probability of an event can then be calculated by the probability formula:
number of favourable outcomes
P(event) = ---------------------------------------------------------------------------total number of outcomes

Counting techniques
The number of ways that n objects can be arranged in order is:
n! or n (n 1) (n 2) . . . 2 1.
In an ordered selection, a number of objects are chosen and are arranged in order.
The number of ordered selections can be calculated by multiplying the number of
first choices that can be made by the number of second choices possible and so on
until all choices have been included.
In an unordered selection, the order in which the objects have been chosen is not
important. The number of unordered selections that are possible is calculated by
dividing the number of ordered selections by the number of ways the ordered
selection can be arranged.
Once the number of selections has been determined, the probability of particular
selections can be determined.

Probability trees
When each outcome is not equally likely, you draw a probability tree.
On each branch of the tree is written the probability of that outcome.
To calculate any probability you multiply along the branches.

Chapter 6 Multi-stage events

213

CHAPTER
review
1 Two coins are tossed in the air.
a Draw a tree diagram.
b Use the tree to list the sample space for this experiment.

6A

2 The digits 5, 7, 8 and 9 are used to form a two-digit number. Use a tree diagram to list the
sample space if:
a no digit can be used more than once
b digits can be repeated.

6A

3 There are three births in the maternity ward of a hospital. Calculate the probability that the
babies are:
a all boys
b two boys and a girl
c more girls than boys.

6A

4 A two-digit number is formed using the digits 4, 6, 7, 8 and 9. No digit is allowed to be repeated.
a Use a tree diagram to list the sample space.
b Find the probability that the number formed is:
i 86
ii odd
iii greater than 65.

6A

5 In a barrel there are three black marbles and three white marbles. A marble is drawn and its
colour noted, and it is then replaced in the barrel. A second marble is then drawn. Find the
probability of selecting:
a two marbles of the same colour
b at least one black marble.

6A

6 A rowing crew has eight rowers. In how many different ways can the crew be seated in the boat?

6B
6B

7 From the rowing crew of eight, a captain and vice-captain are to be selected. Calculate the
number of different ways the captain and vice-captain can be selected.
8 From the rowing crew of eight, four are to be chosen to crew a four-person boat. How many
crews of four can be chosen from the group of eight?

6B

214
6B

Maths Quest General Mathematics HSC Course

9 From the digits 1, 2, 3, 4 and 5:


a how many five-digit numbers can be formed if repetition is not allowed?
b how many three-digit numbers can be formed if repetition is not allowed?

6C

10 The letters D, S, T, U and Y are shuffled and placed in a line on a table. Calculate the
probability that the word STUDY is formed.

6C

11 Two students from Richard, Sandra, Talia and Ingo have to make a speech. They draw
straws to see who will go first and second.
a How many different ways can the first and second speaker be arranged?
b What is the probability that Ingo speaks first and Talia speaks second?

6C

12 Six teams A, B, C, D, E and F contest a basketball competition. The top four sides play in
the semi-finals, and later two will contest the grand final.
a In how many different ways can the top four sides be arranged?
b What is the probability that the top four teams finish D, C, F and A?
c How many pairs of teams is it possible to meet in the grand final?
d What is the probability of A playing B in the grand final?
e What is the probability that C plays in the grand final?

6C

13 Zita is doing an exam when she realises that she has almost run out of time. She has not
answered the last 10 questions.
a If each question requires True or False as an answer and Zita guesses each answer, what
is the probability that she guesses all 10 correctly?
b If each question is multiple choice and requires the choice of (A), (B), (C) or (D), what is
the probability that Zita will guess all 10 correctly?

6D

14 In a bag there are three red marbles and two green marbles. Two marbles are drawn in
succession without replacement. Find the probability that the two marbles drawn are:
a both red
b both green.

6D

15 In a box there are six batteries. Two of the batteries are flat. If two are chosen from the box,
find the probability that both batteries are charged.

6D

16 The probability that a set of lights show green is 2--- . If I pass through this set of lights three
5
times, find the probability that:
a I catch three green lights
b I catch at least one green light.

6D

17 In a tennis match it is noticed that Roger Federer gets 70% of serves in play. If he has two
serves, find the probability that he gets at least one into play.

6D

18 One in every eight light bulbs are faulty. If I buy three light bulbs, find the probability that
none are faulty.

Chapter 6 Multi-stage events

Practice examination questions


1 multiple choice
Which of the following is an example of an ordered selection?
A A team of four people is chosen from a group of 12.
B Two representatives from a class of 30 students are elected to the SRC.
C A class of 30 students elect a class captain and vice-captain.
D From a barrel of 44 balls, six are chosen.
2 multiple choice
Six people are arranged in a line. The number of ways in which this can be done is:
A 6
B 12
C 120
D 720
3 multiple choice
In a race there are six runners. In how many ways can the first three places be filled?
A 6
B 12
C 120
D 620

215

216

Maths Quest General Mathematics HSC Course

4 multiple choice
A group of six people consists of Darren, Shintaro, Jim, Damien, John and Allan. From these
six people a group of three is chosen. The probability of choosing Darren, Jim and John is:
A

3
--6

1
-----20

1
-----12

1
--------120

5 Three coins are tossed in the air.


a Draw a tree diagram to list the sample space.
b Use your tree diagram to calculate the probability of tossing two heads and one tail.
c Calculate the probability of tossing at least one head.

CHAPTER

test
yourself

6 A basketballer has a probability of 0.4 of landing a three point shot. The basketballer has two
shots at the basket.
a Draw a probability tree showing all possible results of the two shots.
b Calculate the probability that the basketballer:
i lands both shots
ii lands exactly one shot
iii lands at least one shot.

Applications
of probability

7
syllabus reference
Probability 4
Applications of probability

In this chapter
7A Expected outcomes
7B Financial expectation
7C Two-way tables

areyou

READY?

Are you ready?

Try the questions below. If you have difficulty with any of them, extra help can be
obtained by completing the matching SkillSHEET. Either click on the SkillSHEET icon
next to the question on the Maths Quest HSC Course CD-ROM or ask your teacher for
a copy.

7.1

Single event probability

7.2

Tree diagrams

7.3

Probability trees

1 Calculate the probability of each of the following.


a Rolling a die and getting a number greater than 2.
b Winning a raffle after purchasing 10 tickets and knowing there are 500 tickets in the draw.
c Selecting an even number from the numbers 1 to 99 inclusive.

2 Two of the digits 3, 5, 6 and 7 are used to form a two-digit number such that no digit can be
repeated. Draw a tree diagram to list all possible two-digits numbers that can be formed.

3 In any given hour of television there are 12 minutes of advertisements. If Tony turns the
television on at two randomly selected times between 7.00 pm and 8.00 pm.
a use a probability tree to show all possible outcomes
b calculate the probability that on both occasions Tony turns on the television during an
advertisement.

Chapter 7 Applications of probability

219

Expected outcomes
Suppose that we toss a coin 100 times. How many times would you expect the coin to
land Heads? As each outcome is equally likely, we would expect there to be 50 Heads
and 50 Tails. How can this be shown to be true?
The number of times that we expect a certain outcome to occur is found by multiplying the probability of each outcome by the number of trials. In the above case, the
probability of the coin landing heads is --1- , and this is multiplied by the number of trials
2

(100). The result is an expectation of 50 Heads in 100 tosses of the coin.


The expected outcome is the number of times that we expect a particular outcome to
occur in a certain number of trials.

WORKED Example 1

A die is rolled 120 times. How many 6s would you expect to occur in 120 rolls
of the die?
THINK
1
2

WRITE

Calculate the probability of rolling a 6.


Multiply the probability of a 6 by the
number of trials.

P(six) = 1--6
Expected number of 6s =

1
--6

120

Expected number of 6s = 20

If the expected number of 6s is 20 in 120 rolls of a die, this does not mean that this is
what will occur. It may be that on one occasion we may get 25 sixes in 120 rolls,
another occasion we may get only 10 sixes. However, we expect that if we repeat the
experiment often enough, we would get an average of 20 sixes in 120 rolls.

Rolling a die
1 Each person is to take a die
and roll it 120 times and
record the number of 6s
rolled.
2 What is the most number of
6s rolled by anyone in
120 rolls of the die?
3 What is the least number of
6s rolled by anyone in
120 rolls of the die?
4 What is the average number
of 6s rolled by the class in
120 rolls of the die? How
does this compare with the
expected outcome of 20?

220

Maths Quest General Mathematics HSC Course

The expected outcome does not need to be a whole number. In many cases this will
not be so. Consider the example below.

WORKED Example 2
Roger draws a card from a standard deck, notes the suit and replaces the card in the
deck. If Roger repeats this process 50 times, how many spades can Roger expect to
have drawn?
THINK
1

WRITE

Calculate the probability of drawing a


spade.
Calculate the expected number of
spades by multiplying the probability
by the number of trials.

P(spade) =

1
--4

Expected number of spades =

1
--4

50

Expected number of spades = 12.5

Obviously, after drawing 50 cards, Roger could not have drawn 12.5 spades. The
number of spades drawn must of course be a whole number. However, if this experiment were repeated a number of times, we would expect to have drawn an average of
12.5 spades in every 50 cards.
The expected outcome method can be applied to any probability experiment. This
includes multistage events in which it may be necessary to draw a tree diagram or probability tree to calculate the probability of a particular outcome.

WORKED Example 3
A psychologist is conducting a study on the upbringing of boys. For the study, the
psychologist selects 100 couples with exactly three children. How many of these couples
would the psychologist expect to have three boys?
THINK
1

WRITE

Draw a tree diagram showing the


sample space for three children.

Boy
Boy
Girl
Boy
Girl
Girl

2
3

Calculate the probability of three boys.


Calculate the expected number by
multiplying the probability of three
boys by the number of couples in the
study.

Boy
Girl
Boy
Girl
Boy
Girl
Boy
Girl

P(three boys) = 1--8


Expected number of couples with three boys
=

1
--8

100

= 12.5

Chapter 7 Applications of probability

221

remember
1. The number of times an event can be expected to occur in a number of trials is
calculated by multiplying the probability of that event by the number of trials.
2. The number of times we expect an event to occur does not mean the event will
occur that number of times. Rather, this is the average number of times we
would expect this event to occur.

7A
WORKED

Example

1 Calculate the number of times that a coin can be expected to land Tails in 40 tosses.
2 A die is rolled 300 times. Calculate the expected number of 6s to be rolled.
3 A card is drawn from a standard deck, its suit is noted and the card is replaced in
the deck. Calculate the expected number of hearts in 100 selections.

7.1
Single
event
probability

7.2

Tree
diagrams

7.3

SkillS

HEET

Example

5 Lorna spends a night at the greyhounds. There are 10 races, and in each race there
are eight greyhounds. Lorna bets on number 5 in every race. Calculate the number
of winning greyhounds that Lorna can expect to back.

SkillS

HEET

4 A barrel contains five red marbles, four blue marbles and a green marble. A marble
is drawn from the barrel. Its colour is noted, and it is then replaced in the barrel. In
70 selections from the barrel, how many times would we expect to select:
a a red marble?
b a blue marble?
c a green marble?
WORKED

SkillS

HEET

Expected outcomes

Probability
trees

6 A card is drawn from a standard deck; the


card is then noted and replaced in the deck.
This is repeated 100 times. Calculate the
number of times (where necessary, correct
to 2 decimal places) that we could expect
to select:
a a club
b a red card
c an ace
d a court card (ace, king, queen or jack)
e the king of diamonds.
7 Kevin buys a ticket in a meat raffle every week. There are 100 tickets and four prizes.
a Calculate the probability of Kevin winning a prize in the raffle.
b How many prizes can Kevin expect to win in one year?
8 Janice buys a ticket in every lottery. In each lottery there are 180 000 tickets, a first
prize and 3384 cash prizes. One lottery is drawn every weekday for 52 weeks a year.
Calculate the number of times in 10 years that Janice can expect to win:
a first prize (as a decimal, correct to 3 significant figures)
b a cash prize (as a decimal, correct to 3 significant figures).

222

Maths Quest General Mathematics HSC Course

9 multiple choice
A meeting is attended by 350 men and 150 women. At the meeting 100 people will be
chosen to make a speech. What is the expected number of women to make speeches?
A 15
B 30
C 50
D 150
10 multiple choice
A tennis club runs a raffle each week with 100 tickets. Fumiko buys one ticket each
week. The expected number of raffle wins over a period of 50 weeks is:
A 0.01
B 0.5
C1
D 20
11 Four coins are tossed simultaneously in the air. If this were repeated 80 times, on how
many occasions would you expect the coins to land with four Heads?
3
12 The digits 2, 5, 6, 7 and 9 are written on cards and placed face down. Three are then
chosen and arranged to form a three-digit number. If this is repeated 150 times, what
is the expected number of:
a odd numbers?
b numbers greater than 600?
c multiples of five?

WORKED

Example

EXCE

et

reads
L Sp he

Die
rolling

13 Two dice are rolled 100 times. Copy and complete the table below to calculate the
expected number of occurrences of each total in 100 rolls of the dice. Give each answer
correct to 1 decimal place.
Outcome

10

11

12

Probability
Expected no.
14 A barrel contains 15 blue marbles and 5 red marbles. Two marbles are selected from
the barrel, the first not being replaced in the barrel before the second is chosen. This
experiment is repeated 100 times. On how many occasions (correct to 2 decimal places)
would you expect the two marbles chosen to be:
a both blue?
b both the same colour?
c different colours?
d selected with at least one being blue?

Computer Application 1 Simulations


EXCE

et

reads
L Sp he

Simulations

A simulation is where a computer gives results to an experiment that are similar to


those that would occur if the experiment were actually performed. For example, if a
coin is tossed 100 times, a computer can randomly choose Heads or Tails in a fraction
of a second. In each case, the probability of each outcome is 1--- and we are saved the
2
process of actually tossing the coin.
1. Access the spreadsheet Simulations from your Maths Quest General Mathematics
HSC Course CD-ROM.

Chapter 7 Applications of probability

223

2. The first worksheet has a coin toss simulation. In cell B3 enter the number of times
you wish to toss the coin, in cell F4 enter the expected number of heads and in cell
F5 enter the expected number of tails.
3. How do the simulation results compare with the expected outcome? Complete 10
simulations and average the results. Is this answer closer to the expected number of
outcomes that you have calculated?
4. Repeat this process for each one of the other simulations on rolling a die and rolling
two dice.

Financial expectation
We can use expected outcomes to make an assessment of financial situations where
probability is concerned. In particular, this applies to many forms of gambling. The
average financial outcome from such a situation is called the financial expectation.
Consider a simple game where two people are betting $1 on the toss of a coin. The
probability of winning the toss is

1
--2

and this will give a financial return of $1, while the

probability of losing the game is

1
--2

and this will lead to a financial loss of $1. We need

to consider a financial loss as being negative.


To calculate the financial expectation, we multiply each financial outcome by the
probability of that outcome and then add the results together. In the above example:
Financial expectation =

1
--2

= $0

$1 +

1
--2

$1

224

Maths Quest General Mathematics HSC Course

This financial expectation tells us that we can expect to neither gain nor lose money
in this game over a long period of time. This does not mean that this will be the outcome, but it is the average expected outcome.

WORKED Example 4
A game is played where a die is rolled. If a six is rolled, the player wins $6; if a five is
rolled, the player wins $3; and if any other number is rolled, the player loses $3. What is
the financial expectation from this game?
THINK

WRITE

Financial expectation is calculated by


multiplying the financial result of each
outcome by the probability of each
outcome and adding the results together.

Financial expectation =

1
--6

Financial expectation =

$3 +
$3 +

1
--6

$3 +

1
--6

$3 +

Financial expectation = $3 + 1--6


Financial expectation = $0.50

1
--6

1
--6

$6

In worked example 4, the financial expectation is negative. This means that over an
extended period of time we can expect to lose 50c per game.
This type of calculation can be applied to other financial situations such as the share
market.

WORKED Example 5
Over the past 10 years the price of a particular share has risen by $2 on five occasions,
by $1 on two occasions and has fallen by $3 on three occasions. What is the financial
expectation for this share price in the next year?
THINK
1

WRITE
5
------ ,
10

Calculate the (experimental) probability


of each outcome.

P($2 profit) =

Calculate the financial expectation


using the experimental probabilities.

Financial expectation =

P($3 loss) =

P($1 profit) =

2
------ ,
10

3
-----10
5
-----10

Financial expectation = +

$2 +
3
-----10

2
-----10

$1

$3

Financial expectation = $0.30


In this example, where the financial expectation is positive, we can expect to make a
profit. Again this does not mean we will make a profit but the average share price
fluctuation is a gain of 30c.

remember
1. Financial expectation is the average return in a financial situation.
2. The financial expectation is calculated by multiplying each possible financial
outcome by the probability of that financial outcome and adding the results
together.
3. A financial loss is indicated by a negative financial outcome while a financial
gain is a positive financial outcome.

Chapter 7 Applications of probability

7B
WORKED

Example

225

Financial expectation

1 A game is played where a die is rolled. If a 1 or a 6 is rolled, the player wins $2; if
any other number is rolled, the player loses $1. What is the financial expectation from
this game?
2 There are five cards labelled 1, 2, 3, 4 and 5. A card is selected. If it is even, you win
$5, and if it is odd, you lose $4. Calculate the financial expectation.
3 Soon-Jung plays a game in which two coins are tossed. If he throws two Heads, he
wins $5; if he throws two Tails, he loses $3. For one Head and one Tail, he loses $2.
Calculate the financial expectation from this game.
4 In a card game, the player selects a card from a standard deck. The player then wins
$5 for an ace and $2 for a king, queen or jack. If any other card is selected, $1 is lost.
Calculate the financial expectation from this game.
5 A raffle has 1000 tickets that sell for $1 each. There is a first prize of $400, a second
prize of $200 and a third prize of $100. Calculate the financial expectation from the
purchase of one ticket in the raffle.

WORKED

Example

6 Over the past 20 years shares in the company FIA have increased by $5 on eight
occasions, increased by $2 on six occasions and fallen by $3 on six occasions.
Calculate the financial expectation for a person who buys FIA shares for the coming
year.
7 Look at the roulette wheel on the right.
a How many slots are on the wheel?
b How many of these slots are:
iii black?
iii red?
iii green?
c Francis bets $10 on black. If a black number
is spun, he wins $10; otherwise, he loses
$10. Calculate Franciss financial expectation.
8 multiple choice
A game is played where a die is rolled. The cost of the game is $1. The players are
returned their $1 plus an extra $5 if they can roll a 6. The financial expectation from
this game is:
A0
B 0.17
C 0.17
D 1
9 multiple choice
Which of the following games has the greatest financial expectation?
A A coin is tossed. Players win $1 if they toss a Head; otherwise, $1 is lost.
B Two coins are tossed. Players win $2 if they toss two Heads; otherwise, $1 is lost.
C A die is rolled. The player wins $6 for rolling a 6; otherwise, $1 is lost.
D Two dice are rolled. The player wins $6 for rolling a total of six; otherwise, $1 is
lost.

226

Maths Quest General Mathematics HSC Course

Work

10 In a dice game, two dice are rolled.


The player wins $1 for rolling a total of 7 or 11.
The player loses $1 for rolling a total of 2, 3 or 12.
If any other total is rolled, the dice are rolled again.
What is the financial expectation from this game?

T
SHEE

7.1

11 In the Jackpot lottery there are 180 000 tickets sold at $2 each. The prizes are shown
below.
1st prize $100 000
2nd prize $10 000
3rd prize $5000
2 prizes of $1000
2 prizes of $500
5 prizes of $200
12 prizes of $100
60 prizes of $50
600 prizes of $20
2700 prizes of $10
Calculate the financial expectation from purchasing a $2 lottery ticket.

1
1 Calculate the expected number of sixes in 120 rolls of a die.
Information for questions 2 to 5.
A pack of cards is shuffled, a card is chosen and then returned to the deck. The cards are
then shuffled again. If this process is repeated 100 times, calculate (correct to 1 decimal
place) the expected number of:
2 clubs
3 red cards
4 kings
5 court cards.
6 A game is played where a die is rolled. The player wins $3 for a six, $2 for a five and
loses $1 for any other result. Calculate the financial expectation for this game.
7 A game is played where two dice are rolled. The player wins $20 for a total of 12, $10
for a total of 2 and loses $1 for any other total. Calculate the financial expectation for
this game.
8 A game is played where the financial expectation is 0.2. Explain what this means.
9 A game is played where the financial expectation is 0.2. Explain what this means.
10 Over the past 10 years the share price in a company has risen by $5 in three of the
years and has fallen by $1.50 in the other seven years. Based upon these results, if I
purchase shares in this company, what would be my financial expectation for the year
ahead?

Chapter 7 Applications of probability

227

Two-way tables
A two-way table is a two-dimensional grid that shows the outcome of an experiment in
terms of two variables. A two-way table is used to display information and allows for
predictions to be made based on this information.
Consider an example where 400 newborn babies are tested for a genetic condition.
The two-way table below displays these results.
Test results
Accurate

Not accurate

Total

85

94

Without condition

304

306

Total

389

11

With condition

The information that is given to us by this two-way table is that:


94 babies have the condition of which 85 were diagnosed and 9 were not
306 babies did not have the condition of which 304 were shown not to have the condition by the test and 2 who were told they had the condition but they did not (these
are known as false positives).
From such a two-way table we can tell the total number of babies with and without the
condition and the total number of correct and incorrect diagnoses made.

WORKED Example 6
A new test was designed to assess the reading ability of students entering high school.
The results were used to determine if the students reading level was adequate to cope
with high school. The students results were then checked against existing records.
150 adequate readers sat for the test and 147 of them passed.
50 inadequate readers sat for the test and 9 of them passed.
Present this information in a two-way table.
THINK

WRITE

Draw up the table showing the number of


students whose reading was adequate and
the number of students for whom the
results of the new test were confirmed.

Test results

Adequate
readers
Inadequate
readers
Total

Passed

Did not
pass

Total

147

150

41

50

156

44

228

Maths Quest General Mathematics HSC Course

When information is presented in a two-way table, conclusions can be made about the
accuracy of such a test and calculations can be made about the probability that such a
test is accurate.

WORKED Example 7

A batch of sniffer dogs is trained by customs to smell drugs in suitcases. Before they are
used at airports they must pass a test. The results of that test are shown in the two-way
table below.
Test results
Detected

Not detected

Total

No. of bags with drugs

24

25

No. of bags without drugs

11

164

175

Total

35

165

a
b
c
d

How many bags did the sniffer dogs examine?


In how many bags did the dogs detect drugs?
In what percentage of bags without drugs did the dogs incorrectly detect drugs?
Based on the above results, what is the probability that the dogs will not detect a bag
carrying drugs?

THINK

WRITE

a Add both total columns; they should


give the same result.

a 200 bags were examined.

b The total of the detected column.

b The dogs detected drugs in 35 bags.

c There were 175 bags without drugs but


dogs incorrectly detected them in
11 bags. Write this as a percentage.

c Percentage incorrectly detected

d Use the probability formula. Of 25 bags


with drugs, 1 went undetected.

d P(bag going undetected) =

11
--------175

100%

= 6.3%
1
-----25

As a result of studying a two-way table, we should also be able to make judgements


about the information given in the tables. In the above worked example only one bag
out of 25 with drugs went undetected. Although the dogs incorrectly detected drugs in
11 bags that did not have them, they still have an overall accuracy of 94%.
Many two-way tables will require you to make your own value judgements about the
conclusions established by the test. For example, the 94% overall accuracy recorded
above may be considered very acceptable.

229

Chapter 7 Applications of probability

remember
1. A two-way table is used to display test results and examine the accuracy of
these results.
2. The table displays horizontally the numbers with and without a certain
condition, and vertically displays information about accuracy.
3. The table can be used to make calculations about the accuracy of the test and
about the probability of those test results being accurate in an individual case.

7C

Two-way tables

1 A test is developed to test for the flu virus. To test the accuracy, the following 500
L Spre
XCE ad
people
are
tested.
6
100 people who are known to have the flu are tested and the test returns 98 positive Two-way
frequency
results.
tables
400 people who are known not to be infected with the virus are tested with 12 false
positives being returned.

WORKED

Display this information in the two-way table below.


Test results
Accurate

Not accurate

Total

With virus
Without virus
Total
2 One thousand people take a lie detector test. Of 800 people known to be telling the
truth, the lie detector indicates that 23 are lying. Of 200 people known to be lying, the
lie detector indicates that 156 are lying. Present this information in a two-way table.
3 The two-way table shown below displays the information gained from a medical test
screening for a virus. A positive test indicates that the patient has the virus.
7
Test results

WORKED

Example

Accurate

Not accurate

Total

45

48

Without virus

922

30

952

Total

967

33

With virus

a How many patients were screened for the virus?


b How many positive tests were recorded? (that is, in how many tests was the virus
detected?)
c What percentage of test results were accurate?
d Based on the medical results, if a positive test is recorded, what is the probability
that you actually have the virus?

sheet

Example

230

Maths Quest General Mathematics HSC Course

4 The two-way table below indicates the results of a radar surveillance system. If the
system detects an intruder, an alarm is activated.
Test results
Alarm activated
Intruders
No intruders
Total

Not activated

Total

40

48

148

152

44

156

a Over how many nights was the system tested?


b On how many occasions was the alarm activated?
c If the alarm was activated, what is the probability that there actually was an
intruder?
d If the alarm was not activated, what is the probability that there actually was an
intruder?
e What was the percentage of accurate results over the test period?
f Comment on the overall performance of the radar detection system.
The information below is to be used in questions 5 to 7.
A test for a medical disease does not always produce the correct result. A positive test
indicates that the patient has the condition. The table indicates the results of a trial on a
number of patients who were known to either have the disease or known not to have the
disease.
Test results
Accurate

Not accurate

Total

57

60

Without disease

486

54

540

Total

543

57

With disease

5 multiple choice
The overall accuracy of the test is:
A 90%

B 90.5%

C 92.5%

D 95%

6 multiple choice
Based on the table, what is the probability that a patient who has the disease has it
detected by the test?
A 90%

B 90.5%

C 92.5%

D 95%

Chapter 7 Applications of probability

231

7 multiple choice
Which of the following statements is correct?
A The test has a greater accuracy with positive tests than with negative tests.
B The test has a greater accuracy with negative tests than with positive tests.
C The test is equally accurate with positive and negative test results.
D There is insufficient information to compare positive and negative test results.
8 Airport scanning equipment is tested by scanning 200 pieces of luggage.
Prohibited items were placed in 50 bags and the scanning equipment detected 48 of
them.
The equipment detected prohibited items in five bags that did not have any forbidden
items in them.
a Use the above information to complete the two-way table below.
Test results
Accurate

Not accurate

Total

Bags with prohibited


items
Bags with no
prohibited items

b Use the table to answer the following:


i What percentage of bags with prohibited items were detected?
ii What was the percentage of false positives among the bags that had no
prohibited items?
iii What is the probability of prohibited items passing through the scanning equipment undetected?
iv What is the overall percentage accuracy of the scanning equipment?

Work

Total

T
SHEE

7.2

232

Maths Quest General Mathematics HSC Course

summary
Expected outcomes
The expected number of times that an event will occur in a number of trials is
calculated by multiplying the number of trials by the probability of that event
occurring.
The expected number of outcomes is the average number of times that the event is
expected to occur. It does not mean this is the number of times the event will occur.

Financial expectation
Financial expectation is the average financial position at the end of a situation
where either a profit or loss will be made.
The financial expectation is calculated by multiplying each possible financial
outcome by the probability of that outcome and then adding the results together.

Two-way tables
A two-way table is used to display the results of a test and assesses the accuracy of
such a test.
The table can be used to calculate the overall probability of the test achieving its
objectives.

Chapter 7 Applications of probability

233

CHAPTER
review
1 Thirty-six coins are tossed in the air. Calculate the expected number of coins landing Heads.
2 A die is rolled 60 times. Calculate the expected number of:
a 6s
b even numbers

c numbers less than 3.

7A
7A

3 A card is chosen from a standard deck, noted and replaced in the deck. In 100 trials,
calculate (where necessary, correct to 2 decimal places) the expected number of:
a red cards
b spades
c aces
d court cards
e black jacks.

7A

4 Two dice are rolled. The score in each roll is the total of the two dice. In 90 rolls of the dice,
calculate the expected number of:
a twos
b sevens
c tens
d doubles
e totals greater than 8.

7A

5 In a game, three coins are tossed in the air. In 100 tosses of the coins, on how many
occasions would you expect the coins to land:
a three Heads?
b two Tails and one Head?
c more Heads than Tails?

7A

6 Two-digit numbers are formed using the digits 2, 4, 7 and 8, and no digit may be repeated.
If 60 such numbers are formed, how many numbers can be expected to be:
a 47?
b even?
c less than 40?

7A

7 Alex bets $10 on the toss of a coin. He calls Heads. If the coin lands Heads, Alex wins $10;
if it lands Tails, he loses $10. What is his financial expectation?

7B

8 A bag contains 10 marbles, each with an amount of money written on it. Five marbles have
$1 written on them, two have $2 written on them and the others have $5, $10 and $20
written on them. A player pays $5 to draw a marble from the bag and is then returned the
amount of money on the marble that is drawn. Calculate the financial expectation from this
game.

7B

9 Explain the difference between a positive and negative financial expectation.

7B
7B

10 A roulette wheel is spun (see photograph page 210). Carly bets $1 on number 25. If 25 is the
number spun, Carly will win $35 and have her $1 returned; if not, she will lose $1. Calculate
the financial expectation from this game.
11 Jason plays a game where he rolls two dice. If he rolls a total greater than 9, he wins $5;
otherwise, he loses $1. Calculate the financial expectation from this game.

7B

12 A bag contains 20 marbles of which 10 are black, 9 are white and 1 is red. Kerry draws a
marble from the bag at random. If a black marble represents a $5 loss, a white marble a $4
gain and a red marble a $20 gain, calculate the financial expectation from this game.

7B

13 Over the past 15 years the share price of PHB has risen by $4 in 12 of the years, fallen by
$5 in two years and fallen by $10 in the others. If I buy shares in PHB, what would my
financial expectation be for the coming year?

7B

234
7C

Maths Quest General Mathematics HSC Course

14 A medical test screens 200 people for a virus. A positive test result indicates that the patient
has the virus.
Of 50 people known to have the virus, the test produced 48 positive results.
Of the remainder who were known not to have the virus, the test produced one positive result.
Use the above information to complete the table below.
Test results
Accurate

Not accurate

Total

With virus
Without virus
Total

7C

15 The results of a lie detector test are given below.


Of 80 people known to be telling the truth, the lie detector indicates that three are lying.
Of 20 people known to be lying, the lie detector indicates that 17 are lying.
Display this information in a two-way table.

Chapter 7 Applications of probability

235

16 Below are the results of a test screening for a disease. A positive test indicates that the
patient has the disease.

7C

Test results
Accurate

Not accurate

Total

18

20

Without disease

108

12

120

Total

126

14

With disease

a How many people were tested for the disease?


b How many positive test results were recorded?
c What percentage of those people with the disease were correctly diagnosed by
the test?
d If a person without the disease is chosen at random, what is the probability that
they returned a positive test?
17 A reading test for people with dyslexia is given and the results are shown in the two-way
table below.
Test results
Accurate

Not accurate

Total

With dyslexia

39

40

Without dyslexia

85

90

124

Total

a How many people were tested?


b What percentage of people tested positive to dyslexia?
c Based on the above results, if a person with dyslexia takes the test, what is the probability
that they will be accurately diagnosed?

Practice examination questions


1 multiple choice
A bag contains 3 red marbles, 13 blue marbles and 4 yellow marbles. A marble is chosen
from the bag and then replaced in the bag. In 90 selections, the expected number of blue
marbles selected is:
A 13
B 20
C 58.5
D 59
2 multiple choice
A game is played where the player tosses four coins in the air. If all four coins have the same
face up, the player wins $6. Otherwise the player loses $1. The financial expectation from this
game is:
A $1.00
B $0.125
C $0.125
D $6.00

7C

236

Maths Quest General Mathematics HSC Course

3 multiple choice
The two-way table below shows the results of a trial on new metal detectors for aircraft. The
metal detector scans a piece of hand luggage and lights up if metal is found.
Test results
Accurate

Not accurate

Total

10

Without metal

87

90

Total

96

With metal

Based on the above results, the probability of metal going undetected in a piece of hand
luggage is:
A 10%
B 25%
C 75%
D 90%
4 A game is played where two dice are rolled.
a Calculate the probability of rolling a total of 7.
b How many times would you expect to roll a total of 7 in 90 rolls of two dice?
c Calculate the probability of rolling a total of 11.
d Xiao plays a game where he wins $3 for rolling a total of 7 and $7 for rolling a total of 11.
Otherwise he loses $1. Calculate the financial expectation for this game.
5 A medical test for a disease does not always give the correct result. A positive test indicates
that the patient has the disease. The two-way table below shows the results of a new screening
test for the disease. It was tested on a group of people, some of whom were known to be
suffering from the disease, some of whom were not.
Test results
Accurate

Not accurate

Total

28

30

Without disease

164

170

Total

192

With disease

CHAPTER

test
yourself

a
b
c
d
e

How many people were tested for the disease?


What percentage of the results were accurate?
How many patients tested positive to the disease?
What percentage of patients with the disease were correctly diagnosed by the new test?
Based on the above results, what is the probability that a patient with the disease will have
the disease detected by this test?

Annuities
and loan
repayments

8
syllabus reference
Financial mathematics 5
Annuities and loan
repayments

In this chapter
8A Future value of an annuity
8B Present value of an
annuity
8C Future and present value
tables
8D Loan repayments

areyou

READY?

Are you ready?

Try the questions below. If you have difficulty with any of them, extra help can be
obtained by completing the matching SkillSHEET. Either click on the SkillSHEET icon
next to the question on the Maths Quest HSC Course CD-ROM or ask your teacher for
a copy.

8.1

Finding values of n and r in financial formulas

8.2

Calculating simple interest

1 Find the value of n and r in for each of the following investments.


a Interest of 8% p.a. for 5 years, with interest calculated annually
b Interest of 6% p.a. for 4 years, with interest calculated six-monthly
c Interest of 7.6% p.a. for 3 years, with interest calculated quarterly
d Interest of 9.6% p.a. for 10 years, with interest calculated monthly
e Interest of 24% p.a. for November, with interest calculated daily

2 Find the simple interest on each of the following investments.


a $25 000 invested at 5% p.a. for 4 years
b $15 500 invested at 8.2% p.a. for 6 years
c $42 000 invested at 9.4% p.a. for 18 months

8.3

Calculating compound interest

3 Find the compound interest earned on each of the following investments.


a $12 000 invested at 6% p.a. for 3 years, with interest compounded annually
b $35 000 invested at 8% p.a. for 5 years, with interest compounded six-monthly
c $56 000 invested at 7.2% p.a. for 4 years, with interest compounded quarterly

8.4

Reading financial tables

4 The table below shows the amount to which $1 will grow under compound interest.
Interest rate per period
Periods

6%

7%

8%

9%

1.060

1.070

1.080

1.090

1.123

1.145

1.166

1.188

1.191

1.225

1.260

1.295

1.262

1.311

1.360

1.412

Use the table to find the future value of each of the following investments.
a $8000 at 6% for 2 years, with interest compounded annually
b $12 500 at 8% p.a. for 3 years, with interest compounded annually
c $18 000 at 12% p.a. for 2 years, with interest compounded six-monthly

Chapter 8 Annuities and loan repayments

239

Future value of an annuity


An annuity is a form of investment involving regular periodic contributions to an
account. On such an investment, interest compounds at the end of each period and the
next contribution to the account is then made.
Superannuation is a common example of an annuity. Here, people invest in a fund on
a regular basis, the interest on the investment compounds, while the principal is added
to for each period. The annuity is usually set aside for a persons entire working life
and is used to fund retirement. It may also be used to fund a long-term goal, such as a
trip in 10 years time.
To understand the growth of an annuity, we need to revise compound interest. The
compound interest formula is:
A = P(1 + r)n
where A is the final balance, P is the initial quantity, r is the interest rate per
compounding period and n is the number of compounding periods.

WORKED Example 1

Calculate the value of a $5000 investment made at 8% p.a. for 4 years.


THINK
1
2
3
4

WRITE

Write the values of P, r and n.


Write the formula.
Substitute values for P, r and n.
Calculate the value of A.

P = $5000, r = 0.08, n = 4
A = P(1 + r)n
A = $5000 (1.08)4
A = $6802.44

An annuity takes the form of a sum of compound interest investments. Consider the
case of a person who invests $1000 at 10% p.a. at the end of each year for five years.
To calculate this, we would need to calculate the value of the first $1000 that is
invested for four years, the second $1000 that is
invested for three years, the third $1000 that is
invested for two years, the
fourth $1000 that is invested for
one year and the last $1000 that
is added to the investment.

240

Maths Quest General Mathematics HSC Course

WORKED Example 2
Calculate the value of an annuity in which $1000 is invested at the end of each year at
10% p.a. for 5 years.
THINK
1 Use the compound interest formula to
calculate the amount to which the first
$1000 will grow.
2 Use the compound interest formula to
calculate the amount to which the
second $1000 will grow.
3 Use the compound interest formula to
calculate the amount to which the third
$1000 will grow.
Use
the compound interest formula to
4
calculate the amount to which the
fourth $1000 will grow.
5 Find the total of the separate $1000
investments, remembering to add the
final $1000.

WRITE
A = P(1 + r)n
A = $1000 1.14
A = $1464.10
A = P(1 + r)n
A = $1000 1.13
A = $1331.00
A = P(1 + r)n
A = $1000 1.12
A = $1210.00
A = P(1 + r)n
A = $1000 1.1
A = $1100.00
Total value = $1464.10 + $1331.00 + $1210.00
Total value = + $1100.00 + $1000
Total value = $6105.10

In most cases it is more practical to calculate the total value of an annuity using a formula. The amount to which an annuity grows is called the future value of an annuity
and can be calculated using the formula:
( 1 + r )n 1
A = M ---------------------------r
where M is the contribution per period paid at the end of the period, r is the interest rate
per period expressed as a decimal, and n is the number of deposits.
( 1 + r )n 1
For the above example: A = M ---------------------------r

1.1 5 1
= $1000 -----------------0.1

= $6105.10

WORKED Example 3
Bernie invests $2000 in a retirement fund at 5% p.a. interest compounded annually at the
end of each year for 20 years. Calculate the future value of this annuity at retirement.
THINK
1 Write the values of M, r, and n.

WRITE
M = $2000, r = 0.05, n = 20

Write the formula.

( 1 + r )n 1
A = M ---------------------------r

Substitute values for M, r and n.

1.05 20 1
A = $2000 -----------------------0.05

Calculate.

A = $66 131.91

Chapter 8 Annuities and loan repayments

241

In some examples, calculations will need to be made when contributions are made
more often than once a year and when interest compounds more often than once a year.

WORKED Example 4
Christina invests $500 in a fund every 6 months at 9% p.a. interest, compounding
six-monthly for 10 years. Calculate the future value of the annuity after 10 years.
THINK

WRITE
9% p.a. = 4.5% for 6 months
So, r = 0.045 and n = 20.

Write the values of M, r and n by


considering the interest rate as 4.5% per
interest period and 20 interest periods.

Write the formula.

( 1 + r )n 1
A = M ---------------------------r

Substitute for M, r and n.

1.045 20 1
A = $500 --------------------------0.045

Calculate.

A = $15 685.71

If we rearrange the formula for an annuity to make M (the contribution per period) the
subject of the formula, we have:
Ar
M = --------------------------( 1 + r )n 1
This formula would be used when we know the final amount to be saved and wish to
calculate the amount of each regular deposit.

WORKED Example 5
Vikki has the goal of saving $10 000 in the next five years. The best interest rate that she
can obtain is 8% p.a., with interest compounded annually. Calculate the amount of each
annual contribution that Vikki must make.
THINK

WRITE

Write the values of A, r and n.

A = $10 000, r = 0.08, n = 5

Write the formula.

Ar
M = --------------------------( 1 + r )n 1

Substitute for A, r and n. Hint: insert


brackets when using your calculator.

( 10 000 0.08 )
M = -------------------------------------( 1.08 5 1 )

Calculate the value of M.

M = $1704.56

242

Maths Quest General Mathematics HSC Course

remember
1. The compound interest formula is:
A = P(1 + r)n
where A is the final balance, r is the interest rate per period expressed as a
decimal and n is the number of compounding periods.
2. An annuity is a form of investment where periodical equal contributions are
made to an account, with interest compounding at the end of each period.
3. The value of an annuity is calculated by adding the value of each amount
contributed as a separate compound interest investment.
4. We can calculate the value of an annuity by using the formula:
( 1 + r )n 1
A = M ---------------------------r
where M is the contribution per period, paid at the end of the period, r is the
interest rate per period expressed as a decimal and n is the number of deposits.
5. The amount of each contribution to annuity to reach a certain goal can be
calculated using the formula:
Ar
M = --------------------------( 1 + r )n 1

SkillS

8A
HEET

8.1

WORKED

Example

SkillS

Finding
values of
n and r in
financial
formulas
HEET

8.2
Calculating
simple
interest WORKED

SkillS

Example

HEET

8.3
Calculating
compound
interest

Future value of an annuity

1 Calculate the value after 5 years of an investment of $4000 at 12% p.a., with interest
compounded annually.
2 Calculate the value to which each of the following compound interest investments
will grow.
a $5000 at 6% p.a. for 5 years, with interest calculated annually
b $12 000 at 12% p.a. for 3 years, with interest calculated annually
c $4500 at 8% p.a. for 4 years, with interest compounded six-monthly
d $3000 at 9.6% p.a. for 3 years, with interest compounded six-monthly
e $15 000 at 8.4% p.a. for 2 years, with interest compounded quarterly
f $2950 at 6% p.a. for 3 years, with interest compounded monthly
3 At the end of each year for four years Rodney invests $1000 in an investment fund
that pays 7.5% p.a. interest, compounded annually. By calculating each investment of
$1000 separately, use the compound interest formula to calculate the future value of
Rodneys investment after four years.
4 Caitlin is saving for a holiday in two years and so every six months she invests $2000
in an account that pays 7% p.a. interest, with the interest compounding every six months.
a Use the compound interest formula to calculate the amount to which the:
i first investment of $2000 will grow
ii second investment of $2000 will grow
iii third investment of $2000 will grow
iv fourth investment of $2000 will grow.
b If Caitlin then adds a final deposit of $2000 to her account immediately before her
holiday, what is the total value of her annuity?

Chapter 8 Annuities and loan repayments

WORKED

Example

( 1 + r )n 1
5 Use the formula A = M ---------------------------r

243

to find the future value of an annuity in

which $1000 is invested each year for 25 years at an interest rate of 8% p.a.
6 When baby Shannon was born, her grandparents deposited $500 in an account that
pays 6% p.a. interest, compounded annually. They added $500 to the account each
birthday, making the last deposit on Shannons 21st birthday.
a How many deposits of $500 were made?
b The investment was given to Shannon as a 21st birthday present. What was the
total value of the investment at this point? (Hint: Use the answer to part a.)
c Shannons grandparents advised Shannon to keep adding $500 to the investment
each birthday so that she had a retirement fund at age 60. If Shannon follows this
advice, what will the investment be worth at age 60? (Assume Shannon makes the
last deposit on her 60th birthday.)
7 Calculate the future value of each of the following annuities.
a $2000 invested at the end of each year for 10 years, at an interest rate of 5% p.a.
b $5000 invested at the end of each year for 5 years, at an interest rate of 8% p.a.
c $10 000 invested at the end of each year for 20 years, at an interest rate of
7.5% p.a.
d $500 invested at the end of each year for 30 years, at an interest rate of 15% p.a.
e $25 000 invested at the end of each year for 4 years, at an interest rate of 9.2% p.a.
8 Darlene is saving for a deposit on a unit. She hopes to buy one in four years and needs
a $30 000 deposit, so she invests $5000 per year in an annuity at 7.5% p.a. starting on
1 January 2007.
a After the last deposit is made on 1 January 2011, how many deposits has Darlene
made?
b Use the annuity formula to calculate if Darlene would have saved enough for her
deposit.
c How much interest was paid to Darlene on this annuity?
WORKED

Example

9 At the end of every six months Jason invests $800 in a retirement fund which pays
interest at 6% p.a., with interest compounded six-monthly. Jason does this for
25 years. Calculate the future value of Jasons annuity after 25 years.
10 Calculate the future value of each of the following annuities on maturity.
a $400 invested at the end of every six months for 12 years at 12% p.a., with interest
compounded six-monthly
b $1000 invested at the end of every quarter for 5 years at 8% p.a., with interest compounded every quarter
c $2500 invested at the end of each quarter at 7.2% p.a., for 4 years with interest
compounded quarterly
d $1000 invested at the end of every month for 5 years at 6% p.a., with interest compounded monthly
11 multiple choice
The interest earned on $10 000 invested at 8% p.a. for 10 years, with interest compounded annually, is:
A $11 589.25
B $21 589.25
C $134 865.62
D $144 865.62

244

Maths Quest General Mathematics HSC Course

12 multiple choice
Tracey invests $500 in a fund at the end of each year for 20 years. The fund pays
12% p.a. interest, compounded annually. The total amount of interest that Tracey
earns on this fund investment is:
A $4323.15
B $4823.23
C $26 026.22
D $36 026.22
13 Thomas has the goal of saving $400 000 for his retirement in 25 years. If the best
interest rate that Thomas can obtain is 10% p.a., with interest compounded annually,
5
calculate the amount of each annual contribution that Thomas will need to make.

WORKED

Example

14 Calculate the amount of each annual contribution needed to obtain each of the
following amounts.
a $25 000 in 5 years at 5% p.a., with interest compounded annually
b $100 000 in 10 years at 7.5% p.a., with interest compounded annually
c $500 000 in 40 years at 8% p.a., with interest compounded annually
15 Leanne is 24 years old and invests $30 per week in her superannuation fund. Leannes
employer matches this amount.
a If Leanne plans to retire at 60, calculate the total that Leanne will contribute to the
fund at this rate.
b Calculate the total contributions that will be made to the fund at this rate.
c If the fund returns an average 4% p.a. interest, compounded annually, calculate the
future value of Leannes superannuation.

Computer Application 1 Annuity calculator


EXCE

et

reads
L Sp he

Annuity
calculator

Access the spreadsheet Annuity calculator from the Maths Quest General
Mathematics HSC Course CD-ROM. The spreadsheet will show you the growth of an
annuity in which $1000 is invested at the end of each year for 20 years at a rate of
8% p.a. interest, compounding annually.

Chapter 8 Annuities and loan repayments

245

1. The spreadsheet shows that after 20 years the value of this investment is $45 761.96.
Below is the growth of the annuity after each deposit is made. This will allow you to
see the growth for up to 30 deposits. From the Edit menu, use the Fill Down functions on the spreadsheet to see further.
2. Click on the tab, Chart1. This is a line graph that shows the growth of the annuity
for up to 30 deposits.
3. Change the size of the deposit to $500 and the compounding periods to 2. This will
show how much benefit can be achieved by reducing the compounding period.
4. Check your answers to the previous exercise by using the spreadsheet.

1
1 Find the future value of $5000 invested at 10% p.a. for 6 years, with interest compounded annually.
2 Find the total amount of interest earned on an investment of $3200 invested for
4 years at 8% p.a., with interest compounded every six months.
3 Find the future value of an annuity of $1600 invested every year for 5 years at
12% p.a., with interest compounded annually.
4 Find the future value of an annuity of $2000 invested every year for 30 years at
7.2% p.a., with interest compounded annually.
5 Find the future value of an annuity in which $400 is invested every three months for
12 years at 8% p.a., with interest compounded quarterly.
6 Find the future value of an annuity in which $350 is invested each month for 10 years
at 9.2% p.a. interest, compounding every six months.
7 Find the interest earned on an annuity of $750 invested per year for 10 years at
8.5% p.a., with interest compounding annually.
8 Find the amount of each annual contribution needed to achieve a future value of
$100 000 if the investment is made for 10 years at an interest rate of 11% p.a., with
interest compounding annually.
9 Find the amount of each quarterly contribution needed to save $15 000 in five years at
12% p.a., with interest compounding quarterly.
10 Find the amount of each six-monthly contribution to an annuity if the savings goal is
$50 000 in 15 years and the interest rate is 8% p.a., with interest compounding sixmonthly.

246

Maths Quest General Mathematics HSC Course

Present value of an annuity


To compare an annuity with a single sum investment, we need to use the present
value of the annuity. The present value of an annuity is the single sum of money
that, invested on the same terms as the annuity, will produce the same financial result.
To calculate the present value of an annuity, N, we can use the formula:
A
N = ------------------n(1 + r )
where A is the future value of the annuity
r is the percentage interest rate per compounding period, expressed as a decimal
n is the number of deposits to be made in the annuity

WORKED Example 6
Ashan has an annuity that has a future value of $500 000 on his retirement in 23 years.
The annuity is invested at 8% p.a., with interest compounded annually. Calculate the
present value of Ashans annuity.
THINK

WRITE

Write the values of A, r and n.

A = $500 000, r = 1.08, n = 23

Write the formula.

A
N = ------------------n(1 + r )

Substitute for A, r and n.

Calculate.

500 000
N = -----------------1.08 23
N = $85 157.64

In many cases you will not know the future value of the annuity when calculating the
present value. You will know only the amount of each contribution, M. We know that:
A
N = ------------------n(1 + r )
( 1 + r )n 1
Using the formula A = M ---------------------------r

to substitute for A gives:

( 1 + r )n 1
N = M --------------------------r ( 1 + r )n
This formula allows us to calculate the single sum needed to be invested to give the
same financial result as an annuity where we are given the size of each contribution.

Chapter 8 Annuities and loan repayments

247

WORKED Example 7
Jenny has an annuity to which she contributes $1000 per year at 6% p.a. interest,
compounded annually. The annuity will mature in 25 years. Calculate the present value of
the annuity.
THINK

WRITE

Write the values of M, r and n.

M = $1000, r = 0.06, n = 25

Write the formula.

( 1 + r )n 1
N = M --------------------------r ( 1 + r )n

Substitute for M, r and n.

N = 1000

Calculate.

N = $12 783.36

1.06 25 1
------------------------------0.06 1.06 25

This present value formula can be used to compare investments of different types. The
investment with the greater present value will produce the greater financial outcome
over time.

WORKED Example 8
Which of the following investments would give the greater financial return?
Investment A: an annuity of $100 deposited per month for 20 years at 12% p.a. interest,
compounding six-monthly
Investment B: a single deposit of $10 000 invested for 20 years at 12% p.a., with interest
compounding six-monthly
THINK

WRITE

The investments can be compared by


calculating the present value of the
annuity.
Consider the deposits of $100 per
month to be $600 every six months.
Write the values of M, r and n.

Write the formula.

( 1 + r )n 1
N = M --------------------------r ( 1 + r )n

Substitute for M, r and n.

N = $600

Calculate.
Make a conclusion.

N = $9027.78
The annuity has a lower present value than the
single investment. Therefore, the investment of
$10 000 will produce a greater outcome over
20 years.

M = $600, r = 0.06, n = 40

1.06 40 1
------------------------------0.06 1.06 40

248

Maths Quest General Mathematics HSC Course

remember
1. The present value of an annuity is the single sum that can be invested under the
same terms as an annuity and will produce the same financial outcome.
2. The present value of an annuity can be calculated using the formula:
A
N = ------------------n(1 + r )
when we know the future value of the annuity.
3. If we know the amount of each contribution of the annuity, we can calculate the
present value using the formula
( 1 + r )n 1
N = M --------------------------r ( 1 + r )n
where M is the contribution per period, paid at the end of the period
r is the percentage interest rate per compounding period (expressed as a
decimal)
n is the number of interest periods
4. Investments can be compared using the present value formula. The investment
with the greater present value will produce the greater financial outcome over
time.

8B
WORKED

Example

Present value of an annuity

1 Calculate the present value of an investment that is needed to have a future value of
$100 000 in 30 years time if it is invested at 9% p.a., with interest compounded
annually.
2 Calculate the present value of an investment required to generate a future value of:
a $20 000 in 5 years time at 10% p.a., with interest compounded annually
b $5000 in 4 years time at 7.2% p.a., with interest compounded annually
c $250 000 in 20 years time at 5% p.a., with interest compounded annually.
3 Calculate the present value of an investment at 7.2% p.a., with interest compounded
quarterly, if it is to have a future value of $100 000 in 10 years time.
4 Calculate the present value of the investment required to produce a future value of
$500 000 in 30 years time at 9% p.a., with interest compounded:
a annually
b six-monthly
c quarterly
d monthly

WORKED

Example

5 Craig is paying into an annuity an amount of $500 per year. The annuity is to run for
10 years and interest is paid at 7% p.a., with interest compounded annually. Calculate
the present value of this annuity.
6 Calculate the present values of each of the following annuities.
a $1000 per year for 30 years at 8% p.a., with interest compounded annually
b $600 per year for 20 years at 7.5% p.a., with interest compounded annually
c $4000 per year for 5 years at 11% p.a., with interest compounded annually
d $200 per month for 25 years at 8.4% p.a., with interest compounded annually

Chapter 8 Annuities and loan repayments

249

7 Darren pays $250 per month into an annuity that pays 5.6% p.a. interest, compounded
quarterly. If the annuity is to run for 10 years, calculate the present value of the annuity.
8 Calculate the present value of a 40-year annuity with interest at 9.6% p.a.,
compounded monthly, if the monthly contribution to the annuity is $50.
9 multiple choice
An annuity is at 12% p.a. for 10 years, with interest compounded six-monthly, and
has a future value of $100 000. The present value of the annuity is:
A $31 180.47
B $32 197.32
C $310 584.82
D $320 713.55
10 multiple choice
An annuity consists of quarterly deposits of $200 that are invested at 8% p.a., with
interest compounded quarterly. The annuity will mature in 23 years. The present value
of the annuity is:
A $1236.65
B $2074.21
C $8296.85
D $8382.72
11 Which of the following investments will have the greater financial outcome?
Investment A: an annuity of $400 per year for 30 years at 6.9% p.a., with interest
8
compounded annually
Investment B: a single investment of $5000 for 30 years at 6.9% p.a., with interest
compounded annually

WORKED

Example

12 multiple choice
Which of the following investments will have the greatest financial outcome?
A An annuity of $1200 per year for 30 years at 8% p.a., with interest compounded annually
B An annuity of $600 every six months for 30 years at 7.9% p.a., with interest compounded six-monthly
C An annuity of $300 every quarter for 30 years at 7.8% p.a., with interest compounded quarterly
D An annuity of $100 per month at 7.5% p.a., for 30 years with interest compounded
monthly.

Work

13 Kylie wants to take a world trip in 5 years time. She estimates that she will need
$25 000 for the trip. The best investment that Kylie can find pays 9.2% p.a. interest,
compounded quarterly.
a Calculate the present value of the investment needed to achieve this goal.
b Kylie plans to save for the trip by depositing $100 per week into an annuity.
Calculate if this will be enough for Kylie to achieve her savings goal (take
13 weeks = 1 quarter).

T
SHEE

8.1

250

Maths Quest General Mathematics HSC Course

Future and present value tables


Problems associated with annuities can be simplified by creating a table that will show
either the future value or present value of an annuity of $1 invested per interest period.

Computer Application 2 Future value of $1


Consider $1 is invested into an annuity each interest period. The table we are going to
construct on a spreadsheet shows the future value of that $1.
1. Open a new spreadsheet.
2. Type in the following information as shown in step 3.
3. In cell B4 enter the formula =((1+B$3)^$A4-1)/B$3. (This is the future value formula
from exercise 2A with the value of M omitted, as it is equal to 1.) Format the cell,
correct to 4 decimal places.
4. Highlight the range of cells B3 to M13. From the Edit menu, use Fill Down and Fill
Right functions to copy the formula to all other cells in this range.

This completes the table. The table shows the future value of an annuity of $1 invested
for up to 10 interest periods at up to 10% per interest period. You can extend the
spreadsheet further for other interest rates and longer investment periods.
The following table is the set of future values of $1 invested into an annuity. This is
the table you should have obtained in computer application 2.
A table such as this can be used to find the value of an annuity by multiplying the
amount of the annuity by the future value of $1.

Chapter 8 Annuities and loan repayments

251

Future values of $1
Interest rate (per period)
Period

1%

2%

3%

4%

5%

6%

7%

8%

9%

10%

11%

12%

1.0000 1.0000 1.0000 1.0000 1.0000 1.0000 1.0000 1.0000 1.0000 1.0000 1.0000 1.0000

2.0100 2.0200 2.0300 2.0400 2.0500 2.0600 2.0700 2.0800 2.0900 2.1000 2.1100 2.1200

3.0301 3.0604 3.0909 3.1216 3.1525 3.1836 3.2149 3.2464 3.2781 3.3100 3.3421 3.3744

4.0604 4.1216 4.1836 4.2465 4.3101 4.3746 4.4399 4.5061 4.5731 4.6410 4.7097 4.7793

5.1010 5.2040 5.3091 5.4163 5.5256 5.6371 5.7507 5.8666 5.9847 6.1051 6.2278 6.3528

6.1520 6.3081 6.4684 6.6330 6.8019 6.9753 7.1533 7.3359 7.5233 7.7156 7.9129 8.1152

7.2135 7.4343 7.6625 7.8983 8.1420 8.3938 8.6540 8.9228 9.2004 9.4872 9.7833 10.0890

8.2857 8.5380 8.8923 9.2142 9.5491 9.8975 10.2598 10.6366 11.0285 11.4359 11.8594 12.2997

9.3685 9.7546 10.1591 10.5828 11.0266 11.4913 11.9780 12.4876 13.0210 13.5795 14.1640 14.7757

10

10.4622 10.9497 11.4639 12.0061 12.5779 13.1808 13.8164 14.4866 15.1929 15.9374 16.7220 17.5487

WORKED Example 9

Use the table to find the future value of an annuity into which $1500 is deposited at the
end of each year at 7% p.a. interest, compounded annually for 9 years.
THINK
1
2

WRITE

Look up the future value of $1 at


7% p.a. for 9 years.
Multiply this value by 1500.

Future value = $1500

11.9780

Future value = $17 967

Just as we have a table for the future value of an annuity, we can create a table for the
present value of an annuity.

Computer Application 3 Present value table


The table we are about to make on a spreadsheet shows the present value of an annuity
of $1 invested per interest period.
1. Open a new spreadsheet.
2. Enter the following information.
3. In cell B4 type the formula =((1+B$3)^$A4-1)/(B$3*(1+B$3)^$A4).
4. Drag from cell B4 to K13, and then from the Edit menu use the Fill Down and Fill Right
functions to copy this formula to the remaining cells in your table.

252

Maths Quest General Mathematics HSC Course

The table created in computer application 3 shows the present value of an annuity of
$1 per interest period for up to 10% per interest period and for up to 10 interest periods.
The table that you have generated is shown below.
Present values of $1
Interest rate (per period)
Period

1%

2%

3%

4%

5%

6%

7%

8%

9%

10%

11%

12%

0.9901 0.9804 0.9709 0.9615 0.9524 0.9434 0.9346 0.9259 0.9174 0.9091 0.9009 0.8929

1.9704 1.9416 1.9135 1.8861 1.8594 1.8334 1.8080 1.7833 1.7591 1.7355 1.7125 1.6901

2.9410 2.8839 2.8286 2.7751 2.7232 2.6730 2.6243 2.5771 2.5313 2.4869 2.4437 2.4018

3.9020 3.8077 3.7171 3.6299 3.5460 3.4651 3.3872 3.3121 3.2397 3.1699 3.1024 3.0373

4.8534 4.7135 4.5797 4.4518 4.3295 4.2124 4.1002 3.9927 3.8897 3.7908 3.6959 3.6048

5.7955 5.6014 5.4172 5.2421 5.0757 4.9173 4.7665 4.6229 4.4859 4.3553 4.2305 4.1114

6.7282 6.4720 6.2303 6.0021 5.7864 5.5824 5.3893 5.2064 5.0330 4.8684 4.7122 4.5638

7.6517 7.3255 7.0197 6.7327 6.4632 6.2098 5.9713 5.7466 5.5348 5.3349 5.1461 4.9676

8.5660 8.1622 7.7861 7.4353 7.1078 6.8017 6.5152 6.2469 5.9952 5.7590 5.5370 5.3282

10

9.4713 8.9826 8.5302 8.1109 7.7217 7.3601 7.0236 6.7101 6.4177 6.1446 5.8892 5.6502

This table can be used in the same way as the future values table.

Chapter 8 Annuities and loan repayments

253

WORKED Example 10
Liam invests $750 per year in an annuity at 6% per annum for 8 years, with interest
compounded annually. Use the table to calculate the present value of Liams annuity.
THINK
1 Use the table to find the present value of a
$1 annuity at 6% for 8 interest periods.
Multiply
this value by 750.
2

WRITE

Present value = $750 6.2098


Present value = $4657.35

remember
1. A table of future values shows the future value of an annuity in which $1 is
invested per interest period.
2. A table of present values shows the present value of an annuity in which $1 is
invested per interest period.
3. A table of present or future values can be used to compare investments and
determine which will give the greater financial return.

8C

Future and present value


tables

1 Use the table of future values on page 251 to determine the future value of an annuity 8.4
of $800 invested per year for 5 years at 9% p.a., with interest compounded annually.
9
2 Use the table of future values to determine the future value of each of the following Reading
financial
annuities.
tables
a $400 invested per year for 3 years at 10% p.a., with interest compounded annually
b $2250 invested per year for 8 years at 8% p.a., with interest compounded annually
c $625 invested per year for 10 years at 4% p.a., with interest compounded annually
d $7500 invested per year for 7 years at 6% p.a., with interest compounded annually
3 Samantha invests $500 every 6 months for 5 years in an annuity at 8% p.a., with
interest compounded every 6 months.
a What is the interest rate per interest period?
b How many interest periods are there in Samanthas annuity?
c Use the table to calculate the future value of Samanthas annuity.
4 Use the table to calculate the future value of each of the following annuities.
a $400 invested every 6 months for 4 years at 14% p.a., with interest compounded sixmonthly
b $600 invested every 3 months for 2 years at 12% p.a., with interest compounded
quarterly
c $100 invested every month for 5 years at 10% p.a., with interest compounded sixmonthly
5 Use the table of future values to determine whether an annuity at 5% p.a. for 6 years or
an annuity at 6% p.a. for 5 years will produce the greatest financial outcome. Explain
your answer.

WORKED

Example

SkillS

HEET

254

Maths Quest General Mathematics HSC Course

6 multiple choice
Use the table of future values to determine which of the following annuities will have
the greatest financial outcome.
A 6% p.a. for 8 years, with interest compounded annually
B 8% p.a. for 6 years, with interest compounded annually
C 7% p.a. for 7 years, with interest compounded annually
D 10% p.a. for 5 years, with interest compounded six-monthly
WORKED

Example

10

7 Use the table of present values on page 252 to determine the present value of an
annuity of $1250 per year for 8 years invested at 9% p.a.
8 Use the table of present values to determine the present value of each of the following
annuities.
a $450 per year for 5 years at 7% p.a., with interest compounded annually
b $2000 per year for 10 years at 10% p.a., with interest compounded annually
c $850 per year for 6 years at 4% p.a., with interest compounded annually
d $3000 per year for 8 years at 9% p.a., with interest compounded annually

2
1 Calculate the amount of interest earned on $10 000 invested for 10 years at 10% p.a.,
with interest compounding annually.
2 Calculate the future value of an annuity of $1000 invested every year for 10 years at
10% p.a., with interest compounding annually.
3 Calculate the future value of an annuity where $200 is invested each month for
5 years at 5% p.a., with interest compounding quarterly.
4 Calculate the amount of each annual contribution to an annuity that will have a future
value of $15 000 if the investment is for 8 years at 7.5% p.a., with interest
compounding annually.
5 Calculate the amount of each annual contribution to an annuity that will have a future
value of $500 000 in 25 years when invested at 10% p.a., with interest compounding
annually.
6 Calculate the present value of an annuity that will have a future value of $50 000 in
10 years at 10% p.a., with interest compounding annually.
7 Calculate the present value of an annuity that will have a future value of $1 000 000 in
40 years at 10% p.a., with interest compounding annually.
8 Calculate the present value of an annuity where annual contributions of $1000 are
made at 10% p.a., with interest compounding annually for 20 years.
9 Use the table on page 251 to find the future value of $1 invested at 16% p.a. for
4 years, with interest compounding twice annually.
10 Use the answer to question 9 to calculate the future value of an annuity of $1250
every six months for 4 years, with interest of 16% p.a., compounding twice annually.

Chapter 8 Annuities and loan repayments

255

Loan repayments
When a loan is taken out and is repaid in equal monthly instalments, the pattern of
repayments works similar to an annuity. Each month interest compounds on the balance
owing on the loan and then a repayment is made.
Consider a loan where the amount borrowed is equal to the present value of the
annuity, N, and the amount paid on the loan each month is equal to the contribution to
( 1 + r )n 1
- .
the annuity per period, M. Use the formula for present value, N = M --------------------------r ( 1 + r )n
To calculate the amount of each monthly repayment, we need to make M the subject of
this formula. When we do this the formula becomes:
r ( 1 + r )n
M = N --------------------------( 1 + r )n 1
In this formula, M is the amount of each repayment, N is the amount borrowed, r is the
interest rate per repayment period as a decimal and n is the number of repayments to be
made.
This formula is not given to you on the formula sheet but will be given to you if it is
needed to solve a problem in the exam.

WORKED Example 11
r( 1 + r )n
Use the formula M = N --------------------------( 1 + r )n 1

to calculate the monthly repayments on a loan of

$5000 to be repaid in monthly instalments over 4 years at an interest rate of 12% p.a.
THINK

WRITE

Calculate the values of r and n.

r = 0.01 and n = 48

Write the formula.

r ( 1 + r )n
M = N --------------------------( 1 + r )n 1

Substitute for N, r and n.

M = 5000

Calculate.

M = $131.67

0.01 1.01 48
------------------------------1.01 48 1

Having worked out the amount of each monthly repayment, we are also able to
calculate the total cost of repaying a loan by multiplying the amount of each repayment
by the number of repayments.

256

Maths Quest General Mathematics HSC Course

WORKED Example 12
Calculate the total cost of repaying a $100 000 home loan at 9% p.a. in equal monthly
repayments over a 25-year term.
THINK

WRITE

Calculate the values of r and n.

r = 0.0075, n = 300

Write the formula.

r ( 1 + r )n
M = N --------------------------( 1 + r )n 1

Substitute for N, r and n.

M = 100 000

Calculate the amount of each monthly repayment.


Calculate the total repayments on the loan.

M = $839.20
Total repayments = $839.20 300
Total repayments = $251 760

0.0075 1.0075 300


---------------------------------------------1.0075 300 1

By increasing the amount of each repayment, we are able to shorten the term of the
loan. There is no easy method to calculate the amount of time that it will take to repay
a loan. To do this we use a guess and refine method. We adjust the value of n in the
formula until the amount of the repayment is reached.

WORKED Example 13
A $100 000 home loan is taken out over a 25-year term at an interest rate of 12% p.a.
reducible interest. The minimum monthly repayment on the loan is $1053.22. How long
will it take the loan to be repaid at $1200 per month?
THINK
1

Calculate the value of r.

Write the formula.

Take a guess for the value of n (we will take 200


since for the original loan n = 300) and substitute.

WRITE
r = 0.01
r ( 1 + r )n
M = N --------------------------( 1 + r )n 1
If n = 200,
M = 100 000

Calculate the repayment with n = 200. As this is


less than $1200 we need to further reduce the value
of n.
Substitute into the formula with n = 150.

= $1158.33
If n = 150,
M = 100 000

Calculate the repayment. As the result is greater


than $1200, we need to increase the value of n.

0.01 1.01 200


---------------------------------1.01 200 1

= $1289.99

0.01 1.01 150


---------------------------------1.01 150 1

Chapter 8 Annuities and loan repayments

THINK
7

WRITE

Substitute into the formula with n = 180.

If n = 180,
M = 100 000

8
9

257

As this is approximately equal to $1200, it will


take 180 months to repay the loan.
Give a written answer.

0.01 1.01 180


---------------------------------1.01 180 1

= $1200.17
It will take 15 years to repay the
loan.

remember
1. By considering the amount borrowed in a loan as the present value of an
annuity, we can use the present value formula to calculate the amount of each
repayment.
2. The formula used to calculate the amount of each monthly repayment is:
r ( 1 + r )n
M = N --------------------------( 1 + r )n 1
where N is the amount borrowed, r is the interest rate per period expressed as a
decimal and n is the number of interest periods.
3. The total cost of a loan can be calculated by multiplying the amount of each
repayment by the number of repayments to be made.
4. The length of time that it will take to repay a loan can be calculated by using
guess and refine methods.

8D

Loan repayments

r ( 1 + r )n
- .
For questions 1 to 3 use the formula, M = N --------------------------( 1 + r )n 1
1 Yiannis takes out a $10 000 loan over 5 years at 10% p.a. reducible interest with five
equal annual repayments to be made. Use the formula to calculate the amount of each
annual repayment.
WORKED

Example

11

2 Use the formula to calculate the amount of each monthly repayment on a loan of
$8000 to be repaid over 4 years at 12% p.a.
3 Use the formula to calculate the amount of each monthly repayment on each of the
following loans.
a $2000 at 12% p.a. over 2 years
b $15 000 at 9% p.a. over 5 years
c $120 000 at 6% p.a. over 20 years
d $23 000 at 9.6% p.a. over 5 years
e $210 000 at 7.2% p.a. over 25 years

258

Maths Quest General Mathematics HSC Course

4 Javier and Diane take out a $175 000 home loan. If the interest rate on the loan is
8.4% p.a. reducible and the term of the loan is 25 years, calculate the amount of each
monthly repayment.
5 Jiro purchases a computer on terms. The cash price of the computer is $3750. The
terms are a deposit of 10% with the balance paid in equal monthly instalments at
9% p.a. reducible interest over 3 years.
a Calculate Jiros deposit on the computer.
b What is the balance owing on the computer?
c Calculate the amount of each monthly repayment.
6 Jeremy and Patricia spend $15 000 on new furnishings for their home. They pay a
15% deposit on the furnishings with the balance paid in equal monthly instalments at
18% p.a. interest over 4 years. Calculate the amount of each monthly repayment.
7 Thanh is purchasing a car on terms. The cash price of the car is $35 000 and he pays
a $7000 deposit.
a What is the balance owing on the car?
b If the car is to be repaid in equal weekly instalments over 5 years at an interest rate
of 10.4% p.a. reducible interest, calculate the amount of each weekly payment.
WORKED

Example

12

8 Ron borrows $13 500 to purchase a car. The loan is to be repaid in equal monthly
instalments over a 3-year term at an interest rate of 15% p.a. Calculate the total
repayments made on the loan.
9 Calculate the total repayments on each of the following loans.
a $4000 at 8.4% p.a. reducible interest to be repaid over 2 years in equal monthly
repayments
b $20 000 at 13.2% p.a. reducible interest to be repaid over 6 years in equal monthly
instalments
c $60 000 at 7.2% p.a. reducible interest to be repaid over 15 years in equal monthly
instalments
d $150 000 at 10.8% p.a. reducible interest to be repaid over 20 years in equal
monthly instalments
10 multiple choice
A loan of $5000 is taken out at 9% p.a. reducible interest over 4 years. Which of the
following will give the amount of each monthly repayment?
A M = 5000

0.09 1.09 4
----------------------------1.09 4 1

B M = 5000

0.09 1.09 48
------------------------------1.09 48 1

C M = 5000

0.0075 1.0075 4
----------------------------------------1.0075 4 1

D M = 5000

0.0075 1.0075 48
------------------------------------------1.0075 48 1

11 multiple choice
A loan of $12 000 is taken out at 12% p.a. reducible interest in equal monthly instalments over 5 years. The total amount of interest paid on the loan is:
A $266.93
B $4015.80
C $7200
D $16 015.80

Chapter 8 Annuities and loan repayments

259

12 A loan of $75 000 is taken out over 15 years at 9% p.a. reducible interest. The minimum
monthly repayment is $760.70. Calculate how long it will take to repay the loan at
13
$1000 per month.

WORKED

13 A $150 000 loan is taken out over a 25-year term. The interest rate is 9.6% p.a.
a Calculate the minimum monthly repayment.
b Calculate the total repayments on the loan.
c Calculate the length of time that it will take to repay the loan at $1600 per month.
d Calculate the total saving on the loan by repaying the loan at $1600 per month.

Work

Example

T
SHEE

8.2

Types of loan arrangements


Research one example of each of the following types of loans.
A. Hire purchase agreement.
This is the type of loan where a major item is purchased on terms. Usually a
deposit is paid and then the balance plus interest is repaid over an agreed period
of time.
B. Personal loan
This is a loan taken out from a bank or other financial institution. It can be used for
any purpose and is unsecured. This means that there is no item of property that the
bank can claim if repayments are not made.
C. Home loan
This is a secured loan, which means that, if the repayments are not made, the bank
can claim the property and sell it to reclaim the amount outstanding on the loan.
For each of the above loans, answer the following questions.
1 What is the interest rate? Is interest calculated at a flat or reducible rate?
2 Over what term can the loan be repaid?
3 How regularly must repayments be made?
4 Can additional repayments be made to shorten the term of the loan?
5 Can the interest rate be altered after repayments have begun to be made?
6 What other fees and charges apply to borrowing the money?

Most financial institutions will provide graphs that show the growth of an annuity and
the declining balance of a loan. These graphs can be obtained by either visiting the
bank or by going to the internet site for the relevant financial institution.
Obtain a copy of a graph showing the growth of an investment and the declining
balance of a loan.
Alternatively, develop a spreadsheet that shows the growth of an annuity and the
declining balance of a loan and use the charting function of the spreadsheet to draw the
graph.
Access the Word file Annuities, Loans, Graphs from the Maths Quest General
Mathematics HSC Course CD-ROM.

Computer Application 4 Graphs of annuities and loans


ord

260

Maths Quest General Mathematics HSC Course

summary
Future value of an annuity
An annuity is where regular equal contributions are made to an investment. The
interest on each contribution compounds as additions are made to the annuity.
The future value of an annuity is the value that the annuity will have at the end of a
fixed period of time.
The future value of an annuity can be calculated using the formula:
( 1 + r )n 1
A = M ---------------------------r
where M is the contribution per period paid at the end of the period, r is the
percentage interest rate per compounding period (expressed as a decimal) and n is
the number of compounding periods.
The amount of each contribution per period in an annuity can be found using the
Ar
-.
formula M = --------------------------( 1 + r )n 1

Present value of an annuity


The present value of an annuity is the single sum that would need to be invested at
the present time to give the same financial outcome at the end of the term.
The present value of an annuity can be calculated using the formula:
A
N = ------------------n(1 + r )
where A is the future value of the annuity.
An alternative formula to use is:
( 1 + r )n 1
N = M --------------------------r ( 1 + r )n
where M is the contribution made to the annuity per interest period.

Use of tables
A table can be used to find the present or future value of an annuity.
The table shows the present or future value of $1 under an annuity.
The present or future value of $1 must be multiplied by the contribution per period
to calculate its present or future value.

Loan repayments
The present value of an annuity formula can be used to calculate the amount of
each periodical repayment in a reducing balance loan. This is done by considering
the present value of an annuity as the amount borrowed and making M the subject
of the formula.
r ( 1 + r )n
- .
The formula to be used is M = N --------------------------( 1 + r )n 1
The total amount to be repaid during a loan is calculated by multiplying the amount
of each monthly repayment by the number of repayments to be made.

Chapter 8 Annuities and loan repayments

261

CHAPTER
review
1 Calculate the amount to which each of the following investments will grow.
a $3500 at 12% p.a. for 3 years, with interest compounded annually
b $2000 at 8% p.a. for 5 years, with interest compounded six-monthly
c $15 000 at 9.2% p.a. for 8 years, with interest compounded quarterly
d $4200 at 13.2% p.a. for 2 years, with interest compounded monthly

8A

2 $400 per year is invested into an annuity at 7% p.a., with interest compounded annually. Use

8A

( 1 + r )n 1
the formula A = M ---------------------------r

to calculate the value of the annuity after 20 years.

( 1 + r )n 1
3 Use the formula A = M ---------------------------r

to calculate the future value of each of the

8A

following annuities.
a $500 invested per year for 25 years at 12% p.a., with interest compounded annually
b $1000 invested every 6 months for 10 years at 9% p.a., with interest compounded
six-monthly
c $600 invested every 3 months for 5 years at 7.2% p.a., with interest compounded
quarterly
d $250 invested per month for 20 years at 12% p.a., with interest compounded monthly
4 An annuity consists of $100 deposits every month for 15 years. The interest rate is 9% p.a.
and interest is compounded six-monthly. Find the future value of the annuity.
Ar
- to calculate the amount of each annual contribution to an
5 Use the formula M = --------------------------( 1 + r )n 1
annuity to achieve a savings goal of $800 000 in 40 years at an interest rate of 8% p.a., with
interest compounded annually.
6 Calculate the amount of each contribution to the following annuities.
a $50 000 in 10 years at 6% p.a., with interest compounded annually and annual deposits
b $250 000 in 30 years at 12% p.a., with interest compounded six-monthly and
contributions made every six months
c $120 000 in 20 years at 16% p.a., with interest compounding quarterly and contributions
made quarterly
A
7 Use the formula N = ------------------n- to calculate the present value of an annuity if it is to have a
(1 + r )
future value of $350 000 in 30 years time at an interest rate of 10% p.a., with interest
compounded annually.
8 Calculate the present value of the following annuities with a future value of:
a $10 000 after 10 years at 5% p.a., with interest compounded annually
b $400 000 after 40 years at 12% p.a., with interest compounded annually
c $5000 after 5 years at 9% p.a., with interest compounded six-monthly
d $120 000 after 8 years at 15% p.a., with interest compounded quarterly.

8A
8A
8A

8B
8B

262
8B
8B

Maths Quest General Mathematics HSC Course

9 Phuong wants to purchase a car in 3 years. He feels that he will need $15 000. The best
investment he can find is at 8.5% p.a., interest compounded quarterly. What is the present
value of this investment?
10 Gayle invests $400 per year in an annuity. The investment is at 6% p.a., with interest
compounded annually. Gayle plans to invest in the annuity for 25 years. Use the formula
( 1 + r )n 1
N = M --------------------------r ( 1 + r )n

to calculate the present value of this annuity.

8B

11 When Joanne begins work at 18, she invests $100 per month in a retirement fund. The
investment is at 9% p.a., with interest compounded six-monthly.
a If Joanne is to retire at 60 years of age, what is the future value of her annuity?
b What is the present value of this annuity?

8C

12 Use the table of future values of $1 on page 251 to calculate the future value of an annuity
of $4000 deposited per year at 7% p.a. for 8 years, with interest compounded annually.

8C

13 Use the table of future values of $1 to calculate the future value of the following
annuities.
a $750 invested per year for 5 years at 8% p.a., with interest compounded annually
b $3500 invested every six months for 4 years at 12% p.a., with interest compounded
six-monthly
c $200 invested every 3 months for 2 years at 16% p.a., with interest compounded quarterly
d $1250 invested every month for 3 years at 10% p.a., with interest compounded
six-monthly

8C

14 Use the table of present values of $1 on page 252 to calculate the present value of an annuity
of $500 invested per year for 6 years at 9% p.a., with interest compounded
annually.

8C

15 Use the table of present values to calculate the present value of each of the following
annuities.
a $400 invested per year for 5 years at 10% p.a., with interest compounded annually
b $2000 invested every six months for 5 years at 14% p.a., with interest compounded
six-monthly
c $500 invested every three months for 2 1--- years at 16% p.a., with interest compounded
2
quarterly
d $300 invested every month for 4 years at 12% p.a., with interest compounded half-yearly

8D

r ( 1 + r )n
16 Use the formula M = N --------------------------( 1 + r )n 1

to calculate the amount of each monthly repayment

on a loan of $28 000 to be repaid over 6 years at 12% p.a.

8D

17 Scott borrows $22 000 to purchase a car. The loan is taken out over a 4-year term at an
interest rate of 9.6% p.a., with the loan to be repaid in equal monthly repayments.
a Calculate the amount of each monthly repayment.
b Calculate the total amount that is repaid on the loan.

8D

18 Calculate the total repayments made on a home loan of $210 000 to be repaid in equal
monthly repayments over 25 years at an interest rate of 8.4% p.a.

Chapter 8 Annuities and loan repayments

263

19 Adam buys a new lounge suite for $4400 and pays for it on his credit card. The interest rate
on the credit card is 21% p.a. Adam hopes to pay the credit card off in two years by making
equal monthly repayments.
a Calculate the amount of each monthly repayment that Adam should make.
b Calculate the total amount that Adam will make in repayments.
c Calculate the amount of interest that Adam will pay.

Practice examination questions


1 multiple choice
Jenny invests $1000 per year for 20 years in an annuity. The interest rate is 6.5% p.a. and
interest is compounded annually. The future value of the annuity is:
A $3523.65
B $11 018.51 C $18 825.31 D $38 825.31
2 multiple choice
Madeline invests $1000 per year for 20 years in an annuity. The interest rate is 6.5% p.a. and
interest is compounded annually. The present value of the annuity is:
A $3523.65
B $11 018.51 C $18 825.31 D $38 825.31
3 multiple choice
Which of the following investments has the greatest future value after 10 years?
A An annuity of $500 per year at 7.75% p.a., with interest compounded annually
B An annuity of $250 per six months at 7.6% p.a., with interest compounded six-monthly
C An annuity of $125 per quarter at 7.2% p.a., with interest compounded quarterly
D A single investment of $3400 at 7.9% p.a., with interest compounded annually

8D

264

Maths Quest General Mathematics HSC Course

4 multiple choice
A loan of $80 000 is taken out over a 20-year term at an interest rate of 9% p.a. The monthly
repayment is $719.78. What would the total saving be if the term were reduced to 15 years?
A $91.63
B $16 493.40 C $21 991.20 D $26 693.40
5 Lien invests $2000 per year in an annuity. The term of the annuity is 20 years and the interest
rate is 8% p.a., with interest compounding annually.
a Calculate the future value of this annuity.
b Calculate the present value of this annuity.
c By how much will the future value of the annuity increase if Lien deposits $500 per
quarter and interest is compounded quarterly?
6 Eddie has the goal of saving $1 000 000 over his working life, which he expects to be 40
years. Over the period of his working life, Eddie expects to be able to obtain an average 7%
p.a. in interest with interest compounded every six months.
a Calculate the present value of this annuity.
Ar
- to calculate the amount of each six-monthly
b Use the formula M = --------------------------( 1 + r )n 1
contribution to the annuity.
c For the first 10 years of the annuity Eddie makes no contributions, preferring to direct all
his money into paying off a mortgage. At that time he makes a single contribution to catch
up on the annuity. What amount must Eddie deposit?
7 Jim and Catherine take out a $150 000 loan. The interest rate on the loan is 12% p.a. and the
loan is to be repaid in equal monthly repayments over a 20-year term.
r ( 1 + r )n
a Use the formula M = N --------------------------( 1 + r )n 1

CHAPTER

test
yourself

to calculate the amount of each monthly

repayment.
b Calculate the total amount of interest that Jim and Catherine will need pay on this loan.
c Calculate the saving that Jim and Catherine will make by repaying the loan over a 12-year
term.

Modelling
linear and
non-linear
relationships

9
syllabus reference
Algebraic modelling 4
Modelling linear and
non-linear relationships

In this chapter
9A
9B
9C
9D
9E

Linear functions
Quadratic functions
Other functions
Variations
Graphing physical
phenomena

areyou

READY?

Are you ready?

Try the questions below. If you have difficulty with any of them, extra help can be
obtained by completing the matching SkillSHEET. Either click on the SkillSHEET icon
next to the question on the Maths Quest HSC Course CD-ROM or ask your teacher for
a copy.

9.1

Substitution into a formula

9.2

Recognising linear functions

1 For each of the following linear equations, find the y-values corresponding to x, when x equals
3, 2, 1, 0, 1, 2 and 3. Show the results as a table of values.
a y = 2x
b y = 3x 1
c y = 7 3x

2 State which of the following are linear functions.

9.3

a y = 4x 1

b y = x2

d 2x 3y + 5 = 0

e y = 2x

Gradient of a straight line

3 Calculate the gradient of the line joining the following points.


a (1, 1) and (5, 6)
b (4, 0) and (6, 6)

9.4

1
c y = --x
f 2y = 5x

c (3, 7) and (2, 3)

Graphing linear equations

4 Sketch the graph of the linear functions.


a y = 3x
b y = 2x 3

c y = 5 2x

Chapter 9 Modelling linear and non-linear relationships

267

Linear functions
As discussed in chapter 5, a linear function is a function in which the independent and
dependent variables have only a power of 1. When graphed, these values form a
straight line.
An example of a linear function is y = 2x 1. The function is graphed by creating a
table of values, plotting the pair of coordinates that are formed on a number plane and
joining them with a straight line. The independent variable is x, and as such values of x
are substituted into the equation to find the corresponding values of y.
If we recognise the function as linear, we need to plot only three points. Two points
are sufficient to fix a line and the third is a check. If all three points are not in a straight
line, we know that an error has been made.

WORKED Example 1

Graph the equation y = 2x

1.

THINK
1

WRITE

Draw a table of values for x.


(Choose three easy values of x.)

x
y

Substitute each value of x into the


equation to find the corresponding
values of y.
Plot each of the points formed on a
number plane.

y
5
4
3
2
1
5 4 3 2 1 0
1
2
3
4
5

Join the points formed with a straight


line and label the line with the
equation.

1 2 3 4 5

y
5
4
3
2
1
5 4 3 2 1 0
1
2
3
4
5

y = 2x 1
1 2 3 4 5

The straight line in worked example 1 has the equation y = 2x 1, which is


written in gradientintercept form. Any equation in the form y = mx + b
is said
to be in gradientintercept form, because the gradient of the straight line is
represented by m and the y-intercept is represented by b.

268

Maths Quest General Mathematics HSC Course

This can be used to sketch any straight line. Considering worked example 1, we can
begin by plotting the point (0, 1) as the y-intercept. Other points can then be plotted
using the gradient, by plotting points 1 across and 2 up. That is, starting with (0, 1),
we plot (1, 1), (2, 3), (3, 5) and so on.
At this point it is worth remembering the gradient formula:
vertical change in position
m = --------------------------------------------------------------------horizontal change in position
We use this formula when we know two points on the graph, and this is useful on many
occasions to help us find the equation of a straight line.

Graphics Calculator tip! Graphing a linear function


1. From the MENU select GRAPH.

2. Delete any existing equation and enter Y1 = 2X 1.

3. Press SHIFT F3 [V-Window]. This allows you to


set the lower and upper limits to draw on both the xand y-axes. Enter the setting shown on the screen at
right.
4. Press EXE to return to the previous screen, and
then press F6 (DRAW) to draw the graph.

WORKED Example 2

A store owner finds that the number of televisions sold each week, N, decreases as the
price, P, increases. This relationship can be given by the rule N = 200 0.2P.
a Complete the table below.
P

100

200

500

N
b Graph the relationship between the number of televisions sold and their price.
c How many televisions will be sold if they are priced at $900 each?
d The store can sell only a maximum of 50 televisions each week. At what price should the
televisions be sold?

Chapter 9 Modelling linear and non-linear relationships

THINK

WRITE

a Substitute the given values of P into the


equation to find the corresponding
values of N.

Plot the points given by the table.


(Note: Only positive values of N and
P are needed in this example.)

100

200

500

180

160

100

269

N
200
160
120
80
40
0

Join the points with a straight line


and label the equation.

0
60
0
80
0
10
00

40

20

N
200
N = 200 0.2P

160
120
80
40
0

80
0
10
00

0
60

0
40

20

c Use the graph to find N when P = 900.

c When P = 900, N = 20; they will sell


20 televisions at $900 each.

d Use the graph to find P when N = 50.

d When N = 50, P = 750; the televisions


should be sold for $750 each.

N
200
160

N = 200 0.2P

120
80
40

N = 0.05P

0
0

20
0
40
0
60
0
80
0
10
00

When two linear functions are graphed on the same pair


of axes, the intersection of the two graphs shows the
point where both equations hold true. This can have
applications in a practical context.
Consider worked example 2, in which the number of
televisions sold each week was given by N = 200 0.2P.
Now consider that the company producing the televisions
is prepared to produce more if the price is higher. This
is given by the rule N = 0.05P. When these two functions
are graphed on the same pair of axes, we can see that the
point of intersection is (800, 40).

270

Maths Quest General Mathematics HSC Course

The company would want to sell all of the televisions that they produce and similarly
would want to produce enough to meet this demand. This will be done if the televisions
are sold at $800 each, as the company would be prepared to produce 40 per week at
this price, and this would be the number that would be sold.
Graphing linear functions can be used to determine profit, loss or break-even points.
If cost and receipts are graphed, the difference between the y-values at any point will
determine the profit or loss. The point where the graphs intersect will be the break-even
point, where no profit or loss is made.

Graphics Calculator tip! Graphing a practical linear function


Many practical functions need to have different limits than worked example 1, which
we have drawn, with the simple limits of 5 to 5 on both the x- and y-axes. In worked
example 2, negative values are not realistic, while the maximum possible value on the
vertical axis is 200. We need a value on the horizontal axis that will show the greatest
possible value of P.
1. From the MENU select GRAPH.
2. Delete any existing equation and enter Y1 = 200 0.2X.
Note: We replace N with Y1 and P with X.

3. Press SHIFT F3 [V-Window]. This allows you to


set the lower and upper limits to draw on both the xand y-axes. Enter the setting shown on the screen at
right.
4. Press EXE to return to the previous screen, and
then press F6 (DRAW) to draw the graph.

WORKED Example 3
The cost of producing shoes in Asia is given by the equation C = 2000 + 15n, where n is the
number of pairs of shoes produced per day. The cost of producing shoes in Australia is
given by the equation C = 1000 + 20n.
a On the same pair of axes, graph the cost equations for producing shoes in Asia and
Australia.
b When is it more cost efficient to produce the shoes in Asia?

Chapter 9 Modelling linear and non-linear relationships

THINK

WRITE

a C = 2000 + 15n

Draw a table of values for each cost


equation.

100

200

2000

3500

5000

271

C = 1000 + 20n

Plot a pair of points generated by


each cost equation.
Join each with a straight line
labelling each with its equation.

100

200

1000

3000

5000

C
10 000

C = 1000 + 20n

8000
C = 2000 + 15n
6000
4000
2000
0

b It will be more efficient to produce the


shoes in Asia after the point of
intersection.

0
00
10

80

60

40

20

b If more than 200 pairs of shoes are produced


per day, it will be cheaper to produce the
shoes in Asia. This is because if n > 200 the
value of C is less, if the shoes are produced
in Asia.

the intersection of
Graphics Calculator tip! Finding
two graphs
Consider the two graphs drawn in worked example 3. It will be cheaper to produce the
shoes in Asia when the value of C = 200 + 15n is less than C = 1000 + 20n. To find
when this occurs we need to locate the point of intersection of the two graphs as shown
below.
1. From the MENU select GRAPH.

2. Delete any existing equations and enter


Y1 = 2000 + 15X and Y2 = 1000 + 20X. Note that we
replace C with Y1 and Y2 and n with X.

272

Maths Quest General Mathematics HSC Course

3. Press SHIFT F3 [V-Window]. This allows you to


set the lower and upper limits to draw on both the xand y-axes. Enter the setting shown on the screen at
right.
4. Press EXE to return to the previous screen, and
then press F6 (DRAW) to draw the graphs.

5. Press SHIFT F5 [G-Solv], followed by F5 [ISCT]


(intersection). This will find the point of intersection
and display the coordinates of this point. Be patient:
this may take a moment.
From this we can see that the intersection occurs at x = 200 and y = 5000. Interpreting
this result in terms of the question shows us that when 200 pairs of shoes are produced
the cost will be $5000 in either Australia or Asia. From that point on it will be cheaper
to produce the shoes in Asia.

remember
1. A linear function has powers of only 1 for both the independent and dependent
variables.
2. Linear functions, when graphed, will appear as a straight line and can be
written in the form y = mx + b , where m is the gradient and b is the yintercept.
3. To graph a linear function, draw a table with at least three values for the
independent variable and calculate the corresponding value for the dependent
variable. Plot the pairs of coordinates generated and join with a straight line.
4. Linear functions can also be graphed using a graphics calculator.
5. Many practical situations can be graphed using linear functions. When two
linear functions are graphed on the same pair of axes, the point of intersection
will give some important information about the question.

SkillS

9A
HEET

9.1

WORKED

Example

SkillS

Substitution
into a
formula
HEET

9.2
Recognising
linear
functions

Linear functions

1 Graph the function y = x + 3.


2 Graph each of the following linear functions on separate axes.
a y = 2x
b y = 3x 2
c y= x
d y = 5 2x
e y = 1--- x + 3
f y=1
2

3 Consider the linear function 3x + 2y 6 = 0.


a Copy and complete the table at right.
b Graph the function 3x + 2y 6 = 0.

x
y

1
--- x
4

Chapter 9 Modelling linear and non-linear relationships

WORKED

Example

4 The cost, C, of a taxi hire is given by the linear equation C = 3 + 1.5d, where d is the 9.3
distance travelled in kilometres.
a Copy and complete the table below.
Gradient of
d

10

30

SkillS
HEET

273
a straight
line

9.4

HEET

b Graph the cost function for the taxi hire.


c Use the graph to determine the cost of a 20 km taxi journey.
d Katie has $24. How far can Katie afford to travel in a taxi?

SkillS

Graphing
linear
equations
E

5 A concert promoter finds that the profit made on a performance is given by the
Plotting
equation P = 3n - 24 000, where n is the number of people who attend the concert.
linear
a Complete this table of values, and use it to graph the profit equation.
graphs
n
P

10 000
0

b What profit will the promoter make if 20 000 people attend the concert?
c What will be the financial outcome for the promoter if 5000 people attend the
concert?
d How many people will need to attend the concert for the promoter to break even?
6 It is found that the number of ice-creams that will be sold during a day at the beach
decreases as the price of the ice-creams increases. The number of ice-creams that will
be sold can be determined by the equation N = 1000 - 5P, where P is the price of the
ice-creams in cents.
a Graph the function.
b How many ice-creams will be sold at $1 each?
c If the ice-cream salesman has only 100 ice-creams to sell, at what price should he
sell them?
7 Two linear functions are represented by y = 4 - x and y = 3x.
a Graph both linear functions on the same pair of axes.
b What is the point of intersection of the two graphs?
8 By graphing both functions on the same pair of axes, find the point of intersection of
the graphs y = 2x - 6 and y = x - 1.
9 Find the point of intersection of the graphs x + 2y - 4 = 0 and y = 2x + 2.
10 A factory produces two types of computer games: game A and game B.
a The factory can produce a maximum of 120 games per week. This can be repre3
sented by the linear equation A + B = 120. Graph this function.
b Sales research shows that twice as many copies of game A will sell as game B. This
can be represented by the equation 2A = B. On the same pair of axes graph this
function.
c Find the point of intersection of the two graphs and make a conclusion about the
number of each game that should be produced by the factory each week.

WORKED

Example

sheet

L Spre
XCE ad

274

Maths Quest General Mathematics HSC Course

11 The cost of running an old refrigerator is $1.20 per day. This can be represented by the
equation C = 1.2d. A new refrigerator will cost $900 but the cost to run will be only
30c per day. This can be represented by the equation C = 900 + 0.3d.
a Copy and complete the table below.
0

1000

2000

C (old)
C (new)
b Graph both linear functions on the same pair of axes.
c Find the point of intersection of the two graphs; hence, state after how many days
it will become more economical to purchase a new refrigerator.
12 The cost, in dollars, of producing calculators can be given by the equation
C = 15n + 1500, where n is the number of calculators produced. When selling the
calculators the receipts can be given by the equation C = 20n.
a Graph both linear functions on the same pair of axes.
b Determine the number of calculators that need to be sold in order for the manufacturer to break even.

Conversion of temperature
To convert a temperature from degrees Celsius to degrees Fahrenheit, you can use
9C
the formula F = ------- + 32 . A simpler but less accurate way is to double degrees
5
Celsius and add 30. This approximation written as a formula becomes F = 2C + 30.
1 Use a spreadsheet or graphics calculator to graph each function on the same set
of axes.
2 Describe the accuracy of the simpler formula and state the values for which it is
most accurate.

Quadratic functions
A quadratic function is a function that involves the independent variable (x) to the
power of 2. The graph of a quadratic function is a parabola, a curved line that comes to
either a minimum or maximum point.
The graph of a quadratic function is again drawn by creating a table of values and
plotting the pairs of coordinates generated. Because the graph is not a straight line, it is
necessary to plot more than just three points to show the shape of the curve.

275

Chapter 9 Modelling linear and non-linear relationships

The most basic quadratic function is y = x2. The table of values is drawn showing at
least nine values of x.
x

1
--2

1
--2

1
--4

1
--4

9
y

Plotting these points gives the graph shown on


the right.
This graph has a minimum at (0, 0) and
forms the basic shape for all parabolas.
In general, the form of a quadratic function
is y = ax2 + bx + c, and we need consider only
positive values of x.

9
8
7
6
5
4
3
2
1
4 3 2 1 0
1

WORKED Example 4

Consider the quadratic function y = x2


a Complete the table of values below.
x
y

b Graph the function for x 0.


c State the minimum value of y = x2

4x + 7.
WRITE

a Substitute each value of x into the


function.

1 2 3 4

4x + 7.

THINK

y = x2

Plot the points generated by the table


of values.
Join the points plotted with a smooth
curve.

x
y

1
4

2
3

y
12
11
10
9
8
7
6
5
4
3
2
1
1 0
1

c The minimum value is the y-value at the


point where the graph turns.

0
7

1 2 3 4 5

For y = x2 4x + 7,
minimum value = 3.

3
4

4
7

5
12

276

Maths Quest General Mathematics HSC Course

Graphics Calculator tip! Graphing a quadratic function


Your graphics calculator can graph quadratic functions as well as linear functions (and
many other types of functions as well). The calculator can also find any maximum or
minimum point on the graph. Consider worked example 4.
1. From the MENU select GRAPH.

2. Delete any existing equation and enter


Y1 = X2 4X + 7.

3. Press SHIFT F3 [V-Window]. In this course you


will not need to consider negative values for x. Enter
the setting shown on the screen at right.

4. Press EXE to return to the previous screen, and


then press F6 (DRAW) to draw the graph.

5. Press SHIFT F5 [G-Solv], followed by F3 (MIN).


This will find the minimum point and display the
coordinates of that point. Be patient: this may take a
moment.
Note:
1. When setting the view window you do not have to get the limit right the first time.
It may take a bit of trial and error, especially with the y-values to make sure that you
have the minimum (or maximum) point in your display.
2. Any question that has a negative value of x2 (such as worked example 5) will be concave downwards and as such will have a maximum point and not a minimum point.
In step 5 after pressing SHIFT F5 [G-Solv] you will need to press F2 (MAX).
3. The display on the graphics calculator can sometimes lead to a slight inaccuracy in
the answer. This can be seen in step 6. In cases such as this we can see that the
calculator should display X = 2.
For quadratic functions that have a positive x2 term, the parabola is concave up. This
means that the graph comes to a minimum point. When the x2 term is negative, the
graph is concave down and the graph comes to a maximum point.

277

Chapter 9 Modelling linear and non-linear relationships

WORKED Example 5
Graph the function y = 1 + 4x

x2.

THINK
1
2

3
4

WRITE

Draw a table of values.


Substitute each value of x into the
function.
Plot the points formed by each pair of
coordinates.
Join the points with a smooth curve.
Note: For this function, the maximum
value is 5.

y
5
4
3
2
1
1 0
1
2
3
4
5

1 2 3 4 5

Quadratic models can be used to solve several practical situations.

WORKED Example 6
A ball is thrown in the air. Its height, h, after t seconds can be given by the formula
h = 20t 5t 2. Graph the function to calculate the maximum height the ball will reach.
THINK
1
2

Draw a table of values.


Substitute the values of t to calculate
the corresponding values of h.
Plot the points formed by each pair of
coordinates. Negative values of h can
be ignored because height must be
positive.
Join the points formed with a smooth
curve.

WRITE

15

20

15

25

h
20
16
12
8
4
1 0
1

The maximum height reached by the


ball will be the h value at the turning
point on the curve.

1 2 3 4 5

The maximum height reached by the ball is


20 m.

278

Maths Quest General Mathematics HSC Course

remember
1. A quadratic function is a function where the independent variable is raised to
the power of 2.
2. The graph of a quadratic function is a parabola. The parabola is a curved graph
and can be drawn using a table of values that has several points to allow the
shape of the graph to be formed.
3. If the x2 term is positive, the graph is concave up and has a minimum point. If
the x2 term is negative, the graph is concave down and has a maximum point.
4. The maximum or minimum value in a practical situation is the dependent
variable at the maximum or minimum point.

9B
EXCE

et

reads
L Sp he

WORKED

Example

4
Graphing
quadratics

Quadratic functions

1 For the quadratic function y = x2 2x + 3:


a copy and complete the table of values below
x

y
b draw the graph of the function
c state the minimum value of x2 2x + 3.
2 For the quadratic function y = x2 4x 2, draw up a table of values and use the table
to draw the graph of the function for x 0.
3 Graph each of the following functions for x
a y = x2 6x + 5
b y = x2 + x + 5
c y = (x 2)2

0.

4 On the one set of axes, graph the following quadratic functions for x
a y = x2
b y = 2x2
c y = 1--- x2

0.

5 On the one set of axes, graph each of the following quadratic functions for x
a y = x2
b y = x2 + 2
c y = x2 3

0.

6 Use your answers to questions 5 and 6 to answer the following.


a Describe the effect a coefficient of x2 has on the graph of a quadratic function.
b Describe the effect adding a constant term has on the graph of a quadratic function.
7 Graph the function y = (x
Explain why this occurs.
WORKED

Example

1)2 + 4. Compare this with the graph of y = x2

8 Graph the function y = 2 + 2x

x2 for x

0.

9 Graph each of the following functions for x


a y = 4 + 6x x2
b y = 8 x2

0.
c y = (2

x)2

2x + 5.

Chapter 9 Modelling linear and non-linear relationships

279

10 multiple choice
Which of the following functions is not a quadratic function?
A y = x2 + 5x 4
B y = (x 4)2
x2
C y = (x 2)(x + 2)
D y = -----------x+2
11 multiple choice
The graph drawn on the right could have the equation:
A y = (x 2)2 + 3
B y = (x 2)2 3
2
C y = 4 (2 x)
D y = (2 x)2 3
12 multiple choice
Which of the following functions will produce the same graph as y = (x
A y = x2 4x 1
B y = x2 4x + 19
2
C y = x 8x 1
D y = x2 8x + 19
13 Graph the quadratic function y = 2x2

4x + 8 for x

14 An object dropped from a height falls


to Earth according to the equation
6
d = 5t 2, where d is the distance fallen
in metres and t is the time in seconds
since the object was dropped.
a Draw the graph of d against t.
b How far will the object fall in
4 seconds?
c How long will it take for an object
to fall a distance of 500 m?

WORKED

Example

15 The height of a ball which is thrown


vertically upwards is given by the
equation h = 30t 5t2.
a Draw the graph of h against t.
b Find the maximum height reached
by the ball.
c Find the length of time taken for
the ball to return to Earth.
16 A rectangular field is to be made out of
100 m of fencing. If the length of the field
is x metres:
a show that the width of the field is
(50 x) metres
b show that the area is given by
the quadratic function A = 50x x2
c draw the graph of the function
d find the maximum area of the field
and what dimension the field must
be to give the maximum area.

0.

4)2 + 3?

Work

280

T
SHEE

9.1

Maths Quest General Mathematics HSC Course

17 Another rectangular field is to be built with 100 m


of fencing using a river as one side of the field as
River
shown on the right.
x
a Show that the area of the field can be given by
the equation A = 100x 2x2.
100 2x
b Graph the function.
c Calculate the dimensions of the field so that the area of the field is a maximum.

Maximising areas
1 Sketch ten rectangles that each have a perimeter of 40 m.
2 Show the length, width and area of each rectangle in a table.
3 If the length of the rectangle is x:
a explain why the width of the rectangle will be 20 x
b write a quadratic equation for the area of the rectangle.
4 Use a spreadsheet or graphics calculator to graph your function.
5 What is the maximum area of the rectangle?

1
1 Graph the function y = 2x

3.

2 State the gradient of the function.


3 What is the y-intercept of the function?
4 State the linear function with a gradient of 3 and a y-intercept of 2.
5 Give an example of a linear function with a negative gradient.
6 Copy and complete the table below for the function y = 4
x

x2.

y
x2 for x

0.

8 State the maximum value of the function y = 4

x2.

7 Draw the graph of the function y = 4


9 Is the function y = (x

5)2 concave up or concave down? Explain your answer.

10 Find the minimum value of y = (x

6)2 + 5.

281

Chapter 9 Modelling linear and non-linear relationships

Other functions
There are several other types of function that we should be able to graph. In each of
these cases the graphs are curves and so several points should be found to demonstrate
the shape of the curve.

Cubic functions
A cubic function has the independent variable (x) raised to a power of 3.

WORKED Example 7
Graph the function y = 2x3.
THINK
1 Draw a table of values.
2 Substitute values of x to find the
corresponding values of y.
3 Plot the points generated by the table.
4 Join the points with a smooth curve.

WRITE
x

16

54

y = 2x 3

100
80
60
40
20
0

1 2 3 4

Hyperbolas
a
A hyperbolic function is of the form y = --- , where a is a constant. For hyperbolas,
x
x 0, and so we graph only values of x > 0. As the value of x increases, the value of y
will decrease, and therefore we need to look at values close to 0 when creating our
table of values.

WORKED Example 8
2
Graph the function y = --- .
x
THINK
1 Draw a table of values.
2 Substitute the x-values into the equation
to find the corresponding y-values.
3
4

Plot each pair of coordinates generated


by the table.
Join each point with a smooth curve.

WRITE
x

1
--4

1
--2

2
--3

1
--2

y
9
8
7
6
5
4
3
2
1
0

y = 2x

1 2 3 4

282

Maths Quest General Mathematics HSC Course

Exponential graphs
An exponential function is of the form y = ax or y = b(ax ), where a and b are both
constants. An exponential graph can increase rapidly.

WORKED Example 9

Graph the function y = 2x.


THINK

WRITE

Draw a table of values.

Substitute values of x to find the


corresponding values of y.

Plot the points generated by the table.

Join the points with a smooth curve.

16

y
20
16
12
8
4
0

y = 2x
1 2 3 4 5

Graphics Calculator tip! Graphing other functions


To graph the functions shown in worked examples 7, 8 and 9, enter the functions as
shown below. In each case set the view window as shown in the diagrams on the x- and
y-axes.

An exponential function of the form y = b(ax ) represents an example of exponential


growth. These functions show the growth of an investment over a period of time. In
examples where the value of a is between 0 and 1, the function models exponential
decay. An example of this is the depreciation of an asset over time.

283

Chapter 9 Modelling linear and non-linear relationships

WORKED Example 10
Glenn invests $10 000 at 8% p.a. with interest compounded annually. The growth of
this investment can be given by the exponential function A = 10 000(1.08)n, where n is
the number of years of the investment and A is the amount to which the investment
grows.
Graph the growth of this investment.
THINK

WRITE

Draw a table of values.

Substitute values of n to find the


corresponding values of A.

A 10 800 11 664 12 597 13 605 14 693


6

10

A 15 869 17 138 18 509 19 990 21 589


Plot the points generated by the table.

Join the points with a smooth curve.

35 000
Investment ($)

A = 10 000 (1.08)n

30 000
25 000
20 000
15 000
10 000
5000
0
0

10
15
Number of years

20

remember
1. A cubic equation is of the form y = ax3.
a
2. A hyperbola is an equation of the form y = --- . In such a function x 0, and
x
we need to examine values of x close to 0 to observe the behaviour of the curve
near the y-axis.
3. An exponential function is of the form y = b(ax ). An exponential function can
be used to model a growth function such as the growth of an investment. If
0 < a < 1, the function will model an exponential decay such as the
depreciation of an item.

284

Maths Quest General Mathematics HSC Course

9C
EXCE

et

reads
L Sp he

WORKED

Example

7
Function
grapher

Other functions

1 Graph the cubic function y = x3 for x

0.

2 Graph the following functions for x 0.


a y = 3x3
b y = 1--- x3

c y = x3

WORKED

Example

WORKED

Example

4
3 Graph the hyperbolic function y = --- for x > 0.
x
4 Graph each of the following functions for x > 0.
1
10
a y = --b y = -----x
x

1
c y = --x

5 Graph the function y = 3x.


6 Graph each of the following functions.
a y = 4x
b y = 10x

c y = ( 1--- )x
2

7 Graph the function y = 5(2x ).


8 multiple choice
The equation of the graph shown on the right could be:
A y = x3
B y = 3x
C y = 3x
3
D y = --x

9 multiple choice
2
Which of the graphs shown below could be the graph of y = --- ?
x
A y
B y
C x
D y

10 Ming Lai invests $1000 at 10% p.a. interest with interest compounded annually. This
investment can be represented by the function A = 1000(1.1)n, where A is the amount
10
to which the investment grows and n is the number of years of the investment. Draw
the graph of the function.

WORKED

Example

11 Kevin invests $50 000 at 12% p.a. interest, compounded annually.


a Write an equation for the amount, A, to which the investment will grow in terms of
the number of years of the investment, n.
b Graph the function.
c Use the graph to estimate the amount of time that it will take for the investment to
reach $70 000.
12 A new car is purchased for $40 000. The car depreciates at the rate of 15% p.a. The
value, V, of the car after a number of years, n, can be given by the equation
V = 40 000(0.85)n. Graph this function.

Chapter 9 Modelling linear and non-linear relationships

285

Compound interest
The amount to which an investment will grow under compound interest can be
found using the following formula:
A = P(1 + r)n
.
Consider an investment of $10 000 at an interest rate of 8% p.a.
1 If interest is compounded annually, the amount to which the investment will
grow can be given by the function A = 10 000(1.08)n, where n is the number of
years. Graph this function using graphing software or a graphics calculator.
2 If interest is compounded six-monthly, the function becomes A = 10 000(1.04)2n.
On the same set of axes graph this function.
3 Write a function that will show the amount to which the investment will grow if
interest is compounded quarterly, and graph this function on the same set of axes.
4 Use the graphs drawn to describe the overall effect of shortening the
compounding period.

Variations
From our work on measurement we know that the area of a circle is given by the
formula A = r 2, where A is the area and r is the radius of the circle.
This is an example of a quantity (area) that varies in proportion with the power of
another quantity (radius). This can be written as A
r2. The symbol
means in
proportion to. In this example is the constant of variation, that is, the amount by
which r 2 must be multiplied to calculate the area.
An equation of the form y = ax2 or y = ax3 can be used to model several variations.
In such cases we may need to calculate the constant of variation from some known or
given information.

WORKED Example 11

It is known that y varies directly with the cube of x. It is known that y = 24 when x = 2.
Write an equation connecting the variables x and y.
THINK
1
2
3

WRITE

Write a proportion statement.


Insert a constant of variation (k) to form
an equation.
Substitute the known values of x and y
to find the value of k.

Replace the known value of k in the


equation.

y x3
y = kx3
When x = 2, y = 24.
24 = k 23
= 8k
k=3
y = 3x3

Once we have calculated the constant of variation, we are able to calculate one quantity
given the other.

286

Maths Quest General Mathematics HSC Course

WORKED Example 12
The surface area of a cube varies directly with the square of the length of the cubes edge.
a A cube of edge length 5.5 cm has a surface area of 181.5 cm2. Find the constant of
variation.
b Find the surface area of a cube with an edge length of 7.2 cm.
THINK

WRITE

a s

1
2
3
4
5

1
2
3
4

Write a proportion statement


choosing pronumerals s and e.
Insert the constant of variation, k, to
form an equation.
Substitute known information.

s = ke2
When e = 5.5, s = 181.5
181.5 = k 5.52
181.5 = k 30.25
181.k = 6

Calculate 5.52.
Solve the equation (divide by 30.25).
Rewrite the proportion statement
with k = 6.
Substitute e = 7.2.

e2

b s = 6e2
When e = 7.2,
s = 6 7.22
s = 311.04
The surface area of a cube with an edge of
7.2 cm is 311.04 cm2.

Calculate s.
Give a written answer.

Hyperbolic functions represent inverse variations. These variations occur when one
a
quantity decreases as the other increases. An inverse variation is of the form y = --- .
x

WORKED Example 13
It is known that y varies inversely with x and that when y = 8, x = 4. Write an equation
connecting y with x.
THINK
1

Write an inverse proportion statement.

Insert a constant of variation (k) to form


an equation.
Substitute the known values of x and y
to find the value of k.

Replace the known value of k in the


equation.

WRITE
1
--x
k
y = -x
y

When x = 4, y = 8.
k
8 = --4
k = 32
32
y = -----x

Chapter 9 Modelling linear and non-linear relationships

287

Such variations can also be applied to practical situations.

WORKED Example 14

It is known that the time taken for a journey varies inversely with speed. The trip takes
6 hours at 60 km/h.
a Find the constant of variation.
b How long will it take at 90 km/h?
THINK

WRITE

a t

Write a proportion statement


choosing pronumerals t and s.

2
3

Insert the constant of variation, k, to


form an equation.
Substitute known information.

Solve the equation (multiply by 60).

Rewrite the equation with k = 360.

Substitute s = 90.

Calculate t.
Give a written answer.

1
--s

k
t = -s
When t = 6, s = 60
k
6 = -----60
k = 360
360
b t = --------s
When s = 90,
360
t = --------90
t=4
The trip will take 4 hours at 90 km/h.

Graphics Calculator tip! Graphing variations


Once we have established the value of the constant of variation, we can use the
graphics calculator to graph the variation and then find the value of one variable given
the other. Consider worked example 14. Having found in part a the value of k, we can
360
graph t = --------- .
s
1. From the MENU select GRAPH.

2. Enter the equation Y1 = 360 X, which replaces t with


Y1 and s with X.

288

Maths Quest General Mathematics HSC Course

3. Press SHIFT F3 [V-Window]. In this situation you


will not need to consider negative values for x. Enter
the setting shown on the screen at right.

4. Press EXE to return to the previous screen, and


then press F6 (DRAW) to draw the graph.

5. To find the value of t (Y) when s (X) = 20, press


SHIFT
F5 [G-Solv], then F6 ( ) for more
options and then F1 (Y-CAL). Enter x = 90 and the
calculator will return the y-value.

remember
1. A variation can be expressed as a function.
2. If one quantity varies as the square of another, the variation is of the form
y = ax2.
3. If one quantity varies as the cube of another, the variation is of the form
y = ax3.
a
4. If one quantity varies inversely as another, the variation is of the form y = --- .
x
5. An inverse variation occurs when one quantity decreases while the other
decreases.

9D
WORKED

Example

11

Variations

1 It is known that y varies directly with the square of x. If y = 88 when x = 4, write an


equation connecting y with x.
2 It is known that b varies directly with the cube of a. When a = 6, b = 108. Write an
equation connecting b with a.
3 It is known that the distance, d, an object will fall varies directly with the square of
the time, t, it has been falling. An object that has been falling for 2 seconds falls a
distance of 19.6 metres.
a Write an equation connecting d with t.
b Graph the relationship between d and t.

WORKED

Example

12

4 The surface area of a cube varies directly with the square of its side length.
a A cube of side length 15 cm has a surface area of 1350 cm2. Find the constant of
variation.
b What is the surface area of a cube that has a side length of 6.2 cm?

289

Chapter 9 Modelling linear and non-linear relationships

5 The area of a circle varies directly with the square of its radius.
a If the area of a circle with side length 6 cm is 113.1 cm2, find the constant of
variation. (Give your answer correct to 2 decimal places.)
b What is the area of a circle with a radius of 12 cm?
6 The mass of an egg varies directly as the cube of the
eggs length.
a An egg of length 5 cm has a mass of 31.25 g.
Find the constant of variation.
b What will be the mass of an egg with a
length of 6 cm?
c If an egg has a mass of 70 g, what
would be the eggs length? (Give
your answer correct to 1 decimal
place.)
WORKED

Example

13

7 It is known that y varies inversely with


x. When y = 10, x = 5; write an
equation connecting y with x.
8 It is known that m varies inversely with
n. When m = 0.5, n = 2; write an
equation connecting m and n.
9 The time taken, t, to travel between two points
varies inversely with the average speed, s, for the
trip. If the journey takes 2.5 hours at 60 km/h:
a write an equation that connects t with s
b graph the relationship between t and s.

10 The time, t, taken to dig a trench varies inversely with the number of workers, n,
digging. It takes four workers 5 hours to dig the trench.
14
a Find the constant of variation.
b How long would it take 10 workers to dig the same trench?

WORKED

Example

11 The fuel economy, f, of a car varies inversely with the speed, s, at which it is driven.
A car that averages 40 km/h has a fuel economy of 15 km/L. What will be the fuel
economy of a car that averages 50 km/h?
12 In an electricity circuit, the current (measured in amps, a) is inversely proportional to
the resistance (measured in ohms, r). When the resistance is 40 ohms, the current is
measured at 3 amps. What will be the current when the resistance is 15 ohms?

Graphing physical phenonema


In many cases, an algebraic function can be used to graph a physical situation. Consider the case of a sphere of radius r. The volume of a sphere can be given by the formula V = 4--- r 3. We can create a table of values that allows us to graph the function for
3
volume.
r

4.19

33.51

3
113.10

4
268.08

5
523.60

10
4189

290

Maths Quest General Mathematics HSC Course

We can then plot each pair of points from the table


and join the points with a smooth curve. The
graph shown at right shows the volume for a
sphere of any radius.

4000
3500
3000
Volume

2500
2000
1500
1000

4 r3
V =
3

500
0

5 10 15
Radius

WORKED Example 15
The surface area of a sphere is given by the formula A = 4 r2.
a Complete the table below.
r

10

A
b Graph the surface area function.
THINK

WRITE

a Substitute each
value of r into the
formula.

Plot each pair of


points generated
by the table.
Join the points
with a smooth
curve and
extrapolate the
graph.

0 12.57 50.27 113.10 201.06 314.16 452.39 615.75 804.25 1017.88 1256.64

3000

10

A = 4 r2

2500
Area

2000
1500
1000
500
0

10
15
Radius

Many graphs have physical restrictions placed on them.


Consider the case of a ball that is thrown vertically
upwards. The height, h, of the ball at any time, t, can be
given by the equation h = 15t 5t 2. The height of the ball
must always be positive, and when the ball returns to Earth
the graph stops as shown on the right.

h
h = 15t 5t 2

12
9
6
3
0

291

Chapter 9 Modelling linear and non-linear relationships

When we graph several points, we try to estimate other values by interpolating


(estimating values between given points by drawing the graph joining the points) or
extrapolating (estimating values by extending the graph beyond the points given).
Other graphs need to have restrictions placed upon them when we try to interpolate
or extrapolate. There may be a limit placed upon one or both of the variables, and this
will indicate a change in the graph.

WORKED Example 16
A cinema owner believes that more people will attend the movies on cold days and so
believes the number of people attending each session of a movie varies inversely with the
temperature of the day. When the temperature is 15C, 80 people attend a movie. The
cinema has a maximum of 120 seats, and the cinema owner believes that a minimum of 40
people will attend, regardless of temperature.
a Write an equation connecting the number of people attending the movie, N, with the
temperature, T.
b Graph the relationship between attendance and temperature.
THINK
WRITE
a 1 Write an inverse proportion
1
a N --statement.
T
k
2 Insert a constant of variation, k, to
N = --form an equation.
T
When T = 15, N = 80.
3 Substitute the known values of N and
k
T to find the value of k.
80 = -----15
k = 1200
1200
4 Replace the known value of k in the
N = -----------equation.
T
b

1
2
3

Create a table of values.


Substitute each value of T into the
equation.
If the value of N > 120, then we
enter 120 for N (maximum
attendance); if N < 40, enter 40 for N
(minimum attendance).

10 15

20

25

30

35

N 120 120 80

60

48

40

40

N
120
100
80
60
40
20

Plot the points and join with a


smooth curve. The minimum and
maximum attendance is drawn with
a straight line.

10

20

30

40

292

Maths Quest General Mathematics HSC Course

remember
1. An algebraic model can be used to represent many physical situations.
2. When modelling a situation, there may be restrictions on one or both of the
variables.

9E

Graphing physical
phenomena

1 The surface area of a cube is given by the formula A = 6s2.


a Complete the table of values below.
15

WORKED

Example

A
b Draw the graph to represent the surface area of a cube of a given side length.
2 The distance that an object will fall when dropped from a height can be given by the
formula d = 5t 2, where d is in metres and t is in seconds. Draw a graph of the function.
3 A car is travelling at v km/h and the driver needs to brake. It takes 2.5 seconds to react
and in that time the car will travel a distance of 0.7v m. The total stopping distance, d,
can be given by the function d = 0.01v2 + 0.7v.
a Copy and complete the table below.
v

10

20

30

40

d
b Draw the graph of the stopping distance of a vehicle.
4 Lorraine organises a lottery syndicate at her work. If they win a prize of $100 000, the
amount is shared equally between the members of the syndicate. There must be at least
16
one member of the syndicate and a maximum of 10.
a Write an equation putting the amount, A, each person receives in terms of the
number of members, n.
b Graph the function.

WORKED

Example

5 A car is purchased new for $40 000. After one year the depreciated value of the car is
$30 000. After the first year the car depreciates at a rate of 20% p.a.
a Copy and complete the table below.
Age (years)

Value
b The car will always be worth a minimum of $2000 in scrap metal and accessories.
Graph the value of the car against the age of the car.
6 The mass of a newborn baby increases by 20% per month for the first four months of
life. If the average mass of a newborn baby is 3.3 kg, graph the mass function up to
n = 4.

Chapter 9 Modelling linear and non-linear relationships

7 A square piece of sheet metal has a side length of 12 m. A


square of side length x m is to be cut from each corner of the
sheet metal and the sides bent up to form an open rectangular
prism.
a What is the maximum possible value of x?
b Show that the volume of the prism formed can be given by
the function V = x(12 2x)2.
c Graph the volume function.

293

12 m
xm

8 The population of a city is growing at a rate of 5% p.a. If the population in 2000 is


1.5 million, the population function can be given by the function P = 1.5(1.05)n, where
P is the population, in millions. The city cannot sustain a population greater than
4 000 000.
a Complete the table below.
Year

2007

2008

2009

2010

2011

b Plot the points given and extrapolate to graph the population function.
c Use your graph to state when the population will reach its maximum sustainable
level.
d What will happen to the graph when it reaches this level?

Force of gravity
When an object is dropped, the
distance that it will fall in t
seconds can be approximated by
the formula d = 5t 2. The
coefficient of t 2 is half the force of
gravity (10 m/s2) and so will
change if an object were to be
dropped on another planet. For
example, on the moon this
equation would become d = 0.8t2.
1 Use a graphics calculator or
graphing software to graph the
equations for both the Earth
and the moon.
2 Find out the force of gravity on
other planets and compare the
graphs formed with that for the
Earth.

Work

Population (million)
T
SHEE

9.2

294

Maths Quest General Mathematics HSC Course

summary
Linear functions
Linear functions have powers of only 1 for both the independent and dependent
variables and are graphed as straight lines.
To graph a linear function, a table of at least three values is drawn; the points
generated are plotted on a number plane and then joined with a straight line.
The intersection of two linear functions will give the point where both conditions
hold true.

Quadratic functions
A quadratic function is a function where the independent variable is raised to the
power of 2.
The graph of a quadratic function is a parabola, a curved graph with either a
minimum (positive x2 term) or a maximum (negative x2 term).
A quadratic function is graphed by plotting the points formed from a table of at
least seven values.

Other functions
A cubic function uses a power of 3 for the independent variable. It is of the form
y = ax3.
a
A hyperbola is a function of the form y = --- . In a hyperbolic function, as one
x
variable increases the other decreases.
An exponential function is of the form y = ax. When a > 1, an exponential function
models exponential growth, while if 0 < a < 1, the function models exponential
decay.
Each of these functions is graphed by plotting points from a table of values.

Variations
A variation occurs when one quantity changes in proportion with another.
If one quantity varies directly with another, as one increases so does the other.
If the quantity varies directly with the square of the other, it can be expressed as a
function in the form y = ax2. If it varies with the cube of another, it can be
expressed in the form y = ax3.
An inverse variation occurs when one quantity decreases, while the other increases.
a
An inverse variation can be expressed in the form y = --- .
x
The constant of variation, a, is calculated by using a known quantity of each
variable. Once this has been calculated, if we know one quantity we can calculate
the other.

Graphing physical phenomena


Algebraic models can be used to represent many physical situations.
When graphing physical phenomena, we need to consider any restrictions that may
exist on one or both of the variables.

Chapter 9 Modelling linear and non-linear relationships

295

CHAPTER
review
1 Graph each of the following linear functions.
a y = 3x
b y=x+3
d y = 5 3x
e 2y = 4x 3

c y=2 x
f 3x 2y + 6 = 0

2 The cost, C, of a taxi fare is given by the formula C = 3 + 0.4d, where d is the distance
travelled by the car, in kilometres.
a Copy and complete the table below.
d

10

15

9A
9A

20

C
b Graph the cost function.
3 At a fete, 400 cans of soft drink are purchased for $320. The cans are then sold for $1.25
each.
a Write, as a linear function, an expression for the profit on the sale of the cans, where n is
the number of cans sold.
b Graph the profit function.
c What will be the financial outcome if:
i 300 cans are sold?
ii 142 cans are sold?
d How many cans will need to be sold for the drink stall to break even?

9A

4 Graph the linear functions y = 6

9A
9A

x and y = x + 2, and hence state the point of intersection.

5 Andrew needs to purchase a new washing machine.


a A brand new washing machine will cost $1000, and running costs will be approximately
20c per wash. Express this as a linear function.
b Alternatively, Andrew could purchase a second-hand washing machine for $200, but
running costs will be about $1.00 per wash. Express this as a linear function.
c Graph both linear functions on the same pair of axes.
d By finding the point of intersection, find out after how many washes does it become more
economical to purchase the new machine.
6 For the quadratic function y = x2 4x + 5:
a copy and complete the table of values below
x

9B
5

y
b draw the graph of the function for x 0
c state the minimum value of the function y = x2
7 For the quadratic function y = x2
the graph for x 0.

2x

4x + 5.

2, draw a table of values and use the table to sketch

9B

296

Maths Quest General Mathematics HSC Course

9B

8 Sketch each of the following quadratic functions for x


a y = (x 4)2
b y = 5 x2

9B

9 An object is dropped from a height of 500 m. Its height above the ground at any time, t, is
given by the function h = 500 5t 2.
a Draw the graph of the function.
b How many seconds does it take for the object to fall to Earth?

9B

10 A team of workers are digging a mine shaft. The number of kilograms of earth moved each
hour by the team is given by the function E = 24n n2, where n is the number of workers
digging the shaft.
a Graph the function.
b What is the maximum amount of earth that can be moved by the team of workers in one
hour? How many workers are needed to move this amount of earth?
c Explain possible reasons why the amount of earth moved each hour then begins to
decrease as more workers are used.

9C

11 Graph each of the following cubic functions for x


a y = x3
b y = 1--- x3

0.
c y = 4 + 2x

x2

0.

9C

12 Graph each of the following hyperbolic functions for x > 0.


2
1
a y = --b y = --x
x

9C

13 Graph each of the following exponential functions.


a y = 2x
b y = ( 1--- )x
2

9C

14 The average inflation rate is 4% p.a. In 2006 it cost the average family $500 per week in
living expenses. The future cost of living, C, can be estimated using the function
C = 500(1.04)n where n is the number of years since 2006.
a Graph the cost of living function.
b Use the graph to estimate the cost of living in 2016.
c When will the cost of living first reach $1000 per week?

9C

15 If the value of a computer purchased for $5000 depreciates by 20% p.a., the future value of
the computer, V, can be given by the equation V = 5000(0.8)n, where n is the age of the
computer, in years.
a Graph the function.
b Find when the value of the computer is approximately $1000.

9D

16 It is known that y varies directly with the square of x. When x = 4, y = 80. Write an equation
connecting x with y.

9D

17 The mass, m, of an egg varies directly with the cube of its length, l. An egg of length 5.5 cm,
has a mass of 75 g.
a Write an equation connecting m with l.
b Find the mass of an egg with a length of 5 cm.
c Find the length of a 50 g egg.

Chapter 9 Modelling linear and non-linear relationships

297

18 It is known that y varies inversely with x. When x = 8, y = 8; write an equation connecting y


with x.

9D

19 The amount of food in a camp varies inversely with the number of people to feed. There is
enough food to feed 100 campers for 10 days.
a Write an equation connecting the amount of food, A, with the number of campers, n.
b Calculate how long the food would last 125 campers.
c If the food lasts for four days, calculate the number of campers.

9D

20 The area of a circle is given by the formula A = r2.


a Complete the table of values below.

9E

A
b Draw the graph of A against r.
21 A ball is thrown directly up in the air. The height, h, of the ball at any time, t, can be found
using the equation h = 20t 5t 2.
a Draw a graph of the height equation.
b Use the graph to find:
i the maximum height of the ball
ii the time taken for the ball to fall back to earth.

9E

22 An investment of $10 000 at 6% p.a. can be modelled using the equation A = 10 000(1.06)n,
where n is the number of years of the investment.
a Graph the function.
b Use your graph to estimate the value of the investment after 8 years.
c Use your graph to find the amount of time that it will take for the investment to grow to
$15 000.

9E

Practice examination questions


1 multiple choice
Which of the following equations is not an example of a linear function?
2
A y = 2x + 1
B y = --C 2y = x + 1
D x + 2y + 1 = 0
x
2 multiple choice
Which of the following quadratic equations is equivalent to y = (x 3)2 + 7?
A y = x2 3x 2
B y = x2 3x + 16
2
C y = x 6x 2
D y = x2 6x + 16
3 multiple choice

The graph shown at right could have the equation:


2
A y = x2
B y = --x
x
1 x
Cy=2
D y = ( --- )
2

298

Maths Quest General Mathematics HSC Course

4 multiple choice
It is known that y varies inversely with x. The variation can be modelled by the equation:
A y = ax
B y = ax2
a
C y = ax3
D y = --x
5 As a fundraising activity, a school hires a cinema to show the premiere of a movie. The cost
of hiring the cinema is $500. People are then charged $10 to attend the movie.
a Write a function for the profit or loss made on the movie in terms of the number of people
attending.
b Graph the function.
c Use the graph to calculate the number of people who must attend the movie for the school
to break even.
d A rival cinema offers to waive the hire fee but the school will receive only $5 per person
attending. On the same axes graph the function P = 5n.
e The school chose to pay the $500 and receive $10 per person. How many people must
attend the premiere to make this the better of the two options?
6 A rock is thrown from a cliff 20 m above ground level. The height of the rock at any time is
given by the quadratic function h = 20 + 15t 5t 2.
a Copy and complete the table below.
t

h
b Graph the function and use your graph to find the maximum height reached by the ball.
2
7 a On the one set of coordinate axes, sketch the graphs of y = 2x3 and y = --- .
x
2
b Use your graphs to find the point of intersection of the graphs y = 2x3 and y = --- .
x

CHAPTER

test
yourself

8 The growth of an investment made at 8% p.a. can be modelled by the equation y = 1.08x.
a Graph the function.
b Use your graph to determine the amount of time that it will take for the investment to
double in value.
c The depreciation of an item at 8% p.a. can be modelled by the equation y = 0.92x. Graph
this function.
d Use your graph to determine the amount of time that it will take for the item to halve in
value.

Depreciation

10
syllabus reference
Financial mathematics 6
Depreciation

In this chapter
10A Modelling depreciation
10B Straight line depreciation
10C Declining balance method
of depreciation
10D Depreciation tables

areyou

READY?

Are you ready?

Try the questions below. If you have difficulty with any of them, extra help can be
obtained by completing the matching SkillSHEET. Either click on the SkillSHEET icon
next to the question on the Maths Quest HSC Course CD-ROM or ask your teacher for
a copy.

10.1

Graphing linear equations

1 Draw the graphs of the following equations.


a y = 2x 1
b y = 8 4x

10.2

Graphing exponential functions

2 Draw the graphs of the following equations for x 0.


a y = 2x
b y = (0.8)x

10.3

Solving linear equations

3 Solve the equations.


a 7x 5 = 79

10.4

c y = 5( 1--- )x

b 3000x 500 = 12 500

c 6000 500x = 3500

Calculating compound interest

4 Calculate:
a The amount to which $10 000 will grow at 6% p.a. over 5 years with interest compounded
annually
b The amount to which $50 000 will grow at 8.2% p.a. over 4 years with interest compounded
six-monthly.

10.6

Reading financial tables

5 The table below shows the amount to which $1 will grow under compound interest.
Interest rate per period
Periods

6%

7%

8%

9%

1.060

1.070

1.080

1.090

1.123

1.145

1.166

1.188

1.191

1.225

1.260

1.295

1.262

1.311

1.360

1.412

Use the table to find:


a the amount to which $10 000 will grow at 7% p.a. over 4 years with interest compounded
annually.
b the amount to which $50 000 will grow at 12% p.a. over 2 years with interest compounded sixmonthly.

Chapter 10 Depreciation

301

Modelling depreciation
An asset is an item that has value to its owner. Many assets such as cars and computers
lose value over time. This is called depreciation.
Consider the case of a new motor vehicle. The value of the car depreciates the
moment that you drive the car away from the showroom. This is because the motor
vehicle is no longer new and if it were sold, it would have to be sold as a used car. The
car then continues to lose value steadily each year.

Depreciation of motor vehicles


Choose a make of car and find out the price for a new vehicle of this make and
model. Look through NRMAs Open Road magazine or the classified advertisements in the newspaper to find the price of the same model as a second-hand car.
Age of car (years)

Price

New (0)
1
2
3
4
5
Draw a graph that shows the price of this car as it ages.

There are two types of depreciation: the straight line method and the declining
balance method. The straight line method is where the asset depreciates by a constant
amount each year. When this type of depreciation is graphed, a straight line occurs and
the asset will reduce to a value of 0.
In such a case, a linear function can be derived that will allow us to calculate the
value of the item at any time. The function can be found using the gradientintercept
method. The purchase price of the asset (V0) will be the vertical intercept, and the
gradient will be the negative of the amount that the item depreciates, D, each period.
The equation of this linear function will be:
V = V0

Dn

where V is the salvage value of the item and n is the age of the asset, in years.
Note: Gradients for depreciation will always be negative.

302

Maths Quest General Mathematics HSC Course

WORKED Example 1

The table below shows the declining


value of a computer. Graph the value
against time and write an equation
for this function.
Age (years)

Value ($)

New (0)

4000

3500

3000

2500

2000

1500

WRITE

Draw a set of axes with age on the


horizontal axis and value on the
vertical.

Plot each point given by the table.

Join all points to graph the function.

Value ($)

THINK

4500
4000
3500
3000
2500
2000
1500
1000
500
0
0123456789
Age (years)

Write the initial value as V0 and use the


gradient to state D.

V0 = 4000, D = 500

Write the equation using V = V0

V = V0 Dt
V = 4000 500t

Dt.

Note: To solve worked example 1 you can use the graphics calculator methods demonstrated in chapter 9.
In worked example 1, how long does it take for the computer to depreciate to a value
of $0? The computer is said to be written off when it reaches this value.
The other method of depreciation used is the declining balance method of
depreciation. Here, the value of the item depreciates each year by a percentage of its
current value. Under such depreciation, the value of the item never actually becomes
zero. This type of depreciation is an example of exponential decay that we saw in
chapter 9.

Chapter 10 Depreciation

303

WORKED Example 2

The table below shows the value of a car that is purchased new for $40 000.
Age of car (years)

Value ($)

New (0)

40 000

32 000

25 600

20 480

16 384

13 107

Plot the points on a set of axes and graph the depreciation of the car. Use the graph to
estimate the value of the car after 10 years.

2
3

WRITE

Draw a set of axes with age on the


horizontal axis and value on the
vertical.
Plot the points from the table.
Join the points with a smooth curve.

Value ($)

THINK

40 000
35 000
30 000
25 000
20 000
15 000
10 000
5 000
0
0 1 2 3 4 5 6 7 8 9 10
Age (years)

Estimate the value after 10 years from


the graph you have drawn.

From the graph, the approximate value of the


car after 10 years is $4000.

remember
1. Depreciation is the loss in the value of an item over time.
2. Depreciation can be of two types:
(a) Straight line depreciation.
The item loses a constant amount of value each year
(b) Declining value depreciation.
The value of an item depreciates by a percentage of its value each year.
3. Straight line depreciation can be graphed using a linear function in which the
new value of the item is the vertical intercept and the gradient is the negative of
the annual loss in value.
4. Declining value depreciation is an example of exponential decay and is
graphed with a smooth curve.

304

Maths Quest General Mathematics HSC Course

SkillS

10A
HEET

10.1

WORKED

Example

SkillS

Graphing
linear
equations
HEET

10.2
Graphing
exponential
functions

Modelling depreciation

1 The table below shows the depreciating value of a tractor.


Age (years)

Value ($)

New (0)

100 000

90 000

80 000

70 000

60 000

50 000

a Draw a graph of the value of the tractor against the age of the tractor.
b Write a function for the value of the tractor.
2 The table below shows the depreciating value of a tow truck.
Age (years)

Value ($)

New (0)

50 000

42 000

34 000

26 000

18 000

10 000

Draw a graph of value against age; hence, write a value as a linear function of age.
3 The function V = 50 000 6000A shows the value, V, of a car when it is A years old.
a Draw a graph of this function.
b Use the graph to calculate the value of the car after 5 years.
c After how many years would the car be written off?
4 A computer is bought new for $6400 and depreciates at the rate of $2000 per year.
a Write a function for the value, V, of the computer against its age, A.
b Draw the graph of this function.
c After how many years does the computer become written off?

Chapter 10 Depreciation

WORKED

Example

305

5 The table below shows the declining value of a new motorcycle.


Age (years)

Value ($)

New (0)

20 000

15 000

11 250

8 450

6 350

4 750

a Plot the points shown by the table, and draw a graph of the value of the motorcycle
against age.
b Use your graph to estimate the value of the motorcycle after 8 years. Give your
answer correct to the nearest $1000.
6 The table below shows the declining value of a semi-trailer.
Age (years)

Value ($)

New (0)

600 000

420 000

295 000

205 000

145 000

100 000

a Plot the points as given in the table, and then draw a curve of best fit to graph the
depreciation of the semi-trailer.
b Use your graph to estimate the value of the semi-trailer after 10 years.
c After what number of years will the value of the semi-trailer fall below $50 000?
7 a A gymnasium values its equipment at $200 000. Each year the value of the equipment depreciates by 20% of the value of the previous year. Calculate the value of the
equipment after:
i 1 year
ii 2 years
iii 3 years
iv 4 years.
b Plot these points on a set of axes and draw a graph of the value of the equipment
against its age.

306

Maths Quest General Mathematics HSC Course

8 multiple choice
Which of the tables below shows a straight line depreciation?
A

Age (years)

Value ($)

Age (years)

Value ($)

New (0)

4000

New (0)

4000

3600

3600

3240

3200

2916

2800

2624

2400

2362

2000

Age (years)

Value ($)

Age (years)

Value ($)

New (0)

4000

New (0)

4000

3600

3000

3300

2500

3100

1500

3000

1000

2950

500

9 A car is bought new for $30 000.


a The straight line method of depreciation sees the car lose $4000 in value each year.
Complete the table below.
Age (years)

Value ($)

New (0)

30 000

1
2
3
4
5
b Draw a graph of this depreciation.

Chapter 10 Depreciation

307

c The declining balance method of depreciation sees the value of the car fall by 20%
of the previous years value. Complete the table below.
Age (years)

Value ($)

New (0)

30 000

1
2
3
4
5
d On the same set of axes draw a graph of this depreciation.
e After how many years is the car worth more under declining balance depreciation
than under straight line depreciation?

Straight line depreciation


We have already seen that the method of straight line depreciation is where the value of
an item depreciates by a constant amount each year. The depreciated value of an item is
called the salvage value, S. The salvage value of an asset can be calculated using the
formula:
S = V0 Dn
where V0 is the purchase price of the asset, D is the amount of depreciation apportioned
per period and n is the number of periods.

WORKED Example 3
A laundry buys dry-cleaning
equipment for $30 000. The
equipment depreciates at a
rate of $2500 per year.
Calculate the salvage value
of the equipment after
6 years.

THINK
1
2
3

WRITE

Write the formula.


Substitute the values of V0, D and n.
Calculate the value of S.

S = V0 Dn
S = $30 000
S = $15 000

$2500

By solving an equation we are able to calculate when the value of an asset falls below
a particular amount.

308

Maths Quest General Mathematics HSC Course

WORKED Example 4

A plumber purchases equipment for a total of $60 000. The value of the equipment is
depreciated by $7500 per year. When the value of the equipment falls below $10 000 it
should be replaced. Calculate the number of years after which the equipment should be
replaced.
THINK

WRITE

Write the formula.

Substitute for S, V0 and D.

10 000 = 60 000

Solve the equation to find the value


of n.

7500n = 50 000
n = 6 2---

Give a written answer, taking the value


of n up to the next whole number.

The equipment must be replaced after 7 years.

S = V0

Dn
7500n

Graphics Calculator tip! Solving a depreciation equation


Your graphics calculator can be used as shown below to solve equations such as that
arising in worked example 4.
1. From the MENU select EQUA.

2. Press F3 (Solver).

3. Delete any existing equation, and enter the equation


that arises after the substitution is made.
Note: You may have a different value of N, but at this
stage this can be ignored.
4. Press F6 (SOLV) to solve the equation.

Chapter 10 Depreciation

309

remember
1. Straight line depreciation occurs when the value of an asset depreciates by a
constant amount each year.
2. The formula to calculate the salvage value, S, of an asset is:
S = V0 Dn
where V0 is the purchase price of the asset, D is the amount of depreciation
apportioned per period and n is the total number of periods.
3. To calculate a value of V0, D or n we substitute all known values and solve the
equation that is formed.

10B
WORKED

Example

1 A car that is purchased for $45 000 depreciates by $5000 each year. Calculate the
salvage value of the car after 5 years.

10.3 SkillS

Solving
2 Calculate the salvage value:
linear
a after 5 years of a computer that is purchased for $5000 and depreciates by $800 equations
per year
b after 7 years of a motorbike that is purchased for $25 000 and depreciates by $2100
per year
c after 6 years of a semi-trailer that is purchased for $750 000 and depreciates by
$80 000 per year
d after 2 years of a mobile phone that is purchased for $225 and depreciates by $40
per year
e after 4 years of a farmers plough that is purchased for $80 000 and depreciates by
$12 000 per year.

3 A bus company buys 15 buses for $475 000 each.


a Calculate the total cost of the fleet of buses.
b If each bus depreciates by $25 000 each year, calculate the salvage value of the
fleet of buses after 9 years.
4 The price of a new car is $25 000. The value of the car depreciates by $300 each
month. Calculate the salvage value of the car after 4 years.
WORKED

Example

5 An aeroplane is bought by an airline for $600 million. If the aeroplane depreciates by


$40 million each year, calculate when the value of the aeroplane falls below $300
million.
6 Calculate the length of time for each of the following items to depreciate to the value
given.
a A computer purchased for $5600 to depreciate to less than $1000 at $900 per year
b An electric guitar purchased for $1200 to depreciate to less than $500 at $150 per
year
c An entertainment unit purchased for $6000 to become worthless at $750 per year
d Office equipment purchased for $12 000 to depreciate to less than $2500 at $1500
per year

HEET

Straight line depreciation

310

Maths Quest General Mathematics HSC Course

7 A motor vehicle depreciates from $40 000 to $15 000 in 10 years. Assuming that it is
depreciating in a straight line, calculate the annual amount of depreciation.
8 Calculate the annual amount of depreciation in an asset that depreciates:
a from $20 000 to $4000 in 4 years
b from $175 000 to $50 000 in 10 years
c from $430 000 to $299 500 in 9 years.
9 A computer purchased for $3600 is written off in 4 years. Calculate the annual
amount of depreciation.

Work

10 A car that is 5 years old has an insured value of $12 500. If the car is depreciating at
a rate of $2500 per year, calculate its purchase price.

T
SHEE

11 Calculate the purchase price of each of the following assets given that:
a after 5 years the value is $50 000 and is depreciating at $12 000 per year
b after 15 years the value is $4000 and is depreciating at $1500 per year
c after 25 years the value is $200 and is depreciating at $50 per year.

10.1

12 An asset that depreciates at $6500 per year is written off after 12 years. Calculate the
purchase price of that asset.

Declining balance method of


depreciation
The declining balance method of depreciation occurs when the value of an asset
depreciates by a given percentage each period.
Consider the case of a car purchased new for $30 000, which depreciates at the rate
of 20% p.a. Each year the salvage value of the car is 80% of its value at the end of the
previous year.
After 1 year: S = 80% of $30 000
= $24 000
After 2 years: S = 80% of $24 000
= $19 200
After 3 years: S = 80% of $19 200
= $15 360

WORKED Example 5
A small truck that was purchased for $45 000 depreciates at a rate of 25% p.a. By
calculating the value at the end of each year, find the salvage value of the truck after
4 years.
THINK
1

The salvage value at the end of each


year will be 75% of its value at the end
of the previous year.
Find the value after 1 year by
calculating 75% of $45 000.

WRITE

After 1 year: S = 75% of $45 000


= $33 750

Chapter 10 Depreciation

THINK
3
4
5

311

WRITE

Find the value after 2 years by


calculating 75% of $33 750.
Find the value after 3 years by
calculating 75% of $25 312.50.
Find the value after 4 years by
calculating 75% of $18 984.38.

After 2 years: S = 75% of $33 750


= $25 312.50
After 3 years: S = 75% of $25 312.50
= $18 984.38
After 4 years: S = 75% of $18 984.38
= $14 238.28

The salvage value under a declining balance can be calculated using the formula:
S = V0(1

r)n

where S is the salvage value, V0 is the purchase price, r is the percentage depreciation
per period expressed as a decimal and n is the number of periods.
This formula can be considered as being similar to the compound interest formula.
In the case of depreciation, however, you need to subtract rather than add the depreciation expressed as a decimal from 1.

WORKED Example 6
The purchase price of a boat is $15 000. The value of the boat depreciates by 10% p.a.
Calculate the salvage value of the boat after 8 years.
THINK
1
2
3

WRITE

Write the formula.


Substitute values for V0, r and n.
Calculate the salvage value.

S = V0(1 r)n
S = $15 000 0.98
S = $6457.00

To calculate the amount by which the asset has depreciated, we subtract the salvage
value from the purchase price.

WORKED Example 7
The purchase price of a motor vehicle is $40 000. The vehicle depreciates by 12% p.a.
Calculate the amount by which the vehicle depreciates in 10 years.
THINK
1
2
3
4

Write the formula.


Substitute the value of V0, r and n.
Calculate the value of S.
Calculate the amount of depreciation by
subtracting the salvage value from the
purchase price.

WRITE
S = V0(1 r)n
S = $40 000 0.8810
S = $11 140.04
Depreciation = $40 000 $11 140.04
Depreciation = $28 859.96

312

Maths Quest General Mathematics HSC Course

remember
1. The declining method of depreciation occurs when the value of an asset
depreciates by a fixed percentage each year.
2. The salvage value of an asset can be calculated by subtracting the percentage
depreciation each year.
3. The salvage value can be calculated using the formula:
S = V0(1 r)n
where S is the salvage value, V0 is the purchase price, r is the percentage
depreciation per period expressed as a decimal and n is the number of periods.
4. To calculate the amount of depreciation, the salvage value should be subtracted
from the purchase price.

SkillS

10C
HEET

10.4

WORKED

Example

SkillS

Calculating
compound
interest
HEET

Declining balance method


of depreciation

1 The purchase price of a forklift is $50 000. The value of the forklift depreciates by
20% p.a. By calculating the value of the forklift at the end of each year, find the
salvage value of the forklift after 4 years.
2 A trailer is purchased for $5000. The value of the trailer depreciates by 15% each
year. By calculating the value of the trailer at the end of each year, calculate:
a the salvage value of the trailer after 5 years (to the nearest $10)
b the amount by which the trailer depreciates:
i in the first year
ii in the fifth year.

10.5
Finding a
percentage
of a
quantity
(money)

3 A company purchases a mainframe computer for $3 000 000. The value of the
computer depreciates by 15% p.a. By calculating the value at the end of each year,
find the number of years that it takes for the salvage value of the mainframe to fall
below $1 000 000.
WORKED

Example

4 Use the formula S = V0(1 r)n to calculate the


salvage value after 7 years of a power generator
purchased for $800 000 that depreciates at a rate
of 10% p.a. (Give your answer correct to the
nearest $1000.)
5 Calculate the salvage value of an asset (correct to
the nearest $100) with a purchase price of:
a $10 000 that depreciates at 10% p.a. for 5 years
b $250 000 that depreciates at 15% p.a. for
8 years
c $5000 that depreciates at 25% p.a. for 5 years
d $2.2 million that depreciates at 30% p.a. for
10 years
e $50 000 that depreciates at 40% p.a. for
5 years.

Chapter 10 Depreciation

313

6 A plumber has tools and equipment valued at $18 000. If the value of the equipment
depreciates by 30% each year, calculate the value of the equipment after 3 years.
WORKED

Example

7 A yacht is valued at $950 000. The value of the yacht depreciates by 22% p.a.
Calculate the amount that the yacht will depreciate in value over the first 5 years
(correct to the nearest $1000).
8 A new car is purchased for $35 000. The owner plans to keep the car for 5 years and
then trade the car in on another new car. The estimate is that the value of the car will
depreciate by 16% p.a. Calculate:
a the amount the owner can expect as a trade in for the car in 5 years (correct to the
nearest $100)
b the amount by which the car will depreciate in 5 years.
9 multiple choice
A shop owner purchases fittings for her store that cost a total of $120 000. Three years
later, the shop owner is asked to value the fittings for insurance. If the shop owner
allows for depreciation of 15% on the fittings, which of the following calculations
will give the correct estimate of their value?
A 120 000 0.853
B 120 000 0.153
C 120 000 0.55
D 120 000 0.45
10 multiple choice
A computer purchased for $3000 will depreciate by 25% p.a. The salvage value of the
computer after 4 years will be closest to:
A $0
B $10
C $950
D $2000
11 An electrician purchases tools of trade for a total of $8000. Each year the electrician
is entitled to a tax deduction for the depreciation of this equipment. If the rate of
depreciation allowed is 33%, calculate:
a the value of the equipment at the end of one year (correct to the nearest $1)
b the tax deduction allowed in the first year
c the value of the equipment at the end of two years (correct to the nearest $1)
d the tax deduction allowed in the second year.
12 An accountant purchased a computer for $6000. The value of the computer depreciates
by 33% p.a. When the value of the computer falls below $1000, it is written off and a
new one is purchased. How many years will it take for the computer to be written off?

Rates of depreciation
In the previous investigation you chose a make and model of car and researched the
salvage value of this car after each year.
1 Calculate the percentage depreciation for each year.
2 Calculate if this percentage rate is approximately the same each year.
3 Using the average annual depreciation, calculate a table of salvage values for the
first 5 years of the cars life.
4 Draw a graph showing the depreciating value of the car.

314

Maths Quest General Mathematics HSC Course

1
1 The price of a new DVD player is $1250. The player will depreciate under straight
line depreciation at a rate of $200 per year. Calculate the value of the player after
3 years.
2 An asset that was valued at $39 000 when new depreciates to $22 550 in 7 years.
Calculate the annual amount of depreciation under straight line depreciation.
3 A computer that is purchased new for $9000 depreciates at a rate of $1350 per year.
Calculate the length of time before the computer is written off.
4 A car dealer values a used car at $7000. If the car is 8 years old and the rate of
depreciation is $1750 per year, calculate the value of the car when new.
5 Write the formula for depreciation under the declining balance method.
6 A truck is valued new at $50 000 and depreciates at a rate of 32% p.a. Calculate the
value of the truck after 5 years (correct to the nearest $50).
7 An asset that has a purchase price of $400 000 depreciates at a rate of 45% p.a.
Calculate the assets value after 6 years (correct to the nearest $1000).
8 For the asset in question 7, calculate the amount by which it has depreciated in
6 years.
9 Office equipment valued at $250 000 depreciates at a rate of 15% p.a. Calculate the
amount by which it depreciates in the first year.
10 Calculate the length of time it will take for the salvage value of the office equipment
in question 9 to fall below $20 000.

Depreciation tables
The computer application below will prepare a table that will show the depreciated
value of an asset with a purchase price of $1 over various periods of time and various
rates of depreciation.

Computer Application 1 Depreciation table


1. Open a new spreadsheet and enter the following information.
2. In cell B3 enter the formula =(1-B$2)^$A3.
3. Highlight the range of cells B3 to J12. Then use the Edit and then the Fill and Right
and Fill and Down functions to copy the formula throughout the table.
4. The table that you now have should have the values shown in the table on page 315.

Chapter 10 Depreciation

315

Rate of depreciation (per annum)


Time
(years)

5%

10%

15%

20%

25%

30%

35%

40%

45%

50%

0.9500 0.9000 0.8500 0.8000 0.7500 0.7000 0.6500 0.6000 0.5500 0.5000

0.9025 0.8100 0.7225 0.6400 0.5625 0.4900 0.4225 0.3600 0.3025 0.2500

0.8574 0.7290 0.6141 0.5120 0.4219 0.3430 0.2746 0.2160 0.1664 0.1250

0.8145 0.6561 0.5220 0.4096 0.3164 0.2401 0.1785 0.1296 0.0915 0.0625

0.7738 0.5905 0.4437 0.3277 0.2373 0.1681 0.1160 0.0778 0.0503 0.0313

0.7351 0.5314 0.3771 0.2621 0.1780 0.1176 0.0754 0.0467 0.0277 0.0156

0.6983 0.4783 0.3206 0.2097 0.1335 0.0824 0.0490 0.0280 0.0152 0.0078

0.6634 0.4305 0.2725 0.1678 0.1001 0.0576 0.0319 0.0168 0.0084 0.0039

0.6302 0.3874 0.2316 0.1342 0.0751 0.0404 0.0207 0.0101 0.0046 0.0020

10

0.5987 0.3487 0.1969 0.1074 0.0563 0.0282 0.0135 0.0060 0.0025 0.0010
5. Use the spreadsheets graphing facility to draw a depreciation graph for each of the
depreciation rates shown in the table.
The table produced by the computer application shows the depreciated value of $1 and
can be used to make calculations about depreciation.

316

Maths Quest General Mathematics HSC Course

WORKED Example 8
An item is purchased for $500 and depreciates at a rate of 15% p.a. Use the depreciation
table on page 315 to calculate the value of the item after 4 years.
THINK
1

WRITE

Look up the table to find the


depreciated value of $1 at 15% p.a. for
4 years.
Multiply the depreciated value of $1 by
$500.

Depreciated value = 0.5220


Depreciated value = $261

$500

The computer application on pages 31415 will produce a general table for a declining
balance depreciation. We should be able to use the formula to create a table and graph
showing the salvage value of an asset under both straight line and declining balance
depreciation.

WORKED Example 9
A car is purchased new for $20 000. The depreciation can be calculated under straight line
depreciation at $2500 per year and under declining balance at 20% p.a.
a Complete the table below. (Give all values to the nearest $1.)
Age of car (years)

Straight line value ($)

Declining balance value ($)

New (0)

20 000

20 000

1
2
3
4
5
6
7
8
b Draw a graph of both the straight line and declining balance depreciation and use the
graph to show the point at which the straight line value of the car falls below the
declining balance value.
THINK
a

1
2

Copy the table.


Complete the straight line column by
subtracting $2500 from the previous
years value.

WRITE

Chapter 10 Depreciation

WRITE

Complete the declining balance by


multiplying the previous years value
by 0.8.

Plot the points generated by the


table.
Join the points for the straight line
depreciation with a straight line.
Join the points for the declining
balance depreciation with a smooth
curve.

a
Age of
car
(years)

Straight line
value ($)

Declining
balance value
($)

New (0)

20 000

20 000

17 500

16 000

15 000

12 800

12 500

10 240

10 000

8 192

7 500

6 554

5 000

5 243

2 500

4 194

3 355

b
25 000
Value ($)

THINK

317

20 000

Straight line
value
Declining
balance value

15 000
10 000
5 000
0
(New)0 1 2 3 4 5 6 7 8
Age (years)

The graph shows the straight line


going below the curve after 6 years.

The straight line depreciation value becomes


less than the declining balance depreciation
value after 6 years.

Depreciation is an allowable tax deduction for people in many occupations. A tax


deduction for depreciation is allowed when equipment used in earning an income
depreciates in value and will eventually need replacing. Depending on the equipment
and the occupation, either straight line or declining balance depreciation may be used.
Under declining balance depreciation, when the salvage value falls below a certain
point the equipment may be written off. This means that the entire remaining balance
can be claimed as a tax deduction and as such is considered worthless. From this point
on, no further tax deductions can be claimed for this equipment.

318

Maths Quest General Mathematics HSC Course

WORKED Example 10
A builder has tools of trade that are purchased new for $14 000. He is allowed a
tax deduction of 33% p.a. for depreciation of this equipment. When the salvage
value of the equipment falls below $3000, the builder is allowed to write the
equipment off on the next years return. Complete the depreciation table below.
(Use whole dollars only.)
Years

Salvage value ($)

Tax deduction ($)

1
2
3
4
5
THINK
1

WRITE

Calculate the salvage value by


multiplying the previous years value
by 0.67.
Calculate the tax deduction by
multiplying the previous years value
by 0.33.
When the salvage value is less than
$3000, claim the entire amount as a tax
deduction.

Year

Salvage value
($)

Tax deduction
($)

9380

4620

6285

3095

4211

2074

2821

1390

2821

remember
1. Graphs can be drawn to compare the salvage value of an asset under different
rates of depreciation, or to compare declining balance and straight line
depreciation.
2. The amount by which an asset depreciates can, in many cases, be claimed as a
tax deduction.

Chapter 10 Depreciation

10D

319

Depreciation tables

1 Use the table of depreciated values of $1 to calculate:


a the value of a computer purchased for $5000 after 5 years, given that it depreci8
ates at 20% p.a.
b the value of a car after 8 years with an initial value of $35 000, given that it
depreciates at 15% p.a.
c the value of a boat with an initial value of $100 000 after 10 years, given that it
depreciates at 10% p.a.

WORKED

10.5 SkillS

Example

HEET

Finding a
percentage
of a
quantity
(money)

Example

SkillS

HEET

2 A taxi owner purchases a new taxi for $40 000. The taxi depreciates under straight line 10.6
depreciation at $5000 per year and under declining balance depreciation at 20% p.a.
9
Reading
a Copy and complete the table below. Give all values to the nearest $100.

WORKED

financial
tables

Straight line value ($)

Declining balance
value ($)

New (0)

40 000

40 000

10.7 SkillS
HEET

Age of car
(years)

Increase or
decreasee by
a percentage

1
2
3
4
5
6
7
8
b Draw a graph of the salvage value of the taxi under both methods of depreciation.
c State when the value under straight line depreciation becomes less than under
declining balance depreciation.
3 A company has office equipment that is valued at $100 000. The value of the equipment can be depreciated at $10 000 each year or by 15% p.a.
a Draw a table that will show the salvage value of the office equipment for the first ten
years using both methods. (Give all values correct to the nearest $50.)
b Draw a graph of the depreciating value of the equipment under both methods of
depreciation.

320

Maths Quest General Mathematics HSC Course

4 A computer purchased new for $4400 can be depreciated at either 20% p.a. or 35% p.a.
Draw a table and a graph that compare the salvage value of the computer at each rate of
depreciation over a 6-year period.
5 A teacher purchases a laptop computer for $6500. A tax deduction for depreciation of
the computer is allowed at the rate of 33% p.a. When the value of the computer falls
10
below $1000, the computer can be written off. Copy and complete the table below.
(Give all values correct to the nearest $1.)

WORKED

Example

Year

Salvage value ($)

Tax deduction ($)

1
2
3
4
5
6
6 A plumber purchases a work van for $45 000. The van can be depreciated at a rate of
25% p.a. for tax purposes, and the van can be written off at the end of 8 years. Copy
and complete the depreciation schedule below. (Give all answers correct to the nearest
$1.)
Year
1
2
3
4
5
6
7
8

Salvage value ($)

Tax deduction ($)

Chapter 10 Depreciation

321

7 A truck is purchased for $250 000. The truck can be depreciated at the rate of $25 000
each year or over 10 years at 20% p.a.
a Copy and complete the table below. (Give all values correct to the nearest $1.)
Age of truck
(years)

Straight line value ($)

Declining balance
value ($)

New (0)

250 000

250 000

1
2
3
4
5
6
7
8
9
10
b Draw a graph of the depreciating value of the truck under both methods of
depreciation.
c Complete a depreciation schedule for each method of calculation.

9 Calculate the amount of depreciation on each of the following assets.


a A tractor with an initial value of $80 000 that depreciates at 15% p.a. for 3 months
b A bicycle with an initial value of $600 that depreciates at 25% p.a. for 6 months
c Office furniture with an initial value of $8000 that depreciates at 30% p.a. for
8 months
d A set of encyclopedias with an initial value of $2500 that depreciates at 40% p.a. for
9 months

Work

8 Tony is a plumber and on 1 March purchases a panel van for work purposes. The cost
of the panel van is $40 000, and for tax purposes the panel van depreciates at the rate of
25% p.a.
a Calculate the amount that the panel van will depreciate in the first year.
b The financial year ends on 30 June. For what fraction of the financial year did Tony
own the panel van?
c Tony is allowed a tax deduction for depreciation of his work van. Calculate the
amount of tax deduction that Tony is allowed for the financial year ending on
30 June.

T
SHEE

10.2

322

Maths Quest General Mathematics HSC Course

summary
Modelling depreciation
Depreciation can be calculated in two ways. The depreciation can be straight line
depreciation or declining balance depreciation.
Straight line depreciation occurs when the value of an asset decreases by a constant
amount each year. The graph of the salvage value is a straight line, the vertical
intercept is the purchase price and the gradient is the negative of the annual
depreciation.
Declining balance depreciation occurs when the salvage value of the item is a
percentage of the previous years value. The graph of a declining balance
depreciation will be an exponential decay graph.

Straight line depreciation


The salvage value of an asset under straight line depreciation can be calculated
using the formula:
S = V0 Dn
where S is the salvage value, V0 is the purchase price of the asset, D is the amount
of depreciation apportioned per period and n is the number of periods of
depreciation.
Values of V0, D or n can be calculated by substitution and solving the equation
formed.

Declining balance depreciation


Under declining balance depreciation the salvage value of an asset can be
calculated using the formula:
S = V0(1 r)n
where r is the percentage depreciation per period expressed as a decimal.
To calculate the amount by which an asset depreciates in a year, we subtract the
salvage value at the end of the year from the salvage value at the beginning of the
year.

Depreciation tables
Depreciation can be compared using either a table or a graph.
Tax deductions are allowed for depreciation of assets that are used as part of
earning an income.
A depreciation schedule is used to calculate tax deductions over a period of years
on an asset.

Chapter 10 Depreciation

323

CHAPTER
review
1 The table below shows the depreciating value of a pleasure cruiser.
Age (years)

Value ($)

New (0)

200 000

180 000

160 000

140 000

120 000

100 000

10A

a Draw a graph of the value of the pleasure cruiser against its age.
b Write a function for the value of the pleasure cruiser.
2 The table below shows the depreciating value of a racing bike.
Age (years)

Value ($)

New (0)

3500

3250

3000

2750

2500

2250

10A

a Draw a graph of the value of the bike against age.


b Write a function for the straight line depreciation.
c Use your graph to estimate the value of the bike after 9 years.
3 The function V = 15 000 900A shows the value, V, of a motorcycle when it is
A years old.
a Draw a graph of this function.
b Use the graph to calculate the value of the motorcycle after 5 years.
c After how many years would the motorcycle be written off?

10A

324
10A

Maths Quest General Mathematics HSC Course

4 The table below shows the declining value of a delivery van.


Age (years)

Value ($)

New (0)

60 000

48 000

38 400

30 720

24 576

19 660

a Plot the points as given in the table, and then draw a curve of best fit to graph the
depreciation of the van.
b Use your graph to estimate the value of the van after 10 years.
c After what number of years will the value of the van fall below $10 000?

10A

5 A laundry buys dry-cleaning equipment for $8000. Each year the equipment depreciates by
25% of the previous years value. Calculate the value of the equipment at the end of the first
five years, and use the results to draw a graph of the depreciation.

10B

6 The purchase price of a car is $32 500. The car depreciates by $3250 each year. Use the
formula S = V0 Dn to calculate the salvage value of the car after 8 years.

10B

7 Calculate the salvage value of an asset:


a after 6 years, that was purchased for $4000 and depreciates by $450 each year
b after 10 years, that was purchased for $75 000 and depreciates by $6000 each year
c after 9 years, that was purchased for $640 000 and depreciates by $45 000 each year.

10B

8 A movie projector is purchased by a cinema for $30 000. The projector depreciates by $2500
each year. Calculate the length of time it takes for the projector to be written off.

10B

9 A camera that was purchased new for $1500 has a salvage value of $500 four years later.
Calculate the annual amount of depreciation on the camera.

10B

10 Arthur buys a car for $25 000. The depreciation on the car is $2250 each year. He decides
that he will trade the car in on a new car in the final year before the salvage value falls below
$10 000. When will Arthur trade the car in?

10C

11 The purchase price of a mobile home is $40 000. The value of the mobile home depreciates
by 15% p.a. By calculating the value of the mobile home at the end of each year, find the
salvage value of the mobile home after 4 years. (Give your answer correct to the nearest $1.)

10C

12 Use the formula S = V0(1 r)n to calculate the salvage value after 7 years of a crop duster
that was purchased for $850 000 and depreciates at 8% p.a. (Give your answer correct to the
nearest $1000.)

10C

13 Calculate the salvage value of an asset (correct to the nearest $10) with a purchase price of:
a $40 000 that depreciates at 10% p.a. for 5 years
b $1500 that depreciates at 4% p.a. for 10 years
c $180 000 that depreciates at 12.5% p.a. for 15 years
d $4.5 million that depreciates at 40% p.a. for 10 years
e $250 000 that depreciates at 33 1--- % p.a. for 4 years.
3

Chapter 10 Depreciation

325

14 A company buys a new bus for $600 000. The company keeps buses for 10 years and then
trades them in on a new bus. The estimate is that the value of the bus will depreciate by
12% p.a. Calculate:
a the amount the owner can expect as a trade-in for the bus in 10 years
b the amount by which the bus will depreciate in 10 years.

10C

15 A company has office equipment that is valued at $100 000. The value of the equipment can
be depreciated at $10 000 each year or by 15% p.a.
a Draw a table to show the salvage value of the office equipment for the first ten years.
b Draw a graph of the depreciating value of the equipment under both depreciation methods.

10D

16 A personal computer is purchased for $4500. A tax deduction for depreciation of the
computer is allowed at the rate of 33% p.a. When the value of the computer falls below
$1000, the computer can be written off. Copy and complete the table below.

10D

Year

Salvage value ($)

Tax deduction ($)

1
2
3
4
5

Practice examination questions


1 multiple choice
Which of the following tables gives an example of declining balance depreciation?
A

Year

Salvage value

Year

Salvage value ($)

New (0)

20 000

New (0)

20 000

18 000

18 200

16 200

16 400

14 580

14 600

13 122

12 800

Year

Salvage value

Year

Salvage value ($)

New (0)

20 000

New (0)

20 000

18 000

17 000

16 500

15 000

15 500

14 000

15 000

13 500

326

Maths Quest General Mathematics HSC Course

2 multiple choice
A helicopter is purchased by a company for $3.3 million. The salvage value of the helicopter
depreciates in a straight line at a rate of $240 000 per year. After how many years will the
value of the helicopter be less than $1 million?
A 8
B 9
C 10
D 11
3 multiple choice
Trevor purchases a new computer for $5000. It depreciates under declining balance
depreciation at a rate of 20% p.a. Each year Trevor claims the amount of depreciation on the
computer as a tax deduction. The amount of Trevors tax deduction in the third year is:
A $640
B $1000
C $2560
D $3200
4 multiple choice
The value of a new car depreciates by 12.5% p.a. The salvage value in 5 years of a car that
was purchased new for $37 500 is (to the nearest $100):
A $9375
B $18 300
C $19 200
D $32 800
5 The value of a home theatre system when purchased new is $3000. The system depreciates at
the rate of 15% p.a. under declining balance depreciation.
a Calculate the salvage value of the system in 4 years (correct to the nearest $1).
b By how much has the system depreciated in this time?
c Calculate the equivalent rate of straight line depreciation over the four years.
d Graph the salvage value of the home theatre system under both declining balance and
straight line depreciation.
6 An office is fitted with $200 000 of office equipment. The company claims tax deductions for
the depreciation of the equipment at the rate of 12% p.a.
a Calculate the amount of tax deduction claimed by the company in the first year.
b Complete the depreciation schedule below.
Year

Salvage value ($)

Tax deduction ($)

1
2
3
4

CHAPTER

test
yourself

10

5
c When the value of the equipment falls below $50 000, the equipment is written off and
replaced. After how many years will the equipment be written off?

The normal
distribution

11
syllabus reference
Data analysis 6
The normal distribution

In this chapter
11A z-scores
11B Comparison of scores
11C Distribution of scores

areyou

READY?

Are you ready?

Try the questions below. If you have difficulty with any of them, extra help can be
obtained by completing the matching SkillSHEET. Either click on the SkillSHEET icon
next to the question on the Maths Quest HSC Course CD-ROM or ask your teacher for
a copy.

11.1

11.2

Finding the mean

1 Find the mean of each of the following data sets.


a 4, 6, 2, 7, 9, 3, 6, 9
b 45, 72, 90, 70, 69, 48, 62, 99, 47, 55
c
d
Score
Frequency
Score
Frequency
6

15

12

13

16

36

16

17

42

12

18

15

10

19

11

20

Finding the standard deviation

2 Find the population standard deviation of each of the data sets in question 1. Give each answer
correct to 1 decimal place.

11.3

Choosing the appropriate standard deviation

3 In each of the following choose the appropriate measure of standard deviation.


a On entering a certain music store people are asked how many CDs they own.
b The number of parliamentarians who vote in favour of bills brought before parliament.

Chapter 11 The normal distribution

329

z-scores
A normal distribution is a statistical occurrence where a data set
of scores is symmetrically distributed about the mean. Most
continuous variables in a population, such as height, mass and
time, are normally distributed. In a normal distribution, the
frequency histogram is symmetrical and begins to take on a
x
bell shape as shown by the figure on the right.
The normal distribution is symmetrical about the mean, which has the same value as
the median and mode in this distribution. The graph of a normal distribution will extend
symmetrically in both directions and will always remain above the x-axis.
The spread of the normal distribution will depend on the standard deviation. The lower
the standard deviation, the more clustered the scores will be around the mean. The figure
below left shows a normal distribution with a low standard deviation, while the figure
below right shows a normal distribution with a much greater standard deviation.

x
x

To gain a comparison between a particular score and the rest of the population, we
use the z-score. The z-score (or standardised score) indicates the position of a
particular score in relation to the mean. z-scores are a very important statistical measure
and later in the chapter some of their uses will be explained.
A z-score of 0 indicates that the score obtained is equal to the mean, a negative
z-score indicates that the score is below the mean and a positive z-score indicates a
score above the mean.
The z-score measures the distance from the mean in terms of the standard deviation.
A score that is exactly one standard deviation above the mean has a z-score of 1. A
score that is exactly one standard deviation below the mean has a z-score of 1.
To calculate a z-score we use the formula:
xx
z = ----------s
where x is the score, x is the mean and s is the standard deviation.

WORKED Example 1

In an IQ test the mean IQ is 100 and the standard deviation is 15. Dales test results give
an IQ of 130. Calculate this as a z-score.
THINK
1

Write the formula.

Substitute for x, x and s.

Calculate the z-score.

WRITE
xx
z = ----------s
130 100
z = -----------------------15
z=2

330

Maths Quest General Mathematics HSC Course

Dales z-score is 2, meaning that his IQ is exactly two standard deviations above the mean.
Not all z-scores will be whole numbers; in fact most will not be whole numbers. A
whole number indicates only that the score is an exact number of standard deviations
above or below the mean.

WORKED Example 2
A sample of professional basketball players gives the mean height as 192 cm with a
standard deviation of 12 cm. Dieter is 183 cm tall. Calculate Dieters height as a z-score.
THINK

WRITE

Write the formula.

Substitute for x, x and s.

Calculate the z-score.

xx
z = ----------s
183 192
z = -----------------------12
z = 0.75

The negative z-score in worked example 2 indicates that Dieters height is below the
mean but, in this case, by less than one standard deviation.
When examining z-scores, care must be taken to use the appropriate value for the
standard deviation. If examining a population, the population standard deviation ( n)
should be used and if a sample has been taken, the sample standard deviation ( n 1
or sn) should be used.
Remember: Your graphics calculator displays all of this information once data is stored
and calculated using the statistics function.

WORKED Example 3
To obtain the average number of hours study done by students in her class per week,
Kate surveys 20 students and obtains the following results.
12 18 15 14 9 10 13 12 18 25
15 10 3 21 11 12 14 16 17 20
a Calculate the mean and population standard deviation (correct to 3 decimal places).
b Robert does 16 hours of study each week. Express this as a z-score based on the above
results. (Give your answer correct to 3 decimal places.)
THINK

WRITE

1
2
3

Enter the data into your calculator.


Obtain the mean from your calculator.
Obtain the standard deviation from your
calculator using the sample standard deviation.

Write the formula.

Substitute for x, x and s.

Calculate the z-score.

x = 14.25
sn = 4.753

xx
b z = ----------s
16 14.25
z = ------------------------4.753
z = 0.368

Chapter 11 The normal distribution

331

Graphics Calculator tip! Finding the z-score


Your graphics calculator can be used to find a z-score once the data is stored and the
calculator has in its memory the mean and standard deviation. The z-score calculated
here is found using the population standard deviation. Make sure that this is the appropriate standard deviation for the question that you are doing. Consider worked example 3.
1. From the MENU select STAT.

2. Delete any existing data and enter the scores from


worked example 3 in List 1.

3. Press F2 (CALC). You may need to press F6 first


for more options.

4. Press F6 (SET). Check that 1Var Xlist is set to List 1


and 1Var Freq is set to 1.

5. Press EXE to return to the previous screen, and


then press F1 (1VAR) to display the summary
statistics.

6. Press MENU and then select RUN.

7. Press OPTN
(PROB).

F6 for more options and then F3

8. Again press F6 for more options and then F4 t().


This is the z-score function, so enter 16, close
brackets and press EXE .

332

Maths Quest General Mathematics HSC Course

remember
1. A data set is normally distributed if it is symmetrical about the mean.
2. The graph of a normally distributed data set is a bell-shaped curve that is
symmetrical about the mean. In such a distribution the mean, median and mode
are equal.
3. A z-score is used to measure the position of a score in a data set relative to the
mean.
4. The formula used to calculate a z-score is:
xx
z = ----------s
where x is the score, x is the mean, and s is the standard deviation.

SkillS

11A
HEET

11.1

WORKED

Example

SkillS

3 Tracy is a nurse and samples the mass of


50 newborn babies born in the
hospital in which she works.
She finds that the mean
mass is 3.5 kg, with a
standard deviation of
0.4 kg. What would be
the standardised score of
a baby whose birth mass
was:
a 3.5 kg?
b 3.9 kg?
c 2.7 kg?
d 4.7 kg?
e 3.1 kg?

11.2

SkillS

Finding
the
standard
deviation
HEET

1 In a Maths exam the mean score is 60 and the standard deviation is 12. Chifunes
mark is 96. Calculate her mark as a z-score.
2 In an English test the mean score was 55 with a standard deviation of 5. Adrian scored
45 on the English test. Calculate Adrians mark on the test as a z-score.

Finding
the
mean
HEET

z-scores

11.3
Choosing the
appropriate
standard
deviation

4 Ricky finds that the mean number of hours spent watching television each week by
Year 12 students is 10.5 hours, with a standard deviation of 3.2 hours. How many
hours of television is watched by a person who has a standardised score of:
a 0?
b 1?
c 2?
d 1?
e 3?
WORKED

Example

5 IQ tests have a mean of 100 and a standard deviation of 15. Calculate the z-score for
a person with an IQ of 96. (Give your answer correct to 2 decimal places.)
6 The mean time taken for a racehorse to run 1 km is 57.69 s, with a standard deviation
of 0.36 s. Calculate the z-score of a racehorse that runs 1 km in 58.23 s.

Chapter 11 The normal distribution

333

7 In a major exam every subject has a mean score of 60 and a standard deviation of
12.5. Clarissa obtains the following marks on her exams. Express each as a z-score.
a English 54
b Maths 78
c Biology 61
d Geography 32
e Art 95
8 The mean time for athletes over 100 m is 10.3 s, with a standard deviation of 0.14 s.
What time would correspond to a z-score of:
a 0?
b 2?
c 0.5?
d 3?
e 0.35?
f 1.6?
WORKED

Example

9 The length of bolts being produced by a machine needs to be measured. To do this, a


sample of 20 bolts are taken and measured. The results (in mm) are given below.
20 19 18 21 20 17 19 21 22 21
17 17 21 20 17 19 18 22 22 20
a Calculate the mean and standard deviation of the distribution.
b A bolt produced by the machine is 22.5 mm long. Express this result as a z-score.
(Give your answer correct to 2 decimal places.)
10 A garage has 50 customers who have credit accounts with them. The amount spent by
each credit account customer each week is shown in the table below.
Amount ($)

Class centre

Frequency

020

2040

4060

19

6080

15

80100

a Copy and complete the table.


b Calculate the mean and standard deviation.
c Calculate the z-score that corresponds to a customers weekly account of:
i $50
ii $100
iii $15.40.
11 multiple choice
In a normal distribution, the mean is 21.7 and the standard deviation is 1.9. A score of
20.75 corresponds to a z-score of:
A 1
B 0.5
C 0.5
D1
12 multiple choice
In a normal distribution, the mean is 58. A score of 70 corresponds to a standardised
score of 1.5. The standard deviation of the distribution is:
A6
B 8
C 10
D 12
13 multiple choice
In a normal distribution, a score of 4.6 corresponds to a z-score of 2.4. It is known
that the standard deviation of the distribution is 0.8. The mean of the distribution is:
A 2.2
B 2.68
C 6.52
D 6.8

334

Maths Quest General Mathematics HSC Course

14 The results of 24 students sitting a Maths exam are listed below.


95 63 45 48 78 75 80 66 60 58 59 62
52 57 64 75 81 60 65 70 65 63 62 49
a Calculate the mean and standard deviation of the exam marks.
b Calculate the standardised score of the highest score and the lowest score, correct
to 2 decimal places.

Work

15 The results of Lukes exams are shown in the table below.

T
SHEE

11.1

Subject

Lukes mark

Mean

Standard deviation

English

72

60

12

Maths

72

55

13

Biology

76

64

Computing studies

60

70

Visual arts

60

50

15

Music

50

58

10

Convert each of Lukes results to a standardised score.

Comparison of scores
An important use of z-scores is to compare scores from different data sets. Suppose that
in your Maths exam your result was 74 and in English your result was 63. In which
subject did you achieve the better result?
It may appear, at first glance, that the Maths result is better, but this does not take
into account the difficulty of the test. A mark of 63 on a difficult English test may in
fact be a better result than 74 if it was an easy Maths test.
The only way that we can fairly compare the results is by comparing each result with
its mean and standard deviation. This is done by converting each result to a z-score.
If for Maths x = 60 and

= 12, then

xx
z = ----------s
74 60
= -----------------12
= 1.17

And if for English x = 50 and

xx
= 8, then z = ----------s
63 50
= -----------------8
= 1.625

The English result is better because the higher z-score shows that the 63 is higher in
comparison to the mean of each subject.

Chapter 11 The normal distribution

335

WORKED Example 4
Janine scored 82 in her Physics exam and 78 in her Chemistry exam. In Physics, x = 62
and n = 10, while in Chemistry, x = 66 and n = 5.
a Write both results as a standardised score.
b Which is the better result? Explain your answer.
THINK

WRITE

xx
xx
a Physics: z = ----------- Chemistry: z = ----------s
s
82 62
78 66
= -----------------= -----------------10
5
=2
= 2.4

Write the formula for each subject.

Substitute for x, x and s.

Calculate each z-score.

b Explain that the subject with the highest


z-score is the better result.

b The Chemistry result is better because of the


higher z-score.

In each example the circumstances must be read carefully to see whether a higher or
lower z-score is better. For example, if we were comparing times for runners over
different distances, the lower z-score would be the better one.

WORKED Example 5
In international swimming the mean time for the mens 100 m freestyle is 50.46 s with a
standard deviation of 0.6 s. For the 200 m freestyle, the mean time is 1 min 51.4 s with a
standard deviation of 1.4 s. Sams best time is 49.92 s for 100 m and 1 min 49.3 for 200 m.
At a competition Sam can enter only one of these events. Which event should he enter?
THINK

WRITE

Write the formula for both events.

Substitute for x, x and s. (For 200 m


convert time to seconds.)
Calculate the z-scores.
The best event is the one with the lower
z-score.

3
4

xx
100 m: z = ----------200 m: z =
s
49.92 50.46
= --------------------------------=
0.6

xx
----------s
109.3 111.4
--------------------------------1.4

= 0.9
= 1.5
The z-score for 200 m is lower, indicating that
Sams time is further below the mean and that
this is the event that he should enter.

remember
1. Scores can be compared by their z-scores as they compare the score with the
mean and the standard deviation.
2. Read each question carefully to see if a higher or lower z-score is a better
outcome.

336

Maths Quest General Mathematics HSC Course

11B
EXCE

et

reads
L Sp he

One
variable
statistics

WORKED

Example

Comparison of scores

1 Kens English mark was 75 and his Maths mark was 72. In English the mean was 65
with a standard deviation of 8, while in Maths the mean mark was 56 with a standard
deviation of 12.
a Convert the mark in each subject to a z-score.
b In which subject did Ken perform better? Explain your answer.
2 In the first Maths test of the year the mean mark was 60 and the standard deviation
was 12. In the second test the mean was 55 and the standard deviation was 15.
Barbara scored 54 in the first test and 50 in the second test. In which test did Barbara
do better? Explain your answer.
3 multiple choice
The table below shows the mean and standard deviation in four subjects.
Subject

Mean

Standard deviation

English

60

12

Maths

65

Biology

62

16

Geography

52

7.5

Kellys marks were English 66, Maths 70, Biology 50 and Geography 55. In which
subject did Kelly achieve her best result?
A English
B Maths
C Biology
D Geography
4 multiple choice
The table below shows the mean and standard deviation of house prices in four
Australian cities. The table also shows the cost of building the same three-bedroom
house in each of the cities.
City

Mean

Standard deviation

Cost

Sydney

$230 000

$30 000

$215 000

Melbourne

$215 000

$28 000

$201 000

Adelaide

$185 000

$25 000

$160 000

Brisbane

$190 000

$20 000

$165 000

In which city is the standardised cost of building the house least?


A Sydney
B Melbourne
C Adelaide
D Brisbane

Chapter 11 The normal distribution

WORKED

Example

337

5 Karrie is a golfer who scored 70 on course A, which has a mean of 72 and a standard
deviation of 2.5. On course B, Karrie scores 69. The mean score on course B is 72
and the standard deviation is 4. On which course did Karrie play
the better round? (In golf the lower score is better.)
6 Steve is a marathon runner. On the Olympic course in Sydney
the mean time is 2 hours and 15 minutes with a standard deviation
of 4.5 minutes. On Athens Olympic course the mean time is
2 hours and 16 minutes with a standard deviation of 3 minutes.
In Sydney Steves time was 2 hours 17 minutes and in Athens his
time was 2 hours 19 minutes.
a Write both times as a z-score.
b Which was the better performance? Explain your
answer.
7 multiple choice
The table below shows the mean and standard deviation
of times in the 100 m by the same group of athletes on
four different days. It also shows Matts time on each of these days.
Day

Mean

Standard deviation

Matts time

8 Jan.

10.21

0.15

10.12

15 Jan.

10.48

0.28

10.30

22 Jan.

10.14

0.09

10.05

29 Jan.

10.22

0.12

10.11

On what day did Matt give his best performance?


A 8 Jan.
B 15 Jan.
C 22 Jan.

D 29 Jan.

8 multiple choice
In which of the following subjects did Alyssa achieve her best standardised result?
Subject

Alyssas mark

Mean

Standard deviation

English

54

60

12

Maths

50

55

15

Biology

60

65

Music

53

62

A English

B Maths

C Biology

D Music

9 Shun Mei received a mark of 64 on her Maths exam and 63 on her Chemistry exam.
To determine how well she actually did on the exams, Shun Mei sampled 10 people
who sat for the same exams and the results are shown below.
Maths:
56 45 82 90 41 32 65 60 55 69
Chemistry: 55 63 39 92 84 46 47 50 58 62

338

Maths Quest General Mathematics HSC Course

a Calculate the mean and standard deviation for Shun Meis sample in each subject.
b By converting each of Shun Meis marks to z-scores, state the subject in which she
performed best.
10 Ricardo scored 85 on an entrance test for a job. The test has a mean score of 78 and a
standard deviation of 8. Kory sits a similar exam and scores 27. In this exam the mean
is 18 and the standard deviation is 6. Who is better suited for the job? Explain your
answer.

1
1 In a normal distribution the mean is 32 and the standard deviation 6. Convert a score
of 44 to a z-score.
2 In a normal distribution the mean is 1.2 and the standard deviation is 0.3. Convert a
score of 0.6 to a z-score.
3 The mean of a distribution is 254 and the standard deviation is 39. Write a score of
214 as a standardised score, correct to 2 decimal places.
4 The mean mark on an exam is 62 and the standard deviation is 9.5. Convert a mark of
90 to a z-score. (Give your answer correct to 2 decimal places.)
5 Explain what is meant by a z-score of 1.
6 Explain what is meant by a z-score of 2.
7 In a distribution, the mean is 50 and the standard deviation is 10. What score
corresponds to a z-score of 0?
8 In a distribution the mean score is 60. If a mark of 76 corresponds to a standardised
score of 2, what is the standard deviation?
9 Cynthia scored a mark of 65 in English where the mean was 55 and the standard
deviation is 8. In Maths Cynthia scored 66 where the mean was 52 and the standard
deviation 10. Convert the mark in each subject to a z-score.
10 In which subject did Cynthia achieve her best result?

Comparison of subjects
1 List all the subjects that you study. Arrange the subjects in the order that you
feel is from your strongest subject to your weakest.
2 List your most recent exam results in each subject.
3 From your teachers, find out the mean and standard deviation of the results in
each subject.
4 Convert each of your marks to a standardised score.
5 List your subjects from best to worst based on the standardised score and see
how this list compares with the initial list that you wrote.

339

Chapter 11 The normal distribution

Distribution of scores
In any normal distribution, the percentage of scores that lie within a certain number
of standard deviations of the mean is always the same, provided that the sample is
large enough. This is true irrespective of the values of the mean and standard
deviation.
In any normal distribution, approximately
68% of the values will lie within one standard
deviation of the mean. This means 68% of
scores will have a z-score between 1 and 1.
68%
This can be shown on a normal curve as:
z
3 2 1

Approximately 95% of the values lie within


2 standard deviations, or have a z-score of
between 2 and 2.

95%
z
3 2 1

Approximately 99.7% of scores lie within 3


standard deviations, or have a z-score that lies
between 3 and 3.
99.7%
z
3 2 1

If we know that a random variable is approximately normally distributed, and we


know its mean and standard deviation, then we can use this rule to quickly make some
important statements about the way in which the data values are distributed.

WORKED Example 6
Experience has shown that the scores obtained on a commonly used IQ test can be
assumed to be normally distributed with a mean of 100 and a standard deviation of 15.
Approximately what percentage of the distribution lies:
a between 85 and 115?
b between 70 and 130?
c between 55 and 145?
THINK

WRITE

85 100
115 100
a z = --------------------z = -----------------------15
15
= 1
=1
68% of the scores will lie between
85 and 115.

Calculate the z-scores for


85 and 115.

68% of scores have a z-score


between 1 and 1.

Continued over page

340

Maths Quest General Mathematics HSC Course

THINK

WRITE

70 100
130 100
b z = --------------------z = -----------------------15
15
= 2
=2
95% of the scores will lie between 70 and 130.

Calculate the z-scores for


70 and 130.

95% of scores have a z-score


between 2 and 2.

Calculate the z-scores for


55 and 145.

99.7% of scores have a z-score


between 3 and 3.

55 100
145 100
c z = --------------------z = -----------------------15
15
= 3
=3
99.7% will lie between 55 and 145.

We can also make statements about the percentage of scores that lie in the tails of the
distribution by using the symmetry of the distribution and remembering that 50% of
scores will have a z-score greater than 0 and 50% will have a z-score less than 0.

WORKED Example 7

In an exam x = 60 and
above 84?

= 12. What percentage of candidates in the exam scored

THINK

WRITE

Calculate 84 as a z-score.

Draw a sketch showing 95% of z-scores


lie between 2 and 2.
5% of z-scores therefore lie outside this
range. Half of these scores lie below 2
and half are above 2.

xx
z = ----------s
84 60
z = -----------------12
z=2

2.5%

95%
95%
60

Give a written answer.

2.5%
84

2.5% of scores are greater than 84.

Some important terminology is used in connection with this rule. We can say that if
95% of scores have a z-score between 2 and 2, then if one member of the population
is chosen, that member will very probably have a z-score between 2 and 2.
If 99.7% of the population has a z-score between 3 and 3, then if one member of
that population is chosen, that member will almost certainly have a z-score between 3
and 3.

Chapter 11 The normal distribution

341

WORKED Example 8
A machine produces tyres that have a mean thickness of 12 mm, with a standard deviation
of 1 mm. If one tyre that has been produced is chosen at random, within what limits will
the thickness of the tyre:
a very probably lie?
b almost certainly lie?
THINK

WRITE

Tyre thickness will very probably


have a z-score between 2 and 2.
A z-score of 2 corresponds to a tyre
of 10 mm thickness.
A z-score of 2 corresponds to a tyre
of 14 mm thickness.

a If z = 2
If z = 2
x = x 2s
x = x + 2s
= 12 2 1
= 12 + 2 1
= 10
= 14
A tyre chosen will very probably have a
thickness of between 10 and 14 mm.

Tyre thickness will almost certainly


have a z-score between 3 and 3.
A z-score of 3 corresponds to a tyre
of 9 mm thickness.
A z-score of 3 corresponds to a tyre
of 15 mm thickness.

b If z = 3
If z = 3
x = x 3s
x = x + 35
= 12 3 1
= 12 + 3 1
= 9
= 15
A tyre chosen will almost certainly have a
thickness of between 9 and 15 mm.

Because it is almost certain that a member of the data set will lie within three standard
deviations of the mean, if a possible member of the data set is found to be outside this
range one should suspect a problem.
For example, if a machine is set to deposit 200 mL of liquid into a bottle, with a
standard deviation of 5 mL, and then a bottle is found to have contents of 220 mL, one
would expect there to be a problem with the settings on the machine.
This knowledge of z-scores is then used in industry by the quality control department. In the above example a sample of bottles would be tested and the z-scores
recorded. The percentage of z-scores between 1 and 1, 2 and 2, and 3 and 3 are
checked against the above rule. If these percentages are not correct, the machinery
needs to be checked for faults.

remember
1. In a normal distribution:
68% of scores will have a z-score between 1 and 1
95% of scores will have a z-score between 2 and 2
99.7% of scores will have a z-score between 3 and 3.
2. The symmetry of the normal distribution allows us to make calculations about
the percentage of scores lying within certain limits.
3. If a member of a normally distributed population is chosen, it will:
very probably have a z-score between 2 and 2
almost certainly have a z-score between 3 and 3.
4. Any score further than three standard deviations from the mean indicates that
there may be a problem with the data set.

342

Maths Quest General Mathematics HSC Course

11C
WORKED

Example

Distribution of scores

1 The temperature on a January day in a city is normally distributed with a mean of 26


and a standard deviation of 3. What percentage of January days lie between:
a 23 and 29?
b 20 and 32?
c 17 and 35?
2 The marks of students sitting for a major exam are normally distributed with x = 57
and sn = 13. What percentage of marks on the exam were between:
a 44 and 70?
b 31 and 83?
c 18 and 96?
3 The mean thickness of bolts produced by a machine is 2.3 mm, with a standard
deviation of 0.04 mm. What percentage of bolts will have a thickness between
2.22 mm and 2.38 mm?

WORKED

Example

4 Experience has shown that the scores obtained on a commonly used IQ test can be
assumed to be normally distributed with a mean = 100 and a standard deviation
s = 15. What percentage of scores lie above 115?
5 The heights of young women are normally distributed with a mean x = 160 cm and a
standard deviation sn = 8 cm. What percentage of the women would you expect to
have heights:
a between 152 and 168 cm?
b greater than 168 cm?
c less than 136 cm?
6 The age at which women give birth to their first child is normally distributed with
x = 27.5 years and sn = 3.2 years. From these data we can conclude that about 95%
of women have their first child between what ages?
7 Fill in the blanks in the following statements. For any normal distribution:
a 68% of the values have a z-score between ___ and ___
b ___% of the values have a z-score between 2 and 2
c ___% of the values have a z-score between ___ and ___.
8 multiple choice
Medical tests indicate that the amount of an antibiotic needed to destroy a bacterial
infection in a patient is normally distributed with x = 120 mg and sn = 15 mg. The
percentage of patients who would require more than 150 mg to clear the infection is:
A 0.15%
B 2.5%
C 5%
D 95%
9 multiple choice
The mean mark on a test is 55, with a standard deviation of 10. The percentage of
students who achieved a mark between 65 and 75 is:
A 13.5%
B 22.5%
C 34%
D 95%
10 In a factory, soft drink is poured into cans such that the mean amount of soft drink is
500 mL with a standard deviation of 2 mL. Cans with less than 494 mL of soft drink
are rejected and not sold to the public. What percentage of cans are rejected?

Chapter 11 The normal distribution

343

11 The distribution of IQ scores for the inmates of a certain prison is approximately


normal with a mean of 85 and a standard deviation of 15.
a What percentage of this prison population have an IQ of 100 or higher?
b If someone with an IQ of 70 or less can be classified as mentally disabled, what
percentage of the prison population could be classified as mentally disabled?
12 The distribution of blood pressures (systolic) among women of similar ages is normal
with a mean of 120 (mm of mercury) and a standard deviation of 10 (mm of mercury).
Determine the percentage of women with a systolic blood pressure:
a between 100 and 140
b greater than 130
c between 120 and 130
d between 90 and 110
e between 110 and 150.
13 The mass of packets of chips is normally distributed with x = 100 g and n = 2.5 g. If
I purchase a packet of these chips, between what limits will the mass of the packet:
8
a very probably lie?
b almost certainly lie?

WORKED

Example

14 The heights of army recruits are normally distributed about a mean of 172 cm and a
standard deviation of 4.5 cm. A volunteer is chosen from the recruits. The height of
the volunteer will very probably lie between what limits?

16 The average mass of babies is normally distributed with a mean of 3.8 kg and a standard
deviation of 0.4 kg. A newborn baby will almost certainly have a mass between what
limits?

Examining a normal distribution


Complete a sample of the
heights or masses of 50 people.
1 Calculate the mean and the
standard deviation of your
sample.
2 Calculate the percentage of
people whose height or mass
has a standardised score of
between 1 and 1.
3 Calculate the percentage of
people whose height or mass
has a standardised score of
between 2 and 2.
4 Calculate the percentage of
people whose height or mass
has a standardised score of
between 3 and 3.

Work

15 A machine is set to deposit a mean of 500 g of washing powder into boxes with a
standard deviation of 10 g. When a box is checked, it is found to have a mass of 550 g.
What conclusion can be drawn from this?

T
SHEE

11.2

344

Maths Quest General Mathematics HSC Course

summary
z-scores
A data set is normally distributed if it is
symmetrical about the mean.
A z-score measures the position of a score
relative to the mean and standard deviation.
A z-score is found using the formula
x
xx
z = ----------s
where x is the score, x is the mean, and s is the standard deviation.

Comparison of scores
Standardising both scores best compares scores from different data sets.
When comparing exam marks, the highest z-score is the best result.

Distribution of scores
A data set that is normally distributed will be symmetrical about the mean.
68% of scores will have a z-score of between 1 and 1.
95% of scores will have a z-score between 2 and 2. A score chosen from this data
set will very probably lie in this range.
99.7% of scores will have a z-score of between 3 and 3. A score chosen from the
data set will almost certainly lie within this range.

Chapter 11 The normal distribution

345

CHAPTER
review
1 Measurements of the amount of acid in a certain chemical are made. The results are
normally distributed such that the mean is 6.25% and the standard deviation is 0.25%.
Harlan gets a reading of 5.75%. What is Harlans reading as a z-score?

11A

2 A set of scores is normally distributed such that x = 15.3 and n = 5.2. Convert each of the
following members of the distribution to z-scores.
a 15.3
b 20.5
c 4.9
d 30.9
e 10.1

11A

3 On an exam the results are normally distributed with a mean of 58 and a standard deviation
of 7.5. Jennifer scored a mark of 72 on the exam. Convert Jennifers mark to a z-score,
giving your answer correct to 2 decimal places.

11A

4 A set of scores is normally distributed with a mean of 2.8 and a standard deviation of 0.6.
Convert each of the following members of the data set to z-scores, correct to 2 decimal
places.
a 2.9
b 3.9
c 1
d 1.75
e 1.6

11A

5 Anji conducts a survey on the water temperature at her local beach each day for a month.
The results (in C) are shown below.
20 21 19 22 21 18 17 23 17 16 22 20 20 20 21
20 21 18 22 17 16 20 20 22 19 21 22 23 24 20
a Find the mean and standard deviation of the scores.
b Find the highest and lowest temperatures in the data set and express each as a z-score.

11A

6 The table below shows the length of time for which a sample of 100 light bulbs will burn.

11A

Length of time (hours)

Class centre

Frequency

0500

5001000

28

10001500

59

15002000

10

a Find the mean and standard deviation for the data set.
b A further sample of five light bulbs are chosen. The length of time for which each light
bulb burned is given below. Convert each of the following to a standardised score.
i 1000 hours
ii 1814 hours
iii 256 hours
iv 751 hours
v 2156 hours
7 Betty sat exams in both Physics and Chemistry. In Physics the exam results showed a mean
of 48 and a standard deviation of 12, while in Chemistry the mean was 62 with a standard
deviation of 9.
a Betty scored 66 in Physics. Convert this result to a z-score.
b Betty scored 71 in Chemistry. Convert this result to a z-score.
c In which subject did Betty achieve the better result? Explain your answer.

11B

346

Maths Quest General Mathematics HSC Course

11B

8 In Geography Carlos scored a mark of 56, while in Business studies he scored 58. In
Geography x = 64 and n = 10. For Business studies x = 66 and n = 15.
a Convert each mark to a standardised score.
b In which subject did Carlos achieve the better result?

11B

9 A psychologist records the number of errors made on a series of tests. On a literacy test the
mean number of errors is 15.2 and the standard deviation is 4.3. On the numeracy test the
mean number of errors is 11.7 with a standard deviation of 3.1. Barry does both tests and
makes 11 errors on the literacy test and 8 errors on the numeracy test. In which test did
Barry do better? Explain your answer.

11C

10 A data set is normally distributed with a mean of 40 and a standard deviation of 8. What
percentage of scores will lie in the range:
a 32 to 48?
b 24 to 56?
c 16 to 64?

11C

11 The value of sales made on weekdays at a store appears to be normally distributed with a
mean of $1560 and a standard deviation of $115. On what percentage of days will the days
sales lie between:
a $1445 and $1675?
b $1330 and $1790?
c $1215 and $1905?

11C

12 A data set is normally distributed with a mean of 56 and a standard deviation of 8. What
percentage of scores will:
a lie between 56 and 64?
b lie between 40 and 56?
c be less than 40?
d be greater than 80?
e lie between 40 and 80?

11C

13 A machine is set to produce bolts with a mean diameter of 5 mm with a standard deviation
of 0.1 mm. A bolt is chosen and it is found to have a diameter of 4.5 mm. What conclusion
can be drawn about the settings of the machine?

Practice examination questions


1 multiple choice
The mean time for 12-year-old boys to
swim 50 m is 50.5 s with a standard
deviation of 4.2 s. Kyle swims 50 m in
44.2 s. Kyles time as a standardised score
is:
A 6.3
B 1.5
C 1.5
D 6.3
2 multiple choice
A teacher converts the marks on every test that she gives her class to a standardised score. On
a test the mean mark was 50 and the standard deviation was 10. Adams standardised score on
the test was 0.6. Adams mark on the test was:
A 40
B 44
C 56
D 60

Chapter 11 The normal distribution

347

3 multiple choice
The details of Andreas half-yearly exams are shown in the table below.
Subject

Andreas mark

Mean

Standard deviation

English

65

50

12

Maths

62

52

History

75

58

15

Geography

50

44

In which subject did Andrea achieve her best result?


A English
B Maths
C History

D Geography

4 multiple choice
The details of Bretts half-yearly exams are shown in the table below.
Subject

Bretts mark

Mean

Standard deviation

English

40

50

12

Maths

48

52

History

49

58

15

Geography

42

44

In which subject did Brett achieve his best result?


A English
B Maths
C History

D Geography

5 multiple choice
A data set is normally distributed with x = 25 and
lie in the range 25 to 30 is:
A 34%
B 47.5%
C 68%

= 2.5. The percentage of scores that will


D 95%

6 multiple choice
A fishing boat catches a load of fish and finds the mass of each fish. The masses of the fish are
normally distributed with a mean of 800 g and a standard deviation of 75 g. If a fish is chosen
from the catch, its mass will almost certainly lie between:
A 725 g and 875 g
B 650 g and 950 g
C 575 g and 1025 g
D 800 g and 1025 g

348

Maths Quest General Mathematics HSC Course

7 Theresa attempts to review her exam results in Physics and Chemistry. Theresa samples 10 of
her friends and finds the following results.
Physics:
65 64 67 69 72 50 66 66 63 69
Chemistry: 72 50 69 55 62 68 51 75 78 44
a Find the mean and standard deviation in each subject.
b Theresas marks were 65 in Physics and 67 in Chemistry. Convert each to a standardised
score.
c In which subject did Theresa score her best result? Explain your answer.
d A student is chosen at random from the Physics class. Between what two marks will this
persons result very probably lie?
e If the marks within the class follow a normal distribution, within what two marks will
approximately 99.7% of all Chemistry scores lie?

CHAPTER

test
yourself

11

8 A machine is set to cut lengths of metal such that the mean length of metal cut is 12.5 cm
with a standard deviation of 0.05 cm.
a A piece of metal is measured to have a length of 12.4 cm. Express this as a standardised
score.
b A second piece of metal is measured and found to have a length of 13 cm. What conclusion
can be drawn from this measurement?

Correlation

12
syllabus reference
Data analysis 7
Correlation

In this chapter
12A Scatterplots
12B Fitting a straight line by
eye
12C Fitting a straight line
the 3-median method
12D Correlation

areyou

READY?

Are you ready?

Try the questions below. If you have difficulty with any of them, extra help can be
obtained by completing the matching SkillSHEET. Either click on the SkillSHEET icon
next to the question on the Maths Quest HSC Course CD-ROM or ask your teacher for
a copy.

12.1

12.2

Finding the median

1 Find the median of:


a 3, 5, 6, 3, 4, 2, 5, 2, 7

Using the regression equation to make predictions

2 For the equation y = 5x 2 find:


a y if x = 40

12.3

b 12, 15, 10, 11, 15, 15, 16, 11, 19, 16.

b x if y = 258.

Finding the gradient I

3 Find the gradient of the line joining the points:


a (1, 3) and (4, 12)
b (2, 4) and (6, 2).

12.4

Finding the gradient II

4 Calculate the gradient of the following lines, and state whether the gradient is positive or
negative.
a Vertical rise = 12, horizontal run = 2
b Vertical rise = 6, horizontal run = 4

Chapter 12 Correlation

351

Scatterplots
The manager of a small ski resort has a problem. He wants to be able to predict the
number of skiers using his resort each weekend in advance, so that he can organise
additional resort staffing and catering if needed. He knows that good deep snow will
attract skiers in big numbers but scant covering is unlikely to attract a crowd. To
investigate the situation further, he collects the following data over twelve consecutive
weekends at his resort.
Depth of snow (m)

Number of skiers

0.5

120

0.8

250

2.1

500

3.6

780

1.4

300

1.5

280

1.8

410

2.7

320

3.2

640

2.4

540

2.6

530

1.7

200

Number of skiers

As there are two types of data in this example, they are called bivariate data. For
each item (weekend), two variables are considered (depth of snow and number of
skiers). When analysing bivariate data, we are interested in examining the relationship
between the two variables. In the case of the ski resort data we might be interested in
answering the following questions.
Are visitor numbers related to depth of snow?
800
If there is a relationship between visitor
numbers and depth of snow, is it always true? or
600
is it just a guide? In other words, how strong is
400
the relationship?
200
How much confidence could be placed in the
0
prediction?
0 1 2 3 4
To help answer these questions, the data can be
Depth of snow (m)
arranged on a scatterplot.
Each of the data points is represented by a single visible point on the graph.
When drawing a scatterplot, it is important to choose the correct variable to assign to
each of the axes. The convention is to place the independent variable on the x-axis and
the dependent variable on the y-axis. The independent variable in an experiment or
investigation is the variable that is deliberately controlled or adjusted by the
investigator. The dependent variable is the variable that responds to changes in the
independent variable.

352

Maths Quest General Mathematics HSC Course

Neither of the variables involved in the ski resort data was controlled directly by the
investigator, but Number of skiers would be considered the dependent variable
because it is likely to change depending on depth of snow. (The snow depth does not
depend on numbers of skiers). As Number of skiers is the dependent variable, we
graph it on the y-axis and the Depth of snow on the x-axis.
Notice how the scatterplot for the ski resort data shows a general upward trend. It is
not a perfectly straight line, but it is still clear that a general trend or relationship has
formed: as the depth of snow increases, so too does the number of skiers.

WORKED Example 1

The table below shows the height and mass of ten Year 12 students.
Height (cm)
Mass (kg)

120

124

130

135

142

148

160

164

170

175

45

50

54

59

60

65

70

78

75

80

Display the data on a scatterplot.

1
2

WRITE

Show the height on the horizontal axis


and the mass on the vertical axis.
Plot the point given by each pair.

Mass (kg)

THINK

80
70
60
50
40
30
0

100 110 120 130 140 150 160 170 180


Height (cm)

Graphics Calculator tip! Drawing a scatterplot


Your graphics calculator can draw a scatterplot by storing the two sets of data in
separate lists. Consider worked example 1.
1. From the MENU select STAT.

2. Delete any existing data, and store the data for height
in List 1 and mass in List 2.

Chapter 12 Correlation

353

3. Press F1 (GRPH) (you may have to press F6 for


more options first); then press F6 (SET). Set the
graph type to Scatter by arrowing down to graph type
and pressing F1 (Scat) (again you may have to
press F6 for more options first). Ensure that XList
is List 1, YList is List 2 and Frequency is 1 as shown
at right.
4. Press EXIT to return to the previous screen, and
then press F1 (GPH1). The scatterplot will then be
drawn.

Note that the graphics calculator sets the values on the x- and y-axes automatically.
You can press SHIFT F3 (V-Window) to set the scale as you see fit.

Once the scatterplot has been drawn, we can determine if any pattern is evident.
Worked example 1 shows how, as a general rule, as height increases so does mass.
We can also look to see if the pattern is linear. In worked example 1, although the
points are not in a perfect straight line, they approximate a straight line. The figures
below show examples of linear and non-linear relationships.

Linear relationships
y

Non-linear relationships
y

354

Maths Quest General Mathematics HSC Course

In other cases it may be that there is no relationship


at all between the two variables. Such a scatterplot
would look like the one shown on the right.

WORKED Example 2

The table below shows the length and mass of a dozen eggs.
Length (cm)

6.2

3.9

4.5

5.8

7.2

7.6

6.1

6.7

7.3

5.1

6.0

7.3

Mass (g)

60

15

25

50

95

110

55

75

95

35

54

96

a Display this information in a scatterplot.


b Determine if there is any relationship between the length and mass of the eggs and state
if the relationship is linear.
THINK

WRITE

120
100
Mass (kg)

Display length on the x-axis and mass


on the y-axis.
Plot the point given by each pair.

80
60
40
20
0
0

1
2

Study the scatterplot to see if mass


increases as length increases.
Study the scatterplot to see if the points
seem to approximate a straight line.

4 5 6
Length (cm)

b As length increases, so does the mass of


the egg.
The points do not approximate a straight
line, and so the relationship is not linear.

remember
1. A scatterplot is a graph that is used to compare two variables.
2. One variable (the independent variable) is on the horizontal axis, and the other
variable (the dependent variable) is on the vertical axis.
3. Points are plotted by the pair formed by each variable.
4. A relationship between the variables exists if one increases as the other
increases or if one decreases as the other increases.
5. If the points on the scatterplot seem to approximate a straight line, the
relationship can be said to be linear.

Chapter 12 Correlation

12A

355

Scatterplots
E

65

82

72

58

39

58

74

82

66

Geography

45

78

66

72

50

51

61

70

60

88

L Spre
XCE ad

36

Two
variable
2 The table below shows the maximum temperature each day, together with the number statistics

of people who attend the cinema that day. Display the information on a scatterplot.
Temperature (C)
No. at cinema

25

33

30

22

15

18

27

22

28

20

256

184

190

312

458

401

200

357

312

423

3 The table below shows the wages, W, of 20 people and the amount of money they spend
each week on entertainment, E. Display this information in a scatterplot.
Wages ($)

370

380

500

510

395

430

535

490

495

550

55

85

150

75

145

100

130

115

70

150

Wages ($)

810

460

475

520

530

475

610

780

350

460

Amount spent on
entertainment ($)

220

50

100

150

140

160

90

130

40

50

Amount spent on
entertainment ($)

WORKED

Example

4 The table below shows the marks obtained by nine students in English and History.
English

55

20

27

33

73

18

37

51

79

History

72

37

53

74

73

44

59

55

84

a Display the information on a scatterplot.


b Is there any relationship between the mark obtained in English and in History? If
there does appear to be a relationship, is the relationship linear?
5 The table below shows the daily temperature and the number of hot pies sold at the
school canteen.
Temperature (C)

24

32

28

23

16

14

26

20

29

21

No. of pies sold

56

20

24

60

84

120

70

95

36

63

a Display the information on a scatterplot.


b Determine if there appears to be any relationship between the two variables and if
the relationship appears to be linear.

sheet

History

sheet

L Spre
XCE ad

1 The table below shows the marks obtained by a group of ten students in History and
Example
Scatterplot
Geography. Display this information on a scatterplot.
1
WORKED

356

Maths Quest General Mathematics HSC Course

6 Container ships arriving on a wharf are unloaded by work


teams. The table below shows the number of people in the
work team and the time taken to unload the container ship.
No. in work
team

15 18 12 19 22 21 17 16 18 20

Hours taken

20 16 25 15 14 13 18 20 17 14

a Display the information on a scatterplot.


b Determine if there appears to be a relationship between
the number of people in the work team and the time taken
to unload the container ship. If there is a relationship,
does the relationship appear to be linear?
7 multiple choice
Which of the following scatterplots does not display a linear relationship?
A y

B y

x
x

C y

D y

8 multiple choice
In which of the following is no relationship evident between the variables?
A y

B y

x
x

Cy

Dy

Chapter 12 Correlation

357

9 Give an example of a situation where the scatterplot may look like the ones below.
a y
b y

Collecting bivariate data


1 Choose one of the following and collect data from within your class.
a Each persons hand span and height.
b Each persons resting heart rate and the time it takes for them to run 400 m.
c Each persons mark in Mathematics and in Science.
2 Display the results on a scatterplot.
3 Discuss any relationship that may be evident between the two variables.

Regression lines
The process of fitting straight lines to bivariate data enables us to analyse relationships between the data and possibly make predictions based on the given data set.

Fitting a straight line by eye


Consider the set of bivariate data points shown at right. In this case
the x-values could be heights of married women, while y-values
could be the heights of their husbands. We wish to determine a
linear relationship between these two random variables.

358

Maths Quest General Mathematics HSC Course

Of course, there is no single straight line that would go through all the points, so we
can only estimate such a line.
Furthermore, the more closely the points appear to be on or near a straight line, the
more confident we are that such a linear relationship may exist and the more accurate
our fitted line should be.
Consider the estimate, drawn by eye in the figure below. It is clear that most of the
points are on or very close to this straight line. This line was easily drawn since the
points are very much part of an apparent linear relationship.
However, note that some points are below the line and some are above it. Furthermore, if x is the height of wives and y is the height of husbands, it seems that husbands
are generally taller than their wives.
y
Regression analysis is concerned with finding
these straight lines using various methods so that
the number of points above and below the lines
are balanced.

Method of fitting lines by eye

There should be an equal number of points above and below the line. For example, if
there are 12 points in the data set, 6 should be above the line and 6 below it. This may
appear logical or even obvious, but fitting by eye involves a considerable margin of error.

WORKED Example 3
Fit a straight line to the data in the figure using
the equal-number-of-points method.

THINK
1
2

Note that the number of points (n) is 8.


Fit a line where 4 points are above the line. Using a
clear plastic ruler, try to fit the best line.

DRAW
y

x
3

The first attempt has only 3 points below the line


where there should be 4. Make refinements.

The second attempt is an improvement, but the line is


too close to the points above it. Improve the position
of the line until a better balance between upper and
lower points is achieved.

x
y

Chapter 12 Correlation

359

remember
To fit a straight line by eye, when using bivariate data, make sure there are an
equal number of points above and below the fitted line.

12B

Fitting a straight line by eye

The questions below represent data collected by groups of students conducting different
environmental projects. The students have to fit a straight line to their data sets.
Note: For many of these questions your answers may differ somewhat from those in the
back of the book. The answers are provided as a guide but there are likely to be individual differences when fitting straight lines by eye.
Example

1 Fit a straight line to the data in the scatterplots using the equal-number-of-points method.
a y

b y

d y

c y

e y

f y

x
x

g y

h y

i y

Work

WORKED

Fitting a straight line the 3-median method


Fitting lines by eye is useful, but it is not the most accurate of methods. Greater accuracy
is achieved through closer analysis of the data. Upon closer analysis it is possible to find
the equation of a line of best fit of the form y = mx + c, where m is the gradient and c
is the y-intercept. Several mathematical methods provide a line with a more accurate fit.

T
SHEE

12.1

360

Maths Quest General Mathematics HSC Course

One of these methods is called the 3-median method and


involves the division of the data set into 3 groups, and the
use of the 3 medians in these groups to determine a line of
best fit. It is used when data show a linear relationship. It
can even be used when the data contain outliers. The
3-median method is best described as a step-by-step method.
Step 1. Plot the points on a scatter diagram. This is shown in
figure 1.
Step 2. Divide the points into 3 groups using vertical
divisions (see figure 2). The number of points in a
data set will not always be exactly divisible by 3.
Thus, there will be three alternatives, as follows.
(a) If the number of points is divisible by 3, divide
them into 3 equal groups, for example, 3, 3, 3 or
7, 7, 7.
(b) If there is 1 extra point, put the extra point in the
middle group, for example, 3, 4, 3 or 7, 8, 7.
(c) If there are 2 extra points, put 1 extra point in
each of the outer groups, for example, 4, 3, 4 or
8, 7, 8.
Step 3. Find the median point of each of the 3 groups and
mark each median on the scatterplot (see figure 3).
Recall that the median is the middle value. So the
median point of each group has an x-coordinate that
is the median of the x-values in the group and a
y-coordinate that is the median of the y-values in the
group.
(a) The left group is the lower group and its median
is denoted by (xL, yL).
(b) The median of the middle group is denoted by
(xM, yM).
(c) The right group is the upper group and its median
is denoted by (xU, yU).
Note: Although the x-values are already in ascending order on
the scatterplot, the y-values within each group may need reordering before you can find the median.

y
7
6
5
4
3
2
1
0
0 1 2 3 4 5 6 7 8 x
Figure 1
y
7
6
5
4
3
2
1
0
0 1 2 3 4 5 6 7 8 x
Figure 2
FM Fig 03.13
y
7
6
5
4
3
2
1
0

(xU, yU)
(xM, yM)
(xL, yL)

0 1 2 3 4 5 6 7 8 x
Figure 3
y
7
6
5
4
3
2
1
0

(xM, yM)

(xU, yU)

(xL, yL)
Step 4. Draw in the line of best fit. Place your ruler so that it
passes through the lower and upper medians. Move the
ruler a third of the way toward the middle group
0 1 2 3 4 5 6 7 8 x
median while maintaining the slope. Hold the ruler
Figure 4
there and draw the line.
Step 5. Find the equation of the 3-median regression line (general form y = mx + c).
Draw on your knowledge of finding equations of lines to find the equation of
the line drawn on the scatterplot. If the scale on the axes begins at zero, you
can read off the y-intercept of the line and calculate the gradient of the line.
This will enable you to find the equation of the line.

361

Chapter 12 Correlation

The equation of a straight line can be found using y = mx + b , where m is the


gradient and b is the y-intercept. The gradient of the regression line is best found with
a ruler and using the formula:
vertical change in position
m = --------------------------------------------------------------------horizontal change in position

WORKED Example 4

Find the equation of the regression line for the


data in the table at right using the 3-median
method.
THINK
1

WRITE

Plot the points on a scatterplot, and


divide the data into 3 groups. Note
there are 6 points, so the division will
be 2, 2, 2.

y
7
6
5
4
3
2
1
0
0 1 2 3 4 5 6 7 8 x

Find the median point of each group.


Since each group has only 2 points,
medians are found by averaging them.
Mark in the medians, and place a ruler
on the outer 2 medians. Maintaining the
same slope on the ruler, move it onethird of the way towards the middle
median. Draw the line.

(xL, yL) = (1.5, 2)


(xM, yM) = (3.5, 4)
(xU, yU) = (6, 5.5)
y
7
6
5
4
3
2
1
0
0 1 2 3 4 5 6 7 8 x

Read off the y-intercept from the graph.

Use (xL, yL) and (xU, yU) to calculate the


gradient.

Write the equation of the 3-median


regression line.

y-intercept = 1
5.5 2
Gradient (m) = ---------------6 1.5
3.5
= ------4.5
7
= --9
7
y = --- x + 9 or
9
9y = 7x + 81

362

Maths Quest General Mathematics HSC Course

WORKED Example 5

Weight (kg)

The scatterplot below shows a comparison between the heights and weights of 12 boys.
The median points A and B in the first and last sections have been found for you.

100
90
80
70
60
50
40
30
20
10

10 20 30 40 50 60 70 80 90 100 110 120 130 140 150 160 170 180 190
Height (cm)

a Find the coordinates of median point C, and hence find the median regression line.
b Find the gradient and y-intercept of the regression line, and hence find the equation of
the regression line.
THINK

WRITE

a x-values are: 165, 170, 170 and 175


170 + 170
Median x-value = ------------------------ = 170
2
y-values are: 65, 70, 70 and 80
70 + 70
Median y-value = ------------------ = 70
2
The coordinates of C are (170, 70).

Find the coordinates of point C by


finding the median of the x-values
and finding the median of the yvalues.

Mark point C on the diagram.

Rule a line through points A and B.

Move the line AB one-third of the way


towards C, keeping the new line parallel
to AB.

Weight (kg)

100
90
80
70
60
50
40
30
20
10
0

A
C

10 20 30 40 50 60 70 80 90 100 110 120 130 140 150 160 170 180 190
Height (cm)

Chapter 12 Correlation

THINK

WRITE

rise
b m = -------run
78 70
m = -----------------------190 140
m = 0.16
b = 49

Calculate the gradient, m, by finding


the rise and run between two points on
the line.

Read the value from the graph to state


the y-intercept, b.
Substitute m and b into the formula
y = mx + b to find the equation of the
regression line.

363

The equation is of the form y = mx + b,


where x represents height in cm and
y represents weight in kg.
y = 0.16x + 49

the equation of
Graphics Calculator tip! Finding
a regression line
The Casio graphics calculator can be used to find the equation of a median regression
line. Consider worked example 5.
1. From the MENU select STAT.

2. Enter the data into List 1 and List 2 and draw the scatterplot as shown in the previous section. Since we are
using the calculator it is not necessary to draw the
scatterplot from 0 on the axes.
3. Press F2 (Med) to find the equation of the median
regression line. The value of a is the gradient of the
line and the value of b is the y-intercept.

4. If you want to see the regression line drawn on the


scatterplot, press F6 (DRAW).

In the above example we would give the equation y = 0.15x + 49, which is slightly different from the example done on paper. Because the method relies on the eye to find
two points on the regression line to find the gradient and y-intercept, minor differences
are insignificant and quite acceptable.
Once the regression line has been found, we are able to use the equation to make
predictions about other pieces of data.

364

Maths Quest General Mathematics HSC Course

WORKED Example 6
A casino records the number of people, N, playing a jackpot game and the prize money, p,
for that game and plots the results on a scatterplot. The regression line is found to have the
equation N = 0.07p + 220.
a Find the number of people playing when the prize money is $2500.
b Find the likely prize on offer when there are 500 people playing.
THINK

WRITE

a N = 0.07p + 220

Write the equation of the regression line.

Substitute 2500 for p.

Calculate N.

Give a written answer.

Write the equation of the regression line.

Substitute 500 for N.

500 = 0.07p + 220

Solve the equation.

280 = 0.07p
p = 4000

Give a written answer.

The prize would be approximately $4000.

N = 0.07

2500 + 220

= 395
There would be approximately
395 people playing.
b

N = 0.07p + 220

remember
1. The median regression line is the line of best fit that is drawn on a scatterplot.
2. The median regression line can be drawn using the method of three medians.
3. To find the median regression line:
(a) divide the points into three approximately equal sections. If the number of
points is not divisible by three, make sure there is the same number of
points in the first and last sections.
(b) mark median points in the first and last sections by finding the median of
the x-values and finding the median of the y-values for each section. Label
these points A and B.
(c) find the median point in the middle section and label this point C.
(d) draw the line AB and then move the line one-third of the way towards C,
keeping the line parallel to AB.
4. The equation of the regression line can be found by measuring the gradient and
the y-intercept of the regression line and using the formula y = mx + b .
Sometimes the gradient of the median regression line will be negative.
5. Once the equation of the regression line has been found, it can then be used to
make predictions about the variables.

365

Chapter 12 Correlation

12C
WORKED

Example

Fitting a straight line the


3-median method

1 The table below shows the marks achieved by a class of students in English and Maths.
English

64

75

81

63

32

56

47

59

73

64

Maths

76

62

89

56

49

57

53

72

80

50

12.1 SkillS
HEET

Show these data on a scatterplot, and on the graph show the regression line using the
3-median method.

Finding
the
median

12.2 SkillS

60
50
40
30
20
10
0
0

10

to make
predictions

12.3 SkillS
HEET

60
50
40
30
20
10
0

HEET

2 Position the median regression line, using the 3-median method, through each of the
Using
following graphs, and find the equation of each.
the regression
a 70
b 70
equation

20 40 60 80 100 120

Finding the
gradient I

3000

12.4 SkillS

2500

HEET

2000

Finding the
gradient II

1500
1000

0
0
Example

3-median
regression

10 15 20 25

3 In an experiment, a student measures the length of a spring when different masses


are attached to it. Her results are shown below.
Mass (g)
0
100
200
300
400
500
600
700
800
900

Length of spring (mm)


220
225
231
235
242
246
250
254
259
264

Making
predictions

a Draw a scatterplot of the data, and on it draw the median line of regression, using
the 3-median method.
b Find the gradient and y-intercept of the regression line, and hence find the equation
of the regression line.

sheet

L Spre
XCE ad

WORKED

sheet

L Spre
XCE ad

500

366

Maths Quest General Mathematics HSC Course

4 A scientist who measures the volume of a gas


at different temperatures provides the table of
values at right.
a Draw a scatterplot of the data and on it
draw the line of regression using the
3-median method.
b Give the equation of the line of best fit. Write
your equation in terms of the variables:
volume of gas, V, and its temperature, T.
5 A sports scientist is interested in the
importance of muscle bulk to strength. He
measures the biceps circumference of ten
people and tests their strength by asking them
to complete a lift test. His results are given in
the following table.
Circumference of biceps (cm)
25
25
27
28
30
30
31
33
34
36

Temperature
(C)
40
30
20
0
10
20
30
40
50
60

Volume
(L)
1.2
1.9
2.4
3.1
3.6
4.1
4.8
5.3
6.1
6.7

Lift test (kg)


50
52
58
51
60
62
53
62
61
66

a Draw a scatterplot of the data and draw the median line of regression using the
3-median method.
b Find a rule for determining the ability of a person to complete a lift test, S, from
the circumference of their biceps, B.
WORKED

Example

6 A taxi company adjusts its meters so that the fare is charged according to the
following equation: F = 1.2d + 3, where F is the fare, in dollars, and d is the distance
travelled, in km.
a Find the fare charged for a distance of 12 km.
b Find the fare charged for a distance of 4.5 km.
c Find the distance that could be covered on a fare of $27.
d Find the distance that could be covered on a fare of $13.20.
7 Detectives can use the equation H = 6.1f 5 to estimate the height of a burglar who
leaves footprints behind. (H is the height of the burglar, in cm, and f is the length of
the footprint.)
a Find the height of a burglar whose footprint is 27 cm in length.
b Find the height of a burglar whose footprint is 30 cm in length.
c Find the footprint length of a burglar of height 185 cm. (Give your answer correct
to 2 decimal places.)
d Find the footprint length of a burglar of height 152 cm. (Give your answer correct
to 2 decimal places.)

Chapter 12 Correlation

367

8 A pie seller at a football match finds that the number of pies sold is related to the
temperature of the day. The situation could be modelled by the equation
N = 870 23t, where N is the number of pies sold and t is the temperature of the day.
a Find the number of pies sold if the temperature was 5 degrees.
b Find the number of pies sold if the temperature was 25 degrees.
c Find the likely temperature if 400 pies were sold.
d How hot would the day have to be before the pie seller sold no pies at all?
9 The following table shows the average annual costs of running a car. It includes all
fixed costs (registration, insurance etc.) as well as running costs (petrol, repairs etc.).
Distance (km)

Annual cost ($)

5 000

4 000

10 000

6 400

15 000

8 400

20 000

10 400

25 000

12 400

30 000

14 400

a Draw a scatterplot of the data.


b Using the 3-median method, draw in the line of best fit.
c Find an equation which represents the relationship between the cost of running a
vehicle, C, and the distance travelled, d.
d Use your graph and its equation to find:
i the annual cost of running a car if it is driven 15 000 km
ii the annual cost of running a car if it is driven 1000 km
iii the likely number of kilometres driven if the annual costs were $8000
iv the likely number of kilometres driven if the annual costs were $16 000.
10 A market researcher finds that the number of people
who would purchase Wise-up (the thinking mans
deodorant) is related to its price. He provides the
table of values at right.
a Draw a scatterplot of the data.
b Draw in the line of best fit.
c Find an equation that represents the relationship
between the number of cans of Wise-up sold, N
(in thousands), and its price, p.
d Use the equation to predict the number of cans
sold each week if:
ii the price was $3.10
ii the price was $4.60.
e At what price should Wise-up be sold if the
manufacturers wished to sell 80 000 cans?
f Given that the manufacturers of Wise-up can
produce only 100 000 cans each week, at what
price should it be sold to maximise production?

Price
($)

Weekly sales
( 1000)

1.40

105

1.60

101

1.80

97

2.00

93

2.20

89

2.40

85

2.60

81

2.80

77

3.00

73

3.20

69

3.40

65

368

Maths Quest General Mathematics HSC Course

11 The following table gives the adult return air fares between some Australian cities.
City
MelbourneSydney
PerthMelbourne
AdelaideSydney
BrisbaneMelbourne
HobartMelbourne
HobartAdelaide
AdelaideMelbourne

Distance (km)
713
2728
1172
1370
559
1144
669

Price ($)
580
1490
790
890
520
820
570

a Draw a scatterplot of the data and on it draw the median regression line using the
line of best fit.
b Find an equation that represents the relationship between the air fare, A, and the
distance travelled, d.
c Use the equation to predict the likely air fare (to the nearest dollar) from:
i Sydney to the Gold Coast (671 km)
ii Perth to Adelaide (2125 km)
iii Hobart to Sydney (1024 km)
iv Perth to Sydney (3295 km).
12 Rock lobsters (crayfish) are sized according to the length of their carapace (main body
shell). The table below gives the age and carapace length of 16 male rock lobsters.
Age (years)
3
2.5
4.5
4.5
3.25
7.75
8
6.5
12
14
4.5
3.5
2.25
1.76
10
9.5

Length of
carapace (mm)
65
59
80
80
68
130
150
112
200
210
82
74
51
48
171
160

a Display this information on a scatterplot, and on your scatterplot draw the median
line of regression using the line of best fit.
b Find the equation of the median regression line.

Chapter 12 Correlation

369

c Use the equation to find the likely size of a 5-year-old male rock lobster.
d Use the equation to find the likely size of a 16-year-old male rock lobster.
e Rock lobsters reach sexual maturity when their carapace length is approximately
65 mm. Use the equation to find the age of the rock lobster at this stage.
f The fisheries department wants to set minimum size restrictions so that the rock
lobsters have three full years from the time of sexual maturity in which to breed
before they can be legally caught. What size should govern the taking of a male
rock lobster?
Note: Answers for this exercise are approximate and may vary due to the precise
location of the line of best fit.

Relationship between variables


Earlier in the chapter you would have completed an investigation of bivariate data.
You should have displayed the information on a scatterplot.
1 On your scatterplot draw the median regression line.
2 Find the equation of the median regression line.
3 Find a few more people to test your data. See how accurately your equation
predicts the results. (For example measure a persons hand span and use your
equation to predict their height.)

1
An electrical repair business charges its customers using the formula C = 40h + 35,
where C is the cost of the repairs and h is the time taken for the repairs, in hours. Find
the cost of a repair job that took:
1 2 hours

2 5 hours

3 1 hour and 15 minutes.

Estimate the time taken for repairs if the cost of the repairs were:
4 $175

5 $275

6 $145.

The information below is to be used for questions 7 to 10.


A survey relating exam marks to the amount of television watched finds that the median
regression line has the equation M = 95 15t, where M is the mark obtained and t is the
average number of hours of television watched each night by the students.
7 Estimate the mark of a person who averages one hour of television per night.
8 Estimate the mark of a person who watches an average 4 hours of television per night.
9 Estimate the amount of television watched per night by a person who scores a mark of
65.
10 Jodie scored 27.5 on the exam. Estimate the average amount of television that Jodie
watches each night.

370

Maths Quest General Mathematics HSC Course

Correlation
Correlation is a description of the relationship that exists between two variables. When
one variable increases with another, it is said that there is a positive correlation between
the variables. In such a case, the median regression line will have a positive gradient.
Similarly, if one variable decreases while the other increases, the median regression
line will have a negative gradient and the correlation is negative.
Consider the following example in which ten Year 11 students were surveyed to find
the amount of time that they spend doing exercise each week. This was compared with
their blood cholesterol level.
6

12

16

12

Blood cholesterol level

In this example there seems to be a


general downward trend, and the median
regression line therefore has a negative
gradient. As the amount of exercise
increases, the level of blood cholesterol
decreases.
Notice that in this case the points are
not as closely aligned as in the previous
examples. We can say that the relationship (or correlation) between the variables is only weak. In general terms, the
closer that the points are to forming a
straight line, the stronger the relationship
is between the variables.
Sometimes we find that there is no
relationship between the variables. In the
scatterplot below, a researcher was
looking for a link between peoples
heights and their IQs. The points appear
to be randomly dispersed across the scatterplot. In cases like this, it can be concluded that there is no clear relationship
between the variables.
140
IQ

120
100
80
60
120 140 160 180 200
Height (m)

Blood cholesterol level

Period of exercise (h)

10
8
6
4
2
0
0

6 8 10 12 14 16
Period of exercise

Chapter 12 Correlation

371

WORKED Example 7

In the figure on the right, describe the


correlation as being positive or negative.

THINK

WRITE

Add a median regression line to the


scatterplot.

The gradient of the regression line is positive.


Therefore the correlation is positive.

There is a positive correlation.

The strength of a correlation is based on the correlation coefficient. The correlation


coefficient is a measure of a correlation.
Correlation coefficient
1

Description
Perfect positive correlation

Between 0.75 and 1

Strong positive correlation

Between 0.5 and 0.75

Moderate positive
correlation

Between 0.25 and 0.5

Weak positive correlation

Between 0.25 and 0.25

No correlation

Between 0.5 and 0.25

Weak negative correlation

Scatterplot

372

Maths Quest General Mathematics HSC Course

Correlation coefficient
Between 0.75 and 0.5

Description
Moderate negative
correlation

Between 1 and 0.75

Strong negative correlation

Perfect negative correlation

Scatterplot

WORKED Example 8
The operators of a casino keep records of the number of people playing a Jackpot type
game and compare the numbers playing to the size of the jackpot. The correlation
coefficient for this game is calculated to be 0.65. Describe the correlation between the
prize and the number of players.
THINK

WRITE

The correlation coefficient is between 0.5


and 0.75 and so it is a moderate positive
correlation.

There is a moderate positive correlation


between the jackpot and the number of players
in the game.

Causality
Causality refers to one variable causing another.
For example, there is a high correlation between
a persons shoe size and shirt size. However, one
does not cause the other. Similarly, there is a
high correlation between number of cigarettes
smoked and lung cancer but, in this case,
smoking causes lung cancer.
Explain whether a positive or negative
relationship exists and discuss causality in each
of the following.
1. Hours of study and exam marks
2. Hours of exercise and resting pulse rate
3. Weight and shirt size
4. The number of hotels and churches in country
towns
5. The number of motels in a town and the number of flights landing at the nearest
airport

Chapter 12 Correlation

373

It is possible to make a qualitative judgement as to the type of correlation that is


involved in a relationship by the general appearance of the graph. Care must be taken
before making a statement about one variable causing the other.
Just because there is a strong relationship between two variables, it does not mean
that one variable causes the other. For example, there is a very strong positive correlation
in people between their shoe size and their shirt size, but one does not cause the other.
Similarly, there is a very strong correlation between the amount of study done for an
exam and the result achieved on the exam. In this case it can be argued that the study
causes the high exam mark. Each case needs to be considered on its merit.

WORKED Example 9

A manufacturer who is interested in minimising the cost of training gives 15 of his plant
operators different amounts of training and then measures the number of errors made by
each of the operators. The results of the experiment are placed on a scatterplot and the
correlation between the number of hours of training and the number of errors made is
measured to have a correlation coefficient of 0.69.
a What can be said of the correlation between training and errors?
b What conclusion could the manufacturer make about causality in this case?
THINK

WRITE

The correlation coefficient is


between 0.75 and 0.5.
2 A correlation coefficient in this
range indicates a moderate
negative correlation.
b In this case it would seem logical
that those that have undertaken more
training would make fewer errors.
1

There is a moderate negative correlation between


the amount of training and the number of errors
made.
b The manufacturer could reasonably presume that
the more training a person is given, the less likely
they are to make errors with the machinery.

remember
1. The pattern of the scatterplot gives an
indication of the level of association
(correlation) between the variables.
2. When one variable increases with
another, there is a positive correlation
between them.
3. When one variable decreases while the
other increases, there is negative
correlation.
4. The extent of the correlation is then
measured by the correlation coefficient.
The description of the correlation is
given in the figure on the right.
5. Strong correlation between two
variables does not necessarily mean that
one variable causes the other.

Strong positive
correlation
Moderate positive
correlation
Weak positive
correlation
No correlation
Weak negative
correlation
Moderate negative
correlation
Strong negative
correlation

Perfect positive
correlation

0.75
0.5
0.25
0
0.25
0.5
0.75
1

Perfect negative
correlation

374

Maths Quest General Mathematics HSC Course

12D
WORKED

Example

Correlation

1 For each of the following, state whether a positive or negative correlation exists.
a

2 A sample of 10 drivers was taken. Each driver was asked their age and the number of
speeding offences they had committed in the past five years. The results are in the
table below.
Age
Speeding offences

22

36

48

40

58

64

23

25

30

45

a Display the information on a scatterplot.


b State if there is a positive or a negative correlation between age and speeding
offences.
3 Match each of the following scatterplots with the correlation that it shows.
a

Strong positive correlation


Weak positive correlation
Weak negative correlation
Strong negative correlation

Moderate positive correlation


No correlation
Moderate negative correlation

375

Chapter 12 Correlation

4 A pie seller at a football match notices that there seems to be a relationship between
the number of pies that he sells and the temperature of the day. He collects the
following data.
Daily temperature (C)

12

Number of pies sold

22

26

11

18

14

16

15

16

620 315 295 632 660 487 512 530 546 492

a Draw a scatterplot of the data.


b State the type of correlation that the scatterplot shows and draw a conclusion from
the graph.
5 A researcher is investigating the effect of living in airconditioned buildings upon
general health. She records the following data.
Hours spent each week in
airconditioned buildings

13

48 40

10

Number of days sick due


to flu and colds

15 13

14

16

18 10

a Plot the data on a scatterplot.


b State the type of correlation the graph shows and draw a conclusion from it.
c The researcher finishes her experimental report by concluding that airconditioning
is the cause of poor health. Is she correct to say this? What other factors could have
influenced the relationship shown by the scatterplot?
6 The data below show the population and area of the Australian states and territories.
State

Area ( 1000 km2)

Population ( 1000)

Vic.

228

5092

NSW

802

6828

ACT

329

Qld

1727

4053

NT

1346

207

WA

2526

2051

SA

984

1555

Tas.

68

489

a Plot the data on a scatterplot.


b State the type of correlation the graph shows and draw a conclusion from it.

376

Maths Quest General Mathematics HSC Course

7 In an experiment, 12 people were administered different doses of a drug. When the


drug had taken effect, the time taken for each person to react to a set stimulus was
measured. The results are detailed below.
Amount of drug (mg)

Reaction time (s)

0.1

0.030

0.2

0.025

0.3

0.028

0.4

0.036

0.5

0.040

0.6

0.052

0.7

0.046

0.8

0.068

0.9

0.085

1.0

0.092

1.1

0.084

1.2

0.096

a Plot the data on a scatterplot.


b State the type of correlation the graph shows, and draw a conclusion from it.
8 multiple choice
What type of correlation is shown by the graph on
the right?
A No correlation
B Weak negative correlation
C Moderate negative correlation
D Strong negative correlation
9 multiple choice
What type of correlation is shown by the graph on
the right?
A No correlation
B Weak positive correlation
C Moderate positive correlation
D Strong positive correlation
10 What type of correlation would be represented by scatterplots that had the following
correlation coefficients?
a 1.0
b 0.4
c 0.8
d 0.7
e 0.35
f 0.21
g 0.75
h 0.50
i 0.25
j 1.0

Chapter 12 Correlation

377

11 A researcher investigating the proposition that tall mothers have tall sons measures
the heights of 12 mothers and the heights of their adult sons. The correlation coefficient
8
is found to be 0.67. Describe the correlation between tall mothers and tall sons.

WORKED

Example

12 A teacher who is interested in the amount of time students


spend doing homework asks 15 students to record the amount
of time that they spend on homework and on watching
television. The correlation coefficient is found to be 0.45.
Interpret the correlation between homework and television
watching.
13 A psychologist asked 20 people to rate their level of
contentment on a scale of 0 to 10 (10 representing
perfectly content). This rating is compared to annual
income.
a The correlation coefficient is found to be 0.18.
Describe the correlation between income and level
of contentment.
b The researcher then intends to write an essay entitled
Money cant buy happiness. Do the results confirm
this statement?
14 An experimenter who is investigating the
relationship between exercise and obesity
9
measures the weights of 30 boys (of
equal height) and also documents the
amount of physical exercise that the
boys completed each week.
The correlation coefficient is found to be 0.47.
a What can be said of the correlation between obesity and exercise?
b What conclusion could be made about causality in this case?

WORKED

Example

A researcher is interested in the association between the work rate of production


workers and the level of incentive that they are offered under a certain scheme. After
drawing a scatterplot, she calculates the correlation between the two variables at 0.82.
The researcher can conclude that:
A There is a strong positive correlation between the variables; the greater the incentive, the lower the work rate.
B There is a strong positive correlation between the variables; the greater the incentive, the greater the work rate.
C There is a strong negative correlation between the variables; the greater the
incentive, the lower the work rate.
D There is a strong negative correlation; incentives cause an increase in the work rate.

Work

15 multiple choice

T
SHEE

12.2

378

Maths Quest General Mathematics HSC Course

summary
Scatterplots
When looking for a relationship between two variables, data can be represented on
a scatterplot.
One variable (the independent variable) is on the x-axis and the other variable (the
dependent variable) is on the y-axis.
Points are plotted by the coordinates formed by each piece of data.
If the dependent variable consistently increases or decreases as the independent
variable increases, a relationship exists.
If all points on the scatterplot form a straight line, the relationship is said to be
linear.
The pattern of the scatterplot gives an indication of the strength of the relationship
or level of association between the variables. This level of association is called
correlation.
A strong correlation between variables does not imply that one variable causes the
other to occur.

Median regression lines


A regression line is the line of best fit on a scatterplot.
By measuring the gradient and the y-intercept on the regression line, we can use the
formula y = mx + b to find the equation.
When the equation of a regression line has been found, it can then be used to make
predictions about the data.
We can find the regression line by using the eye method or the method of
3-medians.

Correlation
Correlation is the measure of the relationship between two variables.
A correlation can be positive or negative and has the same sign as the gradient of
the median regression line.
A positive correlation means that one quantity will increase as the other increases.
A negative correlation means that one quantity will decrease as the other increases.
Correlation can be quantified by using a correlation coefficient.
The correlation coefficient may be interpreted as follows:
q=1
Perfect positive correlation
0.75 q < 1
Strong positive correlation
0.5 q < 0.75
Moderate positive correlation
0.25 q < 0.5
Weak positive correlation
0.25 < q < 0.25
No correlation
0.5 < q
0.25
Weak negative correlation
0.75 < q
0.5
Moderate negative correlation
1<q
0.75
Strong negative correlation
q = 1
Perfect negatve correlation
The correlation coefficient will always be a number between 1 and 1 or equal to
1 or 1.

379

Chapter 12 Correlation

CHAPTER
review
1 The table below shows the maximum and minimum temperature on 10 days chosen at
random throughout the year. Display this information on a scatterplot.

12A

Maximum temperature (C)

25

36

21

40

24

26

30

18

20

25

Minimum temperature (C)

12

21

11

23

12

15

19

10

13

2 The table below shows the number of sick days taken by ten employees and relates this to
the number of children that they have.
No. of children

No. of sick days

10

12

12

12A

a Show this information on a scatterplot.


b Does a relationship appear to exist between the number of sick days taken and the
number of children they have? If so, is the relationship linear?
3 The table below shows the number of cars and number of televisions in each household.
No. of cars

No. of televisions

12A

a Show this information on a scatterplot.


b Does a relationship appear to exist between the number of televisions in each household
and the number of cars they have? If so, is the relationship linear?
4 The table below shows the relationship between two variables, x and y.

12B

18

12

11

10

16

103

75

20

66

70

50

95

40

27

42

30

a Prepare a scatterplot of the data.


b On the scatterplot, fit a regression line by eye.
c By measuring the gradient and the y-intercept of the median regression line, find its
approximate equation.
5 A survey is conducted comparing household income, I, with house value, V. A scatterplot is
drawn and the regression line is found to have the equation V = 3.7I + 50 000. Use the
equation to find:
a the likely value of a house owned by a family with an income of $52 000
b the likely income (to the nearest $1000) of a family living in a house valued at $320 000.

12C

380
12C

Maths Quest General Mathematics HSC Course

6 An entomologist conducted an experiment in which small amounts of insecticide were


introduced to a container of 100 blowflies. The results are detailed below.
Insecticide (I)
(micrograms)
No. remaining after 2 h (F)

10

99

92

81

74

62

68

52

45

38

24

a Display the above information on a scatterplot and, on the scatterplot, draw the median
line of regression.
b Find the equation of the regression line.
c Use the equation to predict the number of blowflies that would remain after two hours if
4.25 micrograms of insecticide was introduced.
d Estimate the amount of insecticide needed to remove all blowflies.

12D

7 For each of the following scatterplots, state whether the correlation is positive or negative.
a
b
c

12D

8 The table below shows the relationship between the crowd at cricket matches and the
number of matches the home team has won during the season.
No. of wins by home team

Crowd

8 000

10

21 000

11 000

14

22 000

13 000

12 000

12

19 000

a Display this information on a scatterplot.


b On the scatterplot, draw the median regression line.
c State if a positive or negative correlation exists between the number of wins by the home
team and the crowd at their matches.

12D
12D

9 For each of the following, state the type of correlation if the correlation coefficient is:
a 0
b 1
c 0.5
d 0.84
e 0.3
10 An experiment that tested the strength of wooden beams of different thickness demonstrated
a correlation of 0.9 between the variables.
a What type of correlation exists in this case?
b What can be said about causality in this case?

Chapter 12 Correlation

381

11 A survey in which people were asked to state their age and the age of their car revealed a
correlation coefficient of 0.65.
a What type of correlation exists in this case?
b What can be said about causality in this case?

Practice examination questions


1 multiple choice
A researcher administers different amounts of fertiliser to a number of trial plots of potato
crop. She then measures the total mass of potatoes harvested from each plot. When drawing
the scatterplot, the researcher should graph:
A mass of harvest on the x-axis because it is the independent variable, and amount of
fertiliser on the y-axis because it is the dependent variable
B mass of harvest on the y-axis because it is the independent variable, and amount of
fertiliser on the x-axis because it is the dependent variable
C mass of harvest on the x-axis because it is the dependent variable, and amount of fertiliser
on the y-axis because it is the independent variable
D mass of harvest on the y-axis because it is the dependent variable, and amount of fertiliser
on the x-axis because it is the independent variable.
2 multiple choice
Which of the following graphs best depicts a strong negative correlation between variables?
A y
B y

C y

D y

3 multiple choice
What type of correlation is shown by the graph on the right?
A Strong positive correlation
B Moderate positive correlation
C Moderate negative correlation
D Strong negative correlation

4 multiple choice
A researcher finds that there is a correlation coefficient of 0.62 between the number of pedestrian
crossings in a town and the number of pedestrian accidents. The researcher can conclude that:
A Pedestrian crossings cause pedestrian accidents.
B Pedestrian crossings save lives.
C There is evidence to show that pedestrian crossings cause accidents.
D There is evidence to show that the greater the number of pedestrian crossings, the smaller
the number of pedestrian accidents.

12D

382

Maths Quest General Mathematics HSC Course

5 multiple choice
A researcher, who counts the amount of time taken for production line workers to assemble
components, relates it to the number of weeks that each worker has spent on the production
line. He finds a correlation of 0.82 and can conclude that:
A the greater the number of weeks spent on the production line, the quicker the assembly of
components
B the greater the number of weeks spent on the production line, the slower the assembly of
components
C many weeks doing the same task causes production workers to become efficient
D many weeks doing the same task causes production workers to become bored and slow as
a result.
Number of skiers

6 The scatterplot on the right shows the


number of skiers at a resort and the depth of
800
snow. The median regression line has been
600
drawn on the scatterplot and has the equation
400
N = 191s + 25, where N is the number of
200
skiers and s is the snow depth.
a Does a linear relationship exist between
0
0 1 2 3 4 5
depth of snow and number of skiers?
Snow depth (m)
Explain your answer.
b Use the equation of the median regression line to estimate:
i the number of skiers if the depth of snow is 3.6 m
ii the depth of snow if there are 500 skiers (correct to 1 decimal place).
c By studying the scatterplot:
i state if the correlation between depth of snow and number of skiers is positive or
negative
ii describe the correlation as strong, moderate or weak.
7 The table below shows world population from 1955 to 2005.

CHAPTER

test
yourself

12

Year

1955 1960 1965 1970 1975 1980 1985 1990 1995 2000 2005

World pop.
(million)

2750 3000 3400 3700 4000 4400 4800 5300 5750 6073 6451

a Display this information on a scatterplot.


b Draw the median regression line on your scatterplot.
c Let Y be the number of years since 1950 and P the world population. Find the equation of
your median regression line.
d Use your graph to estimate the world population in 2010.
e Estimate from your graph when world population will exceed 10 billion.

Spherical
geometry

13
syllabus reference
Measurement 7
Spherical geometry

In this chapter
13A Arc lengths
13B Great circles and small
circles
13C Latitude and longitude
13D Distances on the Earths
surface
13E Time zones

areyou

READY?

Are you ready?

Try the questions below. If you have difficulty with any of them, extra help can be
obtained by completing the matching SkillSHEET. Either click on the SkillSHEET icon
next to the question on the Maths Quest HSC Course CD-ROM or ask your teacher for
a copy.

13.1

Circumference of a circle

1 Find the circumference of the following circles. Answer correct to 1 decimal place.
a
b
c
11 cm

13.2

6400 km
28 m

Calculating arc length

2 Find the arc length of each of the following. Answer correct to 3 significant figures.
a
b
c
6m
240
120

9.9 m
12.2 cm

13.3

Converting units of time

3 Convert each of the following to the unit indicated in brackets.


a 160 minutes (hours and minutes)
b 3 1--- hours (minutes)
2
c 4 weeks (days)
d 15 months (years)
4 Use 1 nautical mile 1.852 km to convert the following.
a 6 nautical miles to kilometres
b 80 kilometres to nautical miles
c 4.2 nautical miles to metres
d 7530 metres to nautical miles

Chapter 13 Spherical geometry

385

Arc lengths
An arc is a section of the circumference of a circle. To calculate an arc length we must
first revise the circumference of a circle. The circumference of a circle can be found
using either of the formulas:
C = d, where d is the diameter
C = 2 r, where r is the radius.

WORKED Example 1

Calculate the circumference of a circle that has a radius of 6 m. Give your answer correct
to 2 decimal places.
THINK
1
2
3

WRITE

Write the formula.


Substitute the value of r.
Calculate the circumference.

C=2 r
C=2
6
C = 37.70 m

The length of an arc can be calculated as the fraction of


the circle determined by the angle subtended by the arc
at the centre, as shown in the figure on the right.
The arc length, l, can be calculated using the formula:

l = --------- 2 r
360
where

= number of degrees in the central angle.

WORKED Example 2

Calculate the length of the arc shown on the right,


correct to 1 decimal place.
7.1 cm
60

THINK

WRITE

Write the formula.

Substitute the value of

Calculate the arc length.

and r.

l = --------- 2 r
360
60
l = --------- 2
360
l = 7.4 cm

7.1

The arc length formula is then used to make calculations about the distance between
points on the Earths surface.

386

Maths Quest General Mathematics HSC Course

WORKED Example 3
The radius of the Earth at the equator is approximately 6400 km.
a Calculate the circumference of the Earth at the equator, correct to the nearest
kilometre.
b Two points on the equator subtend at a 5 angle at the centre of the Earth. Calculate the
distance between them, correct to the nearest kilometre.
THINK

WRITE

Write the formula.


Substitute the value of r.
Calculate the circumference.

a C=2 r
=2
6400
= 40 212 km

Write the formula.

Substitute for

Calculate the distance.

b d = --------- 2 r
360
5
= --------- 2
360
= 559 km

1
2

and r.

6400

remember
1. The circumference of a circle can be found using either of the formulas C = d
or C = 2 r.
2. An arc length is calculated by using the angle the arc subtends at the centre of
a circle. The arc length is calculated using the formula:
l = --------- 2 r
360
where is the number of degrees in the central angle.
3. The arc length formula can be used to find the distance between points on the
equator.

SkillS

13A
HEET

13.1

WORKED

Example

Arc lengths

1 Calculate the circumference of each of the following circles, correct to 1 decimal place.
a
b
c

Circumference
of a circle

62 mm
4 cm
9m

Cabr

omet
i Ge ry

Arc
length

f
13.9 km

4.1 km

8.3 m

387

Chapter 13 Spherical geometry

2 Calculate the circumference of a circle with a radius of 100 km. Give your answer
correct to the nearest 10 km.
3 Calculate the circumference of each of the following circles. Give your answer correct
to 3 significant figures.
a
b
c
60 km

14 cm

e
117 mm

39 m

f
3.7 km

219 km

4 Calculate the circumference of a circle that has a diameter of 3000 km. Give your
answer correct to the nearest 100 km.
5 Use the formula C = d to find the diameter of a circle with a circumference of
100 cm. Give your answer correct to 2 decimal places.
6 Find (correct to 3 significant figures):
a the diameter of a circle with a circumference of 80 m
b the radius of a circle with a circumference of 42.3 cm
c the diameter of a sphere with a circumference of 2500 km.
Example

7 Calculate the length of the arc shown on the right,


correct to 2 decimal places.

50
10 cm

Calculating
arc
length

45

120
72 cm

13 m

13.2 SkillS
HEET

8 Calculate the lengths of each of the arcs drawn below, correct to 1 decimal place.
a
b
c
mm
43

WORKED

220
m
c
7.2

72 km
150
7.9 km
4

388
WORKED

Example

Maths Quest General Mathematics HSC Course

9 The radius of the Earth at the equator is 6400 km.


a Calculate the circumference of the Earth at the equator, correct to the nearest
100 km.
b Two points on the equator subtend an angle of 40 at the centre. Calculate the
distance between them, correct to the nearest 10 km.
10 A circle has a radius of 30 km.
a Two points on the circle subtend an angle of 100 at the centre. Calculate the
length of the arc joining them, correct to the nearest kilometre.
b The outer arc between the two points subtends an angle of 260. Calculate the
length of the outer arc, correct to the nearest kilometre.
c Show that the sum of the lengths of the two arcs is equal to the circumference of
the circle.
11 A sphere has a radius of 40 cm.
a Calculate the circumference of the sphere, correct to the nearest centimetre.
b Calculate the distance between two points on the sphere that subtend an angle of
90 at the centre, correct to 1 decimal place.
12 Calculate the distance, correct to 1 decimal place, between two points on a sphere:
a of radius 10 cm, which subtend an angle of 30 at the centre
b of radius 2 m, which subtend an angle at 122 at the centre
c of radius 6400 km, which subtend an angle of 51 at the centre.
13 Calculate the distance between two points on the Earths surface that subtend an angle
of 1 at the centre. Give your answer correct to the nearest kilometre.

Chapter 13 Spherical geometry

389

Great circles and small circles


Consider the sphere drawn on the right. The axis of
the sphere is a diameter of that sphere. The ends of
the axis are called the poles.
If we draw any lines around the sphere passing
through both poles, a great circle is formed. A great
circle is the largest possible circle that can be drawn
around the sphere.
Great circle

Pole

Axis

Pole

The length of a great circle is found using the formulas for the circumference of a
circle:
C = d, where d is the diameter of the sphere
or C = 2 r, where r is the radius of the sphere.

WORKED Example 4

Calculate the length of a great circle on a sphere with a radius of 40 cm. Give your answer
correct to the nearest centimetre.
THINK
1
2
3

WRITE

Write the formula.


Substitute the radius of the sphere.
Calculate the length of the great circle.

C=2 r
C=2
40
C = 251 cm

Now consider a circle drawn perpendicular to the axis of


the sphere. Only one circle, called the equator, will be a
great circle. The centre of the equator will be the centre of
the sphere as shown on the right.
Other circles that are perpendicular to the axis of the
sphere will be smaller than a great circle and are called
small circles.
Small circle
Equat o r

Equato r

390

Maths Quest General Mathematics HSC Course

To calculate the length around a small circle, we need to


know the small circles radius. The small circle will have a
radius smaller than that of the great circle, as shown in the
figure on the right.

Radius of small circle

Radius of great circle

WORKED Example 5
Calculate the length of the small circle on the right, correct to
1 decimal place.
2m
1.5 m

THINK
1
2
3

WRITE

Write the formula for the circumference


of a circle.
Substitute for r.
Calculate the circumference.

C=2 r
C=2
1.5
C = 9.4 cm

remember
1. A great circle is the circle of the greatest possible size that lies on the surface
of a sphere.
2. The length of a great circle is calculated using the formula for the
circumference of a circle.
3. A small circle is any circle smaller in size than the great circle.

13B
WORKED

Example

Great circles and


small circles

1 Calculate the length of a great circle around a sphere of radius 7 cm. Give your
answer correct to 2 decimal places.
2 Calculate the length of a great circle on each of the spheres drawn below. Give each
answer correct to 3 significant figures.
a
b
c
9m
30 m
74 mm

3 The Earth is a sphere with a radius of approximately 6400 km. Calculate the length of
a great circle around the Earths surface, correct to the nearest 10 km.

Chapter 13 Spherical geometry

391

4 Calculate the length of a great circle that lies on a sphere with a diameter of 1 m. Give
your answer correct to the nearest centimetre.
5 Below are the diameters of the other planets in our solar system. Calculate the length
of a great circle on the surface of each planet, correct to the nearest 10 km.
a Mercury 4878 km
b Venus 12 100 km
c Mars 6796 km
d Jupiter 142 984 km
e Saturn 120 540 km
f Uranus 51 118 km
g Neptune 49 100 km
5

6 Calculate the length of a small circle on a sphere of radius 4 m if the radius of the
small circle is 2 m. Give your answer correct to 2 decimal places.
7 Calculate the length of each of the small circles below, correct to 1 decimal place.
a
b
c
1100 km
80

6.4 m
25
7.1 m

6400 km

9m
10
8.8 m

8 multiple choice
The diameter of the moon is 3476 km. The length of a great circle on the moons surface is closest to:
A 5460 km
B 10 920 km
C 21 840 km
D 43 680 km
9 The diameter of the sphere drawn on the right
is 50 cm. Calculate the distance along the
surface from one pole to the other. Give your
answer correct to the nearest centimetre.
10 Calculate the distance between the north and
south poles along the Earths surface, correct
to the nearest 100 km. (Take the radius of the
Earth to be 6400 km.)
11 A and B are two points on a small circle of radius
60 cm, as shown in the figure on the right.
a Calculate the circumference of the small
circle, correct to the nearest centimetre.
b Calculate the length of the arc, AB, correct to
the nearest centimetre.
12 Calculate the length of each arc below, correct to
3 significant figures.
a
b
X

60 cm

1000 km
100 Y

50 m
60
Q
P

Work

Example

50 cm

WORKED

T
SHEE

13.1

392

Maths Quest General Mathematics HSC Course

1
1 Calculate the circumference of a circle that has a radius of 4.7 m. Give your answer
correct to 1 decimal place.
2 Calculate the circumference of a circle with a diameter of 56 cm. Give your answer
correct to 2 significant figures.
3 An arc on a circle of radius 9 cm subtends an angle of 60 at the centre. Calculate the
length of the arc, correct to 1 decimal place.
4 What is a great circle?
5 What is a small circle?
6 Calculate the length of a great circle that lies on a sphere of radius 15 cm. Give your
answer correct to 2 decimal places.
7 Calculate the distance between the north and south poles on a planet that has a
diameter of 4500 km. Give your answer correct to the nearest 100 km.
8 Calculate the distance between two points on the great circle of the planet in question 7
that subtend an angle of 80 at the centre. Give your answer correct to the nearest 10 km.
9 A planet has a radius of 2300 km. Calculate the length of a great circle lying on the
surface of the planet, correct to the nearest 10 km.
10 Sydney lies on a small circle of the Earth that has a radius of approximately 5400 km.
Calculate the circumference of the small circle (correct to the nearest 100 km).

393

Chapter 13 Spherical geometry

Latitude and longitude

Equat o r

South Pole

30

30S

Sydney

North Pole
enwich Meridian
Gre

As the Earth is a sphere, great circles and small circles on the surface of the Earth are used to locate
points on the surface.
Consider the axis of the Earth to be the diameter
joining the North Pole and the South Pole. The only
great circle that is perpendicular to this axis is the
equator. The angular distance either north or south of
the equator is the latitude.
Small circles parallel to the equator are called
parallels of latitude. These small circles are used to
describe how far north or south of the equator a place
is. For example Sydney lies close to the small circle
30S.
This means Sydney subtends a 30 angle at the
centre of the Earth and is south of the equator.
The maximum latitude for any point on the Earth
is 90 N or 90 S. The north and south poles lie at
these points.
For latitude, the equator is the line of reference for
all measurements.
To locate a place on the globe in an eastwest
direction, the line of reference is the Greenwich
Meridian. The Greenwich Meridian is half a great
circle running from the North to the South Pole.
The Greenwich Meridian is named after Greenwich, a suburb of London through which the circle
runs.
All other places on the globe are located by the
half great circle on which they lie. These half great
circles are called meridians of longitude.
Each meridian of longitude is identified by the
angle between it and the Greenwich Meridian and by
whether it is east or west of Greenwich.
The meridian of longitude opposite the Greenwich
Meridian is the International Date Line. The International Date Line has longitude 180 either east or
west. On either side of the International Date Line
the day changes. (This will be explained in more
detail later in the chapter.)
For the convenience of some small island nations
and Russia, the International Date Line is bent so not
to pass through them.
World maps or globes are drawn with both parallels of latitude and meridians of longitude shown.
Any location on a map or globe can be giv en a pair
of coordinates: the first is the parallel of latitude that
it lies on, the second is the meridian of longitude. For
example, the coordinates of Sydney are 30S, 150E.

North Pole

Equato r

South Pole

40

Greenwich
Meridian

International
Date Line

30S

30N

60N

30W

ANGOLA

NIGER

30E

60E

MADAGASCAR

Johannesburg

SOUTH AFRICA

Cape Town

Baghdad

IRAQ

Beijing

90E

Perth

Darwin

120E

150E

Hobart

Melbourne

Sydney

Brisbane

Port Moresby

PAPUA
NEW GUINEA

Tokyo

AUSTRALIA

SINGAPORE

Manila

PHILIPPINES

TAIWAN

Shanghai

INDONESIA

150E

JAPAN
NORTH KOREA

120E

CHINA SOUTH KOREA

90E

Colombo

INDIA

RUSSIA

60E

SRI LANKA

30E

Moscow

KENYA

LIBYA Cairo
EGYPT

Rome

ITALY

NAMIBIA

MALI

ALGERIA

SPAIN

Madrid

FRANCE

PORTUGAL

Oslo

Amsterdam
London NETHERLANDS

IRELAND

FINLAND
NORWAY SWEDEN

30W

UK

ICELAND

GREENLAND

60W

180

90W

120W

JAMAICA

90W

Santiago

60W

30W

Buenos Aires

URUGUAY

PARAGUAY

ARGENTINA

CHILE

Rio de Janeiro

BRAZIL
BOLIVIA

Lima

PERU

COLOMBIA
ECUADOR

MEXICO

30W

New York

Montreal

60W

UNITED STATES
OF AMERICA

CANADA

Los Angeles

Vancouver

120W

150W

Hawaii (USA)

Alaska
(USA)

150W

NEW
ZEALAND

Auckland

Suva

FIJI

180

International Date Line

30S

30N

60N

394
Maths Quest General Mathematics HSC Course

Chapter 13 Spherical geometry

395

WORKED Example 6
Identify the major cities closest to each of the following locations using the map on
page 394.
a 30S, 30E
b 30N, 120E
c 45N, 75W
THINK

WRITE

a Look for the city closest to the


intersection of the 30S parallel of
latitude and the 30E meridian of
longitude.

a Johannesburg

b Look for the city closest to the


intersection of the 30N parallel of
latitude and the 120E meridian of
longitude.

b Shanghai

c Look for the city closest to the


intersection of the 45N parallel of
latitude and the 75W meridian of
longitude.

c Montreal

WORKED Example 7
Write down the approximate coordinates of each of the following cities using the map on
page 394.
a Singapore
b Perth
c Los Angeles
THINK

WRITE

Use the parallels of latitude drawn to


estimate the latitude.
Use the meridians of longitude
drawn to estimate the longitude.

Use the parallels of latitude drawn to


estimate the latitude.
Use the meridians of longitude
drawn to estimate the longitude.

Use the parallels of latitude drawn to


estimate the latitude.
Use the meridians of longitude
drawn to estimate the longitude.

1
2

1
2

1
2

1N, 104E

32S, 115E

35N, 118W

396

Maths Quest General Mathematics HSC Course

remember
1. The Earths axis runs from the North Pole to the South Pole.
2. The only great circle perpendicular to the Earths axis is the equator.
3. Places are located either north or south of the equator using parallels of
latitude. Each parallel of latitude is a small circle parallel to the equator and is
identified by the angle it subtends at the centre.
4. Places are located as being east or west of the Greenwich Meridian. The
Greenwich Meridian is half a great circle from the North Pole to the South
Pole.
5. All meridians of longitude are half great circles and are measured by the angle
made to the east or west of the Greenwich Meridian.
6. A pair of coordinates can identify every point on the Earths surface. These are
called its latitude and longitude.

13C

Latitude and longitude

For the following questions use the map on page 394.


1 Write down the name of the city closest to each of the following pairs of coordinates.
a 30N, 30E
b 30N, 120E
6
c 15S, 135E
d 45N, 75W
e 50N, 0
f 37S, 175E
g 35N, 140E
h 40N, 115E
1
i 22 --- S, 43W
j 60N, 11E

WORKED

Example

2 State the approximate latitude and longitude of each of the following major cities or
Example
islands.
7
a Melbourne
b New York
c Jamaica
d Johannesburg
e Rome
f Buenos Aires
g Baghdad
h Moscow
i Singapore
j Suva
WORKED

Chapter 13 Spherical geometry

397

Important parallels of latitude


Four significant parallels of latitude on the surface of the Earth are the:
1 Arctic Circle
2 Antarctic Circle
3 Tropic of Cancer
4 Tropic of Capricorn.
Find out the latitude of these small circles and state the significance of each.

Distances on the Earths surface


From the previous section on latitude and longitude
it can be seen that angular geometry is of great
importance when making measurements on the
Earths surface.

Now consider a meridian of longitude on the


Earths surface with two points on it. The angular
distance between them will be the difference
between their latitudes.
The angular distance is calculated by subtracting the latitudes of points if both are on
the same side of the equator and adding the latitudes if on opposite sides of the equator.

WORKED Example 8
The coordinates of A are (20S, 130E) and the coordinates of B are (15N, 130E).
Find the angular distance between them.
THINK

WRITE

A and B are on opposite sides of the


equator so add the latitudes.

Angular distance = 20 + 15
Angular distance = 35

398

Maths Quest General Mathematics HSC Course

Now consider two points on the same great circle that have an angular distance of
1 minute. (Remember 60 = 1.) The distance between these two points is defined to be
1 nautical mile (M). Therefore, an angular distance of 1o on a great circle will equal
60 nautical miles.
1 nautical mile 1.852 km
Using this information, we are able to calculate the distance between two points on a
great circle on the Earths surface in both nautical miles and kilometres.

WORKED Example 9
P and Q are two points on the Earths surface with coordinates (27N, 30W) and
(39N, 30W) respectively.
a Calculate the distance between P and Q in nautical miles.
b Use 1 M 1.852 km to give the distance, PQ, to the nearest kilometre.
THINK

WRITE

P and Q are on the same great circle.


Calculate the angular distance, PQ.
Convert the angular distance to
nautical miles using 1 = 60 M.

Angular distance = 39
Angular distance = 12
Distance = 12 60
Distance = 720 M

b Multiply 720 by 1.852 to convert to


kilometres.

b Distance = 720 1.852


Distance = 1333 km

2
3

27

We can also calculate the distance between two points on the same great circle, in kilometres, using the fact that the radius of the Earth is 6400 km.

WORKED Example 10
X and Y are two points on the Earths surface with coordinates (32N, 120E) and
(45S, 120E). Calculate the distance, XY, correct to the nearest 100 km.
THINK

WRITE

Calculate the angular distance, XY.

Angular distance = 32 + 45
Angular distance = 77

Use the arc length formula to calculate


the distance between X and Y, correct
to the nearest 100 km.

l = --------- 2 r
360
77
l = --------- 2 r
360
77
l = --------- 2
360
l = 8600 km

6400

In marine and aerial navigation, speed on the Earths surface is measured in knots.
1 knot = 1 nautical mile/hour

Chapter 13 Spherical geometry

399

WORKED Example 11

The coordinates of two points on the Earths surface are given by the coordinates
A(50N, 120W) and B(30S, 120W). Calculate the time taken for a ship to sail the
shortest distance between these two points at an average speed of 40 knots.
THINK
1
2

WRITE

Calculate the angular distance between


A and B.
Calculate the distance, AB, in nautical
miles.
distance
Use the formula time = ------------------- to
speed
calculate the time taken for the journey.

Angular distance = 50 + 30
Angular distance = 80
Distance = 80 60
Distance = 4800 M
distance
Time = ------------------speed
4800
Time = -----------40
Time = 120 hours (5 days)

remember
1. The angular distance between two points on the same great circle (meridian of
longitude) can be found by:
subtracting the latitudes if the points are on the same side of the equator
adding the latitudes if the points are on opposite sides of the equator.
2. An angular distance can be converted to a linear distance using the relationship,
1 = 1 nautical mile (M).
3. 1 M 1.852 km
4. The distance between two points on the same great circle can also be found
using the arc length formula and taking the radius of the Earth as being
6400 km.
5. Speed can be measured in knots. A speed of 1 knot = 1 M/hour.

13D
WORKED

Example

Distances on the Earths


surface

1 Two points, A and B, on the Earths surface are at (30N, 25W) and (20S, 25W).
Calculate the angular distance between A and B.
2 In each of the following calculate the angular distance between the pairs of points
given.
a (70N, 150E) and (30N, 150E)
b (25N, 40W) and (15S, 40W)
c (64N, 0) and (7S, 0)
d (42S, 97W) and (21S, 97W)
e (0, 60E) and (0, 20W)
3 The city of Durban is at approximately (30S, 30E) while Cairo is at approximately
(30N, 30E). What is the angular distance between Durban and Cairo?

400
WORKED

Example

Maths Quest General Mathematics HSC Course

4 P and Q are two points on the Earths surface with coordinates (45N, 10W) and
(15N, 10W) respectively.
a Calculate the distance between P and Q in nautical miles (M).
b Use 1 M = 1.852 km to calculate the distance, PQ, correct to the nearest km.
5 Calculate the distance between each of the points below in nautical miles.
a A (10N, 45E) and B (25S, 45E)
b C (75N, 86W) and D (60S, 86W)
c E (46S, 52W) and F (7S, 52W)
d G (34N, 172E) and H (62S, 172E)
6 The city of Osaka is at (37N, 135E) while Alice Springs is at (23S, 135E).
a Calculate the distance between Osaka and Alice Springs in nautical miles.
b Use 1 M = 1.852 km to write this distance, correct to the nearest kilometre.
7 The Tropic of Cancer is at latitude 22 1--- N while the Tropic of Capricorn is at latitude
2
22 1--- S. Calculate the distance between these two tropics along the same great circle in:
2
a nautical miles
b kilometres (correct to the nearest km).

WORKED

Example

10

8 M and N are two points on the Earths surface with coordinates (56N, 122W) and
(3S, 122W). Calculate the distance, MN, correct to the nearest 100 km, using the
arc length formula. (Take the radius of the Earth to be 6400 km.)
9 Calculate the distance between each of the points below, correct to the nearest
kilometre, using the arc length formula and taking the radius of the Earth to be 6400 km.
a P (85S, 89E) and Q (46S, 89E)
b R (24N, 0) and S (12S, 0)
c T (34S, 17W) and U (0, 17W)
10 multiple choice
Perth is at approximately (31S, 115E) while Hong Kong is at approximately (22N,
115E). The distance between Perth and Hong Kong is approximately:
A9M
B 53 M
C 540 M
D 3180 M

11 multiple choice
Rachel is a flight navigator. She is responsible for calculating the distance between
Stockholm (60N, 18E) and Budapest (47N, 18E). Rachel calculates the distance
using the arc length formula, assuming the radius of the Earth is 6400 km. Rachels
answer would be closest to:
A 1445 km
B 1452 km
C 11 952 km
D 11 890 km
WORKED 12 The coordinates of two points on the Earths surface are X (40S, 30E) and Y (10S,
Example
30E). Calculate the time taken for a plane to fly from X to Y at a speed of 240 knots.
11
13 Quito (0, 78W) and Kampala (0, 32E) are two cities on the Equator.
a Calculate the angular distance between Quito and Kampala.
b Calculate the distance between them in nautical miles.
c Use 1 M = 1.852 km to find the distance, correct to the nearest 100 km.
d Calculate the time taken to fly from Quito to Kampala at a speed of 480 knots.
14 Calculate the distance between the North Pole and the South Pole in nautical miles.
15 The city of Kingston is at approximately (18N, 76W). Ottawa is at approximately
(46N, 76W).
a Calculate the angular distance between Kingston and Ottawa.
b Calculate the distance between Kingston and Ottawa in nautical miles.
c Use 1 M = 1.852 km to calculate the distance, correct to the nearest kilometre.
d Use the arc length formula to calculate the distance between the two cities, correct
to the nearest kilometre.
e Explain the discrepancy between the two answers.

Chapter 13 Spherical geometry

401

2
1 Calculate the length of a great circle lying on the surface of a sphere of radius 24 cm.
Give your answer correct to the nearest centimetre.
Using the map on page 394, give the coordinates of the following cities.
2 Amsterdam
3 Brisbane
4 Vancouver
Using the same map, state the name of the major city closest to the following
coordinates.
5 (40N, 10E)
6 (12N, 77E)
Two points on the Earths surface are P (13N, 75W) and Q (49N, 75W).
7 What is the angular distance between P and Q?
8 Calculate the distance, PQ, in nautical miles.
9 Use 1 M = 1.852 km to give the distance, PQ, correct to the nearest 10 km.
10 Use the arc length formula to calculate the distance, PQ, correct to the nearest 10 km.

Time zones
As the Earth rotates, different parts of the globe are experiencing day and night. This
means that each meridian of longitude on the Earths surface should have a different
time of day. To simplify this, the Earth is divided into time zones.
Time zones are all calculated in relation to Greenwich. The time on the Greenwich
Meridian is known as Greenwich Mean Time (GMT). Time zones are then stated in
terms of the number of hours they are ahead or behind GMT. All places with longitudes
east of Greenwich are ahead of GMT, while all place with longitudes west of Greenwich are behind GMT. For example, Eastern Standard Time is GMT +10, meaning that
Sydney is 10 hours ahead of GMT. When GMT is noon, EST is 10.00 pm.
The International Date Line is 12 hours ahead of Greenwich when travelling east and
12 hours behind when travelling west, so this totals 24 hours, or one day. Therefore, the
day changes on either side of the International Date Line.
The time difference between two places is calculated by subtracting the comparative
time with GMT.

402

Maths Quest General Mathematics HSC Course

WORKED Example 12
Sydney is GMT +10 while New York is GMT 5. Calculate the time difference between
Sydney and New York.
THINK

WRITE

Subtract the comparative times.

State the difference and which city is


ahead in time.

Time difference = 10 ( 5)
Time difference = 15
Sydney is 15 hours ahead of New York.

Once we have calculated the time difference, we are able to calculate the time in one
place given the time in another. To calculate the time in a city further ahead of GMT we
add time, or to calculate the time in a city further behind GMT we subtract time.

WORKED Example 13
Perth is GMT +8 while Cape Town is GMT +1. When it is 11.00 am in Cape Town, what is
the time in Perth?
THINK

WRITE

Calculate the time difference and state


which city is ahead.

Add the time difference to the time in


Cape Town to calculate the time in
Perth.

Time difference = 8 1
Time difference = 7 hours
Perth is 7 hours ahead of Cape Town.
Time in Perth = 11.00 am + 7 hours
Time in Perth = 6.00 pm

The time as calculated by the longitude is called the standard time. Time zones are calculated to approximate all the standard times within a region.

Australian time zones


At times when daylight saving time does not apply, Australia is divided into three
time zones, Eastern Standard Time (EST), Central Standard Time (CST) and
Western Standard Time (WST).
1 Which states are in each of the three time zones?
2 What is the time difference between each of these zones?
3 In which states does daylight saving time apply in summer?
4 When daylight saving is in force in each of the states that have daylight saving,
how many time zones is Australia divided into?

Chapter 13 Spherical geometry

403

As there are 24 hours in a day and 360 of longitude (180E and 180W), we can
calculate that:
1 hour = 15 of longitude
1 = 4 minutes
We are now able to compare the time in various cities given the longitude of each.

WORKED Example 14
Calculate the time in Los Angeles (34N, 120W) when it is 8.00 am on Wednesday in
Sydney (33S, 150E).
THINK
1 Calculate the difference in longitudes.
2

Convert this angular distance into hours


using 1 = 4 minutes.

Subtract the time difference from the


time in Sydney.

WRITE
Longitude difference = 150 + 120
Longitude difference = 270
Time difference = 270 4
Time difference = 1080 minutes
Time difference = 18 hours
Time in Los Angeles = 8.00 am Wednesday
Time in Los Angeles = 18 hours
Time in Los Angeles = 2.00 pm Tuesday

It is important to note that, for convenience, places that have almost the same longitude
have the same time. An example of this is Australias time zones where all of Queensland, New South Wales, Victoria and Tasmania are in the same standard time zone
although there is a difference of 12 in longitude from the easternmost and westernmost
points in this zone.
These calculations can then be used to calculate the arrival and departure times for
international travel.

WORKED Example 15
A plane leaves London (50N, 0) at 9.00 am Sunday, London time, and flies to Sydney
(33S, 150E). The flight takes 20 hours. Calculate the time in Sydney when the plane
arrives.
THINK
1 Calculate the longitude difference
between Sydney and London.
2 Use 1 = 4 minutes to calculate the time
difference.
3

Calculate the time in Sydney when the


plane is departing London by adding
the time difference.

Add the flying time to calculate the


time when the plane lands.

WRITE
Longitude difference = 150 0
Longitude difference = 150
Time difference = 150 4
Time difference = 600 minutes
Time difference = 10 hours
When the plane leaves London at 9.00 am
(London time)
Time in Sydney = 9.00 am Sunday + 10 hours
Time in Sydney = 7.00 pm Sunday
Plane arrives at 7.00 pm Sunday + 20 hours
Time in Sydney = 3.00 pm Monday.

More challenging examples will require you to allow for daylight saving time. When
daylight saving time applies, we add one hour to the standard time at that location.

404

Maths Quest General Mathematics HSC Course

remember
1. Time zones are calculated by comparison with Greenwich Mean Time (GMT).
2. For places to the east we add time, for places to the west we subtract time to
calculate the time zone.
3. The time zone is calculated using the longitude.
1 hour = 15 of longitude
1 of longitude = 4 minutes
4. To calculate the time, subtract the time when heading west and add time when
heading east.
5. Care must be taken when crossing the International Date Line as you will need
to add a day when travelling east or subtract a day when heading west.
6. Time differences need to be used when calculating departure time and arrival
time for international travel.
7. When daylight saving time applies, we add one hour to the standard time.

SkillS

13E
HEET

13.3

WORKED

Example

12

Time zones

1 The time zone in New Zealand is GMT +12 while in Turkey it is GMT +2. Calculate
the time difference between New Zealand and Turkey.
2 Calculate the time difference between each of the following locations.
a Tokyo GMT +9 and New York GMT 5
b Los Angeles GMT 8 and Dakar GMT 1
c Rio De Janeiro GMT 3 and Perth GMT +8
d Hawaii GMT 11 and Fiji GMT +11

Converting
units of
time

WORKED

Example

13

3 Sydney is GMT +10, while San Francisco is GMT 8. When it is 5.00 pm on Tuesday
in Sydney, what is the time in San Francisco?
4 For each of the following calculate:
a the time in Perth (GMT +8) when it is 10.00 pm in Sydney (GMT +10)
b the time in Washington (GMT 5) when it is 4.00 am Saturday in Sydney
(GMT +10)
c the time in Auckland (GMT +12) when it is 7.00 am Wednesday in Johannesburg
(GMT +2)
d the time in Sydney (GMT +10) when it is 6.00 am Tuesday in Salt Lake City
(GMT 7)
e the time in Adelaide (GMT +9.5) when it is 8.15 pm Sunday in the Cook Islands
(GMT 10).
5 Jane is in Sydney (GMT +10) and wants to telephone her friend in Paris (GMT) at
7.00 pm Friday, Paris time. At what time must she call from Sydney?
6 Carl is holidaying in Hawaii (GMT 11). If he wants to call his parents in Sydney
(GMT +10) at 8.00 pm on Wednesday, what time must he call from Hawaii?
7 Neville is in Sydney (GMT +10). He wants to set his video recorder to tape the
Superbowl which is being played in Atlanta (GMT 5) and televised live in Sydney.
The Superbowl is due to begin at 7.00 pm on Sunday in Atlanta. At what day and time
will Neville need to set his video to begin taping?

Chapter 13 Spherical geometry

405

8 Sydney is GMT +10.


a What is the time in Sydney when it is noon GMT?
b When daylight saving time is applied, describe the time zone in Sydney as
compared to GMT.
c During daylight saving time, what will the time be in Sydney when it is:
i 4.00 am Monday GMT?
ii 9.00 pm Thursday GMT?
9 Sydney is GMT +10 and Los Angeles is GMT 8. Calculate the time difference
between Sydney and Los Angeles when:
a both cities are on standard time
b Sydney has daylight saving time and Los Angeles is on standard time
c Los Angeles has daylight saving time and Sydney is on standard time.
10 Beijing is at approximately (40N, 120E). Rome is at approximately (40N, 15E).
Calculate the difference in standard time between Beijing and Rome.
14
11 Calculate the time difference in standard time between each of the following cities.
a Mumbai (19N, 73E) and Casablanca (23N, 82W)
b Tokyo (36N, 140E) and Adelaide (23S, 134E)
c Miami (26N, 80W) and Seattle (47N, 122W)

WORKED

Example

12 multiple choice
At a point on the Earths surface, the coordinates are (45N, 135W). The standard
time at this point would be:
A GMT 3
B GMT +3
C GMT 9
D GMT +9
13 multiple choice
It is 11.00 am Tuesday at a point X with coordinates (32S, 90W). At a point, Y, with
coordinates (51N, 120E), what is the time if daylight saving time applies at Y?
A 9.00 pm Monday
B 10.00 pm Monday
C 1.00 am Wednesday
D 2.00 am Wednesday
14 A plane leaves Sydney (32S, 150E) at 2.00 pm on Tuesday. If it is an 18-hour flight
to Los Angeles (33N, 120W), at what time will the plane touch down in Los Angeles?
15
15 A plane leaves Perth (32S, 120E) on an 8-hour flight to Cape Town (33S, 15E) at
3.00 pm Wednesday.
a At what time will the plane arrive in Cape Town?
b The return flight leaves Cape Town at 5.00 pm Saturday. At what time will it arrive
in Perth?

WORKED

16 A flight leaves Melbourne (40S, 150E) at 5.00 pm Tuesday on an 18-hour flight to


Frankfurt (50N, 15E). Calculate the time of arrival in Frankfurt if it is:
a daylight saving time in Melbourne
b daylight saving time in Frankfurt.

The keepers of time


Information about the world time zones can be found at the internet site for the
Royal Observatory Greenwich, which is recognised worldwide as the keeper of
time. Information can be found here about all time zones throughout the world and
other facts about time. Their address is www.rog.nmm.ac.uk.

Work

Example

T
SHEE

13.2

406

Maths Quest General Mathematics HSC Course

summary
Arc lengths
The circumference of a circle is found using either C = d or C = 2 r.
The length of an arc can be found using the formula:

where

l = --------- 2 r
360
is the number of degrees in the central angle.

Great circles and small circles


An axis of the sphere is any diameter. A diameter must pass through the centre of
the sphere. The endpoints of the axis are called the poles.
A great circle is any circle of maximum diameter that can be drawn on the surface
of a sphere. The circumference of a great circle can be found using the formula
C = 2 r.
A small circle is any circle on the surface of the sphere that is smaller than a great
circle.

Latitude and longitude


A point on the Earths surface is located using a pair of coordinates.
A point is located as being either north or south of the equator using latitude.
Latitude is the angle subtended at the centre of the sphere by the small circle on
which the point lies.
A point is located as being either east or west of the Greenwich Meridian by the
meridian of longitude. Each meridian of longitude is measured by the angle
between the meridian of longitude and the Greenwich Meridian.

Distances on the Earths surface


If two points lie on the same great circle, the angular distance between them can be
found by finding the difference between their latitudes.
The distance between two points can then be found in nautical miles using
1 = 60 M.
1 M 1.852 km.
The distance between two points can also be found in kilometres using the arc
length formula.
Speed can be measured in knots, where 1 knot = 1 M/h.

Time zones
The longitude of a city determines its time zone.
The standard time at any location can be calculated using 15 = 1 hour or
1 = 4 minutes.
Points to the east of the Greenwich Meridian have standard time ahead of GMT.
Points to the west of the Greenwich Meridian have standard time behind GMT.

Chapter 13 Spherical geometry

407

CHAPTER
review
1 Calculate the circumference of each of the following circles, correct to 1 decimal place.
a
b
c
8.6 cm

19.2 cm

92 mm

2 Calculate the length of each of the following arcs, correct to 1 decimal place.
a
b
c
9m

13A

13A

260
6.2 cm

40

135
9.3 m

3 A sphere has a radius of 7.5 cm.


a Calculate the circumference of the sphere, correct to 1 decimal place.
b Calculate the distance between two points on the surface of the sphere that subtend an
angle of 60 at the centre. Give your answer correct to 2 decimal places.

13A

4 Calculate the circumference of each of the following spheres, correct to 1 decimal place.
a
b
c

13B

7 cm

33 cm

9.2 m

5 Calculate the circumference of a great circle that lies on the surface of a sphere with a radius
of 9 km. (Give your answer correct to 1 decimal place.)

13B

6 A sphere has a diameter of 45 cm. Calculate the distance between the poles on this sphere.
Give your answer correct to the nearest centimetre.

13B

7 Calculate the circumference of each of the small circles drawn below, correct to 1 decimal place.
a
b
c
2.8 m

13B

48
8.1 m
7 cm
20
6.6 cm

1500 km
3000 km

408

Maths Quest General Mathematics HSC Course

13C

8 Use the world map on page 394 to identify the cities at each of the following locations.
a (14N, 121E)
b (12S, 76W)
c (33S, 71W)

13C

9 Use the world map on page 394 to give the approximate coordinates of each of the following
cities.
a Madrid
b Singapore
c Hobart

13D

10 The points X and Y on the Earths surface have coordinates (32N, 120E) and (26S,
120E). Calculate the angular distance between X and Y.

13D

11 A ship gives its coordinates as (56N, 14W) and is sailing to a port at (40N, 14W).
a Calculate the angular distance through which the ship must sail to reach port.
b Calculate the distance the ship must sail, in nautical miles.
c Use 1 M = 1.852 km to calculate the distance, correct to the nearest kilometre.

13D

12 The angular distance between two points on the same great circle is 120. Calculate the time
that it would take to sail between these points at a speed of 48 knots.

13D

13 Santiago has approximate coordinates (33S, 70W), while Santo Domingo has approximate
coordinates (18N, 70W).
a Calculate the distance between Santiago and Santo Domingo:
i in nautical miles
ii correct to the nearest kilometre.
b Calculate the time taken to fly from Santiago to Santo Domingo at a speed of 480 knots.

13D

14 The coordinates of Perth are approximately (32S, 115E), while Hong Kong is
approximately at (22N, 115E). Taking the radius of the Earth to be 6400 km, use the arc
length formula to calculate the distance between Perth and Hong Kong, correct to the
nearest 100 km.

Chapter 13 Spherical geometry

409

15 Calculate the time difference between each of the following cities.


a Sydney (GMT +10) and Istanbul (GMT +2)
b Perth (GMT +8) and New York (GMT 3)
c Ottawa (GMT 5) and Fiji (GMT +12)

13E

16 In Dhahran (GMT +4) the time is 10.00 pm on Wednesday. Calculate the time in Tokyo
(GMT +9).

13E

17 Ann is on a skiing holiday in Winnipeg, Canada (GMT 6). She needs to call her parents at
7.30 pm on Tuesday night, Sydney time. At what time should she make the call from
Winnipeg?

13E

18 Kingston, Jamaica is at approximately (18N, 75W) while Oslo, Norway is at


approximately (60N, 15E). Calculate the time:
a in Oslo when it is 5.00 am in Kingston
b in Kingston when it is 5.00 pm in Oslo.

13E

19 A plane is flying from Munich (48N, 15E) to New York (41N, 75W). The flight departs
Munich at 6.00 pm and takes 7 hours. Calculate the time of arrival in New York.

13E

Practice examination questions


1 multiple choice
A circle has a diameter of 12 cm. An arc is drawn on the circumference of the circle such that
the arc subtends an angle of 45 at the centre of the circle. The length of the arc is closest to:
A 4.7 cm
B 9.4 cm
C 14.1 cm
D 37.7 cm
2 multiple choice
A great circle on the surface of a planet has a circumference of approximately 10 700 km. The
diameter of the planet would be closest to:
A 850 km
B 1700 km
C 3400 km
D 6800 km
3 multiple choice
Point X on the Earths surface has coordinates (69S, 12E), while point Y is at (8S, 12E).
The distance between X and Y is:
A 61 M
B 77 M
C 3660 M
D 4620 M
4 multiple choice
The coordinates of two points, M and N, on the surface of the Earth are (45N, 45W) and
(30S, 60E). Which of the following statements is correct about the time difference between
M and N?
A M is 5 hours behind N.
B M is 5 hours ahead of N.
C M is 7 hours behind N.
D M is 7 hours ahead of N.

410

Maths Quest General Mathematics HSC Course

5 The city of St Petersburg is at approximately (60N, 30E), while the city of Johannesburg
has approximate coordinates of (25S, 30E).
a Calculate the angular distance between St Petersburg and Johannesburg.
b Taking the radius of the Earth to be equal to 6400 km, calculate the distance between
St Petersburg and Johannesburg, correct to the nearest 100 km.
c In Sydney (30S, 150E) daylight saving time applies. Calculate the time difference
between St Petersburg and Sydney.

CHAPTER

test
yourself

13

6 The approximate coordinates of Tokyo are (36N, 140E), while San Francisco is at
approximately (36N, 120W).
a Do San Francisco and Tokyo lie on the same great circle or the same small circle? Explain
your answer.
b An aeroplane takes 8 hours to fly between Tokyo and San Francisco. If a plane leaves
Tokyo at 10.00 pm on Saturday, Tokyo time, what day and time will it arrive in San
Francisco?
c If the return flight leaves San Francisco at 8.00 am Tuesday, what day and time will it
arrive in Tokyo?

Glossary

411

Glossary
Annuity A form of investment involving
regular periodical contributions to an account.
On such an investment, interest compounds at
the end of each period and the next contribution to the account is made.
Annulus The area between two circles that
have the same centre (concentric).
Area chart When line graphs are stacked on
top of each other so that the area between
each of the lines can be used to compare each
data set.
Asset An item that is of value to its owner.
Causality When the occurrence of one
variable causes another. For example there is
a strong positive correlation between a
persons shirt size and shoe size but one does
not cause the other. On the other hand, there
is a strong positive correlation between the
amount of a Lottery jackpot and the number
of tickets sold. In this case, it would seem
that one does cause the other.

Cosine rule A trigonometric formula used in


non-right-angled triangles that allows:
a side length to be found when given the
two other side lengths and the size of the
included angle
an angle to be found when given the three
side lengths of the triangle.
Cubic function A function where the
independent variable is raised to the power of
3. A cubic function is in the form y = x3.
Declining balance method The depreciation
of an item is calculated as a percentage of the
value at the beginning of the period.
Depreciation The reduction in value of an
asset.
Ellipse An oval shaped figure. The shortest
distance from the centre of an ellipse to the
circumference is called the semi-minor axis
and the longest distance from the centre to
the circumference is called the semi-major
axis.

Compass radial survey A survey taken from


a central point in a field. Critical points are
sighted from this central point and radial lines
are drawn. The distance from the central
point to each critical point is then measured
and the compass bearing of each radial line is
noted on the survey.

Expected outcome The number of times that


an outcome can be expected to occur in a
given number of trials.

Correlation A statement reflecting the


relationship between two variables. A positive correlation indicates that as one variable
increases so does another, while a negative
correlation indicates that as one variable
increases, the other decreases.

Extrapolate To extend a graph so as to make


predictions about future trends.

Correlation coefficient A figure between 1


and 1 which indicates the strength of a
correlation. The closer to 1 the coefficient is,
the stronger the positive correlation; the
closer to 1 the coefficient is, the stronger the
negative correlation. A coefficient close to
zero indicates no correlation or, at best, a
very weak correlation.

Exponential function A function where the


independent variable is in the exponent
(index). An exponential function is of the
form y = ax or y = b(ax).

Financial expectation The average financial


result from a situation where money will be
gained or lost.
Flat rate loan A loan where the interest is
calculated using only the initial amount
borrowed.
Future value of an annuity The value of an
annuity at the end of a given period of time.
Great circle A circle of the greatest possible
diameter that can be drawn on the surface of
a sphere.

412

Maths Quest General Mathematics HSC Course

Greenwich Mean Time (GMT) The standard time in Greenwich which is used as the
basis for calculating the time in all other parts
of the world.
Greenwich Meridian The meridian of longitude from which angular distances in the
eastwest direction are measured. Using the
longitude calculated from the Greenwich
Meridian, time in different places on the
Earths surface is calculated.
Hyperbolic function A function where the
independent variable is in the denominator of
the function. A hyperbolic function is of the
a
form y = --- .
x
International Date Line The meridian of
longitude opposite to the Greenwich Meridian.
The International Date Line is, however, bent
for convenience. When crossing the International Date Line, the date changes.
Interpolate Drawing a graph using data
found at the end points.
Latitude The angular distance of a point on
the Earths surface either north or south of the
equator.
Line of best fit A line drawn on a scatterplot
that passes through or is close to as many
points as possible.
Meridian of longitude A line on the Earths
surface that runs from the North Pole to the
South Pole. Each meridian of longitude is
measured by the number of degrees east or
west it is of the Greenwich Meridian.
Nautical mile (M) A distance on the surface
of the Earth that is equal to an angular distance
of 1 minute on a great circle. 1 M = 1.852 km.
Normal distribution Occurs when a data set
of scores is symmetrically distributed about
the mean.
Offset survey A survey where critical points
are measured when they are sighted at right
angles to a traverse line.
Outlier A score in a data set which is an
extreme value. The outlier can be much
greater or much less than all other scores in
the data set. The effect of an outlier is to
greatly increase or decrease the mean such
that the mean is no longer a reliable indicator
of a typical score in the data set.

Parallel of latitude A line on the Earths


surface parallel to the equator. Each parallel
of latitude is measured in terms of the angular
distance either north or south of the equator.
Plane table radial survey A survey taken
from a central point in a field where critical
points are sighted from this central point and
radial lines are drawn. The distance from the
plane table to each critical point is then
measured and the angle between the radial
lines is measured with a protractor.
Present value of an annuity The single sum
of money which, if invested at the present
time, will produce the same financial outcome as an annuity.
Quadrant A quarter of a circle.
Quadratic function A function where the
independent variable is raised to the power of
2. A quadratic function will take the form
y = ax2 or y = ax2 + bx + c.
Reducing balance loan A loan where the
interest is calculated periodically on the outstanding balance at that point in time.
Salvage value The current value of an item
that has depreciated in value.
Scatterplot A graph that shows two
variables, one on each axis, and their relationship by plotting the points generated by each
data pair.
Sector The area between any two radii of a
circle.
Simulation An experiment where a computer
or other device replicates the results that
would be obtained if an experiment were performed in reality.
Sine rule A trigonometric formula used in
non-right-angled triangles that allows:
a side length to be found when given one
other side length and the size of two angles
an angle to be found when given two side
lengths and one non-included angle.
Skewness The distribution of a set of scores
in a data set.
(a) Positively skewed more scores are
gathered at the lower end of the
distribution.

Glossary

(b) Negatively skewed more scores are


distributed at the upper end of the
distribution.
Small circle A circle that is drawn on the
surface of a sphere that is of a smaller diameter than a great circle.
Standardised score (also called z-score)
A figure which represents the distance of a
score from the mean in terms of the standard
deviation. The standardised score is the
number of standard deviations that a member
of the data set is from the mean.
Straight line method The depreciation of an
item is calculated by subtracting a constant
amount from the value each year.

413

Term of the loan The length of time that a


loan is repaid over.
3-median regression line A line of best fit
that is extrapolated to make predictions about
data. This is found by using the 3-median
method.
Two-way table A 2-dimensional table that
displays the results of an experiment in terms
of two variables.
z-score see standardised score

414

Maths Quest General Mathematics HSC Course

FORMULA SHEET
Simple interest

Sine rule

I = Prn
P = initial quantity
r = percentage interest rate per period,
expressed as a decimal
n = number of periods

a
b
c
------------ = ------------ = ------------sin A sin B sin C

Area of a triangle
A = 1--- ab sin C
2

Area of a sector

Cosine rule
c2 = a2 + b2 2ab cos C

A = --------- r
360
= number of degrees in central angle

Area of an annulus
A = (R2 r2)
R = radius of outer circle
r = radius of inner circle

or
2

a +b c
cos C = ---------------------------2ab

Mean of a sample

Area of an ellipse
A = ab
a = length of semi-major axis
b = length of semi-minor axis

Simpsons rule for area approximation


h
--- (df + 4dm + dl)
3
h = distance between successive measurements
df = first measurement
dm = middle measurement
dl = last measurement
A

Surface area

x
x = -----n
fx
x = -------f
x = mean
x = individual score
n = number of scores
f = frequency

Probability of an event
The probability of an event where outcomes are
equally likely is given by:
number of favourable outcomes
P(event) = ---------------------------------------------------------------------------total number of outcomes

Future value (A) of an annuity


2

Sphere
A=4 r
Closed cylinder A = 2 rh + 2 r2
r = radius
h = perpendicular height

Volume
Cone
V = --1- r 2h
3
Cylinder
V = r 2h
Pyramid
V = 1--- Ah
3
Sphere
V = 4--- r 3
3
r = radius
h = perpendicular height
A = area of base

(1 + r ) 1
A = M ---------------------------r
M = contribution per period, paid at the end of
the period

Compound interest
A = P(1 + r)n
A = final balance
P = initial quantity
n = number of compounding periods
r = percentage interest rate per compounding
period, expressed as a decimal

Formula sheet

415

FORMULA SHEET
Present value (N) of an annuity
n

(1 + r ) 1
N = M --------------------------n
r(1 + r )

Formula for a z-score


xx
z = ----------s
s = standard deviation

or
A
N = ------------------n(1 + r )

Straight-line formula for depreciation


S = V0 Dn
S = salvage value of asset after n periods
V0 = purchase price of the asset
D = amount of depreciation apportioned per
period
n = number of periods

Declining balance formula for


depreciation
S = V0(1 r)n
S = salvage value of asset after n periods
r = percentage interest rate per period,
expressed as a decimal

Gradientintercept form of a straight line


y = mx + b
m = gradient
b = y-intercept

Gradient of a straight line


vertical change in position
m = --------------------------------------------------------------------horizontal change in position

Arc length of a circle


l = --------- 2 r
360
= number of degrees in central angle

417

Answers
CHAPTER 1 Credit and
borrowing

4 a

Are you ready?


1 a
d
g
2 a
d
3 a
4 a
b
c
d
e
5 a
c
6 a

0.40
b 0.12
c 0.08
0.024
e 0.003
f 0.075
0.0025
h 0.0002
$1500
b $369.50 c $9250
$16.20
e $410
f $46.80
$2520
b $7475
c $16 875 d $2340
n = 5, r = 6% = 0.06
n = 8, r = 4.5% = 0.045
n = 12, r = 2.2% = 0.022
n = 120, r = 0.6% = 0.006
n = 30, r = 0.058% = 0.000 58
$9881.07
b $108 184.55
$5439.07
d $194 445.84
d = 10
b A = 45
c s = 32.75

Exercise 1A Flat rate interest


1 a $700
b
d $2850
e
2 $1584
4 a $4060
b
d $308.25
e
5 a $1650
b
d $6193
6 a $1600
b
7 a $800
b
8 a $2000
b
d $8160
e
9 $43.33
10 B
13 a $2400
b
c $16 319.88
d
14 15%

$1200
c $7500
$390
3 $5000
$21 330 c $1803.75
$275 000
$3850
c $693
$6600
c $137.50
$2800
c $53.85
$6000
c $2160
$226.67
11 C
12 8%
$9600
15%

Exercise 1B Home loans


1 a $800
2 a $1125, $179 456.38
c $1121.60, $178 909.36
3
Principal

Principal
($)

Interest
($)

Balance
($)

255 000.00

1912.50

254 618.19

254 618.19

1909.64

254 233.52

254 233.52

1906.75

253 845.96

253 845.96

1903.85

253 455.49

253 455.49

1900.92

253 062.10

253 062.10

1897.97

252 665.75

252 665.75

1894.99

252 266.44

252 266.44

1892.00

251 864.13

251 864.13

1888.98

251 458.80

10

251 458.80

1885.94

251 050.43

11

251 050.43

1882.88

250 639.00

12

250 639.00

1879.79

250 224.48

Month

Principal
($)

Interest
($)

Balance
($)

255 000.00

1912.50

254 412.50

254 412.50

1908.09

253 820.59

253 820.59

1903.65

253 224.25

253 224.25

1899.18

252 623.43

252 623.43

1894.68

252 018.11

252 018.11

1890.14

251 408.24

251 408.24

1885.56

250 793.80

250 793.80

1880.95

250 174.76

Month

b $79 950
b $543.62
d $547.02

($)

Interest ($)

Balance
($)

150 000.00

1200.00

149 791.99

149 791.99

1198.34

149 582.32

149 582.32

1196.66

149 370.96

250 174.76

1876.31

249 551.07

149 370.96

1194.97

149 157.92

10

249 551.07

1871.63

248 922.70

149 157.92

1193.26

148 943.18

148 943.18

1191.55

148 726.71

11

248 922.70

1866.92

248 289.62

148 726.71

1189.81

148 508.51

12

248 289.62

1862.17

247 651.79

148 508.51

1188.07

148 288.57

148 288.57

1186.31

148 066.87

10

148 066.87

1184.53

147 843.40

Month

c $2572.69
5 $243 123
6 a $302 308.80

b $241 500 c $60 808.80

answers

Answers

1A
1B

answers

418

Answers

7 A
8 B
9 a $112 034
b $86 072
c $61 789.40
d $39 329.60
10 a Smith family pays $24 000; Jones family pays $36 000
b

Smith family
Month

Principal ($)

Interest ($)

Balance ($)

Principal ($)

Interest ($)

Balance ($)

200 000.00

1583.33

199 583.33

200 000.00

1583.33

198 583.33

199 583.33

1580.04

199 163.37

198 583.33

1572.12

197 155.45

199 163.37

1576.71

198 740.08

197 155.45

1560.81

195 716.27

198 740.08

1573.36

198 313.44

195 716.27

1549.42

194 265.69

198 313.44

1569.98

197 883.42

194 265.69

1537.94

192 803.62

197 883.42

1566.58

197 450.00

192 803.62

1526.36

191 329.98

197 450.00

1563.15

197 013.14

191 329.98

1514.70

189 844.68

197 013.14

1559.69

196 572.83

189 844.68

1502.94

188 347.62

196 572.83

1556.20

196 129.03

188 347.62

1491.09

186 838.70

10

196 129.03

1552.69

195 681.72

186 838.70

1479.14

185 317.84

11

195 681.72

1549.15

195 230.87

185 317.84

1467.10

183 784.94

12

195 230.87

1545.58

194 776.44

183 784.94

1454.96

182 239.91

c $12 536.53

10 Quick Questions 1
1
3
5
7
9

Jones family

$420
$7025
$5525
$8342.75
$1640

2
4
6
8
10

$1135.20
$975
$2817.75
$231.74
$265 577

Exercise 1C The cost of a loan


1 11.6%
2 a 11.6%
b 8.32%
c 15.2%
d 10.6%
e 12.2%
3 a 8.32%
b 8.66%
c 9.01%
d 9.39%
e 11.6%
f 18.3%
4 a $213 996
b $128 996 c 6.0704%
5 9.01%
6 Loan 1
7 a $231 546
b $200 745.60
c $145 593.60
8 Loan 2 they will save $6041
9 C
10 a $341 376
b $337 578
11 D
12 a $562 279.20 b 6.25%
c 5.8%

Exercise 1D Credit cards


1 $136.50
2 a $175
d $10
3 $27.75

b $59.73
e $46.03

c $24.64

4 $18.13
5 a $1800
b $31.50
c $1831.50
d $1863.55
6 a $1767.50
b $88.38
c $20.26, $2046.68
7 a $296.40, $256.40
b $4.12, $260.52
8 a 0.0452%
b $19.34
c $73.34
d $1411.37
9 a $1000
b $15
c $530.23
d $17.50
e $11.87
f $535.55
g The card with the interest-free period

10 Quick Questions 2
1
3
5
6

$2835
2 160% p.a.
$299 191.20
4 10.3%
6.0%
The customer has until the due date of the next
statement to pay the whole balance before any
interest is charged.
7 $164.88
8 $41.96
9 $32.98
10 $61.28

Exercise 1E Loan repayments


1 $674.25
2 a $90.46
b $341.25 c $819.84
d $1101.00
e $1515.54
3 a $400
b $3600
c $123.05
4 They will not need to increase their repayments.
5 a $1510.20
b $1620.14
6 Yes. The repayment is $744 and the most he can
afford is $750.

7 a $7000
8 a $733.40
9 a $2895
d $19 740
10 a $1696.80
d 4.481%

b $1750
c $178 000
b $174.80
b $868 500 c $493 500
e 5.262%
b $509 040 c $269 040
e i 4.3% ii 4.12%

Chapter review
1 a $1120
d $39.60
2 $6760
3 $191.02
4 6.15%
5 a $1250
6 a
Month
1
2
3
4
5
6
7
8
9
10
11
12
b

7
8
9
10
11
12
13

b $7187.50 c $1281.60
e $12 285.00

b $124 873.64
Principal
($)
130 000.00
129 779.30
129 557.12
129 333.47
129 108.32
128 881.67
128 653.51
128 423.83
128 192.62
127 959.87
127 725.56
127 489.70

Interest
($)
866.67
865.20
863.71
862.22
860.72
859.21
857.69
856.16
854.62
853.07
851.50
849.93

Balance
($)
129 779.30
129 557.12
129 333.47
129 108.32
128 881.67
128 653.51
128 423.83
128 192.62
127 959.87
127 725.56
127 489.70
127 252.26

Principal
Interest
Balance
Month
($)
($)
($)
1
130 000.00
866.67
129 366.67
2
129 366.67
862.44
128 729.11
3
128 729.11
858.19
128 087.31
4
128 087.31
853.92
127 441.22
5
127 441.22
849.61
126 790.83
6
126 790.83
845.27
126 136.10
7
126 136.10
840.91
125 477.01
8
125 477.01
836.51
124 813.52
9
124 813.52
832.09
124 145.61
10
124 145.61
827.64
123 473.25
11
123 473.25
823.16
122 796.40
12
122 796.40
818.64
122 115.05
c $5137.21
a $596 844
b $18 884
a 7.25%
b 13.70%
c 25.65%
d 14.11%
a $18 223.20 b $4723.20 c 7%
Loan 2
$21.15
$17.00
$3.21

14 a
c
15 a
c
16 a

b
d
b
d
b

0.0534%
$102.98
$316.75
$2369.11
$2453.49

419

$34.82
$32.82
$599.40
$5100
$2618.06

Practice examination questions


1 C
2 C
3 A
4 D
5 a $4140
b $6292.80
c $30.25
d 15.76%
6 a $1678.50
b $402 840
c $1825.34
d $422 193.55
7 a $13.78
b $261.72 c $4.58
d The card with this interest-free period is cheaper as
the card without the interest-free period would
have charged $6.75 interest.

answers

Answers

CHAPTER 2 Further applications


of area and volume
Are you ready?

1 a 50.3 cm2
b 1206.9 cm2
c 63.6 cm2
d 304.8 cm2
2
2 a 20.25 cm
b 40.33 cm2 c 52.44 cm2
3 a 5832 cm2
b 2376 cm2 c 624 cm3
3
4 a 3820 cm
b 7238.2 cm3
5 163.3 cm3
6 a 14.5 cm15.5 cm
b 8.25 m8.36 m
c 4750 km4850 km

Exercise 2A Area of parts of the


circle
1 128.7 cm2
2 a 254.47 cm2
c 172.03 m2
e 46.32 m2
3 176.7 m2
4 40.7 m2
5 a 14.16 cm2
c 92.33 m2
e 796.39 m2
6 827.3 cm2
7 339.3 cm2
8 a 239 cm2
9 a 2.5 m
10 188.5 cm2
11 a 5892 mm2
12 B
13 B
14 A
15 a 78.5 m2
16 a 314.16 cm2

b 3421.19 mm2
d 2206.18 cm2
f 113.85 m2
b 6451.26 mm2
d 110.79 mm2
f 955.67 mm2
b 240 m2
b 3.5 m

c 18 100 mm2
c 18.8 m2

b 308 m2

c 924 cm2

b 122.5 m2
b 5 cm

c 25.5 m2
c 157.08 cm2

Exercise 2B Area of composite


shapes
1 248 m2
2 a 222 cm2
d 228.5 cm2
3 a 8 cm

b 375 cm2 c 335 cm2


e 44.6 cm2 f 130.3 cm2
b 84 cm2

1B
2B

answers

420
4
5
6
7
8
9

Answers

a 5.3 m
a 120 m2
a 174 cm2
A
B
a

b 31.8 m2
b 168 cm2 c 6658 mm2
b 510 m2
c 4032 mm2

b 250 m

50 m
10 a 80 m2
c 12.2 cm2
e 4600 mm2
11 2513 m2
12 a
1m
25 m

c 3582.5 m2

b 109.7 cm2
d 58.4 cm2
f 20.1 m2

11 B
12 a 565 cm2
13 a 113 cm2
c 28 cm2

b 452 cm2
b r = 3 cm, h = 18 cm
d 368 cm2 e 339 cm2

Exercise 2E Volume of composite


solids

1 a 178 cm2
b 712 cm3
2 a 700 cm3
b 3000 cm3 c 3720 cm3
d 2.128 m3
e 12.75 m3 f 18 m3
3 a 8 m3
b 2 m3
c 10 m3
4 a 22 619 cm3 b 6032 cm3 c 28 651 cm3
5 a 19 000.4 cm3
b 103.7 cm3
c 157 724.9 cm3
6 B
7 D
8 a 16 875 cm3 b 16.875 L
9 a
2.5 cm

40 m
2

b 1000 m
c 134 m2

10 Quick Questions 1
1
3
5
7
9

452.4 cm2
171.3 cm2
306 cm2
1428.3 m2
17 121.8 mm2

2
4
6
8
10

31.2 m2
13 939.2 mm2
625 cm2
1147.6 cm2
670.9 cm2

Exercise 2C Simpsons rule


1 a 30 m
b df = 40 m, dm = 9 m, dl = 18 m
c 940 m2
2 a 1296 m2
b 1560 m2 c 936 m2
2
3 a 620 m
b 880 m2
c 1500 m2
4 a 2535 m2
b 1184 m2 c 2934 m2
5 C
6 B
7 2514 m2
8 a 2970 m2
b 11 840 m2 c 1386 m2
9 a 768 m2
b 640 m2
2
10 2484 m

Exercise 2D Surface area of


cylinders and spheres
1 502.7 cm2
2 a 282.7 cm2
d 785.4 cm2
3 37.4 m2
4 452 cm2
5 a 395.4 cm2
c 2199.1 cm2
e 640.9 cm2
6 a 231 cm2
7 113 cm2
8 a 804.2 cm2
c 2463.0 cm2
e 145.3 cm2
9 5542 cm2
10 A

5 cm

d $2345

b 18.7 m2
c 3141.6 cm2
e 437.4 cm2 f 54.9 m2
b
d
f
b

1616.5 cm2
367.9 cm2
52.8 m2
154 cm2

b 55.4 cm2
d 12.6 m2
f 40.7 m2

b 50 mL
10 a 8.64 m2
11 a 8 cm
12 5.76 g

c 20
b 86.4 m3
b 332 cm3

10 Quick Questions 2
1
3
5
7
9

58.1 cm2
129.2 cm2
77.47 cm2
2073 cm2
2100.6895 cm3

2
4
6
8
10

c 35%

226.19 cm2
452 cm2
3710 m2
21.237 cm2
536 cm3

Exercise 2F Error in measurement


1 a 1152 cm3
b
2 a 0.05 cm
b
c 2.4 cm2
d
3 a 4096 mm3
b
c 4492.125 mm3
d
e 1536 mm2
f
g 1633.5 mm2
h
4 a 302 cm3 b 212 cm3 c
5 24%
6 a 11.5 m3
b 1.3 m3
2
d 24.6 m
e 1.8 m2
7 a 382 L
b 89 L
8 a 39 032 cm3
b
c 60 cm 40 cm 20 cm
d 23%
e
9 a 27.72 m2
b 28 L
10 a 17 m 10 m
b
c $5142.50
d

Chapter review
1 a
c
2 a
3 a
4 a

43.0 cm2
2.3 m2
2215.9 mm2
7147.1 mm2
4241.2 mm2

118.625 cm3
181.5 cm2
1.3%
3723.875 mm3
9.7%
1441.5 mm2
6.3%
414 cm3 d 37%
c 11.1%
f 7.3%
c 23%
7890 cm2
11.5%
c 1L
170 m2
$221.43

b 8494.9 mm2
b 18.1 m2
b 37.7 m2
b 329.5 m2

c 323.2 cm2
c 2723.8 cm2
c 19.8 cm2

705 cm2
a 5.75 m2
1722 m2
a 840 m2
4190 m2
2010 m2
a 747.7 cm2
488 cm2
a 314 cm2
3.438 m3
a 5797 cm3
c 1260 cm3
16 5343.85 cm3
17 a 0.25 cm
18 a 500 mL

2 a
b 27 cm2

c 1804.94 cm2

b 2672 m2

c 5548 m2

b 728.8 cm2 c 11.7 m2


c 76 454 mm2
3

b 2145 cm3 c 9.7%


b 20%

b 30.4 M
c 56.3008 km

B
x

43 M

45

4 B
7 a

5 B
d
8M

b 14 283 cm

45
x

3 a

b 72 m2

b 34 km
34 km

5
6
7
8
9
10
11
12
13
14
15

CG
70

290

b 19.30 km c 20.62 m
b 56
c 37
b 53
c 47
b 27 M
d 1.92 M
b 46
c 7
b x = 1.5
d x = 22.5

14.26 m
44
63
7.408 km
2222.4 m
32
x = 15
x = 14.25

Exercise 3A Review of right-angled


triangles
1 a 12.2 cm
d 409.9 mm
2 a 27
3 62 m
4 a R

b 110.9 mm c 10.0 m
e 29.8 m
f 19.3 cm
b 56
c 57

28

5 km

b 2.659 km
5 15
6 a

b 16
239 m

b 25

Exercise 3B Bearings
b 2.970 km

x
4.2 km

3.5 km

Exercise 3C Using the sine rule to


find side lengths
a
b
c
1 a ------------ = ------------ = ------------sin A
sin B
sin C
x
y
z
b ------------ = ------------ = -----------sin X
sin Y
sin Z
p
q
r
c ------------ = ------------- = -----------sin P
sin Q
sin R
2 a 14.8 cm
b 1.98 km
3 a 10.0 mm
b 22.1 cm
4 B
5 C
6 9.8 cm
7 27.0 m
8 37.8 m
9 a B
b 43.2 m

45

c 112 mm
c 39.6 km

c 33 m

34
49

7 a 28.01 m
8 52
1 a

4 km

c 5.663 km

65 m

11 A
b 229

13 342

Are you ready?


a
a
a
a
c
5 a
6 a
c

b 11.1 km
3.8 km

8 437 km
9 a CP

10 D
12 a

1
2
3
4

6 6.4 km
b 13.9 M

60

Practice examination questions


1 D
2 B
3 A
4 C
5 a 12.6 m2 b 62.8 m3 c 35 300 L d 151 m2
6 a 292 m2
b 233 600 m3 c 6.25%

CHAPTER 3 Applications of
trigonometry

421

answers

Answers

N 20 m

10 a WYX = 40
80
XY
In LWYX ---------------- = ---------------sin 40 sin 30
80 sin 30
\ XY = -----------------------sin 40
h
b In LYXZ sin 70 = -------XY
80 sin 30
\ h = ------------------------ sin 70
sin 40
c 58.5 m

2B
3C

Answers

Exercise 3D Using the sine rule to


find angles
1 a 43
d 75
2 B
5 20
7 a 57
8 54

b
e
3
6

c 27
f 2
4 38

34
37
B
84

2 157 mm
5 860 mm
8 43

26.9 m
60
15 cm
32

3 69.8 cm
6 110 m
9 50

1 a Area = 1--2- ab sin C

b Area = 1--2- yz sin X

c Area = 1--2- am sin G

c Area =

13.6

14

b 25

10 Quick Questions 2
1 452 mm

1
--- bh
2
2

a 42.4 cm
a 133 cm2
D
B
3.865 cm2
a
A

b Area = --12- bh
1
--2

d Area = bh
b 3522.6 mm2 c 4660.9 mm2
b 555.4 cm2
c 608 cm2

b 12.2 m2

70
D

5.2 m

b 59 cm2

9 a 72
10 710 m2

Exercise 3F Using the cosine rule to


find side lengths
1 a a2 = b2 + c2 - 2bc cos A
b r2 = p2 + q2 - 2pq cos R
c n2 = l2 + m2 - 2lm cos N
2 a 8.05 m
b 14.3 cm c 12.0 m
3 a 185.1 cm
b 14.4 m
c 104.4 mm
4 D
5 A
6 C
7 14.4 cm
8 1140 mm
9 6.742 km
10 40 m
11 a
b 8.5 cm c 15.5 cm
5.3

cm

134

46
11.3 cm

12 55 cm

Exercise 3G Using the cosine rule to


find angles
b2 + c2 a2
1 a cos A = ---------------------------2bc
p2

r2

5 12.7 m
7 c2 = a2 + b2 - 2ab cos C

10 49

Exercise 3H Radial surveys


1
2
3
4
5
6
7
8

102 m
a 286 m
12 993 m2
a 4923 m2
a 85
389 m
a 214 m
a 2719 m2

b 432 m

c 540 m

b 8861 m2
b 75 m

c 18 382 m2

b 531 m
c 301 m
b 12 425 m2 c 5809 m2

Chapter review
1
2
3
4
5
6
7
8
9
10
11
12
13
14
15
16
17
18
19
20

a 8.6 cm
a 61
3087 m
a 106 m
56.569 km
21.7 km
a 297
a 1.67 cm
12.4 cm
a 52
809 cm2
3000 m2
a 8.64 m
84.0 cm
985 m
a 60
34
29
a 284 m
a 783 m

b 13.6 km
b 66

+
b cos Q = ---------------------------2 pr

20 m

25 m

c 45
c 10

c 11.2 cm
c 45

b 28

b 117
b 81.7 mm

c 9.81 km

b 21

c 68

b 8.80 m

c 11.8 cm

b 112

c 139

b 4020 m2
b 34 910 m2

Practice examination questions


1 D
2 B
3 A
5 a
b 13
7m

q2

a2 + m2 p2
cos P = -----------------------------2am
2 a 85
b 83
3 a 103
b 137

2 0.782 m
a
b
c
4 ------------ = ------------ = ------------sin A
sin B
sin C
6 32
a2 + b2 c2
9 cos C = ---------------------------2ab

8 210 m

2 a Area = --12- ab sin C

7 23

b 73

3 40

Exercise 3E Area of a triangle

3
4
5
6
7
8

6 42

10 32
11 a 39
12 148

10 Quick Questions 1
1
4
7
10

4 A
5 D
8 81, 54, 44
9 a W 9.2 X

b 63

2.5

answers

422

4 D

6 a 15

d
7 a

CHAPTER 4 Interpreting sets of


data
Are you ready?

25
20
15
10

5
0

8 13 18 23 28
Class centre

5 a 7
b 1
c Mean = 27.3, sn = 1.7
6 a 6
b i 9 ii Lower quartile = 8, upper quartile = 10
iii 2
c i Mean = 9.04 ii sn = 1.44
d
6

7 8

9 10 11 12

7a

1 a Mean = 5
b Mean = 19.5
c Mean = 3.31
2 a 3
b 12 and 30
c 3
3 a Median = 4.5
b Median = 17
c Median = 3
4 a 6
b 21
c 4
5 a 3.5
b 15
c 1
6 a Sample standard deviation = 1.32
b Population standard deviation = 1.41
7 Key: 1 | 9 = 19
Stem Leaf
0 9
1 14599
2 0122336677788
3 012234789
4 0015
8

Crowd

Class
Cumulative
centre Frequency frequency

10 00015 000

12 500

15 00020 000

17 500

13

20 00025 000

22 500

19

25 00030 000

27 500

23

30 00035 000

32 500

26

Cumulative frequency

0 10 20 30 40 50 60 70

25
20
15
10

5
0

0
0
0 0
0
50 50 50 50 50
12 17 22 27 32

Exercise 4A Measures of location


and spread
1 a 1.6
b 1
2 a Mean = 49, median = 44, mode = no mode
b Mean = 3.4, median = 3.5, mode = 1
c Mean = 9.575, median = 9.7, mode = 9.8, 9.9
d Mean = 15.2, median = 15, mode = 13, 15
3 a 6.5
b 6.5
c 7
4 a
Class
Cumulative
Score centre Frequency
frequency
15

610

1115

13

14

1620

18

21

2125

23

24

2630

28

25

b 14.6

Cumulative frequency

Median 14

BT
30
30 sin 20
In LABT ---------------- = ---------------- \ BT = -----------------------sin 20 sin 15
sin 15
h
In LTBC sin 35 = ------BT
\ h = BT sin 35
30 sin 20
h = ------------------------ sin 35
sin 15
22.7 m
100.3 m
b
1625 m2

423

answers

Answers

Class centre

c 9500
d x = 21 000, sn = 6300
8 a Team A = 16, Team B = 16
b i Team A = 10, Team B = 40
ii Team A = 4, Team B = 20
iii Team A = 3.1, Team B = 12.5
c Both teams had the same mean score. However,
Team A was more consistent as shown by a lower
reading in all three measures of spread.
9 B
10 A
11 A
12 C
13 a i 80
ii 80
iii 80
b i 70
ii 79
iii 80
c The outlier had a great effect on the mean, a
small effect on the median and no effect on the
mode.
14 The outlier will greatly increase the mean.
15 A
16 a Mean = $647.00
Median = $397.50
Mode = $397.50
b i Increase
ii None
iii None
c i The median or mode as it is a lower figure,
making it look as though they deserve a rise.
ii The mean as it is a much greater figure, making
it look as though the employees are well paid.

3D
4A

Answers

Exercise 4B Skewness

Frequency

1 a Yes
2 a No
3 a

4
5

b 3
c Yes, both equal 3
b 59 and 2024
c No
b Yes

6
5
4
3
2
1
0

3 a Ford: median = 15, Holden: median = 16


b Ford: range = 26, Holden: range = 32
c Ford: interquartile range = 14,
Holden: interquartile range = 13.5
d
Ford

Holden

0 4 8 12 16 20 24 28 32 36 40 Scale

0 1 2 3 4 5
Number of goals

0 and 5
12
Negatively skewed
Positively skewed
Positively skewed
Frequency

6
7

c
a
a
c
a
a

25
20
15
10
5
0

d Yes, both equal 2.5


b Positively skewed
b Symmetrical
b Negatively skewed
b Negatively skewed

Brand A
Brand B

0 2 4 6 8 10 12 14 16 18 20 22 24 26 Scale

5 a Key: 85 = 8.5
Station A
Station B
8 7 5 8 1 2 4 5 7 7 8 9
9 7 5 4 3 2 2 1 0 9 0 2
1105
112 4
b
Station A

0 0 0 0 0
2 3 4 5 6
11 21 31 41 51

Number of goals

8 A
10 a

Station B

9 A
b No
Frequency

answers

424

10
8
6
4
2
0
50 00 50 00 50
1 11 11 12 12
5 10 15 20

7 7.5 8 8.5 9 9.5 10 10.5 11 11.5 12 Scale

6 a Team A
b Team A: range = 60, Team B: range = 90
c Team A: interquartile range = 13,
Team B: interquartile range = 11
7 a
Emad

Number of people

c
d
11 a
b
c

151200 and 201250


Negatively skewed
Chemistry is symmetrical.
Maths is negatively skewed.
Chemistry: mode = 4150 and 8190,
Maths: mode = 7180
d Maths, because there are more scores further
away from the centre of the distribution.
12 Check with your teacher.

10 Quick Questions 1
1
4
7
9
10

23.3
2 21.5
3 16
29
5 5
6 7.93
Positively skewed
8 Yes, 45 is an outlier.
Median, because the outlier inflates the mean.
The outlier makes the range very large. The outlier
also affects the mean. There is no change to the IQR.

Exercise 4C Displaying multiple data


sets
1 Key: 1.55 = 1.55
Boys
Girls
9 9 7 1.51 2 5 6 7 8 8
9 8 6 6 5 5 4 0 1.64 4 6 7 8 9 9
4 4 2 1 1.70
2 Key: 18 = 18
Team A
Team B
8 0 7 9
9 8 7 1
9 5 4 3 2 0 1 3 4 7
4 2 3 0 5 8
2 0 4 1 6

Larry

0 10 20 30 40 50 60 70 80 90 100 Scale

b Emad: range = 35, Larry: range = 24


c Emad: interquartile range = 12,
Larry: interquartile range = 14
8 C
9 a July
b 13
c 21.7
10 a Supermarket X, range = 111
Supermarket Y, range = 90
b Both supermarkets follow a similar pattern.
There are very few customers from midnight to
6 am. Then the number peaks between 10 am and
noon, remaining fairly constant until 8 pm, when
the number reduces.
11 Month
Sydney Melbourne Brisbane
January
February
March
April
May
June
July
August
September
October
November
December

12
12
13
12
12
12
10
10
10
12
11
12

8
7
9
12
14
14
15
16
15
14
12
11

13
14
15
11
10
8
7
7
7
9
10
12

Boys
Girls
1.4 1.45 1.5 1.55 1.6 1.65 1.7 1.75 1.8 1.85 1.9 Scale

b
c
d
e
3

4
5
6

7
8

9
10

11

12
13
14

Boys 1.64, girls 1.62


Boys 0.34, girls 0.18
Boys 0.14, girls 0.07
The spread of heights is much greater among
boys than among girls.
a Year 7: range = 0.4, Year 12: range = 0.26
b Year 7: interquartile range = 0.15,
Year 12: interquartile range = 0.11
c The range of heights is greater in Year 7 as
shown by the range and the IQR. The heights
become less spread by the time they get to
Year 12.
The pattern of software sales follows after the
pattern of hardware sales with a slight time delay.
a Southern
b Western
c Similar peaks and troughs
a 43.2%
b 1.9%
c 0.9%
d 2.6%
e More evident in males with three times the
number of drivers over the limit
a 90.5%
b 55.6%
c Yes, as a much greater percentage of games are
won with Ashley playing.
a 9.5%
b 9.7%
c i 48.7%
ii 51.3%
d There is no significant difference between the
city and country results.
a Chemistry, 69.25
b Physics, because of the lower standard deviation
a Point A: x = 61, sn = 4.27,
Point B: x = 58.8, sn = 12.06
b Point A because of the higher mean
c Point B because of the greater standard deviation
a Aaron: x = 38.1, Sunil: x = 39.3
b Aaron: range = 76, Sunil: range = 65
c Aaron: interquartile range = 16,
Sunil: interquartile range = 57
d Aaron is more consistent because although he
has a larger range this is caused by one outlier.
Aarons interquartile range is much less, showing
his consistency.
C (based on the interquartile range)
B
A

Matchbox A
Matchbox B
42 44 46 48 50 52 54 56 58 60 62 Scale

b Machine A: x = 49.96, sn = 2.90,


Machine B: x = 50.12, sn = 2.44
c Machine B has a lower standard deviation and so
is more dependable.

Chapter review
1 a 25.4
b 26
2 a
Number of
Cumulative
house calls
Frequency
frequency
0
1
1
1
6
7
2
8
15
3
9
24
4
6
30
5
2
32
b 2.593 75
c 3
d 3
3 a 80.6
b 84
c 20
d 24.4
4 x = 10.3 sn = 1.6
5 a
Waiting
Class
Cumulative
time
centre Frequency frequency
01 minute
0.5
1
1
12 minutes 1.5
4
5
23 minutes 2.5
10
15
34 minutes 3.5
13
28
45 minutes 4.5
9
37
56 minutes 5.5
3
40
b x = 3.35 sn = 1.17
c
Cumulative frequency

1 a English 66, Maths 63.5


b English 32, Maths 53
c The marks are more spread in Maths than in
English.
2 a

15 a

d
6 a
c
7 a
c
8 a

40
35
30
25
20
15
10
5
0

0.5 1.5 2.5 3.5 4.5 5.5


Waiting time (minutes)

1.75
29.5
b 34.5
Outlier reduces the mean greatly.
Yes
b Both are 17.5.
17 and 18
b Positively skewed
Frequency

Exercise 4D Comparison of
data sets

425

answers

Answers

12
10
8
6
4
2
0

1 2 3 4 5 6
Class centre

9 A histogram that shows more columns to the right of


centre than to the left.

4B
4D

Answers

D $25 000 J

N
O

$20 000
$15 000
$10 000
$5 000
$0

M
A

S
J

Day

Economy

Night

Price per minute

30
20
10

Omtus
Company

1 a 2300
2 a 5r
d a 5b
3 a r10
d 35q6
4 a d4

5 a a8
6 a z = 118
d v = 5 1--3-

Tel One

13 a Home: 23 000
b Home: 27 000
c Home: 19 000
d

Away: 16 000
Away: 19 000
Away: 9000

Scale

b 161.2
c 3120
d 375.9
b 10m
c 16x + 15
e 4y x
f 9m 8n
b 18a3
c 28p6
e 48m6
f 27r3s11
b 6m4
c 42x5
2

e 8s5

k
----8
c 16c8
c w = 13
f n = 34
f

b 64b3
b y = 24
e t = 14

7 a 2.5 104
c 4 105

b 2.36 108
d 2.6 1013

Exercise 5A Substitution
Home

Away

15 a
c
16 a
b
c
d

e Boys: x = $107, sn = 22.4,


Girls: x = $97, sn = 53.1
f Generally, the girls is more consistent. The range
and standard deviation for the girls is inflated by
the outlier so the best measure of consistency is the
interquartile range which is lower for the girls.
7 a Sharks: median = 24, Bulldogs: median = 24
b Bulldogs scores are more clustered around the
median and so can be seen to be more consistent.
c Negatively skewed
d x = 23.96, sn = 5.58

d 2q5

0
Telecomm

14 a
b
c
d

Girls

Are you ready?

b There is a peak in surf sales through summer,


while the ski sales are greatest in winter, with a
short peak occurring around Christmas.
12

Boys

CHAPTER 5 Algebraic skills and


techniques

M
A

20
40
60
80
10
0
12
0
14
0
16
0
18
0
20
0
24
0
26
0

10 a Key: 56 = 56
English
Maths
5 4 9
7 6 6 2 5 0 1 5 8
8 7 6 6 5 4 0 6 0 1 5 6 6 6 8 9
7 7 4 4 7 0 0 1 1 4
4 2 0 8 5
1 9 9
b English: median = 66.5, Maths: median = 66
c English: range = 46, Maths: range = 50
d English: interquartile range = 18.5,
Maths: interquartile range = 11.5
11 a
Surf sales
Ski sales

30
00
60
00
90
12 00
0
15 00
0
18 00
0
21 00
0
24 00
0
27 00
0
30 00
0
33 00
0
36 00
00
0

answers

426

Scale

2003: median = 7, 2004: median = 8


2004
2003
2004 Higher median, lower limit, lower
quartile and upper limit
9.7%
b 8.0%
No significant difference
English: x = 71, maths: x = 69.3
English: range = 43, maths: range = 37
English: sn = 11.6, maths: sn = 12.0
English, because of the lower standard deviation

Practice examination questions


1 B, C and D
2 A
3 C
4 B
5 B
6 a Boys: median = $105, Girls: median = $85
b Boys: range = 75, Girls: range = 200
c Boys: interquartile range = 35,
Girls: interquartile range = 20

1 32.16
2 a 30.75
d 120
3 26.4
4 29.8 m
5 181.5
6 a 1.41
d 82.8
7 126 cm2
8 101.25
9 6.3
10 a 15
d 6.0
11 5.2 cm
12 a 10.5

b 2327.5
e -154

c 27.1

b 12.06
e 254.04

c 137.26

b 16.2
e 30.5

c 8.5

b 16.0 cm2

Exercise 5B Algebraic manipulation


1 a
d
g
j
2 a
d
g
j

12a
b
41d
e
-7g
h
11j
k
11m - 9
b
3r + 11s
e
6w2 - 4w3
h
10x + 4y - 6xy

18b
4e
5h
4k
8n - 4
3t - 6
5xz - xy

c
f
i
l
c
f
i

14c
f
-9i
-2l
10p2 + 4p
-u - 6v
5p2 - 16

3 a
d
g
j
4 a
d

a13
7d4
28m6n5
48u4v5w9
k3
2x4
5 6

g mn
j
5 a
d
6 a
c
e
g
i
7 a
c
e
8 a
d

b b4
e 12p9q5
h 20p5q4

c 12c7
f 63g3h4
i 24x3y3z

b 3m5
e 8m2n2
8 p3
h --------q3

c 6n5
f 5x5
11a
i --------b

Exercise 5D Solution by substitution


1 x = 6.6
2 x=7
3 x=7
4 n = 14
5 a 500 000 = 265 000(1.04)n
b 16 years
6 5 years
7 a

2
4
6
8
10

10
15.3
a - 6b
6g3h2
10x - 5x2

12.7 cm
38 m
a 7.3
b 12.4
5
10.22 cm
a 0.75 b 2.20
y1
7 x = ----------2
A
8 l = --b
2A
9 a a = ------- b
h

8 a

x + 2x
18q6 - 6q2
14a6b4 - 21a2b10
-3m2 + 6mn
18p2q2r - 6pqr2
a2 + 13a - 10
18x - 45
7x2y - 8xy2 - xy
c 16mn5
f m

101.87
-5x
15b6
25p6q8
6a2 - 19ab + 24b2

c 4.1

c 3.19

13 B

V
15 a r = ---p
3V
------4p

r =

a = c2 b2

10

Length

10

15

20

25

30

35

Width

35

30

25

20

15

10

d 26

d 4.70

e 20.4

Area

10 Quick Questions 2
1
3
5
7
9

2
4
6
8
10

10
3
20
20x7y4
3mn2

32
127.3
a - 3b
64m9n12
x = 12

Exercise 5E Scientific notation


1
2
3
4

a
a
a
a
c
e
5 a
c
6 a
c
7

e 0.6

2A
b h = -----------a+b

E
b c = ---m
14 D
b u = v 2 as
T 2
d L = g ------
2 p

175 300 375 400 375 300 175

b 20 m 20 m
c 40 m 20 m

A
10 r = --p
E
11 a m = ----c2
12 D

Exercise 5C Equations and formulas


1
2
3
4
5
6

b 8 seconds
c 27m6

10 Quick Questions 1
1
3
5
7
9

d 5 20 45 80 125 180 245 320 405 500

4qr
a12
b 4b8
4 6
16x y
e 16p4q8
2m + 10
b
9a2 + 6ab
d
5mn - 25n2
f
-3d - 15
h
-12r3 + 18r6
j
10x + 6
b
2m2 + 4mn - 12n2
d
4p2 - 15p + 30
f
a7
b m3
10xy11
e 1

t 1

answers

427

Answers

8 a
c
e
g
i

9 104
b 2 1010
c 7 102
1.458 106 b 2.365 1013 c 2.589 103
2 10-8
b 4.57 10-3 c 4.9321 10-11
9.32 107 km
b 7.85 1010 mm
4
4.59 10 t
d 3.65 10-3 g
2.14 10-1 mL
f 5.69 10-6 s
34 000
b 2 870 000
30 248 000 000
0.000 585
b 0.000 001 97
0.001 002

Planet

Distance
(AU)

Distance in km
(Scientific notation)

Mercury

0.39

5.85 107

Venus

0.72

1.08 108

Earth

1.0

1.50 108

Mars

1.52

2.28 108

Jupiter

5.20

7.80 108

Saturn

9.54

1.43 109

Uranus

19.18

2.88 109

Neptune

30.06

4.51 109

2.35 104 m
6.4 106 mm
7.802 109 kg
1.87 105 kL
5.55 1010 mL

b
d
f
h

8.4 104 km
6.58 103 t
8.29 1013 g
2.178 1010 L

5A
5E

answers

428

Answers

Chapter review
1
2
3
4
5

9
13
14
15
16

17

18

19

162
36.952
13
a 136.00 b 37.78
c 8.57
d 8
a 5m
b 16q
c 12p
d t
e 4m + 4n f 4x - 5
g 3k - 2l
h 8x2 + 14x i ab + 7a - 2b
a 28a9
b 45b2
c 21g4h8
6 7
2
d 12m n
e 6x
f 8s3
g 7q
h 27p6q12
i 4m2
2
a 2a + 18
b 2p - 4p
c -3x5 + x2
d 12m7 - 8m5n
e -16xy + 4xy2
f 12a5b3 - 24a2b5
a 8m + 40
b 2p2 - 3p
c 11x - 52
d 7yz + 8y2 - 6z2
e 2p2q - 4pq2 + 8p
15
10 21.6 m 11 s = 6 12 7.2 cm
a x = 2.9
b x = 3.8
c x = 4.8
8 years
3 years
a 6 105
b 2 10-10
13
c 7.892 10
d 1.25 10-3
e 4.589 10-6
f 1.245 89 1014
4
a 1.26 10
b 1.25 10-4
c 3.21 10-2
d 5.86 108
e 1.24 104
f 5.19 10-8
a 250
b 38 700
c 98 504 000
d 0.289
e 0.000 000 367 02
f 0.0011
a 2.5 108 mm
b 2.8 105 kg
c 3.43 107 L
d 1.45 103 km
10
e 4.243 10 kg
f 1.3 105 L

Practice examination questions


1 A
2 A
3 A
4 B
5 A
6 a 304.8 cm3
b 8.3 cm
SA 2 p r 2
c 6.4 cm
d h = ------------------------2pr
7 a n = 10
b (0.85)n = 0.5
c n = 4.3

CHAPTER 6 Multi-stage events


Are you ready?
1 a
b
c
2 a
3 a
c
4 a
5 a

{hearts, diamonds, spades, clubs}


{red, blue, white}
{a, b, c, x, y, z}
Unlikely
b Probable c Impossible
Equally likely
b Equally likely
Impossible
500
b 240
c 45 697 600
1
1
3
-------b
c ----8
50
14

6 a Selecting a consonant
b Selecting a white or clear marble
c Selecting a number greater than 9
7 a 3--5b 1--5c 0.27

Exercise 6A Tree diagrams


1

1st
child

2nd
child

3rd
child

Boy

Boy
Girl

Girl

Boy
Girl

Boy

Boy
Girl

Girl

Boy
Girl

Boy

Girl

1st die
1

1st bag
Red

Blue

Yellow

Green

Male
George

Frank

Stanisa

Ian

4th
child
Boy
Girl
Boy
Girl
Boy
Girl
Boy
Girl
Boy
Girl
Boy
Girl
Boy
Girl
Boy
Girl

2nd die
1
2
3
4
5
6
1
2
3
4
5
6
1
2
3
4
5
6
1
2
3
4
5
6
1
2
3
4
5
6
1
2
3
4
5
6
2nd bag
R
B
Y
G
R
B
Y
G
R
B
Y
G
R
B
Y
G
Female
Thuy
Petria
Joan
Wendy
Amelia
Thuy
Petria
Joan
Wendy
Amelia
Thuy
Petria
Joan
Wendy
Amelia
Thuy
Petria
Joan
Wendy
Amelia

S = {George Thuy, George Petria, George


Joan, George Wendy, George Amelia, Frank
Thuy, Frank Petria, Frank Joan, Frank
Wendy, Frank Amelia, Stanisa Thuy,
Stanisa Petria, Stanisa Joan, Stanisa
Wendy, Stanisa Amelia, Ian Thuy, Ian
Petria, Ian Joan, Ian Wendy, Ian Amelia}

1st digit

2
4
5
7
1
4
5
7
1
2
5
7
1
2
4
7
1
2
4
5

Exercise 6C Probability and counting


techniques

2nd digit

Belinda

Dean

Belinda
Kate
Adrian

Kate

Belinda
Dean
Adrian

Adrian

Belinda
Dean
Kate

1st digit

3
4
7

11 a
12

1
-----20

1
--4

1
--6

2
--3

2
--3

5 a

1
-----42

1
-----42

1
-----21

6 a

1
-----72

2
--9

2
--9

1
---------------13 800

1
-----------2300

1
-----20

3
-----10

3
--5

1
-----10

1
--6

8 a

9 a 20
10 a

1
-----10

10 Quick Questions 1
1
2
5
8
10

S = {HH, HT, TH, TT}


36
3 120
4 3 628 800
720
6 72
7 70
24
9 1680
An ordered selection occurs when the order in which
each choice is made is important but in an unordered
selection, order is not important.

Exercise 6D Probability trees


1

25
-----64

2 a

7
-----12

1st marble
4

9
5

2nd marble
4

9
White

White
5

9 4

3
-----10

2
--5

3 a

13 B

b 120
3 120
6 12
9 210
12 C
b 6
b 504

P(BW) =

3
--5

c 362 880
4 6
7 12 144
10 15
13 D
c 36

Black
16
-----81

20
-----81

P(WB) =

20
-----81

P(WW) =

25
-----81

6
-----25

4 a 51.2%
1
5 a ----20
6 C
7 B
8 a

Black
White

Black

b P(WW) =

Exercise 6B Counting techniques


a 6
24
720
35
72
B
a 20
a 362 880

1
--------120

Kate
Adrian
Dean
Adrian
Dean
Kate
Kate
Adrian
Belinda
Adrian
Belinda
Kate
Dean
Adrian
Belinda
Adrian
Belinda
Dean
Dean
Kate
Belinda
Kate
Belinda
Dean

14 a S = {Bris Peak Ec, Bris Peak BC, Bris


Peak FC, Bris Off-peak Ec, Bris Off-peak
BC, Bris Off-peak FC, GC Peak Ec, GC
Peak BC, GC Peak FC, GC Off-peak
Ec, GC Off-peak BC, GC Off-peak FC,
Cairns Peak Ec, Cairns Peak BC, Cairns
Off-peak Ec, Cairns Off-peak BC}
b i 3--8ii 1--4iii 1--81
2
5
8
11
14
15
16

2nd digit
3
4
7
2
4
7
2
3
7
2
3
4

1
--------120

3 a

7 S = {357, 358, 375, 378, 385, 387, 537, 538, 573,


578, 583, 585, 735, 738, 753, 758, 783, 785,
835, 837, 853, 857, 873, 875}
8 C
9 1--210 a

2 a 120

President Secretary Treasurer


Dean
Kate
Adrian

1
-----24

answers

429

Answers

1
-----15

b
b

1
--------495

7
-----15

b 38.4%
--------c 893
990

9 a S = {BB, BG, GB, GG}


5
-----b i 14
ii ----ii
39
39
10 a

24
-----49

11 a

2
-----21

12
13
14
15

a 0.3025
a 40.96%
0.01
--------a 624
625

16 D

b
b

12
-----25

3
--7

8
-----15

19
--------198

7
-----15

20
-----39
25
-----49

10
-----21

b 0.2025
b 59.04%
b

369
--------625

6A
6D

answers

430

Answers

1
--2

17 a

1
--5

CHAPTER 7 Applications of
probability

1
--2

3
-----10

18

19 a 0.32

b 0.56

Are you ready?

c 0.88

Chapter review
1 a

1st coin

1 a

2nd coin

2
--3

First digit
3

Tails
Heads

Tails
Tails
6

b S = {HH, HT, TH, TT}


2 a S = {57, 58, 59, 75, 78, 79, 85, 87, 89, 95, 97, 98}
b S = {55, 57, 58, 59, 75, 77, 78, 79, 85, 87, 88, 89,
95, 97, 98, 99}
3 a 1--8b 3--8c 1--2-

3 a

1
--2

1
4
5
6
7

b 60

11 a 12

12 a 360

1
--------360

1
-----12

c 15

1
-----15

13 a

1
-----------1024

1
----------------------1 048 576

14 a

3
-----10

1
-----10

8
--------125

98
--------125

1
--3

Practice examination questions


1 C
2 D
3 C
4 B
5 a 1st coin 2nd coin 3rd coin
Heads
Heads
Tails
Heads
Tails
Tails

1st shot

No ad

Ad

Heads
Tails
Heads
Tails
Heads
Tails
Heads
Tails

3
--8

Lands

0.6

Miss

0.4

Lands

0.6

Miss

ii 0.48

c 7.69

d 30.77 e 1.92

b 48.9
10 B
b 90

11 5
c 30

10

11

12

Probability

1
-----36

1
-----18

1
-----12

1
--9

5
-----36

1
--6

5
-----36

1
--9

1
-----12

1
-----18

1
-----36

Expected
no.

2.8 5.6 8.3 11.1 13.9 16.7 13.9 11.1 8.3 5.6 2.8

b 60.53

c 39.47

d 94.74

7
--8

1 $0.00
2 $0.40
4 $0.15
5 -$0.30
7 a 37
b i 18
ii 18
iii 1
c -$0.27
8 A
9 C
10 $0.11

3 -$0.50
6 $1.70

11 -$1.10

10 Quick Questions 1

2nd shot
0.4

3 25
c 7

Exercise 7B Financial expectation

Miss

b i 0.16

b 50
b 2.08

14 a 55.26

0.4 Lands

0.6

No ad

2 50
b 28

8 a 0.0144
9 B
12 a 90
13
Outcome

17 91%
--------18 343
512

6 a

5
1

No ad

20
a 35
1.25
a 25
1
a ----25

2
--5

16 a

Ad

Ad

Exercise 7A Expected outcomes

1
--------120

15

1
-----25

3
--4

40 320
56
70
a 120

6
7
8
9
10

Second time
1

4 a S = {46, 47, 48, 49, 64, 67, 68, 69, 74, 76, 78, 79,
84, 86, 87, 89, 94, 96, 97, 98}
1
b i ----ii 2--5iii 3--420
5 a

First time

49
-----99

Second digit Sample space


5
35
6
36
7
37
3
53
6
56
7
57
3
63
5
65
7
67
3
73
5
75
6
76

Heads
Heads

1
-----50

iii 0.64

1 20
2 25
3 50
4 7.7
5 30.8
6 $0.17
7 -$0.11
8 The player can expect to have an average profit of
20% per game.
9 The player can expect to have an average loss of
20% per game.
10 $0.45

Exercise 7C Two-way tables


1

14

Test results

Test results

Accurate Not accurate Total

Accurate

Not accurate

Total

98

100

Without virus

388

12

400

Total

486

14

With virus

48

50

Without virus

149

150

Total

197

With virus

15
2

Test results

Test results
Accurate

Not accurate

Total

Telling truth

777

23

800

Telling lies

156

44

200

Total

933

67
3
--5

3 a 1000
b 75
c 96.7%
d
4 a 200
b 44
c 90.9%
d 5.1%
e 94%
f A range of factors should be considered, e.g. for
an expensive system a 6% fail rate might be
unsatisfactory.
5 B
6 D
7 A
8 a

Test results
Accurate

Not
accurate

Total

48

50

Bags with no
prohibited
items

145

150

Total

193

Bags with
prohibited
items

b i 96%

ii 3.3%

iii 4%

iv 96.5%

Chapter review
1
2
3
4
5
6
7
8
9

10
11
12
13

431

18
a 10
b 30
c 20
a 50
b 25
c 7.69 d 30.8 e 3.85
a 2.5
b 15
c 7.5
d 15
e 25
a 12.5
b 37.5
c 50
a 5
b 45
c 15
$0.00
-$0.60
A positive financial expectation means that on
average a profit should be made while a negative
financial expectation means that on average a loss
should be made.
-$0.03
$0.00
$0.30
$1.87

answers

Answers

Accurate

Not accurate

Total

Telling truth

77

80

Telling lies

17

20

Total

94

16 a 140
17 a 130

b 30
b 33.8%

c 90%
39
c ----40

Practice examination questions


1 C
2 B
1
4 a 1--6b 15
c ----18
5 a 200

b 96%

c 34

1
-----10

3 A
d $0.11
d 93 1--3- % e

14
-----15

CHAPTER 8 Annuities and loan


repayments
Are you ready?
1 a
c
e
2 a
3 a
4 a

n = 5, r = 0.08
b n = 8, r = 0.03
n = 12, r = 0.019
d n = 120, r = 0.008
n = 30, r = 0.000 66
$5000
b $7626
c $5922
$2292.19
b $16 808.55
c $18 499.35
$8984
b $15 750
c $22 716

Exercise 8A Future value of an


annuity
1 $7049.37
2 a $6691.13
b $16 859.14
c $6158.56
d $3974.56
e $17 713.21
f $3530.21
3 $4472.93
4 a i $2295.05 ii $2217.44 iii $2142.45
iv $2070.00
b $10 724.94
5 $73 105.94
6 a 22
b $21 696.15
c $283 057.94
7 a $25 155.79
b $29 333.00
c $433 046.81
d $217 372.57
e $114 665.87
8 a 5
b No she has saved $29 041.96.
c $4041.96
9 $90 237.49
10 a $20 326.23
b $24 297.37
c $45 881.32
d $69 770.03
11 A
12 C
13 $4067.23

6D
8A

answers

432

Answers

14 a $4524.37
15 a $56 160

b $7068.59 c $1930.08
b $112 320 c $242 106.74

10 Quick Questions 1
1
4
7
10

$8857.81
$195 857.88
$3626.32
$891.50

2 $1179.42
5 $31 741.41
8 $5980.14

3 $10 164.56
6 $66 574.26
9 $558.24

Exercise 8B Present value of an


annuity
1
2
3
4
5
6
7
8
9
11
12
13

$7537.11
a $12 418.43
b $3786.09 c $94 222.37
$48 987.91
a $37 685.57
b $35 644.50
c $34 623.58
d $33 943.00
$3511.79
a $11 257.78
b $6116.69
c $14 783.59
d $24 767.78
$22 851.87
$6113.60
A
10 D
Investment A
C
a $15 864.53
b Yes, Kylie will have $32 547.59.

Exercise 8C Future and present


value tables
1 $4787.76
2 a $1324.00
b $23 932.35
c $7503.81
d $62 953.50
3 a 4%
b 10
c $6003.05
4 a $4103.92
b $5535.38 c $7546.74
5 5% for 6 years. $1 will grow to $6.8019 but at 6%
for 5 years it will grow to $5.6371.
6 D
7 $6918.50
8 a $1845.09
b $12 289.20
c $4455.79
d $16 604.40

Chapter review
1 a $4917.25
c $31 053.57
2 $16 398.20
3 a $66 666.94
c $14 291.59
4 $36 604.24
5 $3088.13
6 a $3793.40
7 $20 057.99
8 a $6139.13
c $3219.64
9 $11 654.84
10 $5113.34
11 a $524 573.59
12 $41 039.20
13 a $4399.95
c $1842.84
14 $2242.95
15 a $1516.32
c $4055.45
16 $547.41
17 a $553.76
18 $503 055
19 a $226.10

$15 937.42
$1435.91
$22 094.93
$13 295.75

2 $15 937.42
5 $5084.04
8 $8513.56

3 $13 537.79
6 $19 277.16
9 $10.63

Exercise 8D Loan repayments


1 $2637.97
2 $210.67
3 a $94.15
b $311.38 c $859.72
d $484.17
e $1511.14
4 $1397.37
5 a $375
b $3375
c $107.32
6 $374.53
7 a $28 000
b $138.21
8 $16 847.35
9 a $4359.36
b $29 059.20
c $98 285.40
d $366 700.80
10 D
11 B
12 9 years, 3 months
13 a $1320.99
b $396 297.00
c 14 years, 6 months
d $117 897

b $31 371.42
d $247 313.84
b $468.93

c $217.69

b $4298.72
d $36 945.53
b $13 002.83
b $34 641.25
d $51 014.25
b $14 047.20
d $11 177.64
b $26 580.48
b $5426.40 c $1026.40

Practice examination questions


1 D
2 B
3 B
5 a $91 523.93
b $19 636.30
6 a $63 792.85
b $2384.89
7 a $1651.63
b $246 391.20

4 D
c $5362.05
c $67 443.86
c $112 692.48

CHAPTER 9 Modelling linear and


non-linear relationships
Are you ready?
1 a
x

10

16

13

10

10 Quick Questions 2
1
4
7
10

b $2960.49
d $5461.06

2 a Linear
d Linear
3 a 5--44 a

b Not linear
e Not linear
b 3
b

y
5
3
1

y =3x

5 3 1
1 1
3
5

3 x

c Not linear
f Linear
c 2
y
5
3
1 0
x
5 3 1
1 1 3
3
5 y =2x 3

5 a

y
5
3

Exercise 9A Linear functions

3 x

24 000

6 000

y
4
3 y = x
2
10
3 1 1 2 3 x
2
3
4

y
5
4
3 y = 1 x + 3
2
2
10
x
3 1
1 1 2 3 4

y y = 3x 2
4
3
2
10
4 2 1 1 2 3 4 x
2
3
4
y
5 y = 5 2x
4
3
2
10
x
3 1
1 1 2 3 4
3
y
5
4
3
1
2 y = 1 x
4
1
0
x
3 11 1 2 3 4
2
3

-3

c $9000 loss d 8000 people


b 500

N
1000
800
600
400
200
0
0

7 a

b (1, 3)

y
5
y = 3x
4
3
2
1
3 1 01 2 3 4 5 x
3 y = 4 x

Intersection (5, 4)

y
5
4 y=x1
3
2
1
3 1 0 1 2 3 4 5 x
y = 2x 6

9 (0, 2)
10 a, b

c $1.80

N = 1000 5P

6 a

y
4
3
2 y = 2x
1
4 2 0 1 2 3 x
2
3

3 a

10 000

b $36 000

2 a

8 000

10
5 3 1
1 1

20

y
5
3 y=x+3

12

20

10

000
000

1
x
5 3 1
1 1 3
3
5 y =5 2x

P P = 3n 24 000
50 000
40 000
30 000
20 000
10 000 0
n
10 000
20 000
30 000

40

433

answers

Answers

B
120
100

2A = B

80
60

A + B = 120

40
y
5
4 3x + 2y 6 = 0
3
2
10
x
3 1
1 1 2 3 4 5
2
3

4 a

20
0
0

40

c Game A: 40; Game B: 80.


11 a

1000

2000

1200

2400

900

1200

1500

10

30

C (old)

10.5

18

48

C (new)

120 A

80

C = 3 + 1.5d

c After 1000 days

C = 1.2d

C
2000

40

1500
C = 900 + 0.3d

1000

20

500

c $33

d 14 km

00

40

20

20

10

00

8A
9A

12 a

Answers

6 a The coefficient will make the graph steeper if it is


greater than 1 and flatten the graph if it lies
between 0 and 1.
b Adding a constant will lift the graph while
subtracting a constant will lower the graph.
7
They are the same function.
y
2

b 300

C
10 000

C = 20n

8000

C = 15n + 1500

6000
4000

30

16
12

Exercise 9B Quadratic functions


1 a

11

18

4
0

8
b

9 a

b
y
16 y = 4 + 6x x2
12

2
2 0 1 2 3 4 5 x
2
4
y
10

y
25
20
15

10 D

10

13

5
0

y = (x 2)2

3 4 x

y
25
20
15
10
5

14 a

2
0

1 2

3 4

y = 2x2
(b)

25

11 C

y = x2
(a)

3 4

d = 5t2

3 4

5 t

h = 30t 5t2

h
40
20

2
2
y y=x +3 y=x
9
y = x2 3

0
0

(b)

b 45 m c 6 s
3

0
3

(a)
1

16 a

(c)
2

3 4

b 80 m c 10 s

5 x

2 4 x

5 x

25

15 a
1

50

0
2
4

12 D

75

y = 1_ x2
2
(c)

10

d
125
100

20
15

0 12 45 x
3
6
9
y = 8 x2

2
y y = (2 x)
4
2

y = 2x2 4x + 8

6
4

0 12345 x
4
8

y = x2 + x + 5

y
9
6

8
4

5 x

y
6 y = x2 6x + 5
4

5 x

0 12 45 x
2
4
6
y = 2 + 2x x2

3 a

3 4

y y = x2 4x 2
6

3 0 1 3
3
6

1 2

y
4

c Min. value = 2

y y = x2 2x + 3
18
15
12
9
6
3
10 1 2 3 4 5 6 x
1

y = x 2x + 5

20

50

2000

10

answers

434

x
50 x

Sum of adjacent sides = 50 m

b A=lb
A = x(50 x)
A = 50x x2
c
A
A = 50x x2

2 a

600
400

200
0
0

20

6
4
2
0

4 x

1 2 3

2 3 4 x

2 3 4 x
y = x3

y = 13x3

y = 3x3

0
2
4

d 625 m2 when the field is 25 m 25 m


17 a A = l b
A = x(100 2x)
A = 100x 2x2
b
A = 100x 2x2

y
10

40

y
50
40
30
20
10
0
0

answers

435

Answers

y
4

1200

y = x

2
800
0
400

0 1

4 a
0
0

20

40

10 Quick Questions 1

1
0

2 2
c

-5

-12

y = 4 x2

2
1
0
1
2

2 4

6 8

3 4

5
x

y = 1x

y = 3x

0 1

6 a

2 3 4 x

y
40
30

y = 4x

20
10
0

y
100
80
60

y = 10 x

40
20
0

1 2 3 4 x

20
10

4 5 x

y = x3

10

y=
x

2
0

4 x

2 3

y
40
30

Exercise 9C Other functions


y
8
7
6
5
4
3
2
1
0
0

6
4

0
1

2
3
4

8 4
9 Concave up because the x2 term is positive.
10 5
1

y
4
3

8
1
y = x

0
1

3 -3
4 y = 3x + 2
5 Coefficient of x must be negative: e.g. y = 2x + 7
6

3
2

y y = 2x 3
3
2
10
x
3 1
1 1 2 3
3

y
4

c 25 m 50 m
1

2 3

4 x

2 3

2 3

4 x

y
3
2
1
0

y = (12 )
0 1

2 3

4 5 x

9A
9C

answers

436
7

Answers

10 a 20
b 2 hours
11 12 km/L or 8 1--3- L/100 km
12 8 amps

y
40
30
20
10
0

y = 5(2x)

8 C
9 B
10

Exercise 9E Graphing physical


phenomena

1 2 3 4

1 a
A

1600
1400
1200
1000
0

b
A = 1000(1.1)n

0 2

8 n

4 6

11 a A = 50 000(1.12)n
b
A
65 000

2
A = 50 000(1.12)n

55 000
0
0 1

2 3

4 5 n

c 3 years
12

V
40 000

d
500
400
300
200
100
0

24

54

96

150

A
400
320
240
A = 6s2
160
80
0
0 2 4 6 8 10 s

d = 5t2
0

2 4

6 8 10 t

V = 40 000(0.85)n

30 000

3 a

20 000

10 000
0

0 1

2 3

4 5

Exercise 9D Variations
2

1 y = 5.5x
2 b = 0.5a3
3 a d = 4.9t 2

d
50
40
30
20
10
0

150
9 a t = --------s

10

20

30

40

18

30

44

d
40
30
20
10
0

d = 0.01v 2 + 0.7v

0 10 20 30 40 50 v

100 000
4 a A = ------------------n

d = 4.9t

2 3

4 t

60 000

b 230.64 cm
b 452.16 cm2
b 54 g
c 6.5 cm

5 a

Age
(years)

V
40 000
V = 30 000(0.8)n1

30 000
150

20 000

t=
s

0 10 20 30 40 50

10 000
0

0 2

4 6

8 10 n

$30 000 $24 000 $19 200 $15 360 $12 288

Value
t
50
40
30
20
10
0

000
A = 100
n

40 000
20 000

A
100 000
80 000

0 1

4 a 6
5 a 3.14
6 a 0.25
50
7 y = -----x
1
8 m = --n

2 4

8 10

W
8 W = 3.3(1.2)n

C
24

6
4

C = 3 + 0.4d

18

0 1

2 3

12
6
0

7 a 6
b V = l b h = (12 2x)(12 2x)x = x(12 2x)2
c

V
125
100

0
100

400
4 5

2007

Year
Population
(million)

1.5

c i $55 profit
d 256

6 x

2008
1.58

2009
1.65

2010
1.74

2011

1500
1000
0

8 12 16 20 n

Chapter review
b

y
4
3 y=x+3

y
4
3 y=2x
2
10
x
3 1
1 1 2 3 4
2
3
4
y
3
2
1 0 2y = 4x 3
x
3 1
1 1 2 3 4
3
4
5

y
5
4
3 3x 2y + 6 = 0
10
x
3 1
1 1 2 3 4
2
3

10

c 1

6
4
2
0

y
5
4 y = 5 3x
3
2
10
x
3 1
1 1 3 4
2
3

y
10
y = x2 4x + 5
8

10
x
5 3 1
1 1 2 3
2
3
4

25
0
50
0
75
0
10
0
12 0
50

6 a
y
4
3
2 y = 3x
1
3 1 01 2 3 4 x

C = 1000 + 0.2w

500

c 2027
d The graph will become a straight, horizontal line.

0 1

4 5 x

y = x2 2x 2

y
6
4
2
0
2

2 3

4 x

8 a

y
20
16

b
y = (x 4)2

10

15

20

11

4
0

y
4
1 0
4
8
12

12

2 a

b C = 200 + w
d 1000 washes

C = 200 + w

2000

y=6x

5 a C = 1000 + 0.2w
c
C

y=x+2

10
3 1 1 2 3 4 5
1

1 a

ii $142.50 loss
Intersection (2, 4)

y
5
4
3

1.82

P
4 P = 1.5(1.05)n
3

200
300

0 1 2 3

P = 1.25n 320

200
100

V = x(12 2x)2

75
50
25
0

8 a

0 10 20 30 40 50 d

3 a P = 1.25n - 320
b
P

24
0
32
0
40
0

2
0

80
16
0

answers

437

Answers

0 2

6 8x

y = 5 x2
1 2

4 x

9C
9E

Answers

y
4
1 0
4
8
12

9 a

10 a

2 3

4 5 x

b 10 s

h
500
400
300
200
100
0

16 y = 5x2
17 a m = 0.45l 3
64
18 y = -----x
1000
19 a A = -----------n

y = 4 + 2x x2

h = 500 5t2

A 0
b
0

E
150

8 10 t

E = 24n n2

90
60
30

21 a
0 4

8 12 16 20 24

b 144 kg, 12 workers


c Too many people getting in each others way etc.
11 a y
b y
10
8
6
4
2

8
y = x3

6
4
2
0
0 1 2

b
1

y = x

y
4
3

1
y = 23x3

2 3 4 x

0 1

y
y = 2x

8
4
0
0 1
C
1000
750

2 3

3.14

12.57

28.27

50.27

78.54

1 2 3 4 t

b i 20 m
22 a
A

1 2 3 4 x

14 000
12 000
10 000
0

y = x

ii 4 s
A = 10 000(1.06)n

2 4

b $16 000
0 1

2 3 4 x

y
2
1 x
y = ( 2)
1
0
0 1 2 3 4 x

4 5

h = 20t 5t2

18 000
16 000
0

2 3

6 8 10 n

c 7 years

Practice examination questions


1 B
2 D
3 B
5 a P = 10n 500
b, d

4 D
P
2000
1500
1000
500
0
500

P = 10n 500

P = 5n

1000

c 2024

5000
4000
3000

6 a

e 100
t

20

30

30

20

8 12 16 20 n

b $740
15 a
V

2000
1000
0

4 x

C = 500(1.04)n

c 250 campers

c 50

500
250
0

h
20
16
12
8
4
0

2
1

16
12

14 a

y
4
3
2

13 a

3 4

b 8 days

A = r 2

1
0

A
80
60

c 4.8 cm

40
20
0

120

12 a

20 a

b 56.25 g

50
10
0
15
0
20
0

answers

438

h
30

h = 20 + 15t 5t2

20
V = 5000(0.8)n
10

b 7 years

6 8 10 n

1 2

3 4

Max. height = 31.25 m when t = 1.5

6
4
2
0

8 a

Exercise 10A Modelling depreciation

b (1, 2)

y
10
8

1 a
y = 2x3
y = 2x

4 5 x

2 3

b 10 years

y
4
x
3 y = 1.08

b V = 100 000 - 10 000A


Value ($)

4 6 8 10 x

0 2

d 8 years

y
1

y = 0.92x

0.75
0.5

4 6 8 x

0 2

y
10
8
6

6
4
2
4 2 0
2

y =2x 1

2 4

4
2

y =8 4x
0

4
6

4 2
2

2 4

6 8 10 12 x

y
5
4

Value ($)
2

3 4

x
y = 5(12 )

2 3

4 a $13 382.26
5 a $13 110

0 2 4 6 8 10
Age (years)

4 5 x

b x = 4 --13-

ii $128 000 iii $102 400


b
200 000
160 000
120 000
80 000
40 000
0

Value ($)

(2, 114 )

3 a x = 12

b $17 000 c 7
620 000
480 000
360 000
240 000
120 000
0

7 a i $160 000
iv $81 920

3
2
1

0 2 4 6 8 10
Age (years)

6 a

(2, 0.64)

0.4
0.2
0

b $2000

y = 0.8x

y
1.0
0.8
0.6

0 1 2 3 4
Age (years)

20 000
16 000
12 000
8 000
4 000
0

Value ($)

0 2 4 6 8 10
Age (years)

5 a

4
6
8
10
12
y
y = 2x
10
9
(3, 8)
8
7
6
5
(2, 4)
4
3
(1, 2)
2
1
0 1 2 3 4 5 6 7 8 9 10 x

50 000
40 000
30 000
20 000
10 000
0

7000
6000
5000
4000
3000
2000
1000
0

Value ($)

1 a

0 2 4 6 8 10
Age (years)

b $20 000
c 9 years
4 a V = 6400 - 2000A
b
c 4

Are you ready?

V = 50 000 - 8000A

3 a

CHAPTER 10 Depreciation

2 a

50 000
40 000
30 000
20 000
10 000
0

Value ($)

0.25
0

0 2 4 6 8 10
Age (years)

2
1
0

100 000
80 000
60 000
40 000
20 000
0

Value ($)

7 a

439

answers

Answers

c x=5

b $68 956.60
b $63 100

0 2 4 6 8 10
Age (years)

10A

Answers

New (0)

30 000

26 000

9 A
10 C
11 a $5360
c $3591
12 5 years

22 000

10 Quick Questions 1

18 000

14 000

10 000

Age (years)

b See part d
c
Age (years)

Value ($)

30 000

24 000

19 200

15 360

12 228

9 830

1 a $1683.50
2 a

e 6 years

Exercise 10B Straight line


depreciation
c $270 000

3 a
b 5 years
d 7 years
b $12 500/year c $14 500/year
b $26 500

c $1450

Exercise 10C Declining balance


method of depreciation
1
2
3
4
5

$20 480
a $2220
7 years
$383 000
a $5900
d $62 100
6 $6174
7 $676 000
8 a $14 600

b i $750
b $68 100
e $3900

$2350/year
$21 000
$7250
$389 000
16 years

b $9537.50 c $34 870

Age of car
(years)

0 2 4 6 8 10
Age (years)

b $10 300
e $32 000
b $3 750 000

2
4
6
8
10

$650
7 years
S = V0(1 - r)n
$11 000
$37 500

Straight line value


Declining balance
value

1 $20 000
2 a $1000
d $145
3 a $7 125 000
4 $10 600
5 8 years
6 a 6 years
c 8 years
7 $2500/year
8 a $4000/year
9 $900/year
10 $25 000
11 a $110 000
12 $78 000

b $2640
d $1769

Exercise 10D Depreciation tables

Value ($)

New (0)

30 000
24 000
18 000
12 000
6 000
0

1
3
5
7
9

Value ($)

8 B
9 a

Value ($)

answers

440

ii $390
c $1200

b $20 400

Straight line
value ($)

Declining
balance
value ($)

New (0)

40 000

40 000

35 000

32 000

30 000

25 600

25 000

20 500

20 000

16 400

15 000

13 100

10 000

10 500

5 000

8 400

6 700

40 000
32 000
24 000
16 000
8 000
0

c After 6 years

Straight line value


Declining balance
value

0 2 4 6 8 10
Age (years)

Age of
equipment
(years)

Straight
line value
($)

Declining
balance
value ($)

100 000

100 000

90 000

85 000

80 000

72 250

70 000

61 400

60 000

52 200

50 000

44 350

40 000

37 700

30 000

32 050

20 000

27 250

10 000

23 150

10

19 700

New (0)

7 a

Value ($)

Straight line value


100 000
Declining balance
value
80 000
60 000
40 000
20 000
0
0 2 4 6 8 10
Age (years)

Age of
computer
(years)

Age of
truck
(years)

Straight
line value
($)

Declining
balance
value ($)

New (0)

250 000

250 000

225 000

200 000

200 000

160 000

175 000

128 000

150 000

102 400

125 000

81 920

100 000

65 536

Salvage
value at 20%
($)

Salvage
value at 35%
($)

4400.00

4400.00

3520.00

2860.00

2816.00

1859.00

2252.80

1208.35

75 000

52 429

1802.24

785.43

50 000

41 943

1441.79

510.53

25 000

33 554

1153.43

331.85

10

26 843

Salvage value
($)

Tax deduction
($)

4355

2145

2918

1437

1955

963

1310

645

Years

878

432

878

b
Value ($)

441

250 000
Straight line
200 000
value
150 000
100 000
Declining
50 000
balance
value
0
0 2 4 6 8 10
Age (years)

Salvage
value
straight line
($)

Tax
deduction
($)

225 000

25 000

200 000

25 000

175 000

25 000

150 000

25 000

125 000

25 000

Age of
truck
(years)

Salvage value
($)

Tax deduction
($)

33 750

11 250

25 313

8 437

18 985

6 328

14 239

4 746

100 000

25 000

10 679

3 560

75 000

25 000

8 009

2 670

50 000

25 000

6 007

2 002

25 000

25 000

4 505

1 502

10

25 000

Years

answers

Answers

10A
10D

Answers

5
Tax
deduction
($)

200 000

50 000

160 000

40 000

128 000

32 000

102 400

25 600

81 920

20 480

65 536

16 384

52 429

13 107

41 943

10 486

33 554

8 389

10

26 844

6 711

Age of
truck
(years)

8 a $10 000

9 a $3000

1
--3

b $75

10 000
8000
6000
4000
2000
0

Value ($)

Salvage
value
declining
balance ($)

$6500
a $1300
12 years
$250/year
After 6 years
$20 880
$474 000
a $23 620
d $27 210
14 a $167 100
15 a

c $3333.33
c $1600

Value ($)

1 a

200 000
160 000
120 000
80 000
40 000
0

d $750

0 2 4 6 8 10
Age (years)

0 2 4 6 8 10
Age (years)

b V = 3500 - 250A
3 a
Value ($)

16 000
12 000
8 000
4 000
0

c $1250

60 000
50 000
40 000
30 000
20 000
10 000
0

b $6500

c 17 years

0 2 4 6 8 10
Age (years)

c 9 years

c $235 000

b $1000
e $49 380
b $432 900

c $24 290

Salvage value
straight line
($)

Salvage
value
15% p.a. ($)

New (0)

100 000

100 000

90 000

85 000

80 000

72 300

70 000

61 500

60 000

52 300

50 000

44 500

40 000

37 800

30 000

32 100

20 000

27 300

10 000

23 200

10

19 700

100 000
80 000
60 000
40 000
20 000
0

0 2 4 6 8 10
Age (years)

b $10 500
4 a

b $15 000

Age
(years)

Value ($)

Value ($)

b V = 200 000 - 20 000A


2 a
3 000
2 000
1 000
0

0 2 4 6 8 10
Age (years)

6
7
8
9
10
11
12
13

Chapter review

Value ($)

answers

442

16

Salvage
value straight line

Salvage
value
15% p.a.
0 2 4 6 8 10
Age (years)

Year

Salvage value
($)

Tax deduction
($)

3015

1485

2020

995

1353

667

907

446

907

Practice examination questions


1 A
2 C
3 A
4 C
5 a $1566
b $1434
c $358.50/year
3000
2000
1000
0

Straight line
value
Declining
balance
value

Value ($)

Year

Salvage value
($)

Tax deduction
($)

176 000

24 000

154 880

21 120

136 300

18 580

119 900

16 400

105 500

14 400

c 11 years

CHAPTER 11 The normal


distribution
b 65.7
b 17.3

c 8.1
d 17.032
c 1.1
d 1.3
b Population

Exercise 11A z-scores


1
2
3
4
5
6
7
8

3
-2
a 0
b 1
c -2
d 3
e -1
a 10.5 b 13.7 c 16.9 d 7.3
e 0.9
-0.27
1.5
a -0.48 b 1.44 c 0.08 d -2.24 e 2.8
a 10.3 s
b 10.58 s
c 10.37 s
d 9.88 s
e 10.251 s
f 10.524 s
9 a x = 19.55, sn = 1.76
b 1.68
10 a

Amount
($)

Class
centre

Frequency

020

$10

2040

$30

4060

$50

19

6080

$70

15

80100

$90

b x = 56, sn = 20.1
c i 0.30
ii 2.2
11 B
12 B

iii -2.0

1 a English 1.25, Maths 1.33


b Maths mark is better as it has a higher z-score.
2 2nd test, Barbaras z-score was -0.33 compared to
-0.5 in the first test.
3 B
4 D
5 Course A, z-score of -0.8 compared to -0.75 on
course B
6 a Sydney 0.44, Athens 1
b In Sydney because of the lower z-score
7 C
8 B
9 a Maths x = 59.5, sn = 17.9
Chemistry x = 59.6, sn = 16.8
b Maths 0.25, Chemistry 0.20. Maths is the better
result.
10 Kory is the better candidate with a z-score of 1.5
compared with 0.875 for Ricardo.

10 Quick Questions 1

Are you ready?


1 a 5.75
2 a 2.4
3 a Sample

13 C
14 a x = 64.7, sn = 11.4
b Highest score z = 2.66, Lowest score z = -1.73
15 English 1, Maths 1.31, Biology 1.5, Computing
studies -2, Visual arts 0.67, Music -0.8

Exercise 11B Comparison of scores

0 2 4 6 8 10
Age (years)

6 a $24 000
b

443

answers

Answers

1
2
3
4
5
6
7
8
9
10

2
-2
-1.03
2.95
One standard deviation above the mean
Two standard deviations below the mean
50
8
English 1.25, Maths 1.4
Maths

Exercise 11C Distribution of scores


1
2
3
4
5
6
7

8
9
10
11
12
13
14
15
16

a 68%
b 95%
c 99.7%
a 68%
b 95%
c 99.7%
95%
16%
a 68%
b 16%
c 0.15%
21.1 and 33.9
a 68% of the values have a z-score between -1
and 1.
b 95% of the values have a z-score between -2
and 2.
c 99.7% of the values have a z-score between -3
and 3.
B
A
0.15%
a 16%
b 16%
a 95%
b 16%
c 34%
d 15.85%
e 83.85%
a 95 g to 105 g
b 92.5 g to 107.5 g
163 cm - 181 cm
Faulty, as the one chosen has a z-score greater than 3
2.6 kg - 5 kg

10D
11C

Answers

Chapter review

80
0
0 20 40
Temperature (C)

b The greater the temperature, the fewer pies are


sold. The points on the scatterplot approximate a
straight line and so the relationship can be said to
be linear.
6 a
Hours taken

-2
a 0
b 1
c -2
d 3
e -1
1.87
a 0.17 b 1.83 c -3
d -1.75 e -2
a x = 20.1, sn = 2.1
b Highest = 1.91, Lowest = -1.98
6 a x = 1130, sn = 334.2
b i -0.39 ii 2.05 iii -2.62 iv -1.13 v 3.07
7 a 1.5
b 1
c Physics, higher z-score
8 a Geography: -0.8, Business studies: -0.53
b Business studies: higher z-score
9 Numeracy: lower z-score
10 a 68%
b 95%
c 99.7%
11 a 68%
b 95%
c 99.7%
12 a 34%
b 47.5%
c 2.5%
d 0.15%
e 97.35%
13 Faulty, as it is more than three standard deviations
from the mean.
Practice examination questions
1 B
2 B
3 B
4 D
5 B
6 C
7 a Physics x = 65.1, sn = 5.9
Chemistry x = 62.4, sn = 11.8
b Physics -0.02, Chemistry 0.39
c Chemistry has a higher z-score.
d 53.3 and 76.9
e 27 and 97.8
8 a -2
b Faulty, more than two standard deviations from the
mean

Number of
pies sold

5 a

1
2
3
4
5

30
20
10
0

0 10 20 30
Number in work team

b The more workers on the team reduces the amount


of time taken to unload the ship, and, as the points
on the scatterplot form a straight line, the
relationship is linear.
7 D
8 A
9 a The scatterplot shows a relationship between 2
quantities. As one increases, the other generally
increases.
b The scatterplot shows a relationship between 2
quantities. As one increases, the other generally
decreases.

Exercise 12B Fitting a straight line


by eye
(Note: Best fit lines are indicated as a guide only.)
1 a y
b y

CHAPTER 12 Correlation
Are you ready?
1 a 4
2 a y = 198
3 a 3

b 15
b x = 52
b 1--4-

4 a 6, positive

b 1 1--2- , negative

Number at
cinema

400
200
0

0 20 40
Temperature (C)

200
100
0

20
0
40
0
60
0
80
0

80
40
0
0 40 80
History

Amount spent on
entertainment ($)

Geography

Exercise 12A Scatterplots

Wages ($)

4 a

History

answers

444

x
80
40
0
0 40 80
English

b The greater the English mark, the greater the


History mark generally is. However, as the points
on the scatterplot do not form a straight line, the
relationship is not linear.

5 a

B
A
C

70
60
50
40
30
20
10
0

Lift (kg)

10 20 30 40
Circum. (cm)

Exercise 12C Fitting a straight line


the 3-median method

Cost ( $1000)

20 40 60 80
English

5 10 15 20 25 30
Distance ( 1000 km)

160
80
0

0.

6 8 10 x

Price ($)

Distance (km)

5 10 15 20 25 x

1 $115
5 6h
9 2h

0 100 200 300 400 500 600 700 800 900


Mass (g)

4 8 12 16
Age (years)

Volume (L)

2
1

40 30 20 10 0 10 20 30 40 50 60
Temp. (C)

b V = 0.05T + 3.3

4 3 h 30 min
8 35

Exercise 12D Correlation

4 a
6
5
4
3

2 $235
3 $85
6 2 h 45 min 7 80
10 4 h 30 min

1 a Negative
2 a

b L = 0.05M + 220

10 Quick Questions 1

100

200
100
0

c 92 mm
d 246 mm
e 3 years
f 106 mm
Note: Some answers may vary slightly depending on the
location of the line of best fit.

Speeding
offences

Length (mm)

300

c i $582 ii $1236 iii $740 iv $1762


12 a
b L = 14A + 22

3 a
A

f $1.75
b A = 0.45d + 280

0
10
00
20
00
30
00

ii 43 000

1400
800
0

20 40 60 80 100 120 x

3000
2500
2000
1500
1000
500

200

d i 73 000
e $2.75
11 a
Price ($)

y
70
60
50
40
30
20
10
0

c y = 80x + 750

00

Length of
carapace (mm)

b y = 70 - 0.4x

c C = 0.4d + 2100
d i $8100 ii $2500 iii 14 750 km iv 34 750 km
10 a, b
c N = 135 - 20p

2 4

20 km
d 8.5 km
178 cm
25.74 cm
20C
d 38C

y
70
60
50
40
30
20
10
0

c
b
d
c

4.

2 a y = 5x + 12

15
10
5

00

A C

Weekly sales
( 1000)

Maths

B
80
60
40
20

2.

b S = 1.1C + 24
6 a $17.40 b $8.40
7 a 159.7 cm
c 31.15 cm
8 a 755
b 295
9 a, b

00

445

answers

Answers

3 a
b
c
d
e
f
g

8
4
0

b Positive
c Positive
b Negative

0 20 40 60
Age (years)

Weak positive correlation


Moderate positive correlation
Strong negative correlation
Weak negative correlation
No correlation
Moderate negative correlation
Strong positive correlation

12A
12D

Answers

0 10 20 30
Temperature (C)

16
8
0
0 20 40 60
Hours

12
8
4
0

4
2
0
0 2
Number of cars

b There is no apparent relationship.


4 a, b y
c y = 99 - 5x
80
40
0

0
20 0
0
30 0
00

0
Area ( 1000 km2)

0.12
0.08
0.04
0
0
Amount of drug (mg)

Crowd

0.
4
0.
8
1.
2

c 73
7 a Negative
b Positive
c Negative
8 a, b

b
d
f
h
j

Weak positive
Moderate negative
No correlation
Moderate negative
Perfect negative

11 There is a moderate positive correlation.


12 There is a weak negative correlation.
13 a No correlation
b There is no evidence to connect money and
happiness which confirms the statement.
14 a There is a weak negative correlation between
obesity and exercise.
b There is little evidence that connects the amount
of exercise and obesity.

d 13.4 mg

c Positive

20 000
10 000
0

0 8 16
Number of wins
by home team

9 B

15 B

b $73 000
b F = 107 - 8I

120
80
40
0

8 C
Perfect positive
Strong positive
Weak positive
Strong negative
Weak negative

0 4 8 12
Amount of insecticide (g)

b There is a strong positive correlation indicating


that increasing the amount of the drug given
increases the reaction time.

10 a
c
e
g
i

0 8 16

5 a $242 400
6 a

b There is no correlation evident.


7 a

0 2 4 6
Number of children

b There appears to be a positive relationship which


is linear.
3 a

6000
4000
2000

10

6 a

Population ( 1000)

b There is a moderate positive correlation between


the time spent in airconditioned buildings and the
number of sick days with colds and flu. As hours
spent increases, there is often some increase in
the number of days sick.
c More aspects would need to be examined.

2 a

Number of blowflies

5 a

Number of
sick days

b There is a strong negative correlation between


temperature and the number of pies sold. As
temperature increases the number of pies sold
decreases.

20
10
0
0 20 40
Maximum
temperature (C)

Number of sick days

600
400
200
0

Minimum
temperature (C)

Chapter review

Number of
televisions

Number of pies sold

4 a

Reaction time (s)

answers

446

9 a
c
e
10 a
b
11 a
b

No correlation
b Perfect positive
Moderate negative
d Strong negative
Weak positive
Strong positive
Thicker beams cause greater strengths.
Moderate negative
There is some evidence that older people own
older cars.

Practice examination questions


1 D
2 D
3 C
4 D
5 A
6 a Yes, the median regression line is straight.
b i 713
ii 2.5 m
c i Positive
ii Moderate

19

Year

d 7 billion

e 2050

CHAPTER 13 Spherical geometry


Are you ready?
1 a 69.1 cm
b 88.0 m
c 40 212.4 km
2 a 9.42 m
b 25.6 cm c 41.5 m
3 a 2 hours 40 min
b 210 minutes
c 28 days
d 1 1--4- years
4 a 11.112 km
c 7778.4 m

b 43.20 M
d 4.07 M

Exercise 13A Arc lengths


1 a 25.1 cm
b 56.5 m
d 25.8 km
e 87.3 km
2 630 km
3 a 44.0 cm
b 123 m
d 368 mm
e 11.6 km
4 9400 km
5 31.83 cm
6 a 25.5 m
b 6.73 cm
7 8.73 cm
8 a 33.8 mm
b 20.4 m
d 27.6 cm
e 5.0 km
9 a 40 200 km
b 4470 km
10 a 52 km
b 136 km
c Check with your teacher.
11 a 251 cm
b 62.8 cm
12 a 5.2 cm
b 4.3 m
13 112 km

c 389.6 mm
f 52.2 m
c 188 km
f 688 km

6
7
8
11
12

1 a Cairo
b Shanghai c Darwin
d Montreal
e London
f Auckland
g Tokyo
h Beijing
i Rio de Janeiro
j Oslo
2 These answers are approximate.
a (38S, 145E)
b (40N, 75W)
c (18N, 76W)
d (26S, 28E)
e (42N, 12E)
f (35S, 57W)
g (33N, 44E)
h (55N, 40E)
i (2N, 104E)
j (18S, 178E)

Exercise 13D Distances on the


Earths surface
1
2
3
4
5
6
7
8
9
10
11
12
13
14
15

50
a 40
b 40
c 71
d 21
e 80
60
a 1800 M
b 3334 km
a 2100 M
b 8100 M
c 2340 M
d 5760 M
a 3600 M
b 6667 km
a 2700 M
b 5000 km
6600 km
a 4356 km
b 4021 km
c 3798 km
D
B
7 hours 30 minutes
a 110
b 6600 M
c 12 200 km
d 13 hours 45 minutes
10 800 M
a 28
b 1680 M
c 3111 km
d 3128 km
e 1 M 1.852 km and radius Earth 6400 km.
We are therefore working with approximations.

10 Quick Questions 2
c 5696.8 km

Exercise 13B Great circles and


small circles
1
2
3
4
5

Exercise 13C Latitude and longitude

c 796 km
c 150.8 cm
f 20.7 km

447

answers

c P = 80Y + 2200

8000
6000
4000
2000
0

5
19 0
7
19 0
90
20
10

7 a, b

World population
(millions)

Answers

43.98 cm
a 56.5 m
b 465 mm
c 188 m
40 210 km
314 cm
a 15 320 km
b 38 010 km
c 21 350 km
d 449 200 km e 378 690 km f 160 590 km
g 154 250 km
12.6 m
a 55.3 m
b 40.2 m
c 6911.5 km
B
9 79 cm
10 20 100 km
a 377 cm
b 94.25 cm
a 1750 km
b 52.4 m

10 Quick Questions 1
1 29.5 m
2 180 cm
3 9.4 cm
4 A great circle is the circle of greatest possible
diameter on the surface of the sphere.
5 A small circle is any circle drawn on the surface that
is smaller than a great circle.
6 94.25 cm
7 7100 km
8 3140 km
9 14 450 km
10 33 900 km

1
4
7
10

151 cm
2 (52N, 5E)
(50N, 125W) 5 Rome
36
8 2160 M
4020 km

3 (28S, 153E)
6 Colombo
9 4000 km

Exercise 13E Time zones


1
2
3
4

5
6
7
8
9
10
11
12
13
14
15
16

10 h
a 14 h
b 7h
c 11 h
d 22 h
11:00 pm Monday
a 8:00 pm
b 1:00 pm Friday
c 5:00 pm Wednesday d 11:00 pm Tuesday
e 3:45 pm Monday
5:00 am Saturday
11:00 pm Tuesday
10:00 am Monday
a 10:00 pm
b GMT +11
c i 3:00 pm Monday
ii 8:00 am Friday
a 18 h
b 19 h
c 17 h
7h
a 10 h 20 min b 24 min
c 2 h 48 min
C
D
2:00 pm Tuesday
a 4:00 pm Wednesday b 8:00 am Sunday
a 1:00 am Wednesday b 3:00 am Wednesday

12D
13E

answers

448

Answers

Chapter review
1
2
3
4
5
6
7
8
9
10
11
12
13

a 120.6 cm
a 6.3 m
a 47.1 cm
a 207.3 cm
56.5 km
71 cm
a 41.5 cm
a Manila
a (41N, 3W)
c (43S, 147E)
58
a 16
6 days 6 hours
a i 3060 M

b
b
b
b

54.0 cm
28.1 cm
7.85 cm
44.0 cm

c 289.0 mm
c 21.9 m
c 57.8 m

b 17.6 m
c 9424.8 km
b Lima
c Santiago
b (1N, 104E)
b 960 M

c 1778 km

ii 5667 km

b 6.375 h

14
15
16
17
18

6000 km
a 8h
b 11 h
3:00 am Thursday
3:30 am Tuesday
a 11:00 am the same day
b 11:00 am the same day
19 7:00 pm the same day

c 17 h

Practice examination questions


1 A
2 C
3 C
4 C
5 a 85
b 9500 km c 9 h
6 a Small circle. They lie on the same small circle
because they lie on the same parallel of latitude.
b 12:40 pm Saturday
c 9:20 am Wednesday

Index

449

Index
addition rule for probability 206
algebraic manipulation 1723
algebraic models 28992
angular distance 3979
annuities 23940
future value 2402
future value table 2501
present value 2468
present value table 2513
annuity calculator 2445
annulus, area 44
arc lengths 3856
area
annulus 44
circle 43
composite shapes 489
ellipse 44
irregular figures, using Simpsons rule
parts of the circle 435
sector 43
triangle 1023
see also surface area
area charts 149
assets 301
Australian time zones 402
bearings 869
bivariate data 357
box-and-whisker plots

credit cards 226


calculating interest on a daily basis
interest-free period 22
cube roots 170
cubic expressions 170
cubic functions 281
cylinder
surface area 578
volume 63

535

145

causality 3723
Central Standard Time (CST) 402
circle
area 43
circumference 385, 389
parts of, area 435
comparing data sets 1525
compass bearings 867
compass radial surveys 1201
complementary events 2078
composite shapes, area 489
composite solids, volume 625
compound interest formula 239, 285
cone, volume 63
constant of variation 285
correlation 3703
correlation coefficient 3712
cosine ratio 81
cosine rule
derivation 106
finding angles 11114
finding side lengths 1069
using equation solver to find angles 112
using equation solver to find side lengths 107
cost of a loan 1619
counting techniques 196200
and probability 2013
ordered arrangements 1969
unordered selection 199200

245

data sets
comparison 1525
measures of location and spread 1316
multiple displays 1459
skewness 1401
daylight saving time 403
declining balance depreciation 301, 303, 31012
depreciation 301
declining balance method 301, 31012
modelling 3013
straight line method 301, 3079
tax deductibility 317
depreciation tables 31418
distances on the Earths surface 3979
Earth
latitude and longitude 3936
time zones 4014
Earths surface, distances on 3979
Eastern Standard Time (EST) 401, 402
effective rate of interest formula 16
ellipse, area 44
equation of a median regression line 3604
equation of a straight line, gradientintercept
form 2678, 361
equations
and formulas 1758
solution by substitution 1802
equator 393
error in measurement 6970
expected outcomes 21921
exponential decay 283, 303
exponential functions 2823
factorial function 197
financial expectation 2234
First Index Law 173
flat rate interest 36
comparison with reducible rates of interest 1619
flat rate interest loan calculator 89
flat rate loan, effective rate of interest 1619
future value of an annuity 2402
future value of an annuity formula 240
future value table 2501
gradient formula 268
gradientintercept form, equation of a straight
line 2678, 361

450

Index

graphing
physical phenomena 28992
variations 2878
graphs
cubic functions 281
exponential functions 2823
hyperbolic functions 281
linear functions 26871
quadratic functions 2748
great circles 389, 393
distance between two points 3979
Greenwich Mean Time (GMT) 401, 402
Greenwich Meridian 393, 401
home loan calculator 10
home loans 912
loan repayment function 21
hyperbolas 281
hyperbolic functions 281, 2867
index laws 173
International Date Line 393
interquartile range (IQR) 132
intersection of two graphs 2712
inverse variation 2867
irregular figures, area 535
knot

398

latitude 3936
linear equations, gradientintercept form 2678, 361
linear expressions 169
linear functions 26772
finding the intersection of two graphs 2712
graphing 26870
graphing practical functions 2701
straight line depreciation 301
linear relationships (scatterplots) 353
loan repayments
calculated using present value formula 2557
reducing balance loan 2931, 33
longitude 3936
and time difference 403
map coordinates 393, 395
mean 131, 140, 329
measurement error 6970
measures of location 131, 1336
measures of spread 132, 1336
median 131, 140, 329
median regression lines 35964
meridians of longitude 393
mode 135, 140, 329
modelling depreciation 3013
multiple data sets
displaying 1459
storing 147
multiplication rule of probability 205
nautical mile 398
negative correlation 372
negatively skewed 141

non-linear relationships (scatterplots)


normal distribution 140, 329
and standard deviation 33941
and z-scores 33941

353

offset surveys 118


ordered arrangements 1969
and tree diagrams 198
ordered selection 1989
outliers 134
parabola 2748
parallels of latitude 393, 397
physical phenomena, graphing 28992
plane table radial surveys 11820
population standard deviation 330
positive correlation 371
positively skewed 141
present value of an annuity 2468
present value of an annuity formula 246, 255
present value table 2513
prism, volume 62
probability
addition rule 206
and counting techniques 2013
complementary events 2078
expected outcomes 21921
financial expectation 2234
multiplication rule 205
two-way tables 2279
probability formula 194
probability trees 2058
pyramid, volume 63
quadrant 43
quadratic expressions 170
quadratic functions 2748
general form 275
graphing 276
maximum value 276
minimum value 275, 276
radar charts 148
radial surveys 11821
range 132
reducible rates of interest
comparison with flat rate interest 1619
home loans 912
reducing balance loan 912
loan repayments 2931, 33
monthly repayment per $1000 borrowed 30
regression lines 35764
finding equation of 3604
fitting a straight line by equal-number-of-points
method 358
fitting a straight line by eye 3579
fitting a straight line by the 3-median method
35963
right-angled triangles, review 814
salvage value 307, 311
and tax deductions 317

Index

sample standard deviation 330


scatterplots 3514
and correlation 3712
drawing 3523
linear relationships 353
no relationship 354
non-linear relationships 353
scientific notation 1845
Second Index Law 173
sector, area 43
simple interest formula 3
Simpsons rule 535
simulations 2223
sine ratio 81
sine rule
derivation 92
finding angle sizes 979
finding side lengths 915
using equation solver to find angles 98
using equation solver to find side lengths
skewness 1401
small circles 38990, 393
sphere
great circles 389, 393
small circles 38990, 393
surface area 58
volume 63
square roots 170
standard deviation 132, 140
and normal distribution 33941
and z-scores 32930
standardised scores see z-scores
statistical data, displaying 152
stem-and-leaf plots 145
straight line, equation of 2678, 361
straight line depreciation 301, 3079
substitution 16971
solution by 1802
solving equations arising from 1757
summary statistics 132
comparing 154

451

surface area
cylinder 578
minimising 61
sphere 58
surveying 11821

93

table function (graphics calculator) 181


tangent ratio 81
term of the loan 3
Third Index Law 173
time difference 403
time zones 4014
tree diagrams 1935, 205
and ordered arrangements 198
see also probability trees
triangle, area 1023
trigonometric ratios 814
using equation solver to find side lengths 82
using equation solver to find the size of an
angle 834
true bearings 879
two-way tables 153, 2279
unordered selection

199200

variations 2858
graphing 2878
volume
composite solids 625
cone 63
cylinder 63
prism 62
pyramid 63
sphere 63
Western Standard Time (WST)
z-scores 32932
comparison 3345
distribution 33941

402

Вам также может понравиться